05.03.2015 Views

clerkship handbook - University of Hawaii – Department of Medicine

clerkship handbook - University of Hawaii – Department of Medicine

clerkship handbook - University of Hawaii – Department of Medicine

SHOW MORE
SHOW LESS

Create successful ePaper yourself

Turn your PDF publications into a flip-book with our unique Google optimized e-Paper software.

UNIVERSITY OF HAWAII<br />

JOHN A. BURNS SCHOOL OF MEDICINE<br />

DEPARTMENT OF MEDICINE<br />

THIRD-YEAR CLERKSHIP IN<br />

INTERNAL MEDICINE<br />

MEDICINE 531 (6B)<br />

MEDICINE 532 (6L)<br />

CLERKSHIP HANDBOOK<br />

Revised 02/25/13<br />

1


TABLE OF CONTENTS<br />

CONTACT INFORMATION ................................................................................................................................................... 3<br />

SUMMARY TABLE OF CLERKSHIP REQUIREMENTS ................................................................................................. 4<br />

TOP 10 WAYS TO EXCEL ON THE INTERNAL MEDICINE CLERKSHIP ................................................................. 5<br />

DESCRIPTION OF THE THIRD-YEAR CLERKSHIP IN INTERNAL MEDICINE ...................................................... 6<br />

INTRODUCTION ..................................................................................................................................................................... 6<br />

Goal <strong>of</strong> the Clerkship ........................................................................................................................................ 6<br />

Design <strong>of</strong> the Clerkship .................................................................................................................................... 6<br />

Work Hours, Designated Study Time, Days Off and Holidays ..................................................................... 6<br />

Absences ........................................................................................................................................................... 7<br />

CURRICULUM ....................................................................................................................................................... 8<br />

Learning Objectives .......................................................................................................................................... 9<br />

Learning Strategies .......................................................................................................................................... 9<br />

EVALUATION IN THE THIRD-YEAR CLERKSHIP IN INTERNAL MEDICINE ..................................................... 9<br />

General Guidelines ........................................................................................................................................... 9<br />

Retaking Exams .............................................................................................................................................. 10<br />

Honors.............................................................................................................................................................. 10<br />

SPECIFIC REQUIREMENTS AND GUIDELINES................................................................................................ 12<br />

Inpatient <strong>Medicine</strong> ........................................................................................................................................... 12<br />

Ambulatory <strong>Medicine</strong> ...................................................................................................................................... 16<br />

PBL Tutorial ..................................................................................................................................................... 18<br />

Role Descriptions for <strong>Medicine</strong> 531/532 ....................................................................................................... 19<br />

MISCELLANEOUS CLERKSHIP INFORMATION ......................................................................................................... 24<br />

GUIDELINES FOR APPROPRIATE APPEARANCE AND ATTIRE .................................................................... 25<br />

RECOMMENDED RESOURCES ......................................................................................................................... 26<br />

REQUIRED EQUIPMENT .................................................................................................................................... 27<br />

EXPOSURE TO BLOOD/BODY FLUIDS PROTOCOL ....................................................................................... 28<br />

HIV MEDICINE ..................................................................................................................................................... 29<br />

MEDICINE T-RES INSTRUCTIONS .................................................................................................................... 32<br />

STUDENT EVALUATION OF TEACHERS .......................................................................................................... 34<br />

EVALUATION FORMS ......................................................................................................................................................... 35<br />

OBSERVED HISTORY AND PHYSICAL ............................................................................................................. 36<br />

THE BASIC PHYSICAL EXAMINATION SEQUENCE (BPES) ........................................................................... 37<br />

OBSERVED PATIENT COUNSELING ................................................................................................................ 48<br />

CASE PRESENTATION ....................................................................................................................................... 49<br />

SMALL GROUP LEARNING EXPERIENCE ....................................................................................................... 50<br />

MID-CLEKSHIP FEEDBACK FORM ................................................................................................................... 51<br />

STUDENT EVALUATION FORM ......................................................................................................................... 52<br />

OTHER FORMS ...................................................................................................................................................................... 56<br />

INPATIENT MEDICINE AGREEMENT FORM ..................................................................................................... 57<br />

INPATIENT WORK HOURS LOG ........................................................................................................................ 59<br />

PHYSICIAN’S ORDERS ...................................................................................................................................... 60<br />

APPENDICES .......................................................................................................................................................................... 61<br />

TRAINING PROBLEMS LIST .............................................................................................................................. 62<br />

GENERAL CLINICAL CORE COMPETENCIES IN INTERNAL MEDICINE ..................................................... 192<br />

LIST OF ERROR-PRONE ABBREVIATIONS, SYMBOLS, AND DOSE DESIGNATIONS ............................... 234<br />

COMPREHENSIVE WRITE-UPS ....................................................................................................................... 238<br />

Definition <strong>of</strong> Comprehensive Write-up ....................................................................................................... 238<br />

Submission Requirements <strong>of</strong> Comprehensive Write-ups ........................................................................ 238<br />

Example <strong>of</strong> Inpatient History and Physical ................................................................................................ 239<br />

Example <strong>of</strong> Ambulatory Note ....................................................................................................................... 246<br />

CLINICAL SKILLS EXAMINATION (CSE) ........................................................................................................ 251<br />

NBME SUBJECT EXAM IN INTERNAL MEDICINE .......................................................................................... 256<br />

2


CONTACT INFORMATION<br />

Clerkship Director<br />

Clerkship Coordinator:<br />

Hospital Site Coordinators:<br />

Laurie M. Tam, M.D., F.A.C.P.<br />

Assistant Pr<strong>of</strong>essor <strong>of</strong> <strong>Medicine</strong><br />

1356 Lusitana Street, 7 th Fl.,<br />

Honolulu, HI 96813<br />

Telephone: (808) 586-7460 FAX: 586-7486<br />

e-mail: lmtam@hawaii.edu<br />

Ms. Julieta Rajlevsky<br />

1356 Lusitana Street, 7 th Fl.<br />

Honolulu, HI 96813<br />

Telephone: (808) 586-7478 FAX: 586-7486<br />

e-mail: jlrajlev@hawaii.edu<br />

Queens Medical Center:<br />

Miki Kiyokawa, M.D.<br />

Assistant Pr<strong>of</strong>essor <strong>of</strong> <strong>Medicine</strong><br />

QET9, 1301 Punchbowl Street,<br />

Honolulu, HI 96813<br />

Telephone: 586-2910 FAX: 586-7486<br />

e-mail: kmcsitecoordinator@yahoo.com<br />

Kuakini Medical Center:<br />

Miki Kiyokawa, M.D.<br />

Assistant Pr<strong>of</strong>essor <strong>of</strong> <strong>Medicine</strong><br />

Office <strong>of</strong> Medical Education<br />

347 N. Kuakini St., Honolulu, HI 96817<br />

Telephone: 547-9226 FAX: 547-9867<br />

e-mail: kmcsitecoordinator@yahoo.com<br />

Tripler Army Medical Center:<br />

Bethany Sonobe, MD<br />

Internal <strong>Medicine</strong><br />

Telephone: 433-2638 Pager: 363-1423<br />

e-mail: bethany.sonobe@amedd.army.mil<br />

3


Third-Year Clerkship in Internal <strong>Medicine</strong><br />

SUMMARY TABLE OF CLERKSHIP REQUIREMENTS<br />

Inpatient Ambulatory<br />

1. Call<br />

a. Every time your team is on call 6B & 6L -<br />

6B & 6L<br />

-<br />

b. Overnight call once (preferably Friday night)<br />

at KMC<br />

2. Comprehensive Write-ups<br />

a. 1 Comprehensive Write-up per week<br />

6B & 6L<br />

(3 - 6 total)<br />

b. 2 Comprehensive Write-ups per week (10 total) 6B<br />

c. 2 Comprehensive Write-ups per month (10 total) - 6L<br />

3. Required Clerkship Activities<br />

a. UH <strong>Department</strong> <strong>of</strong> <strong>Medicine</strong> Grand Rounds<br />

(designated Tues, 12:30-1:30 pm)<br />

6B & 6L<br />

(All students<br />

except TAMC<br />

6B<br />

(All students<br />

except TAMC)<br />

b. PBL Tutorial 6B (1x/week)<br />

6L (1x/week)<br />

c. Bedside Clinical Skills (1-2x/week) 6B & 6L -<br />

d. Chief Medical Resident Rounds (1x/week) 6B & 6L -<br />

e. CVPE (usually Tues pm) 6L 6B<br />

f. EBM 1 and 2 (usually Wed pm) 6L 6B<br />

g. EKG Workshop (usually Tues pm) 6L 6B<br />

h. HIPSTER (Sim Session) 6B & 6L -<br />

6B (1x/week)<br />

6L (1x/month)<br />

i. HIV <strong>Medicine</strong> (Tues 1:30-5:00 pm) 6L 6B<br />

j. Neuro 1 and 2 (usually Tues pm) 6L 6B<br />

k. Site-specific conferences 6B & 6L 6B & 6L<br />

4. Learning Objectives<br />

a. Training Problems (33) 6B & 6L 6B & 6L<br />

b. General Clinical Core Competencies in IM (17) 6B & 6L 6B & 6L<br />

5. Evaluation<br />

a. Observed History and Physical (with BPES)<br />

6B & 6L -<br />

(by end <strong>of</strong> 2 nd wk <strong>of</strong> inpatient)<br />

b. Observed Patient Counseling (2) 6B & 6L<br />

c. Mid-Clerkship Feedback Form 6B & 6L 6B & 6L<br />

d. Student Evaluation Forms 6B & 6L 6B & 6L<br />

e. Clinical Skills Exam (Saturday, ____________ ) End <strong>of</strong> Clerkship<br />

f. NBME Exam (Friday, ____________________) End <strong>of</strong> Clerkship<br />

4


TOP 10 WAYS TO EXCEL ON THE INTERNAL MEDICINE CLERKSHIP<br />

1. Find out what your residents and preceptors expect <strong>of</strong> you. Meet and try to exceed their<br />

expectations. Follow through on every assigned task.<br />

2. Be actively involved in the care <strong>of</strong> your patients to the greatest extent possible. Go the<br />

extra mile for your patients. You will benefit as much as they will.<br />

3. Go the extra mile for your team. Additional learning will follow. The more you put in, the<br />

more you will gain.<br />

4. Read consistently and deeply about the problems your patients face. Raise what you<br />

learn in your discussions with your team and in your notes. Educate your team members<br />

about what you learn whenever possible.<br />

5. Learn to do excellent presentations as early as possible. This will make you more effective<br />

in patient care and gain the confidence <strong>of</strong> your supervisors to allow you more involvement<br />

in patient care.<br />

6. Ask good questions.<br />

7. Speak up—share your thoughts in teaching sessions, share your opinions about your<br />

patients’ care, constructively discuss how to improve the education you are receiving and<br />

the systems around you.<br />

8. Actively seek feedback and reflect on your experiences.<br />

9. Keep your goals focused on the right priorities, in the following order: patient care,<br />

learning, and personal satisfaction. You should always strive to meet all three goals.<br />

10. Always be enthusiastic. Be caring and conscientious and strive to deliver outstanding<br />

quality to your patients as you learn as much as you can from every experience.<br />

From: Primer to the Internal <strong>Medicine</strong> Clerkship, Second Edition, A Guide Produced by the<br />

Clerkship Directors in Internal <strong>Medicine</strong> (CDIM), c2008, 2nd edition<br />

Download free <strong>of</strong> charge from:<br />

http://www.im.org/Publications/PhysiciansInTraining/Pages/Primer.aspx<br />

5


DESCRIPTION OF THE THIRD-YEAR CLERKSHIP IN INTERNAL MEDICINE<br />

INTRODUCTION<br />

Goal <strong>of</strong> the Clerkship<br />

The goal <strong>of</strong> the Third-Year Clerkship in Internal <strong>Medicine</strong> is to provide the opportunity to develop<br />

experience and competence in Internal <strong>Medicine</strong>, which serves as the foundation for all<br />

specialties and their respective residencies. To practice competently, a physician must obtain and<br />

retain a large body <strong>of</strong> knowledge, master the skills <strong>of</strong> clinical practice, develop efficient problem<br />

solving techniques and demonstrate compassion, integrity, self-discipline and life-long learning<br />

skills. It is the student’s responsibility to utilize this <strong>clerkship</strong> experience to accomplish these<br />

goals. It is the <strong>Department</strong> <strong>of</strong> <strong>Medicine</strong>’s responsibility to assure that every graduate <strong>of</strong> the John<br />

A. Burns School <strong>of</strong> <strong>Medicine</strong> has obtained graduation level competency in Internal <strong>Medicine</strong>.<br />

The core <strong>of</strong> this <strong>clerkship</strong> is “Patient-Based Learning,” which occurs as students evaluate patients<br />

through history taking and physical examinations, develop comprehensive assessments including<br />

appropriate differential diagnoses, formulate diagnostic, therapeutic and education plans and then<br />

provide care and follow-up appropriate to the inpatient or ambulatory setting.<br />

Design <strong>of</strong> the Clerkship<br />

Third-Year Clerkship in Internal <strong>Medicine</strong> (MED 531/532)<br />

MED 531 for 6B students is 11 weeks in length and consists <strong>of</strong> 5-1/2 or 6 weeks <strong>of</strong> Inpatient<br />

<strong>Medicine</strong> and 5-1/2 or 5 weeks <strong>of</strong> Ambulatory <strong>Medicine</strong>. MED 532 for 6L students consists <strong>of</strong> 6<br />

weeks <strong>of</strong> Inpatient <strong>Medicine</strong> (block) and 25 half days <strong>of</strong> ambulatory medicine (clinics - one half<br />

day weekly for 21-22 weeks plus 3-4 additional half days).<br />

Work Hours, Designated Study Time, Days Off and Holidays<br />

Work hours<br />

No student should work more than eighty (80) hours per week, averaged over the course <strong>of</strong> the<br />

<strong>clerkship</strong>.<br />

The <strong>clerkship</strong> recognizes that excessive work hours do not promote student well-being and may<br />

endanger students, their patients and others with whom they interact.<br />

For inpatient medicine, the earliest time that any student is permitted to arrive at the hospital is<br />

4:00 a.m. and the earliest time that any student is permitted to see any patient, that is, to actually<br />

talk to and examine, is 4:30 a.m. (excluding emergency situations such as Code Blues). (See<br />

Inpatient <strong>Medicine</strong>)<br />

The Hospital Site Coordinators and Chief Medical Residents are aware <strong>of</strong> these work hour<br />

guidelines.<br />

If a student is not able to complete his/her work within these work hour guidelines, the student is<br />

advised to see his/her Hospital Site Coordinator, Chief Medical Resident or Supervising Resident.<br />

6


If a student does not follow these guidelines, the Hospital Site Coordinator and Chief Medical<br />

Resident are required to advise the student and notify the <strong>clerkship</strong>.<br />

It is the <strong>clerkship</strong>'s goal that students will become more efficient with experience, that they will be<br />

able to arrive at the hospital and see patients at reasonable times and that they will be able to<br />

maintain reasonable work hours.<br />

"Designated study time"<br />

Students should have one (1) afternoon <strong>of</strong> "designated study time" per week, averaged over<br />

the course <strong>of</strong> the <strong>clerkship</strong>.<br />

"Designated study time" is defined as time Monday through Friday and morning through<br />

afternoon away from patient care responsibilities that is devoted to studying which includes<br />

reading, completing write-ups, preparing for required <strong>clerkship</strong> activities such as PBL Tutorials,<br />

Bedside Clinical Skills rounds, Chief Rounds and other inpatient or ambulatory activities.<br />

Students will have "designated study time" when their patient care responsibilities and any<br />

required activities are finished. (See Summary Table <strong>of</strong> Required <strong>Medicine</strong> Clerkship Activities)<br />

In both the inpatient and ambulatory settings, students are advised to notify their team and<br />

preceptor(s), respectively, when they are leaving to study.<br />

"Designated study time" should be used for educational activities, as described above, and not for<br />

leisure activities. Therefore, academic action may be initiated against students who abuse the<br />

"designated study time," as determined by the <strong>Department</strong> <strong>of</strong> <strong>Medicine</strong> Student Education<br />

Committee. Likewise, inpatient and ambulatory sites that do not follow the guidelines on<br />

"designated study time" should be brought to the attention <strong>of</strong> the <strong>clerkship</strong> director.<br />

The one day before the NBME Exam is designated a full day <strong>of</strong> “designated study time” for<br />

<strong>clerkship</strong> students.<br />

Days <strong>of</strong>f<br />

Students must have one (1) day <strong>of</strong>f per week, averaged over the course <strong>of</strong> the <strong>clerkship</strong>.<br />

On Inpatient <strong>Medicine</strong>, there is one (1) day <strong>of</strong>f each week, usually a Saturday or a Sunday. The<br />

day <strong>of</strong>f will be determined by the site’s Hospital Site Coordinator and Chief Medical Resident.<br />

On Ambulatory <strong>Medicine</strong>, there are usually two (2) days <strong>of</strong>f each week, usually Saturday and<br />

Sunday. The days <strong>of</strong>f will be determined by the Ambulatory Preceptor.<br />

Holidays<br />

Students will follow the holiday schedule that is observed at their training site.<br />

Absences<br />

Absences from Inpatient <strong>Medicine</strong><br />

On each day that you are absent, you must notify your Chief Medical Resident and your Hospital<br />

Site Coordinator.<br />

You may be required to make up any time missed. This decision will be made together by your<br />

Chief Medical Resident, Hospital Site Coordinator and Clerkship Director and is dependent on the<br />

reason for absence, length <strong>of</strong> absence, <strong>clerkship</strong> activities missed and your <strong>clerkship</strong><br />

7


performance to date.<br />

Absences from Ambulatory <strong>Medicine</strong><br />

On each half day that you are absent, you must notify your Ambulatory Preceptor and the Clerkship<br />

Director (Dr. Laurie Tam at lmtam@hawaii.edu).<br />

Each half day missed must be made up.<br />

6B students usually have 2-4 weekday afternoons and every Saturday <strong>of</strong>f each week. They<br />

should try to schedule their make up during these times, at the convenience <strong>of</strong> their Ambulatory<br />

Preceptor.<br />

6L ambulatory students usually have 3 half days and every Saturday <strong>of</strong>f each week. They should<br />

try to schedule their make up during these times, at the convenience <strong>of</strong> their Ambulatory<br />

Preceptor.<br />

Notes<br />

Attendance is mandatory for all <strong>Medicine</strong> Clerkship orientations and exams.<br />

If you are absent for more than three (3) days, totaled over the course <strong>of</strong> the <strong>clerkship</strong>, the<br />

<strong>clerkship</strong> is required to report this to the Office <strong>of</strong> Student Affairs.<br />

Required make up for time missed must be completed by the end <strong>of</strong> <strong>Medicine</strong> Clerkship in order<br />

to receive Credit for the <strong>clerkship</strong>. Ideally, required make up should be completed in the same<br />

half <strong>of</strong> the <strong>clerkship</strong> that time was missed.<br />

Questions regarding absences should be directed to the Clerkship Director (Dr. Laurie Tam at<br />

lmtam@hawaii.edu).<br />

CURRICULUM<br />

The <strong>Medicine</strong> Clerkship Curriculum is based on the Clerkship Directors in Internal <strong>Medicine</strong><br />

(CDIM)-Society <strong>of</strong> General Internal <strong>Medicine</strong> (SGIM) Core <strong>Medicine</strong> Clerkship Curriculum<br />

Guide Version 3.0, which was released in 2006 to all <strong>Medicine</strong> <strong>clerkship</strong> directors in the United<br />

States. The third edition outlines thirty-three (33) Training Problems and seventeen (17)<br />

General Clinical Core Competencies, which are aligned with the Accreditation Council for<br />

Graduate Medical Education (ACGME) general competencies. The Training Problems and<br />

General Clinical Core Competencies are printed in the <strong>clerkship</strong>’s Student Handbook and<br />

website.<br />

The <strong>Medicine</strong> Clerkship Curriculum, which has been reviewed by JABSOM’s <strong>Department</strong><br />

<strong>of</strong> <strong>Medicine</strong> Student Education Committee (SEC), is aligned with and fulfills JABSOM’s<br />

Graduation Objectives. The curriculum provides third-year medical students with the<br />

opportunity to develop and demonstrate competence and excellence in the medical<br />

knowledge, clinical skills and pr<strong>of</strong>essionalism expected in the evaluation and care <strong>of</strong> the<br />

adult patient.<br />

8


Learning Objectives<br />

a. Training Problems (See Appendix A)<br />

Appendix B)<br />

b. General Clinical Core Competencies in Internal <strong>Medicine</strong> (See<br />

Learning Strategies<br />

a. After seeing each patient, students should identify which Training<br />

Problems were manifested, read and study those Training Problems and assess whether<br />

they are able to meet the specific learning objectives for each (Appendix A).<br />

b. Students should read and study the General Clinical Core<br />

Competencies in Internal <strong>Medicine</strong> and assess whether they are able to meet the<br />

specific learning objectives for each (Appendix B).<br />

c. Students should refer to the Recommended Resources for their<br />

reading and studying. .<br />

EVALUATION IN THE THIRD-YEAR CLERKSHIP IN INTERNAL MEDICINE<br />

General Guidelines<br />

The <strong>clerkship</strong> uses a "competency-based" system to assess the three domains <strong>of</strong> Medical Knowledge,<br />

Skills and Pr<strong>of</strong>essionalism.<br />

For Medical Knowledge, competency is assessed throughout the <strong>clerkship</strong> by faculty and resident<br />

observations <strong>of</strong> students’ clinical performance. It is also assessed by performance on the NBME<br />

Subject Examination in Internal <strong>Medicine</strong> administered at the end <strong>of</strong> the <strong>clerkship</strong><br />

For Clinical Skills, competency is assessed throughout the <strong>clerkship</strong> by faculty and resident<br />

observations <strong>of</strong> students’ clinical performance. It is also assessed by performance on the Clinical<br />

Skills Exam (CSE) conducted by the Center for Clinical Skills near the end <strong>of</strong> the <strong>clerkship</strong>.<br />

For Pr<strong>of</strong>essionalism, competency is assessed throughout the <strong>clerkship</strong> by faculty and resident<br />

observations <strong>of</strong> students’ clinical performance.<br />

a. Hospital Site Coordinators and Ambulatory Preceptors give students mid-<strong>clerkship</strong> feedback on<br />

the students’ clinical performance, reviewing the students’ strengths and weaknesses and<br />

discussing specific strategies to improve. Therefore the <strong>clerkship</strong> expects students to<br />

demonstrate improvement in their clinical performance by the end <strong>of</strong> the <strong>clerkship</strong>.<br />

b. The UH <strong>Department</strong> <strong>of</strong> <strong>Medicine</strong> Student Education Committee reviews each student’s entire<br />

<strong>clerkship</strong> performance, including Clinical Performance (inpatient and ambulatory), CSE and<br />

NBME Exam to determine each student’s grade.<br />

c. The <strong>clerkship</strong> does not use numerical formulas for evaluation and grading.<br />

d. For questions about <strong>clerkship</strong> grades, refer to JABSOM’s Academic Appeals Policy.<br />

9


Credit<br />

a. To earn Credit for the Third-Year Clerkship in Internal <strong>Medicine</strong> (MED 531 for 6B students;<br />

MED 532 for 6L students), students must demonstrate Clerkship Level Competency in all three<br />

domains, as reflected by their Clinical Performance (inpatient and ambulatory), CSE and NBME<br />

Exam.<br />

For Clinical Performance (inpatient and ambulatory rotations), students need to receive ratings <strong>of</strong><br />

Competent or higher.<br />

On the CSE, students need to score 55 or higher.<br />

On the NBME Exam, students need to score 60 or higher.<br />

Students who do not demonstrate Clerkship Level Competency in all three domains by the end <strong>of</strong><br />

the <strong>clerkship</strong> may be required to repeat part or all <strong>of</strong> the <strong>clerkship</strong> and/or retake the CSE and/or<br />

NBME Exam.<br />

b. To earn Credit for Advanced <strong>Medicine</strong> (MED 541), which is a JABSOM graduation requirement,<br />

students must also demonstrate Graduation Level Competency on their CSE and NBME Exam<br />

by December 31 <strong>of</strong> the fourth year.<br />

On the CSE, students need to score 60 or higher.<br />

On the NBME Exam, students need to score 67 or higher.<br />

Students who do not demonstrate Graduation Level Competency on their CSE and/or NBME<br />

Exam by the end <strong>of</strong> the <strong>clerkship</strong> will be required to retake the respective exam. They may also<br />

be required to complete a designated fourth-year <strong>Medicine</strong> elective before retaking the exam.<br />

Retaking Exams<br />

CSE<br />

Students who score 55-59 on their CSE must retake the CSE and score 60 (Graduation Level<br />

Competency) or higher by December 31 <strong>of</strong> their fourth year. Students will have up to two (2)<br />

opportunities to achieve this.<br />

NBME Exam<br />

Students who score 60 - 66 on their NBME Exam must retake the NBME Exam and score 67<br />

(Graduation Level Competency) or higher by December 31 <strong>of</strong> their fourth year. Students will have up<br />

to two (2) opportunities to achieve this.<br />

Honors<br />

The <strong>Department</strong> <strong>of</strong> <strong>Medicine</strong> gives all students the opportunity to earn an "Honors" grade for exceptional<br />

performance in the Third-Year Clerkship in Internal <strong>Medicine</strong>.<br />

To earn Honors for the Third-Year Clerkship in Internal <strong>Medicine</strong> (MED 531 for 6B students; MED<br />

532 for 6L students), students must demonstrate mostly Exceptional performance in all three domains,<br />

as reflected by their Clinical Performance (inpatient and ambulatory), CSE and NBME Exam by the end<br />

<strong>of</strong> the <strong>clerkship</strong>.<br />

10


SUMMARY OF EVALUATION<br />

1. Medical Knowledge:<br />

o Clinical performance (Student Evaluation Form)<br />

(Interns, Residents, Chief Resident, Inpatient and Ambulatory Attendings)<br />

Credit for Credit for<br />

Clerkship Advanced <strong>Medicine</strong><br />

Competency (in 3 rd yr) (by 12/31 <strong>of</strong> 4 th yr) Consequence<br />

- - - Repeat the <strong>clerkship</strong>,<br />

partially or entirely<br />

Clerkship Level + +<br />

o NBME Subject Exam in Internal <strong>Medicine</strong> (Shelf Exam)<br />

Credit for Credit for<br />

NBME Clerkship Advanced <strong>Medicine</strong><br />

Score Competency (in 3 rd yr) (by 12/31 <strong>of</strong> 4 th yr) Consequence<br />


3. Pr<strong>of</strong>essionalism:<br />

o Clinical performance (reflected on Student Evaluation Form)<br />

(Interns, Residents, Chief Resident, Inpatient and Ambulatory Attending)<br />

Credit for Credit for<br />

Clerkship Advanced <strong>Medicine</strong><br />

Competency (in 3 rd yr) (by 12/31 <strong>of</strong> 4 th yr) Consequence<br />

- - - Repeat the <strong>clerkship</strong>,<br />

partially or entirely<br />

Clerkship Level + +<br />

SPECIFIC REQUIREMENTS AND GUIDELINES<br />

Inpatient <strong>Medicine</strong><br />

Schedule:<br />

1) For 6B students, Inpatient <strong>Medicine</strong> is 5-1/2 weeks (if in the first half <strong>of</strong> the academic year) or 6<br />

weeks (if in the second half <strong>of</strong> the academic year).<br />

2) For 6L students, Inpatient <strong>Medicine</strong> is 6 weeks.<br />

Site: The student will be assigned to Kuakini Medical Center (KMC), Queen’s Medical Center<br />

(QMC) or Tripler Army Medical Center (TAMC).<br />

Orientation: The student will be oriented to the site by its Hospital Site Coordinator(s) (HSC) and<br />

Chief Medical Resident (CMR).<br />

Team Assignment: The student will be assigned to a medical team where an Upper Level<br />

Resident will be directly responsible for the student’s supervision. The student will also work with<br />

the Intern(s) on the team.<br />

Call:<br />

1) Students must take call every day that their team is on call. Students may leave earlier if<br />

their patient care responsibilities are complete, with the Upper Level Resident’s approval.<br />

2) Students at KMC must take one (1) overnight call with their team, preferably a Friday night<br />

or a Saturday night.<br />

Work Hours (See Work Hours, Dedicated Study Time and Days Off):<br />

1) The earliest time that students are permitted to arrive at the hospital is 4:00 a.m. at KMC<br />

and 5:00 a.m. at QMC and TAMC.<br />

2) The earliest time that students are permitted to see patients, that is, to actually talk to and<br />

examine patients, is 4:30 a.m. at KMC and 5:30 a.m. at QMC and TAMC (excluding emergency<br />

situations such as Code Blues).<br />

3) The Hospital Site Coordinators, Chief Medical Residents and Upper Level Residents are<br />

aware <strong>of</strong> these work hour guidelines. If a student is not able to complete his/her work within<br />

these work hour guidelines, the student is advised to see his/her Hospital Site Coordinator, Chief<br />

12


Medical Resident or Upper Level Resident. If a student does not follow these work hour<br />

guidelines, the Hospital Site Coordinator, Chief Medical Resident or Upper Level Resident are<br />

required to advise the student and notify the <strong>clerkship</strong>.<br />

Patient Assignment:<br />

1) The Upper Level Resident will be responsible for assigning patients to the student.<br />

Patients should be selected for their ability to cooperate and communicate, as well as for their<br />

specific medical problems.<br />

2) The student has a list <strong>of</strong> Training Problems on which the Third-Year Clerkship in Internal<br />

<strong>Medicine</strong> curriculum and specific learning objectives are based. The student’s goal is to see at<br />

least one (1) inpatient patient with each <strong>of</strong> thirty-two (32) Training Problems. Note that the Training<br />

Problem does not have to be the patient’s Chief Complaint. In fact, a patient may present with many<br />

Training Problems. (See Training Problems section)<br />

Patient Census: The student should admit 1 – 2 patients per call. The student should actively<br />

follow an average <strong>of</strong> two (2) patients at all times (maximum 5 patients).<br />

Patient Care Responsibilities:<br />

1) The student should complete an Initial History and Physical on each and every patient<br />

assigned to the student, whether or not the student was present for the patient’s admission (i.e.<br />

transfer patients, etc). This Initial History and Physical includes interviewing and examining<br />

patients either independently or while being observed by a resident or attending. If the student<br />

observes a resident or attending obtain the History and Physical, this observation does not qualify as<br />

the student’s own History and Physical. The student would have to go back at another time and<br />

perform his/her own independent History and Physical “Group” work-ups are not allowed or<br />

accepted.<br />

2) The student should perform the physical exam which was taught as the Basic Physical<br />

Exam Sequence (BPES) in the pre-clinical years. Note that the funduscopic exam is included in the<br />

BPES. The student will also be expected to perform appropriate Branch Steps as needed. The<br />

Upper Level Resident is responsible for insuring proper supervision <strong>of</strong> the following parts <strong>of</strong> the<br />

physical exam which may be performed by the student if clinically indicated: female breast exam,<br />

female genital and/or pelvic exam, male genital and/or prostate exam, and female and male<br />

rectal exam. This means that the student must be supervised by a physician (interns, upper level<br />

residents, chief residents or attendings) who is certified or has expertise to competently perform the<br />

exam in question.<br />

4) The student is expected to Pre-Round and write daily Progress Notes on all his/her<br />

assigned patients before the Intern or Upper Level Resident(s) write their note(s). The Intern or<br />

Upper Level Resident should review the notes with the student, give constructive feedback and<br />

countersign the notes. Any missed history or physical exam finding should be noted, corrected and<br />

demonstrated as needed.<br />

5) The student should Round with the team. Since the student will be expected to have<br />

detailed knowledge <strong>of</strong> his/her assigned patients, the student is expected to take the lead in<br />

discussing his/her patients, including the student’s assessment <strong>of</strong> the patients’ problems and<br />

the student’s plans. In addition, the student will be expected to have a general knowledge <strong>of</strong> the<br />

other patients on the team so that he/she can be included in the team’s discussions <strong>of</strong> the care <strong>of</strong><br />

13


all <strong>of</strong> the team’s patients and can assist the team in the care <strong>of</strong> all <strong>of</strong> the team’s patients. The<br />

student will be expected to actively participate in teaching attending rounds with the team.<br />

6) The student is expected to assume as much patient care responsibility and functions<br />

as the team feels are appropriate for the individual student’s level <strong>of</strong> training and competence.<br />

The student is expected to participate in patient education and counseling, work with nursing<br />

staff, dieticians, respiratory therapists, physical and occupational therapists, social workers,<br />

hospital chaplains, etc. and to assist in discharge planning.<br />

7) The student may perform Procedures or assist in the performance <strong>of</strong> procedures that the<br />

team feels are appropriate for the student’s level <strong>of</strong> training and competence. The Upper Level<br />

Resident is responsible for insuring proper supervision <strong>of</strong> any procedure performed or assisted by<br />

the student. This means that the student must be supervised by a physician who is certified or has<br />

expertise to competently perform the procedure in question, which includes interns, upper level<br />

residents, chief residents or attendings. Note that there are no required procedures for third-year<br />

medical students to perform.<br />

8) The student should learn how to write Orders on his/her assigned patients. The student will<br />

observe the residents enter orders electronically. The student should practice writing orders in their<br />

Comprehensive Write-ups and daily Progress Notes (in the Plans section) and on the Physician's<br />

Orders forms (See Physician’s Orders). The Hospital Site Coordinator and residents should<br />

review the student’s orders with the student and correct them as needed.<br />

Comprehensive Write-ups:<br />

1) The student is required to submit 3 write-ups by halfway through the inpatient rotation<br />

(averaging 1 write-up each week - schedule to be determined by the Hospital Site Coordinator).<br />

The HSC will decide whether these 3 write-ups are satisfactory. If they are deemed satisfactory,<br />

then the student will not be required to submit any more write-ups. If, however, they are deemed<br />

unsatisfactory, then the student will be required to submit 1 – 3 additional write-ups, as<br />

determined by the Hospital Site Coordinator, up to a maximum <strong>of</strong> 6 write-ups. (See Appendix)<br />

2) The HSC should read and then review each comprehensive write-up with the student and<br />

give constructive feedback.<br />

3) At the end <strong>of</strong> Inpatient <strong>Medicine</strong>, the student will turn in a total <strong>of</strong> 3 - 6 comprehensive<br />

write-ups, (total number to be determined by the HSC), either the originals or copies <strong>of</strong> the originals<br />

with the HSC’s comments, to the <strong>clerkship</strong> <strong>of</strong>fice.<br />

Required Clerkship Activities specific to Inpatient <strong>Medicine</strong>:<br />

1) Bedside Clinical Skills<br />

week.<br />

a) All students on Inpatient <strong>Medicine</strong> will attend Bedside Clinical Skills 1 - 2 times each<br />

b) Each week, one student will prepare and formally present one <strong>of</strong> his/her patients.<br />

c) Patients should be chosen who are unknown to the other students in the group (ideally)<br />

and who have physical exam findings.<br />

d) The student who is presenting should have the case presentation memorized<br />

(written notes for referral only).<br />

14


e) After presenting the case, the student will take the group to see the patient. (The<br />

student should have already gotten the patient’s consent and determined that the patient would<br />

be available for bedside teaching.)<br />

f) The Bedside Clinical Skills attending will teach/review physical exam skills and<br />

review/clarify important physical exam findings.<br />

g) The Bedside Clinical Skills attending may meet individually with the student who<br />

presented to give constructive feedback on his/her case presentation. (See Case Presentation<br />

Evaluation Form)<br />

h) The Bedside Clinical Skills attending may use the Small Group Learning Experience<br />

Evaluation Form to evaluate the students in the group. (See Small Group Learning<br />

Experience Evaluation Form)<br />

2) Chief Medical Resident Rounds<br />

a) All students on Inpatient <strong>Medicine</strong> will attend weekly Chiefs Rounds with the Chief<br />

Medical Resident (CMR). These may include bedside or didactic teaching.<br />

b) Chief Rounds give the students the opportunity to specify which areas in Internal<br />

<strong>Medicine</strong> they would like the CMR to teach in a small group setting.<br />

c) The CMR, together with the Upper Level Resident, are responsible for insuring that<br />

students are instructed in, but not limited to, the following:<br />

o Case presentations<br />

o Interpretation <strong>of</strong> basic EKGs<br />

o Interpretation <strong>of</strong> common imaging, including chest and abdominal x-rays and<br />

CT, head CT and MRI<br />

o Physiology and management <strong>of</strong> fluids and electrolytes<br />

o Interpretation <strong>of</strong> arterial blood gases<br />

o Basic physiology and management <strong>of</strong> shock<br />

o Basic understanding and management <strong>of</strong> ventilators (at KMC)<br />

d) The CMR may use the Small Group Learning Experience valuation Form to evaluate<br />

the students in the group. (See Small Group Learning Experience Evaluation Form)<br />

3) HIPSTER (<strong>Hawaii</strong> InterPr<strong>of</strong>essional Simulation Training for Emergency<br />

Response)<br />

Evaluation and Feedback to Students:<br />

1) The student is expected to regularly (at least once a week) ask for feedback on his/her<br />

performance and progress from his/her residents and attendings. It is especially important that<br />

this occurs before the student rotates <strong>of</strong>f the team and before the residents and attendings rotate<br />

<strong>of</strong>f the team. This feedback should identify the student’s strengths and weaknesses so that the<br />

student will know what to work on improving.<br />

2) Midway through Inpatient <strong>Medicine</strong>, the Hospital Site Coordinator will complete a Mid-<br />

Clerkship Feedback Form and discuss it with the student. The HSC will indicate whether the<br />

student’s progress to date is satisfactory or unsatisfactory, identify the student’s strengths and<br />

weakness and suggest a remediation plan if necessary. (See Mid-Clerkship Feedback Form)<br />

15


3) Interns, Upper Level Residents and Attendings who work with the student for one (1) week<br />

or more are expected to evaluate the student. During the course <strong>of</strong> the student’s inpatient rotation,<br />

the Intern and Upper Level Resident are expected to give the student verbal feedback on his/her<br />

performance and progress. This feedback should be provided on a regular basis, ideally daily but at<br />

least weekly. Then during the last week <strong>of</strong> the Intern and Upper Level Resident’s block or during the<br />

last week <strong>of</strong> the student’s inpatient rotation, the Intern and the Upper Level Resident will complete<br />

independent evaluations <strong>of</strong> the student which will be turned in to the Chief Medical Resident or<br />

Hospital Site Coordinator. Finally, at the end <strong>of</strong> the student’s inpatient rotation, the HSC will<br />

summarize all evaluations and complete a summary Student Evaluation Form for the site, which is<br />

submitted to the <strong>clerkship</strong> <strong>of</strong>fice. (See Student Evaluation Form)<br />

4) The student will be formally observed performing a history and physical on a patient<br />

unknown to the student by the end <strong>of</strong> the student’s second week <strong>of</strong> Inpatient <strong>Medicine</strong>. It is the<br />

student’s responsibility to arrange for a time when both the student and the Chief Medical<br />

Resident are available. It is the CMR’s responsibility to find an appropriate patient. The CMR will<br />

observe the student perform a history and physical. The CMR may also ask the student to make<br />

a brief case presentation. The CMR will complete the Observed History and Physical and<br />

BPES (Basic Physical Exam Sequence) Evaluation Forms (which should be provided by the<br />

student to the CMR) and give the student immediate feedback. If the student’s performance is<br />

not satisfactory, the student must repeat the exercise until his/her performance is satisfactory to<br />

the CMR. Note that this history and physical may be used for one <strong>of</strong> the student’s<br />

comprehensive write-ups. (See Observed History and Physical and BPES Evaluation Forms –<br />

this is a required <strong>clerkship</strong> activity.)<br />

5) The student will have the opportunity to present his/her patients at various hospital rounds<br />

and conferences. The student should ask the Upper Level Resident or Chief Medical Resident in<br />

advance for assistance in preparing for such presentations at a level that is appropriate for the<br />

student’s training and for the particular rounds or conference. After the presentation, the student<br />

should ask the physician who is supervising the rounds or conference for feedback. (See Case<br />

Presentation Evaluation Form)<br />

Ambulatory <strong>Medicine</strong><br />

Schedule: For 6B students, Ambulatory <strong>Medicine</strong> consists <strong>of</strong> 5 half days per week for 5-1/2<br />

weeks (if in the first half <strong>of</strong> the academic year) or for 5 weeks (if in the second half <strong>of</strong> the<br />

academic year). For 6L students, Ambulatory <strong>Medicine</strong> consists <strong>of</strong> 25 half days (one half day<br />

weekly for 21-22 weeks plus 3-4 additional half days).<br />

Location: The Ambulatory <strong>Medicine</strong> sites include Queen Emma Clinics, TAMC Internal<br />

<strong>Medicine</strong> Clinic, VA Clinics, community health clinics, Kaiser HMO and private physician<br />

<strong>of</strong>fices. The sites for MED 531 for 6B students are all on Oahu, primarily in Honolulu,<br />

while the sites for MED 532 for 6L students are either outside <strong>of</strong> Honolulu on Oahu or on<br />

the Neighbor Islands. Although each site has unique features, the <strong>clerkship</strong>’s goal is to<br />

provide students with a uniform learning experience based on the same learning objectives<br />

while allowing students the opportunity to take advantage <strong>of</strong> the strengths <strong>of</strong> each site.<br />

Due to the distant location <strong>of</strong> their assigned sites and their schedules, 6L students<br />

do not attend any <strong>clerkship</strong> activities while on Ambulatory <strong>Medicine</strong> (UH <strong>Department</strong> <strong>of</strong><br />

<strong>Medicine</strong> Grand Rounds, EBM 1 and 2, EKG Workshop, HIV <strong>Medicine</strong>, Neuro 1 and 2).<br />

16


Patient Assignment: The Ambulatory Preceptor(s) will be responsible for assigning patients<br />

to the student. Patients should be selected for their ability to cooperate and communicate, as<br />

well as for their specific medical problems. The student has a list <strong>of</strong> Training Problems on<br />

which the Third-Year Clerkship in Internal <strong>Medicine</strong> curriculum and specific learning<br />

objectives are based. The student’s goal is to see at least one (1) ambulatory patient with<br />

each <strong>of</strong> thirty-three (33) Training Problems. Note that the Training Problem does not have to<br />

be the patient’s Chief Complaint. In fact, a patient may present with many Training Problems.<br />

(See Training Problems section)<br />

Patient Census: The student should work up at least two (2) patients each half day.<br />

Patient Care Responsibilities:<br />

1) The student may see new or returning patients for complete examinations or<br />

problem-focused visits. Ideally, the student will see some <strong>of</strong> the patients when they return<br />

for their follow-up visits.<br />

2) After reviewing the patient’s chart as necessary, the student will perform an<br />

appropriate history and physical examination. The student will present the case to the<br />

preceptor who should correct and demonstrate any missed history or physical exam<br />

findings and review the student’s assessment and plans.<br />

3) The student will write an appropriate write-up or progress note in a timely manner, as<br />

specified by the preceptor. The preceptor should review the write-up or note with the student and<br />

give constructive feedback.<br />

4) The student is expected to carry out whatever patient care responsibilities and functions<br />

the preceptor feels is appropriate for the student’s level <strong>of</strong> training and competence. The student<br />

should try to assume as much responsibility as is appropriate and possible. The student is<br />

expected to participate in patient education and counseling, work with <strong>of</strong>fice/clinic staff, and to<br />

assist in follow-up planning. If the student’s patient requires any consultations or procedures, the<br />

student is highly encouraged to be present if the patient agrees. If the student’s patient is<br />

hospitalized, the student is highly encouraged to follow the patient during the hospitalization.<br />

Comprehensive Write-ups:<br />

1) The 6B student is required to complete an average <strong>of</strong> two (2) comprehensive writeups<br />

each week. The 6L student is required to complete an average <strong>of</strong> two (2)<br />

comprehensive write-ups each month. These comprehensive write-ups may be different<br />

from those initially written – they are expected to be more detailed and thorough, requiring<br />

additional reading and thought. (See Appendix D and E)<br />

2) The Ambulatory Preceptor should read, correct and review each comprehensive writeup<br />

with the student and give constructive feedback.<br />

3) At the end <strong>of</strong> Ambulatory <strong>Medicine</strong>, the student will turn in a total <strong>of</strong> ten (10)<br />

comprehensive write-ups, either the originals or copies <strong>of</strong> the originals with the preceptor’s<br />

comments, to the <strong>clerkship</strong> <strong>of</strong>fice.<br />

Evaluation and Feedback to Students:<br />

1) The student is expected to regularly ask the Ambulatory Preceptor for feedback on<br />

his/her performance and progress, ideally on a weekly basis for 6B students and on a<br />

monthly basis for 6L students, and definitely before the student finishes Ambulatory<br />

<strong>Medicine</strong>. This feedback will give the student the opportunity to correct mistakes and<br />

17


omissions and to improve skills.<br />

2) Midway through Ambulatory <strong>Medicine</strong>, the student should provide the Ambulatory<br />

Preceptor with a Mid-Clerkship Feedback Form to complete and discuss with the<br />

student. The preceptor will indicate whether the student’s progress to date is satisfactory<br />

or unsatisfactory, identify the student’s strengths and weakness and suggest a remediation<br />

plan if necessary. (See Mid-Clerkship Feedback Form)<br />

3) At the end <strong>of</strong> Ambulatory <strong>Medicine</strong>, the Ambulatory Preceptor will complete a Student<br />

Evaluation Form provided by the Third-Year Clerkship in internal <strong>Medicine</strong>. The Student<br />

Evaluation Form should be completed during the last week <strong>of</strong> the student’s Ambulatory<br />

<strong>Medicine</strong>. The preceptor should review the Student Evaluation Form with the student and the<br />

student should then sign the form. The form is then submitted to the <strong>clerkship</strong> <strong>of</strong>fice. (See<br />

Student Evaluation Form)<br />

4) The student will be formally observed counseling a patient and/or family on two<br />

separate occasions. (See Observed Patient Counseling Evaluation Form – this is a<br />

required <strong>clerkship</strong> activity.)<br />

PBL Tutorial<br />

1) All students will attend PBL Tutorial:<br />

a. Inpatient students, whether 6B or 6L, will attend weekly PBL Tutorial led by<br />

their Hospital Site Coordinator who serves as the Inpatient PBL Tutor.<br />

b. Ambulatory 6B students will attend weekly PBL Tutorial led by an Ambulatory<br />

PBL Tutor.<br />

c. Ambulatory 6L students will attend monthly PBL Tutorial led by an<br />

Ambulatory PBL Tutor.<br />

2) One student will prepare and formally present one <strong>of</strong> his/her patients at each PBL<br />

Tutorial.<br />

3) Patients should ideally be chosen who are unknown to the other students in the<br />

tutorial group and who manifest one or more Training Problems (see Appendix A).<br />

4) The student who is presenting should have the case presentation memorized<br />

(written notes for referral only) and should bring and be familiar with all<br />

pertinent data (labs, pathology reports, EKG, imaging, etc.).<br />

6) The group will agree on appropriate Learning Issues that ideally address Training<br />

Problems and their specific Learning Objectives. (See Appendix A)<br />

7) At the following week’s PBL Tutorial, the Learning Issues from the previous week will<br />

be presented and discussed.<br />

8) The PBL Tutor should meet individually with the student who presented to give<br />

constructive feedback on the student’s case presentation. (See Case Presentation<br />

Evaluation Form)<br />

9) The PBL Tutor may use the Small Group Learning Experience Evaluation Form to<br />

evaluate the students in the group. (See Small Group Learning Experience<br />

Evaluation Form)<br />

18


Role Descriptions for <strong>Medicine</strong> 531/532<br />

The Bedside Clinical Skills Attending’s Role<br />

1. The BCS Attending’s role will be to serve as a role model, instructor and facilitator.<br />

The BCS Attending will make formal rounds. Meetings on the inpatient service will<br />

be scheduled regularly and this time should be held inviolable.<br />

2. The BCS Attending will be familiar with and reinforce the method <strong>of</strong> problem based,<br />

self-directed learning, as established in Units 1-5.<br />

3. BCS Attendings will emphasize bedside clinical teaching during their rounds, never<br />

failing to see the patients that are presented. The Attending will serve as resource<br />

faculty at the bedside, teaching techniques and demonstrating abnormalities. In<br />

addition to the general bedside teaching, they are tasked to specifically ensure<br />

competent student performance <strong>of</strong> pulmonary and cardiac bedside examination<br />

skills.<br />

4. The BCS Attending will be responsible for giving feedback to the student on his/her<br />

performance after each presentation. Any problems identified will be discussed and<br />

a plan to (FIX SPACING! remedy difficulties will be constructed. Progress on<br />

previously identified difficulties will also be discussed.<br />

5. BCS Attendings will be responsible for evaluation and will serve on the<br />

<strong>Department</strong>al Student Evaluation Committee.<br />

The Upper Level Resident’s Role<br />

1. Introduction:<br />

The upper level resident (Levels 2 or 3) is ultimately responsible for the third-year medical<br />

student’s experience with the inpatient medical team and should insure that the student has a<br />

good inpatient experience. At the beginning and throughout the student’s inpatient<br />

experience, the resident should clarify what is expected <strong>of</strong> students.<br />

2. Responsibilities:<br />

The Upper Level Resident will be responsible for assigning 2-3 patients per week<br />

for the student work up and continuing care. Patients should be selected for their<br />

ability to cooperate and communicate, as well as for their specific medical problems.<br />

The Upper Level Resident will be responsible also for reviewing the student’s write<br />

ups, for assistance with development <strong>of</strong> learning issues and for help in execution <strong>of</strong><br />

the learning agenda.<br />

The Upper Level Resident will "sign <strong>of</strong>f" on the student’s write up, confirming that<br />

he/she has reviewed the written note and given feedback.<br />

The Upper Level Resident will confirm that the student is pursuing self-directed<br />

learning by taking advantage <strong>of</strong> "Dedicated Learning Time."<br />

19


The Chief Medical Resident’s Role<br />

1. Introduction:<br />

The Chief Medical Resident (CMR) serves as a key faculty resource for the medical<br />

student on the inpatient service. As supervisor for the medical teams and their<br />

students, the CMR helps insure a good experience for the students.<br />

2. Responsibilities:<br />

a. The CMR will supervise and assist the Upper Level Resident in the<br />

inpatient instruction <strong>of</strong> medical students. They are responsible for insuring<br />

that students are instructed in, but not limited to, the following:<br />

o Case presentations<br />

o Interpretation <strong>of</strong> basic EKGs<br />

o Interpretation <strong>of</strong> common imaging, including chest and abdominal x-<br />

rays and CT, head CT and MRI<br />

o Physiology and management <strong>of</strong> fluids and electrolytes<br />

o Interpretation <strong>of</strong> arterial blood gases<br />

o Basic physiology and management <strong>of</strong> shock<br />

o Basic understanding and management <strong>of</strong> ventilators (at KMC)<br />

b. The CMR will observe and evaluate student’s Observed History and<br />

Physical during the student’s first two weeks <strong>of</strong> inpatient medicine, as<br />

previously described.<br />

c. The CMR will lead weekly Chief Rounds with the students as<br />

previously described<br />

d. The CMR will regularly discuss each student’s clinical performance<br />

with the Hospital Site Coordinator.<br />

The Site Coordinator’s Role<br />

1. Introduction:<br />

As a community-based <strong>clerkship</strong>, the Third-Year Clerkship in Internal <strong>Medicine</strong><br />

utilizes several clinical sites for medical student instruction. A major role <strong>of</strong> the<br />

Site Coordinator is to provide students at his/her site with the best possible<br />

educational experience and to insure that that learning experience is equivalent<br />

to the educational experience provided at the other sites utilized by the<br />

<strong>clerkship</strong>.<br />

The Site Coordinator is selected for his/her knowledge and experience at the site and<br />

the ability to work well with physicians, staff and administrators at the site, as well as<br />

the ability to work well with medical students. The Site Coordinator may be inpatient<br />

(Hospital Site Coordinator) or ambulatory (Ambulatory Site Coordinator).<br />

2. Responsibilities <strong>of</strong> Hospital Site Coordinators and Ambulatory Site<br />

Coordinators include, but are not limited to:<br />

a. Selecting qualified faculty to provide a good learning experience for the<br />

20


student(s) assigned to the site. The site coordinator will serve as a resource for<br />

the faculty and chief medical resident (CMR) at the site. This includes orienting<br />

new faculty and CMRs and assisting with faculty development and education<br />

regarding <strong>clerkship</strong> curriculum, expectations and goals.<br />

b. Identifying learning resources at the site that are available for student use (i.e.<br />

library, photocopying, computer, classrooms, etc.).<br />

c. Orientating new students, which include scheduling and leading orientations to<br />

the site and key personnel including faculty, CMR and administrative staff,<br />

making team/clinic assignments, reviewing <strong>clerkship</strong> requirements and student<br />

responsibilities. Orientations should occur on the student’s first day at the<br />

assigned site.<br />

d. Evaluating case presentations. (See Case Presentation Evaluation Form)<br />

e. Evaluating comprehensive write-ups. The Site Coordinator will read students’<br />

comprehensive write-ups and give constructive feedback at least on a weekly<br />

basis. (See Appendix D and E)<br />

f. Meeting with each student regularly to review the student’s performance and<br />

progress and to provide feedback. These meetings should also be used to<br />

review the number and type <strong>of</strong> patients that students are seeing and their level<br />

<strong>of</strong> patient care involvement. (See Training Problems List) Formal meetings<br />

with each student should be scheduled at least twice during the student’s<br />

rotation at the assigned site, once midway and once before the last day (ideally<br />

during the last week).<br />

g. Completing a Mid-Clerkship Feedback Form, midway through the student’s<br />

rotation at the assigned site, indicating whether the student’s progress to date is<br />

satisfactory or unsatisfactory, identifying the student’s strengths and weakness<br />

and suggesting a remediation plan if necessary. The Site Coordinator will<br />

review the completed form with the student to allow the student time to improve<br />

in the identified areas <strong>of</strong> weakness during the remainder <strong>of</strong> the rotation. If the<br />

student’s progress to date is unsatisfactory, the Site Coordinator must<br />

immediately notify the Clerkship Director who may further discuss the student’s<br />

performance and possible remediation with the <strong>Department</strong> <strong>of</strong> <strong>Medicine</strong> Student<br />

Education Committee. (See Mid-Clerkship Feedback Form)<br />

h. Coordinating the site evaluation <strong>of</strong> students. The Site Coordinator will<br />

review the student’s performance with each physician (including Interns, Upper<br />

Level Residents and Attendings) who worked with the student for one (1) week<br />

or more and also with any Attendings who worked with the student on a weekly<br />

basis. The Site Coordinator will collect and review evaluation forms such as the<br />

Observed History and Physical, Observed Patient Counseling, Case<br />

Presentations and Small Group Learning Experiences. The Site Coordinator<br />

will complete a summary Student Evaluation Form for the site at the end <strong>of</strong> the<br />

rotation (ideally, during the last week), summarizing all the evaluations including<br />

his/her own personal evaluation. The Site Coordinator will be careful to insure that<br />

a single evaluation, incident or conflict will not overshadow a student’s overall<br />

performance. The Site Coordinator will review the form with the student and<br />

then submit it to the <strong>clerkship</strong> <strong>of</strong>fice. In addition, the Site Coordinator will complete<br />

the confidential Honors/AOA Evaluation Form which is not shared with the student<br />

but is submitted to the <strong>clerkship</strong> <strong>of</strong>fice. If the student’s performance is marginal in<br />

21


any area(s), the Site Coordinator should immediately notify the Clerkship<br />

Director who may further discuss the student’s performance and possible<br />

remediation with the <strong>Department</strong> <strong>of</strong> <strong>Medicine</strong> Student Education Committee.<br />

(See Student Evaluation Form and Honors/AOA Evaluation Form)<br />

i. Serving as a member <strong>of</strong> the <strong>Department</strong> <strong>of</strong> <strong>Medicine</strong> Student Education<br />

Committee (SEC). The Site Coordinator attends the committee’s monthly<br />

meetings to act as a spokesperson for the students assigned to his/her site,<br />

reporting on students’ performance and progress and recommending<br />

commendation and remediation when appropriate. In addition, SEC members<br />

participate in the oversight, development and implementation <strong>of</strong> all <strong>of</strong> the<br />

<strong>Department</strong> <strong>of</strong> <strong>Medicine</strong>’s 3 rd and 4 th year clinical rotations.<br />

j. Monitoring students’ work hours, dedicated study time, days <strong>of</strong>f and<br />

absences. (See Work Hours, Dedicated Study Time and Days Off section.)<br />

k. Serving as mediator in the event students encounter problems at their<br />

assigned site. Problems may include <strong>clerkship</strong> requirements, <strong>clerkship</strong><br />

performance, absences, mistreatment, harassment and abuse. The Site<br />

Coordinator should refer significant problems to the Clerkship Director and/or the<br />

<strong>Department</strong> <strong>of</strong> <strong>Medicine</strong> Student Education Committee.<br />

3. Additional responsibilities <strong>of</strong> Hospital Site Coordinators include, but are not<br />

limited to:<br />

a. Assisting in Team Assignments - the Hospital Site Coordinator will assist<br />

the Chief Medical Resident with the assignment <strong>of</strong> students to medical<br />

teams.<br />

b. Leading PBL Tutorial. This should occur on average once a week<br />

throughout the <strong>clerkship</strong> and each session should last on average 2 – 3<br />

hours. (See PBL Tutorial description.)<br />

c. Insuring that Bedside Clinical Skills occur 1 – 2 times each week and<br />

Chief Medical Resident Rounds occur 1 time each week and monitoring<br />

students’ attendance at these as well as at other hospital conferences<br />

and rounds deemed to be <strong>of</strong> educational value by the Hospital Site<br />

Coordinator.<br />

The Ambulatory Attending’s Role<br />

1. Introduction:<br />

The ambulatory attending, or preceptor, is a critical element for student learning in<br />

Internal <strong>Medicine</strong> as it is practiced in the ambulatory milieu. Although sometimes<br />

lacking the drama <strong>of</strong> inpatient medicine, the outpatient setting <strong>of</strong>fers special skills sets<br />

in learning and teaching critical thinking, decision making, clinical skills and judgment,<br />

and patient-physician communication in Internal <strong>Medicine</strong><br />

2. Responsibilities:<br />

a) The ambulatory attending/preceptor will serve as a role model for excellence<br />

in ambulatory primary internal medical practice, to include thoughtful, evidence-<br />

22


ased health care that is patient focused, comprehensive, and which maximizes<br />

and restores health, and is preventative.<br />

b) The attending/preceptor will assign patients for student evaluation that will<br />

cover the depth and breadth <strong>of</strong> ambulatory learning issues, as found in the<br />

Learning Objectives listed under Patient’s Presenting Health issues (Appendix<br />

A).<br />

c) The attending/preceptor will monitor and teach data gathering as it applies to<br />

the clinical focus <strong>of</strong> ambulatory patient care; this includes appropriate<br />

consideration <strong>of</strong> the breadth <strong>of</strong> patient health issues, rather than a simple focus<br />

on chief complaint.<br />

d) The attending/preceptor will model and teach evidence-based problem<br />

solving as it applies to the ambulatory experience.<br />

e) The attending/preceptor will critique student write-ups for accuracy, style,<br />

problem solving and give constructive feedback, by applying the criteria <strong>of</strong> the<br />

“comprehensive write-up” and the write-up examples (see Appendix E).<br />

f) The attending/preceptor will give continuous feedback on student clinical and<br />

analytical skills, and stimulate his/her intellectual growth.<br />

g) The ambulatory preceptor will assure completion the “Observation <strong>of</strong> Patient<br />

Counseling form for a least 2 separate patient encounters.<br />

h) The attending/preceptor will evaluate student performance in areas <strong>of</strong><br />

medical knowledge, clinical skills, and pr<strong>of</strong>essionalism and work habits.<br />

23


MISCELLANEOUS CLERKSHIP INFORMATION<br />

24


U.H. John A. Burns School <strong>of</strong> <strong>Medicine</strong><br />

Third-Year Clerkship in Internal <strong>Medicine</strong><br />

GUIDELINES FOR APPROPRIATE APPEARANCE AND ATTIRE<br />

These guidelines are intended to contribute to your overall pr<strong>of</strong>essional development as students<br />

in training to become physicians. The Third Year Clerkship in Internal <strong>Medicine</strong> expects students<br />

to appear and dress in a pr<strong>of</strong>essional manner. Some things, such as your appearance and attire,<br />

which may have been acceptable in college, may no longer be appropriate in our <strong>clerkship</strong>. Your<br />

appearance and attire should show respect to faculty, staff, classmates, patients and the general<br />

public.<br />

It is recognized that different attire will be necessary for different settings, depending on factors<br />

such as student activities and responsibilities, training sites, patient and public contact. Student<br />

attire should always be appropriate and not interfere with the activities and responsibilities<br />

expected <strong>of</strong> students.<br />

General guidelines for all <strong>Medicine</strong> Clerkship students:<br />

Students should wear:<br />

JABSOM name tag with photo ID<br />

Short white medical coat<br />

Closed-toe footwear<br />

Students should maintain an optimum level <strong>of</strong> personal hygiene and grooming<br />

Strong odors and fragrances should be avoided<br />

Clothes, hair, fingernails and footwear should be clean and neat<br />

Clothing should not be suggestive, revealing or tight-fitting<br />

Clothing should not have <strong>of</strong>fensive images or language<br />

Sites-specific (inpatient and ambulatory) guidelines for <strong>Medicine</strong> Clerkship students:<br />

Students should adhere to the dress code/policy in place at their training site<br />

25


U.H. John A. Burns School <strong>of</strong> <strong>Medicine</strong><br />

Third-Year Clerkship in Internal <strong>Medicine</strong><br />

RECOMMENDED RESOURCES<br />

Highly recommended <strong>clerkship</strong> guidebook:<br />

Primer to the Internal <strong>Medicine</strong> Clerkship, Second Edition, A Guide Produced by the Clerkship<br />

Directors in Internal <strong>Medicine</strong> (CDIM), c2008, 2nd edition – download free <strong>of</strong> charge from<br />

http://www.im.org/Publications/PhysiciansInTraining/Pages/Primer.aspx<br />

Online resources:<br />

Internal <strong>Medicine</strong> Essentials for Clerkship Students - online materials such as photographs, tables,<br />

screening tools and other instruments – access free <strong>of</strong> charge at http://www.acponline.org/acp_press/essentials/<br />

UpToDate – access available at some training sites<br />

Textbooks:<br />

Internal <strong>Medicine</strong> Essentials for Students, American College <strong>of</strong> Physicians (ACP) and Clerkship Directors<br />

in Internal <strong>Medicine</strong> (CDIM), c2011 – discount <strong>of</strong>fered for ACP student members at<br />

http://www.im.org/Publications/PhysiciansInTraining/Pages/Essentials.aspx<br />

Kochar’s Clinical <strong>Medicine</strong> for Students, Lippincott Williams & Wilkins, c2008, 5 th edition<br />

Cecil Essentials <strong>of</strong> <strong>Medicine</strong>, Elsevier, c2011, 8th edition<br />

Reference textbooks:<br />

DeGowin’s Diagnostic Examination, McGraw-Hill, c2008, 9th edition<br />

Spiral bound books and pocket guides:<br />

Pocket <strong>Medicine</strong>, Lippincott Williams & Wilkins, c2010, 4 th edition<br />

Practical Guide to the Care <strong>of</strong> the Medical Patient, Elsevier, c2011, 8 th edition<br />

Washington Manual <strong>of</strong> Medical Therapeutics, Lippincott Williams & Wilkins, c2010, 33rd edition<br />

Washington Manual <strong>of</strong> Outpatient Internal <strong>Medicine</strong>, Lippincott Williams & Wilkins, c2010, 2nd edition<br />

The Sanford Guide to Antimicrobial Therapy, c2012, 42nd edition<br />

Highly recommended self-assessment program:<br />

MKSAP (Medical Knowledge Self-Assessment Program) for Students 5 (Book and Digital), American<br />

College <strong>of</strong> Physicians (ACP) and Clerkship Directors in Internal <strong>Medicine</strong> (CDIM), c2011 – discount <strong>of</strong>fered for ACP<br />

student members at http://www.im.org/Publications/PhysiciansInTraining/Pages/MKSAPforStudents.aspx<br />

26


U.H. John A. Burns School <strong>of</strong> <strong>Medicine</strong><br />

Third-Year Clerkship in Internal <strong>Medicine</strong><br />

REQUIRED EQUIPMENT<br />

1. White coat<br />

2. JABSOM nametag<br />

3. Stethoscope<br />

4. Ophthalmoscope and otoscope<br />

5. Penlight<br />

6. Reflex hammer<br />

7. Tuning fork 128 Hz for vibratory exam<br />

8. Tape measure<br />

9. Visual acuity card<br />

10. Small magnifier lens<br />

11. Disposable tongue blades<br />

12. Cotton-tipped swabs for sensory exam<br />

13. Watch with second and minute marks to record vital signs, etc.<br />

All <strong>of</strong> the above listed equipment, except for the JABSOM nametag and watch, are available for<br />

purchase at the Medical Bookstore.<br />

Students are expected to have ALL <strong>of</strong> the equipment and to bring them to their assigned training<br />

sites on the first day. The equipment should be easily and quickly accessible, i.e. worn, carried in<br />

a bag or in/on a white coat.<br />

All students on inpatient medicine are required to carry ALL <strong>of</strong> the listed equipment AT ALL<br />

TIMES while on the wards.<br />

Students on ambulatory medicine may find that their assigned sites may have some, but not all,<br />

<strong>of</strong> the listed equipment available for student use. If the listed equipment are NOT easily and<br />

quickly accessible at the site, it is the student’s responsibility to carry them AT ALL TIMES.<br />

The possible consequences <strong>of</strong> not having the required equipment are (1) being immediately sent<br />

to the medical bookstore to purchase them (requiring the student to make up the missed time)<br />

and (2) receiving unsatisfactory evaluations in the areas <strong>of</strong> (a) Clinical Skills - physical<br />

examinations and (b) Pr<strong>of</strong>essionalism dependability, pr<strong>of</strong>essional appearance and attire.<br />

27


EXPOSURE TO BLOOD/BODY FLUIDS PROTOCOL<br />

1. IMMEDIATELY following the exposure:<br />

a. Flush the exposed skin or mucous membrane with water or saline.<br />

If exposure to the eyes has occurred, use wash station or nearest sink to flush eyes with water for at<br />

least 5 minutes.<br />

b. Wash any needle stick, puncture, cut or abrasion with soap and water.<br />

2. Initiate the host agency protocol for hazardous exposure to blood/body fluids by following the instructions outlined in<br />

the table below.<br />

3. If the exposure is in a non-hospital setting (for example, ambulatory site not associated with a hospital, in a JABSOM<br />

lab, class, or other non-hospital-based exposure), you or your preceptor/supervisor can call Queens ED (547-4311)<br />

to review current protocol for immediate needs in such a circumstance, and begin the process, (AFTER #1). You<br />

may go to an Emergency <strong>Department</strong>, or during open hours, contact the <strong>University</strong> Health Services (Manoa<br />

Campus) 956-8965, and ask for immediate attention.<br />

FACILITY CONTACT or GO TO PHONE<br />

Castle Medical Center Report incident to supervisor. Obtain care from<br />

Employee Health Coordinator or hospital supervisor who<br />

will assist in filing incident report. Contact JABSOM<br />

OSA to report incident.<br />

263-5159 or<br />

hospital supervisor<br />

263-5329<br />

(5 pm-8 am)<br />

HOME Clinic<br />

Notify attending physician and complete incident report. 221-0685<br />

Kaiser Permanente Medical<br />

Center<br />

Kapiolani Medical Center<br />

Kuakini Medical Center<br />

Pali Momi Medical Center<br />

Call Dr. Jill Omori to report exposure.<br />

Report incident within 2 hours <strong>of</strong> exposure. Call<br />

operator in house “0” and ask for infection control<br />

personnel on duty.<br />

Report to Employee Health. Go to Emergency Dept, if<br />

EH closed, also call on-call Employee Health<br />

Coordinator, 983-6000).<br />

Occupational Health Services.<br />

When closed, go to ED, and also notify Nursing<br />

Supervisor (through Operator, dial “O”).<br />

Employee Health during regular work hours or<br />

Emergency <strong>Department</strong> when exposure occurs after<br />

hours. Notify supervisor. Report incident Work Injury<br />

Line.<br />

432-0000<br />

983-8525<br />

547-9531<br />

535-7200<br />

The Queen’s Medical Center Employee Health/PEP Team. 547-4004<br />

Straub Clinic and Hospital Employee Health during business hours, go directly to 522-3481<br />

ED after business hours.<br />

Tripler Army Medical Center Let care team know <strong>of</strong> exposure. Report to the ER.<br />

Report exposure to, or go to, Occupational Health the<br />

next business day.<br />

433-6235<br />

VA Clinic<br />

Contact EHU during business hours. Go to TAMC ER<br />

after hours.<br />

433-0091<br />

Wahiawa General Hospital<br />

Go to ER; also notify Nursing Supervisor (through<br />

operator) <strong>of</strong> exposure.<br />

621-4230<br />

You may also seek care and information from <strong>University</strong> Health Services (956-8965), your personal physician, or any emergency<br />

department, but seek immediate evaluation and counseling. All follow-up care after immediate evaluation services are the<br />

responsibility <strong>of</strong> the student<br />

4. Report exposure to;<br />

a. Your supervising faculty member and course/<strong>clerkship</strong> director<br />

b. Medical School Office <strong>of</strong> Student Affairs @ 692-1000;<br />

c. For URGENT after hours needs, call 692-0912, ask for Dr Smerz or Administrator on-call<br />

5. Students should be knowledgeable about their health insurance coverage, and should know what their plan will<br />

cover related to occupational exposures. Remind anyone billing for follow-up that it is NOT an Occupational<br />

Exposure, but medical follow-up, or the insurer may not want to pay for services.<br />

JABSOM’s Affiliation Agreement with Health Care Facilities (HCF) state:<br />

“Environmental exposure. In the event a medical student is exposed to an infectious, environmental, or occupational hazard at<br />

the HCF, the HCF shall be responsible for providing immediate evaluation and counseling as with employees <strong>of</strong> the HCF. Followup<br />

after the initial evaluation and counseling will not be the responsibility <strong>of</strong> the HCF, and will proceed according to <strong>University</strong><br />

student health policies.”<br />

28


U.H. John A. Burns School <strong>of</strong> <strong>Medicine</strong><br />

Third-Year Clerkship in Internal <strong>Medicine</strong><br />

HIV MEDICINE<br />

1:30 – 5:00 p.m.<br />

Clint Spencer Clinic<br />

Leahi Hospital<br />

All students in the Third-Year Clerkship in Internal <strong>Medicine</strong> are scheduled to attend one <strong>of</strong> the<br />

upcoming HIV <strong>Medicine</strong> sessions.<br />

This is a required <strong>clerkship</strong> activity.<br />

How to prepare for the HIV <strong>Medicine</strong> session:<br />

1. Read the 4 chapters on HIV at http://uhmed.org/images/HIV_Required_Reading.pdf<br />

2. Read about the Clint Spencer Clinic at http://www.hawaii.edu/hacrp/spencer.htm<br />

3. Read about HACRP (<strong>Hawaii</strong> AIDS Clinical Research Program) at<br />

http://www.hawaii.edu/hacrp/home2.htm<br />

4. If you are on inpatient medicine, please notify your team and chief resident <strong>of</strong> your<br />

scheduled absence to attend this required <strong>clerkship</strong> activity. Likewise, you should discuss<br />

with them whether or not you are expected to return to the hospital after this session.<br />

What to bring to the HIV <strong>Medicine</strong> session:<br />

White coat<br />

JABSOM nametag<br />

Temporary parking pass<br />

After attending the HIV <strong>Medicine</strong> session:<br />

Check <strong>of</strong>f HIV as an “S” (Special” patient encounter) on your Training Problems List<br />

Log patient(s)<br />

Evaluate teaching attending(s)<br />

29


Location and Parking:<br />

The Clint Spencer Clinic is located on the basement floor <strong>of</strong> the Young Building on the grounds <strong>of</strong><br />

Leahi Hospital at 3675 Kilauea Ave (on the block bordered by Kilauea Ave, Makapuu Ave,<br />

Maunalei Ave and Pokole St.) in Kaimuki.<br />

There is free parking in an open lot on Maunalei Ave. You may park in any unmarked stall. (Do<br />

not park in the stalls marked “Reserved for Clinical Research.”) Display your temporary parking<br />

pass on your dashboard.<br />

After parking, cross Maunalei Ave, walk on the main floor (actually the basement floor) <strong>of</strong> the<br />

main building (Young Building) and look for a sign that reads “HACRP” (<strong>Hawaii</strong> AIDS Clinical<br />

Research Program) and then the door that reads “Clint Spencer Clinic.” Check in with Joel<br />

Chapman at the reception desk.<br />

Please plan on arriving early at 1:15 p.m., especially if you are not familiar with Leahi Hospital<br />

and to allow time for parking.<br />

If you get lost, call the Clinic at 737-2751.<br />

30


TEMPORARY<br />

PARKING PASS<br />

FOR<br />

THE CLINT SPENCER<br />

CLINIC<br />

Date _______________<br />

1:30 – 5:00PM<br />

CALL 737-2751 for<br />

QUESTIONS<br />

31


MEDICINE T-RES INSTRUCTIONS<br />

General<br />

Logging all <strong>of</strong> your patients is required for JABSOM accreditation and for your <strong>clerkship</strong> grade.<br />

Failure to log properly and on a timely basis may lead to serious consequences for JABSOM<br />

and for you.<br />

You should log your patients regularly – ideally every workday so that you don’t forget and so<br />

that you don’t fall behind.<br />

You should sync your patient log regularly – at least once a week.<br />

T-Res Data Fields<br />

Complete all data fields (except 2 nd Diag which is not always needed).<br />

Date:<br />

In the Inpatient setting, the date is when you first saw the patient. This may or may not be the<br />

date <strong>of</strong> admission.<br />

In the Ambulatory setting, the date is when you see the patient. If you see a patient again for<br />

a follow-up visit, the patient should be logged again using the date <strong>of</strong> the follow-up visit.<br />

Site:<br />

In the Inpatient setting, select the hospital (KMC, QMC, TAMC)<br />

In the Ambulatory setting:<br />

o If you are working in a clinic, select the clinic (QEC, VA, Kaiser-Mapunapuna, Kaiser-<br />

Waipio, Kalihi Palama, Kokua Kalihi Valley, TAMC Clinic)<br />

or<br />

o<br />

If you are working in a physician’s <strong>of</strong>fice, select <strong>Medicine</strong> – Other Amb (Do not select<br />

Private Outpatient).<br />

Do not select Other<br />

Setting:<br />

For the Inpatient block: Select Inpatient or Special (Do not select Other)<br />

Inpatient is the patient for whom you performed a history and physical exam, and wrote daily<br />

progress notes. In other words, you provided “D” (Direct patient care) to this patient.<br />

You should log each patient only 1 time during the patient’s hospitalization. For example: If<br />

Mr. S was hospitalized for three days, although you wrote daily progress notes, you should log<br />

him only 1 time (upon his admission). If Mr. S. was discharged and then comes back to your<br />

team, you should log him again as a new patient encounter (upon his re-admission). If,<br />

however, Mr. S. went home AMA and then comes back with the same problem, you should<br />

not log him as a new patient encounter. If, however, Mr. S. went home AMA and then comes<br />

back with a different problem, you should log him as a new patient encounter.<br />

Special: Special is any patient that contributed to your education in the setting <strong>of</strong> conferences,<br />

rounds, procedures, etc. In other words, you did not provide “D” (Direct patient care) to this<br />

patient. Instead, this was an “S” (Special setting). If you wrote one or two progress notes to<br />

help out your team (but not on a daily basis), this is still an “S” (Special setting).<br />

For the Ambulatory block: Select Ambulatory or Special (Do not select Other)<br />

Ambulatory is the patient for whom you performed a history and physical exam, and wrote a<br />

note. In other words, you provided “D” (Direct patient care) to this patient.<br />

Each and every visit, including follow-up visits, should be considered a new patient encounter<br />

and logged.<br />

32


Special: Special is any patient that contributed to your education in the setting <strong>of</strong><br />

conferences, rounds, procedures, etc. In other words, you did not provide “D” (Direct patient<br />

care) to this patient. Instead, this was an “S” (Special setting).<br />

Supervisor:<br />

In the Inpatient setting, select last name MEDICINE + first name Inpt-KMC, Inpt-QMC-MTC,<br />

Inpt-QMC-UHS or Inpt-TAMC.<br />

In the Ambulatory setting, select the attending physician from the menu - except for the<br />

following situations:<br />

o If you are at QEC, select MEDICINE, QEC<br />

o If you are at TAMC IM Clinic, select MEDICINE, TAMC Clinic<br />

o If you are at VA Honolulu, select MEDICINE, VA<br />

o If you are at another VA, select your attending physician<br />

If the setting is Special, select MEDICINE, Special<br />

Do not select Other<br />

Birth Date:<br />

Enter the patient’s date <strong>of</strong> birth<br />

Sex:<br />

Select the patient’s gender<br />

Problems:<br />

Select up to 5 Training Problems that apply to the patient<br />

If none <strong>of</strong> the Training Problems apply to the patient, select None<br />

Prim Diag:<br />

Select the primary diagnoses that you addressed for this patient. You may enter up to 5<br />

diagnoses.<br />

If the diagnosis is not listed or if you have a more specific diagnosis than is listed, you can<br />

write in the diagnosis by choosing Other.<br />

2 nd Diag:<br />

Use this field if the patient has more than 5 diagnoses. You may enter up to 5 additional<br />

diagnoses, for a total <strong>of</strong> 10.<br />

Printed Reports<br />

Print an activity report according to the following schedule:<br />

o Mid-inpatient feedback meeting<br />

o Mid-ambulatory feedback meeting<br />

o End <strong>of</strong> the inpatient block<br />

o End <strong>of</strong> the ambulatory medicine block<br />

At the end <strong>of</strong> the inpatient block and at the end <strong>of</strong> the ambulatory block, your printed reports<br />

must reviewed, signed and dated by your Hospital Site Coordinator or Ambulatory Preceptor,<br />

respectively, and then turned in to the <strong>clerkship</strong>.<br />

Your Hospital Site Coordinator or Ambulatory Preceptor may request that you report additional<br />

handwritten information on your printed report (to help identify patients). However, the signed<br />

reports that you turn in to the <strong>clerkship</strong> should be copies without any additional handwritten<br />

information.<br />

Instructions for generating and printing your T-Res <strong>Medicine</strong> Activity Report:<br />

o Login to the T-Res web site: www.t-res.net<br />

o Select Lists under Reports in the left column<br />

o View report 038c – Activity Export Details for <strong>Hawaii</strong> Pediatrics<br />

o Activity Type: Internal <strong>Medicine</strong>, then View Report<br />

o Select Export as Acrobat (PDF) File<br />

o You can save a copy if you wish, otherwise select Open, then print the report<br />

33


U.H. John A. Burns School <strong>of</strong> <strong>Medicine</strong><br />

<strong>Department</strong> <strong>of</strong> <strong>Medicine</strong><br />

STUDENT EVALUATION OF TEACHERS<br />

Please complete an evaluation on each <strong>of</strong> the following individuals with whom you interacted during your<br />

inpatient or ambulatory rotation:<br />

INPATIENT MEDICINE<br />

Clerkship Director - Dr. Tam<br />

Hospital Site Coordinator<br />

PBL Tutor<br />

Bedside Clinical Skills Teacher(s)<br />

Chief Medical Resident<br />

Upper Levels<br />

Interns<br />

CV PE - Dr. Fukuyama<br />

EBM - Dr. Kasuya<br />

EKG - Dr. Azuma<br />

HIV <strong>Medicine</strong> Teacher(s)<br />

HIPSTER - Dr. Ganitano<br />

Neurology - Dr. Yee<br />

AMBULATORY MEDICINE<br />

Clerkship Director - Dr. Tam<br />

Ambulatory Preceptor(s)<br />

PBL Tutor<br />

Upper Levels, if at VA<br />

Interns, if at VA<br />

CV PE - Dr. Fukuyama<br />

EBM - Dr. Kasuya<br />

EKG - Dr. Azuma<br />

HIV <strong>Medicine</strong> Teacher(s)<br />

HIPSTER - Dr. Ganitano<br />

Neurology - Dr. Yee<br />

In addition, you are also encouraged to complete evaluations on any other individuals to help the <strong>Department</strong> <strong>of</strong><br />

<strong>Medicine</strong> recognize our most outstanding teachers as well as identify those who may need further training to improve<br />

their teaching.<br />

These evaluations are anonymous. Your honest and thoughtful feedback is integral in helping the <strong>Department</strong> <strong>of</strong><br />

<strong>Medicine</strong> and its teachers improve students’ educational experiences. Your assistance is greatly appreciated. Thank<br />

you!<br />

INSTRUCTIONS<br />

1. Go to uhmed.org/db<br />

2. Login using the following: Account Name: student<br />

Password: jabsom*08<br />

3. Click on “StudentEval”<br />

4. Select “MS3”, the teacher and the academic year for which you are evaluating<br />

5. Select “Evaluation <strong>of</strong> Teacher”<br />

6. Enter your Block Number which will be emailed to you.<br />

7. Complete the evaluation form and then click submit.<br />

8. You will be given a confirmation number and the option to email it to yourself for record keeping (Do not email<br />

this number directly to Julieta, as she will not be able to identify the sender). Save this confirmation number as<br />

a pro<strong>of</strong> <strong>of</strong> your submission.<br />

9. To complete another evaluation, select “”Email & New Eval”.<br />

10. Email your confirmation number(s) to Julieta at jlrajlev@hawaii.edu<br />

34


EVALUATION FORMS<br />

35


U.H. John A. Burns School <strong>of</strong> <strong>Medicine</strong><br />

Third-Year Clerkship in Internal <strong>Medicine</strong><br />

OBSERVED HISTORY AND PHYSICAL<br />

Evaluation Form<br />

INSTRUCTIONS:<br />

o The Observed History and Physical should be scheduled by halfway through the inpatient block.<br />

o The Chief Medical Resident should select a patient who is unknown to the student.<br />

o The student must perform the Observed History and Physical by memory.<br />

o The student should try to finish the Observed History and Physical in 1 hour.<br />

o The student must turn in at least one (1) completed Observed History and Physical Evaluation Form to the<br />

Hospital Site Coordinator.<br />

Student_____________________________________________ Hospital (circle): KMC QMC TAMC<br />

Start Time ____:____<br />

Faculty (Chief Resident) _______________________________ Date_____________ End Time ____:____<br />

TOTAL Time ____:____<br />

A. PATIENT-PHYSICIAN RELATIONSHIP<br />

Observe whether and how the student:<br />

Maximum Score<br />

Student’s Score<br />

1. Introduces self to the patient and explains his/her role correctly 2 ( )<br />

2. Attends to the patient’s privacy and comfort 3 ( ) before beginning<br />

and throughout the history and physical<br />

3. Demonstrates effective listening skills 5 ( )<br />

(appropriate use <strong>of</strong> eye contact, posture, pauses/silence, etc.)<br />

4. Demonstrates effective verbal skills 5 ( )<br />

(appropriate use <strong>of</strong> open- and closed-ended questions,<br />

repetition, clarifying, restating, explanation, interpretation, etc.)<br />

5. Uses language that the patient understands 5 ( )<br />

B. INTERVIEW<br />

Observe whether and how the student:<br />

1. Obtains the patient’s complete history 5 ( )<br />

systematically and completely<br />

2. Balances allowing the patient to speak while 5 ( )<br />

directing questions to efficiently obtain a<br />

clear and accurate history in the time available<br />

3. Elicits the history <strong>of</strong> the present illness (HPI) 10 ( )<br />

systematically and completely<br />

4. Delineates major symptoms systematically and completely 5 ( )<br />

(location, radiation, quality, intensity, setting, onset, duration, frequency<br />

setting, aggravating and alleviating factors, associated manifestations,<br />

functional impairment, patient’s interpretation <strong>of</strong> symptoms, etc.)<br />

36


5. Obtains information for the other major sections <strong>of</strong> the history<br />

systematically and completely:<br />

Maximum Score<br />

THE BASIC PHYSICAL EXAMINATION SEQUENCE (BPES)<br />

Student’s<br />

A. GUIDELINES<br />

a. Past Medical History (PH) 5<br />

( )<br />

b. M<br />

During the physical examination, the following guidelines should be observed:<br />

e<br />

d<br />

ications (dose, route, frequency, last dose) 2 ( )<br />

c. Allergies (specific reactions and date) 2 ( )<br />

d. Family History (FH) 2 ( )<br />

e. Social History (SH) 2 ( )<br />

f. Substance Use (tobacco, alcohol and Illicit drugs) 2 ( )<br />

g. Review <strong>of</strong> Systems (ROS) 5 ( )<br />

C. PHYSICAL EXAMINATION<br />

Observe whether and how the student:<br />

1. Demonstrates proper hygienic practices 5 ( )<br />

2. Positions the patient and self properly 5 ( )<br />

for each part <strong>of</strong> the physical exam, being<br />

observant and respectful <strong>of</strong> the patient’s modesty and comfort<br />

3. Performs the patient’s complete physical exam 5 ( )<br />

systematically, completely and correctly<br />

4. Performs the basic physical examination sequence (BPES) 10 ( )<br />

systematically, completely and correctly<br />

5. Performs the appropriate branch steps 10 ( )<br />

systematically, completely and correctly<br />

Maximum<br />

Total Score<br />

Student’s<br />

Total Score<br />

100 ( )<br />

If the Student’s Total Score is < 85,<br />

the Observed History and Physical must be repeated with another patient.<br />

Comments:<br />

Chief Medical Resident’s name: _____________________________________<br />

Chief Medical Resident’s signature: __________________________________<br />

Student’s signature: ______________________________________________<br />

I received constructive and immediate feedback on my Observed History and Physical (circle): Yes No<br />

37


1. The patient always comes first - be considerate <strong>of</strong> his/her comfort and privacy.<br />

2. Students WILL wash their hands before each examination.<br />

3. While the patient must adequately disrobe, modesty should be respected at all times.<br />

4. The examiner should explain what he/she is doing during the examination.<br />

5. Never force a procedure on any adult patient.<br />

6. Reassure the patient during the examination.<br />

7. Be serious, yet maintain an appropriate sense <strong>of</strong> humor.<br />

8. Always be gentle -- a physical examination may be slightly uncomfortable if an<br />

abnormality is present. It should never be painful.<br />

9. Be systematic and thorough.<br />

Physical examination techniques and skills that are required for MD 1 are outlined in<br />

the MD 1 Handbook and are described in detail below , and in your physical diagnosis<br />

textbooks.<br />

B. STEPS OF THE BASIC PHYSICAL EXAMINATION SEQUENCE (BPES)<br />

These are the steps included in the student Basic Physical Examination Sequence<br />

(BPES). They should be performed by the student in every complete physical<br />

examination. The BPES should be completed in 30 minutes.<br />

Patient sitting: physician facing patient<br />

1. GENERAL APPEARANCE: Inspect for habitus, level <strong>of</strong> consciousness, movement,<br />

comfort, affect.<br />

2. VITAL SIGNS: Determine body temperature - read thermometer.<br />

Determine radial pulse - rate per minute and<br />

compare bilateral radial pulses simultaneously.<br />

Determine respiratory rate per minute (w hile<br />

comparing radial pulses).<br />

Determine blood pressure - place BP cuff 2.5 cm<br />

above antecubital fossa and at level <strong>of</strong> atria.<br />

Determine BP by unilateral palpation <strong>of</strong> radial pulse<br />

and then by auscultation <strong>of</strong> the brachial artery in the<br />

antecubital fossa.<br />

3. SKIN: Inspect for overall color, hair and gross lesions. (Inspect and palpate skin in<br />

detail under each regional exam below).<br />

4. HEENT: HEAD: Inspect face for symmetry, abnormality. Inspect and<br />

palpate scalp.<br />

38


EYES: Test Visual acuity. Check each eye separately (w ith<br />

corrective lenses) w ith Rosenbaum pocket chart or new sprint.<br />

Cover opposite eye. Inspect external ocular structures - inspect<br />

lids, conjunctivae, sclerae, cornea. Test Ocular muscle function<br />

and alignment (III, IV, VI). Note light reflex in neutral position. Ask<br />

patient to direct eyes laterally, superiorly and inferiorly " up and<br />

out," " dow n and out." Test pupillary response in accommodation<br />

(convergence) and to light (direct and consensual). Inspect<br />

corneas, lenses, ocular fundi (retina <strong>of</strong> each eye) w ith<br />

ophthalmoscope. Stabilize head w ith opposite hand. Ask patient<br />

to fix gaze on a specific point on the w all. Change hands and<br />

sides appropriately for examination <strong>of</strong> the other eye.<br />

EARS: Test Hearing. Use w ristw atch or w hispered voice. Inspect<br />

and palpate auricles and mastoids. Inspect canals and tympanic<br />

membranes on each side.<br />

NOSE: Palpate for tenderness and ask patient to " sniff" to test<br />

for patency <strong>of</strong> each nasal passage. Inspect each passage w ith<br />

light, using nasal speculum.<br />

MOUTH: Palpate masseters as you ask patient to " bite dow n"<br />

(V) bilaterally. Inspect for symmetry as patient " show s teeth"<br />

(VII) & " protrudes tongue" (XII). Inspect lips, teeth, gingivae,<br />

oral mucosa, tongue, under tongue, and posterior pharynx w ith<br />

tongue blade and light. Note uvula motion w hen patient says<br />

" ah" .<br />

Patient sitting: physician in front <strong>of</strong>/behind patient.<br />

(Tutor to demonstrate both techniques.)<br />

5. NECK: Inspect neck for neck veins (see below - #7). Inspect and palpate for lymph<br />

nodes -postauricular, anterior and posterior cervical, submandibular and supraclavicular<br />

bilaterally, simultaneously with both examining hands. Palpate for parotid glands bilaterally.<br />

Inspect and palpate for thyroid - each lobe separately, with patient swallowing, and<br />

palpate for trachea, from front or rear <strong>of</strong> patient.<br />

39


6. THORAX:<br />

POSTERIOR THORAX: Inspect thorax and palpate entire spine. Percuss at costovertebral angles -w ith<br />

fore-w arning Percuss for diaphragmatic excursion, from mid-thorax dow n to dullness before and after<br />

deep inspiration on each side. Percuss bilateral chest w all at equivalent areas looking for<br />

hyperresonance or dullness. Auscultate posterior and lateral lung fields -w ith diaphragm <strong>of</strong><br />

stethoscope, w hile patient is deep-breathing w ith mouth open. Compare side to side in equivalent<br />

areas.<br />

Patient sitting: physician facing patient.<br />

ANTERIOR THORAX: Inspect. Auscultate anterior lung fields beginning w ith supraclavicular areas<br />

bilaterally w ith deep breathing.<br />

Patient supine: physician at right.<br />

AXILLAE: Palpate w ith passive abduction-adduction (R hand for L axilla and<br />

vice versa).<br />

BREASTS: Inspect and palpate in all quadrants and nipples bilaterally.<br />

7. CARDIOVASCULAR:<br />

NECK: Inspect veins. Palpate carotid pulses on each side one at a time.<br />

Auscultate carotids bilaterally.<br />

HEART: Inspect precordium. Palpate apical impulse (PMI) and entire precordium w ith R hand.<br />

Auscultate w ith diaphragm <strong>of</strong> stethoscope all cardiac areas (aortic, pulmonic, LSB and apex) w ith<br />

concurrent left finger palpation <strong>of</strong> right carotid pulse for time <strong>of</strong> the heart sounds. Auscultate at<br />

pulmonic area for split S2 during inspiration. Auscultate at apex w ith bell <strong>of</strong> stethoscope (supine,<br />

and in left lateral decubitus position, for presence <strong>of</strong> S3, S4 and murmur).<br />

8. ABDOMEN:<br />

ABDOMEN: Begin by inquiring about pain. Alert patient. Ask patient to flex hips<br />

and knees to relax abdominal wall muscles.<br />

40


Inspect. Auscultate - epigastrium, left paraumbilical area over aorta for bruit and<br />

bowel sounds. Palpate - superficially and deeply in all four quadrants. Palpate deeply<br />

for aorta and for liver and spleen with deep inspiration. Palpate for spleen: turn<br />

patient to R lateral decubitus if spleen not previously felt with patient supine.<br />

INGUINAL AREAS:<br />

Inspect. Palpate for lymph nodes and/or masses<br />

and femoral arterial pulses.<br />

9. EXTREMITIES:<br />

LOWER EXTREMITIES:<br />

Inspect and palpate feet, legs, and thighs for<br />

edema. Palpate dorsalis pedis and posterior tibial<br />

pulses bilaterally and simultaneously.<br />

Patient sitting: physician facing patient.<br />

UPPER EXTREMITIES: Inspect skin, fingernails, and for tremor. Test strength- <strong>of</strong> grip bilaterally by<br />

asking patient to squeeze 2 <strong>of</strong> your fingers. Ask patient to "shrug shoulders" (XI).<br />

DEEP TENDON REFLEXES:<br />

plantar reflexes bilaterally.<br />

Test biceps, triceps, patellar, achilles reflexes and<br />

Patient standing: physician facing patient and then behind patient.<br />

POSTURE:<br />

Inspect for symmetry <strong>of</strong> shoulders, hips, and<br />

curvature <strong>of</strong> the spine.<br />

LOWER EXTREMITIES: Inspect for varicose veins, other deformities.<br />

GAIT: Inspect gait as patient walks barefoot.<br />

The follow ing is usually omitted for the video examination:<br />

MALE GENITALIA: Inspect penis (ask patient to retract foreskin) and scrotum.<br />

Palpate scrotal contents and inguinal canals.<br />

ANORECTUM: Men – have patient bend over examining table or lie in<br />

left lateral decubitus position w ith right hip and knee flexed.<br />

Women - patient in dorsosacral position w ith<br />

knees flexed and abducted (if for pelvic exam), or<br />

lying in left lateral decubitus position w ith right hip<br />

and knee flexed.<br />

Inspect anus, buttocks, and perineum.<br />

41


Palpate anal canal, rectal ampulla (and prostate in<br />

men), after gently inserting gloved and lubricated<br />

index finger.<br />

PELVIC EXAM: Instruction by obstetrician-gynecologist.<br />

C. BRANCH STEPS: (EXTENDED PHYSICAL EXAMINATION)<br />

If an abnormality is detected, it must be fully described and additional examination<br />

(branch steps) performed to clarify and delineate that finding.<br />

Example: If decreased auditory acuity is noted, the Weber and Rinne tests must be<br />

done.<br />

Such branch steps, how ever, are not part <strong>of</strong> the BPES.<br />

Branch steps are taught in separate laboratory sessions. They are useful in further<br />

elucidating abnormalities uncovered in the history and BPES and are generally not<br />

screening maneuvers (Refer to page 24).<br />

42


COMMON ERRORS OF THE BASIC PHYSICAL EXAM SEQUENCE<br />

EYES:<br />

When testing visual acuity for each eye corrected preferred, use some form <strong>of</strong><br />

measurement so that your evaluator knows that the same distance is being used for both<br />

eyes. For most pocket eye-chart cards, the card should be held 14 inches away from the<br />

eye being tested.<br />

When testing pupillary responses in accommodation and to light (direct and<br />

consensual), review the difference between accommodation and convergence. For<br />

accommodation, ask the patient to look at a distant object or the wall (for example) for<br />

distance, then have the patient look at your finger held at 2-4 inches from the bridge <strong>of</strong> his/her<br />

nose. Convergence should be tested along with ocular motor function testing.<br />

When inspecting cornea, lens and retina <strong>of</strong> each eye with ophthalmoscope, remember to<br />

turn <strong>of</strong>f the room lights. Use your right eye and hold the ophthalmoscope in your right hand when<br />

examining the patient’s right eye. Conversely, use your left eye and hold the ophthalmoscope in<br />

your left hand when examining the patient’s left eye.<br />

EARS: When testing for auditory acuity bilaterally, patient should have their eyes closed. Test<br />

one ear at a time. Begin by snapping your fingers 3 feet (arms length) away from the ear being<br />

tested. Ask the patient to indicate if he/she hears the snapping <strong>of</strong> your fingers. If the patient<br />

doesn’t hear, move closer to ear in six-inch increments and retest.<br />

NECK: Inspect and palpate thyroid, each lobe separately, and trachea with swallowing from<br />

front or back. If you choose to examine the patient from the front, identify the thyroid<br />

isthmus and then each lobe <strong>of</strong> the thyroid gland together and separately, asking the patient<br />

to swallow as you examine each side.<br />

THORAX:<br />

Review technique to percuss the lungs, percussing the posterior chest wall in<br />

symmetrical areas.<br />

Review proper technique to percuss diaphragmatic excursion on each side.<br />

Need to estimate the descent <strong>of</strong> the diaphragm when patient inspires.<br />

Review proper technique to palpate axillae with passive abduction-adduction. Your left<br />

hand should be used to examine the patient’s right axillae, using your right hand to support the<br />

patient’s right arm. Your right hand should be used to examine the patient’s left axillae, using<br />

your left hand to support the patient’s left arm.<br />

ABDOMEN:<br />

Review proper use <strong>of</strong> hands. Your left hand should be used to support the patient’s left<br />

flank and your right should be used to feel for the spleen.<br />

EXTREMITIES:<br />

Review proper technique to test for deep tendon reflexes, biceps, triceps, knee, ankle,<br />

and plantar reflexes bilaterally. Student should check patient’s bicep reflexes on each arm then<br />

tricep reflexes on each arm in order to compare bilaterally versus checking the bicep and tricep<br />

reflexes on one arm then moving to the other arm and checking the bicep and tricep reflexes.<br />

When inspecting the spine, please have patient bend over forward in order to check the<br />

curvature <strong>of</strong> their spine.<br />

43


EVALUATION OF PERFORMANCE OF<br />

BASIC PHYSICAL EXAMINATION SEQUENCE<br />

Student: Date: _________ Evaluator: ___________________<br />

Steps Done Not<br />

Done<br />

Seq.<br />

Incor<br />

.<br />

Tech<br />

.<br />

Incor<br />

.<br />

Comments<br />

1. General Appearance: (sitting)<br />

Inspect overall<br />

2. Vital Signs (sitting)<br />

A. Temperature with thermometer<br />

B. Pulse - Measure rate per minute<br />

Compare bilaterally<br />

C. Respiratory - rate per minute<br />

D. Blood pressure - Palpation (unilateral)<br />

(unilateral)<br />

- Auscultation<br />

3. Skin (sitting)<br />

Inspect overall<br />

4. HEENT<br />

A. Head - Inspect face<br />

Inspect/palpate scalp<br />

B. Eyes (sitting)<br />

Test visual acuity for each eye,<br />

corrected preferred<br />

Inspect lids, conjunctivae, sclerae,<br />

corneas<br />

Test extraocular muscle movements<br />

and convergence<br />

Test pupillary responses to light (direct<br />

and consensual) and to accomodation<br />

Inspect cornea, lens and retina <strong>of</strong> each<br />

eye with ophthalmoscope<br />

C. Ears (sitting)<br />

Test for auditory acuity bilaterally<br />

Inspect and palpate auricles and<br />

mastoids<br />

Inspect canals and tympanic<br />

membranes<br />

44


Steps Done Not<br />

Done<br />

Seq.<br />

Incor<br />

.<br />

Tech<br />

.<br />

Incor<br />

.<br />

Comments<br />

D. Nose (sitting)<br />

Palpate nose and sinuses for<br />

tenderness<br />

Test patency <strong>of</strong> each nasal passage by<br />

asking patient to "sniff"<br />

Inspect each nasal passage with<br />

speculum<br />

E. Mouth (sitting)<br />

Test CN V - "bite down" palpate<br />

masseter<br />

Examine CN VII - "show teeth"<br />

Test CN XII - "protrude tongue"<br />

Inspect lips, teeth, gingivae, tongue,<br />

posterior pharynx: "say ah"<br />

5. Neck (sitting)<br />

Inspect for neck veins (see below also)<br />

Inspect/palpate for postauricular, cervical,<br />

sub-mandibular and supraclavicular nodes<br />

Palpate for parotid gland on each side<br />

Inspect and palpate thyroid (each lobe<br />

separately), (and trachea) with swallowing<br />

6. Thorax (sitting)<br />

A. Posterior Thorax (sitting)<br />

Inspect posterior thorax with<br />

respiration<br />

Inspect and palpate entire spine<br />

Percuss costovertebral angles for<br />

tenderness<br />

Percuss posterior thorax bilaterally and<br />

symmetrically<br />

Percuss diaphragmatic excursion<br />

bilaterally<br />

Auscultate posterior lung fields<br />

bilaterally with deep inspiration<br />

Auscultate lateral lung fields bilaterally<br />

B. Anterior Thorax (sitting)<br />

Inspect anterior thorax with respiration<br />

45


Steps Done Not<br />

Done<br />

Seq.<br />

Incor<br />

.<br />

Tech<br />

.<br />

Incor<br />

.<br />

Comments<br />

Auscultate anterior lung fields<br />

bilaterally (starting with supraclavicular area)<br />

with deep inspiration<br />

C. Axillae (supine) Palpate axillae with<br />

passive abduction- adduction<br />

D. Breasts (supine)<br />

Inspect & palpate breasts including<br />

nipples<br />

7. Cardiovascular (supine)<br />

A. Neck Veins/arteries: Inspect, palpate,<br />

auscultate carotid arteries bilaterally one at a<br />

time<br />

B. Heart (supine) Inspect precordium<br />

Palpate PMI (apical impulse) &<br />

precordium<br />

Auscultate with diaphragm at aortic and<br />

pulmonic areas, LSB and apex<br />

Auscultate for splitting <strong>of</strong> S 2 in pulmonic<br />

area<br />

left<br />

Auscultate with bell at apex (supine &<br />

lateral decubitus)<br />

8. Abdomen (supine; hips and knees flexed)<br />

Alert patient to abdominal exam<br />

Inspect abdomen<br />

for<br />

Auscultate epigastrium and over aorta<br />

bruit, and bowel sounds<br />

Palpate superficially/deeply in 4<br />

quadrants<br />

Palpate for liver<br />

Palpate for aorta<br />

Palpate for spleen - supine position<br />

- R lateral decubitus<br />

Inspect and palpate for inguinal nodes<br />

and femoral pulses in inguinal areas<br />

bilaterally<br />

9. Extremities<br />

A. Lower Extremities (supine)<br />

Inspect and palpate legs and feet for edema<br />

46


Steps Done Not<br />

Done<br />

Seq.<br />

Incor<br />

.<br />

Tech<br />

.<br />

Incor<br />

.<br />

Comments<br />

Palpate dorsalis pedis & posterior tibial<br />

pulses bilaterally<br />

B. Upper Extremities (sitting)<br />

Inspect skin and nails. Observe for<br />

tremor<br />

Examine grip<br />

Examine CN XI ("shrug shoulders")<br />

Test for deep tendon reflexes, biceps,<br />

triceps, knee, ankle, and plantar<br />

reflexes bilaterally<br />

C. Posture (standing) - Inspect spine<br />

D. Lower Extremities (standing)<br />

Inspect for varicosities<br />

E. Gait - Inspect walking<br />

Did student respect patient's privacy?<br />

No<br />

Did student arouse patient's anxiety?<br />

No<br />

Yes<br />

Yes<br />

Did student display appropriate affect?<br />

No<br />

Yes<br />

Did student use appropriate verbal/nonverbal behavior? Yes<br />

No<br />

47


U.H. John A. Burns School <strong>of</strong> <strong>Medicine</strong><br />

Third-Year Clerkship in Internal <strong>Medicine</strong><br />

OBSERVED PATIENT COUNSELING<br />

Evaluation Form<br />

Complete 2 during ambulatory medicine and turn in to your Ambulatory Preceptor.<br />

Student: __________________________ Site: ________________ Date: __________<br />

Setting: _____ Inpatient<br />

Counseling was directed at: _____ Patient<br />

_____ Ambulatory (check one or both) _____ Patient’s family<br />

Counseling:<br />

___ Explain diagnoses<br />

___ Explain tests, procedures or surgery<br />

___ Review results <strong>of</strong> tests, procedures or surgery<br />

___ Review medications (indications, dosing, side effects, etc.)<br />

___ Review diets, exercise or other lifestyle changes<br />

___ Discuss smoking cessation<br />

___ Discuss alcohol use<br />

___ Discuss compliance (medications, follow-up, etc.)<br />

___ Review discharge instructions<br />

___ Other (specify) ___________________________<br />

Good<br />

Needs Improvement<br />

Student used clear and understandable language.<br />

Student adapted to patient’s/family’s readiness to learn.<br />

Student adapted to patient’s/family’s comprehension level<br />

All pertinent information was presented accurately.<br />

Student demonstrated empathy and compassion.<br />

Patient’s/family’s comprehension was assessed.<br />

Evaluator Comments:<br />

Evaluator’s name _________________________<br />

Evaluator’s signature _______________________<br />

Student’s signature _______________________<br />

I received constructive feedback on my Observed Patient Counseling (circle): Yes No<br />

48


U.H. John A. Burns School <strong>of</strong> <strong>Medicine</strong><br />

Third-Year Clerkship in Internal <strong>Medicine</strong><br />

CASE PRESENTATION<br />

Evaluation Form<br />

Before presenting, the student should give this form to the attending or resident leading rounds. Afterwards, the<br />

student should turn the form in to the Hospital Site Coordinator.<br />

Student: _______________________________________<br />

Date: _______________<br />

Rounds:<br />

___ Attending Rounds<br />

___ Bedside Clinical Skills<br />

___ ICU Rounds<br />

___ Morning Report<br />

___ PBL Tutorial<br />

___ Other Rounds/Conference __________________<br />

Major problems/diagnoses <strong>of</strong> case presented:<br />

1. ___________________________<br />

2. ___________________________<br />

3. ___________________________<br />

Presentation was clear<br />

Presentation was organized<br />

Presentation was memorized<br />

All pertinent history was presented<br />

accurately<br />

All pertinent physical findings were<br />

presented accurately<br />

All pertinent labs were presented<br />

accurately<br />

All pertinent problems were correctly<br />

identified<br />

Assessment was appropriate for level <strong>of</strong><br />

training<br />

Plan (diagnostic, therapeutic, education)<br />

was appropriate for level <strong>of</strong> training<br />

Comments:<br />

Exceptional<br />

Very<br />

Competent Competent Marginal Unsatisfactory<br />

Evaluator:<br />

Print _____________________________<br />

Signature _________________________<br />

49


U.H. John A. Burns School <strong>of</strong> <strong>Medicine</strong><br />

Third-Year Clerkship in Internal <strong>Medicine</strong><br />

SMALL GROUP LEARNING EXPERIENCE<br />

Evaluation Form<br />

Student: ____________________ Date: _________ Activity: ____ PBL Tutorial<br />

____ Bedside Clinical Skills<br />

____ Chief Rounds<br />

____ Other: ___________<br />

Check appropriate box on this Rating Scale:<br />

Activity<br />

Participates actively<br />

Shares knowledge<br />

Respects opinions and learning<br />

needs <strong>of</strong> others<br />

Asks thoughtful questions<br />

Facilitates group process<br />

Demonstrates appropriate fund<br />

<strong>of</strong> knowledge for MS3<br />

Demonstrates appropriate<br />

clinical skills for MS 3<br />

Is properly prepared for this<br />

activity<br />

Evaluator Comments:<br />

Exceptional<br />

Very<br />

Competent<br />

Competent Marginal Unsatisfactory<br />

Evaluator’s name: ___________________ Evaluator’s signature: _____________________<br />

Student’s signature _________________<br />

50


U.H. John A. Burns School <strong>of</strong> <strong>Medicine</strong><br />

Third-Year Clerkship in Internal <strong>Medicine</strong><br />

MID-CLEKSHIP FEEDBACK FORM<br />

Student’s name: ___________________________________________________<br />

Considering the three domains <strong>of</strong> Medical Knowledge, Clinical Skills and Pr<strong>of</strong>essionalism,<br />

This student’s STRENGTHS are:<br />

This student NEEDS TO WORK AND IMPROVE ON:<br />

Overall, this student’s progress to date is:<br />

___ Satisfactory<br />

___ Unsatisfactory<br />

Evaluator’s name: ___________________________<br />

Student's signature: __________________________<br />

Signature: _____________________________<br />

Date reviewed with student: _______________<br />

Please make a photocopy <strong>of</strong> this form after it is completed, reviewed and signed by both the Evaluator and the<br />

Student.<br />

Instructions for the Student:<br />

Keep a photocopy <strong>of</strong> this form after your Hospital Site Coordinator or Ambulatory Preceptor completes and reviews it with<br />

you.<br />

You should actively work on and improve the areas identified above.<br />

You will be instructed on how to turn in the completed form at the end <strong>of</strong> your inpatient block and at the end <strong>of</strong> your<br />

ambulatory block.<br />

Instructions for the Evaluator:<br />

Please keep a copy and include the information in your final Student Evaluation Form.<br />

Specifically, if you identified any areas to work on and improve, please comment on whether the student successfully<br />

responded to your feedback and improved by the end <strong>of</strong> the block.<br />

51


Exceptional<br />

Very Competent<br />

Competent<br />

Marginal<br />

Unsatisfactory<br />

Not Evaluated<br />

UH John A. Burns School <strong>of</strong> <strong>Medicine</strong><br />

Third-Year Clerkship in Internal <strong>Medicine</strong><br />

STUDENT EVALUATION FORM<br />

Student: _____________________________________<br />

Ambulatory Site: _______________________<br />

Block: _____ Dates <strong>of</strong> rotation: _____________________ Date <strong>of</strong> this report: ___________________<br />

IMPORTANT:<br />

o Comments are MANDATORY for any Exceptional, Marginal or Unsatisfactory ratings.<br />

<br />

I. Life-Long Learning Skills<br />

Graduates will be life-long learners.<br />

Demonstrates enthusiasm, motivation and self-initiative to learn<br />

Identifies relevant learning issues, searches for and critically appraises information<br />

(self-directed learning)<br />

Comments:<br />

II.<br />

The Biological Sciences<br />

Graduates will understand the biological sciences underlying clinical medicine.<br />

Demonstrates basic science knowledge (including anatomy, physiology,<br />

pathology and pathophysiology)<br />

Demonstrates clinical knowledge (including knowledge <strong>of</strong> the illnesses seen in the<br />

adult patient/population in <strong>Hawaii</strong>, Asia-Pacific region and globally)<br />

Applies basic science knowledge to clinical situations<br />

Comments:<br />

52


Exceptional<br />

Very Competent<br />

Competent<br />

Marginal<br />

Unsatisfactory<br />

Not Evaluated<br />

<br />

<br />

III. The Care <strong>of</strong> Patients<br />

Graduates will be able to care for their patients under the supervision <strong>of</strong> faculty.<br />

Demonstrates respect and compassion for patients and their families<br />

Serves as an effective patient advocate<br />

Interviews patients in a systematic, organized and thorough manner<br />

(History Taking)<br />

Examines patients in a systematic, organized and thorough manner<br />

(Physical Exam)<br />

Formulates appropriate Assessments, Problem Lists and Differential Diagnoses<br />

Formulates appropriate Diagnostic Plans<br />

Formulates appropriate Therapeutic Plans<br />

Educates and counsels patients (and their families) about their health and<br />

medical problems<br />

Demonstrates clinical judgment and problem solving skills<br />

Works efficiently<br />

Comments:<br />

IV. Oral and Written Communication Skills<br />

Graduates will be able to communicate effectively with patients, families and other<br />

healthcare providers.<br />

Uses effective verbal and non-verbal communication skills (including listening<br />

and using language appropriate for each patient, family and clinical setting)<br />

Presents cases in a systematic, organized and thorough manner that is accurate<br />

and appropriate for the clinical setting<br />

Writes notes (write-ups and progress notes, etc.) in a systematic, organized and<br />

thorough manner that is accurate, legible and appropriate for the clinical setting<br />

Comments:<br />

53


Exceptional<br />

Very Competent<br />

Competent<br />

Marginal<br />

Unsatisfactory<br />

Not Evaluated<br />

<br />

V. Pr<strong>of</strong>essionalism<br />

Graduates will be pr<strong>of</strong>essional and ethical, demonstrate an enthusiasm for medicine, and value<br />

honor, integrity, altruism, respect, accountability, excellence, scholarship, and leadership<br />

while delivering compassionate care to their patients.<br />

Demonstrates pr<strong>of</strong>essional appearance and attire<br />

Attends all required <strong>clerkship</strong> and training site-specific activities; is punctual<br />

Listens attentively and participates actively<br />

Completes all <strong>clerkship</strong> and patient care responsibilities on time<br />

Demonstrates pr<strong>of</strong>essional behavior<br />

Demonstrates pr<strong>of</strong>essional relationships (with attendings, residents, students,<br />

nurses, other health care workers, patients and their families)<br />

Seeks, accepts and responds positively and effectively to feedback<br />

Works as an effective team member<br />

Comments:<br />

COMMENTS:<br />

Ambulatory Preceptor’s name: ___________________________________________<br />

Ambulatory Preceptor’s signature: ____________________________ Date: _________________<br />

Student’s signature: ______________________________________<br />

Date: _________________<br />

Please return this form to:<br />

Julieta Rajlevsky<br />

UH <strong>Department</strong> <strong>of</strong> <strong>Medicine</strong><br />

1356 Lusitana St., 7 th Floor, Honolulu, HI 96813<br />

Fax: (808) 586-7486 Phone: (808) 586-7478<br />

54


UNIVERSITY OF HAWAII JOHN A. BURNS SCHOOL OF MEDICINE<br />

THIRD-YEAR CLERKSHIP IN INTERNAL MEDICINE<br />

STUDENT EVALUATION FORM<br />

Ratings Definitions<br />

EXCEPTIONAL:<br />

The student consistently demonstrates medical knowledge, clinical skills or pr<strong>of</strong>essionalism at a level<br />

higher than expected for a third year student. In other words, the student consistently “goes the extra mile”<br />

and performs “above and beyond” in the item rated.<br />

Comments are mandatory for this rating.<br />

VERY COMPETENT:<br />

The student on occasion demonstrates medical knowledge, clinical skills or pr<strong>of</strong>essionalism at a level<br />

higher than expected for a third year student. In other words, the student on occasion, but not consistently,<br />

“goes the extra mile” and performs “above and beyond” in the item rated. The student should continue to<br />

strive for Exceptional ratings.<br />

COMPETENT:<br />

The student demonstrates medical knowledge, clinical skills or pr<strong>of</strong>essionalism at the level expected for a<br />

third year student. Note that the <strong>clerkship</strong> expects that most students will receive Competent ratings for the<br />

majority <strong>of</strong> items. Some students in the past have incorrectly interpreted Competent ratings as punitive.<br />

On the contrary, this rating merely indicates that the student performed at the expected level for that<br />

particular item. The student should continue to strive for Very Competent and Exceptional ratings.<br />

MARGINAL:<br />

The student demonstrates medical knowledge, clinical skills or pr<strong>of</strong>essionalism slightly below the level<br />

expected for a third year student. An item rated Marginal is an area <strong>of</strong> weakness that has been identified.<br />

The student will be expected to work on this area <strong>of</strong> weakness and demonstrate improvement to the level<br />

expected for a third year student. A Marginal rating probably precludes an Honors grade.<br />

Comments are mandatory for this rating.<br />

UNSATISFACTORY:<br />

The student demonstrates medical knowledge, clinical skills or pr<strong>of</strong>essionalism below the level expected<br />

for a third year student. An item rated Unsatisfactory is an area <strong>of</strong> weakness that has been identified.<br />

Formal remediation may be required. An Unsatisfactory rating precludes an Honors grade.<br />

Comments are mandatory for this rating.<br />

55


OTHER FORMS<br />

56


U.H. John A. Burns School <strong>of</strong> <strong>Medicine</strong><br />

Third-Year Clerkship in Internal <strong>Medicine</strong><br />

MS3-RESIDENT<br />

INPATIENT MEDICINE AGREEMENT FORM<br />

(Revised 8/11/12)<br />

Name <strong>of</strong> MS3 _______________________________________________<br />

1. The upper level (UL) resident (Level 2 or 3) is responsible for the third-year medical<br />

student’s inpatient medicine experience. At the beginning and throughout the student’s<br />

inpatient experience, the UL should clarify what is expected <strong>of</strong> the student.<br />

2. The earliest time that the student is permitted to arrive at the hospital is:<br />

o 4:00 a.m. at Kuakini<br />

o 5:00 a.m. at Queen’s<br />

o 5:00 a.m. at Tripler<br />

The earliest time that the student is permitted to see patients is:<br />

o 4:30 a.m. at Kuakini<br />

o 5:30 a.m. at Queen’s<br />

o 5:30 a.m. at Tripler<br />

3. The student must take every call with his/her team throughout the inpatient rotation.<br />

Call at Kuakini:<br />

The student must take every call (q4) with his/her team throughout the inpatient rotation.<br />

Overnight Call - 1 required, preferably a Fri night<br />

If the student is unable to take overnight call on a Fri night, he/she must talk to the<br />

CMR and the Hospital Site Coordinator about taking overnight call on a Sat instead.<br />

Call – q4 days until 10 p.m. at the latest<br />

(May leave earlier depending on admissions and UL approval.)<br />

Non-Call Days – The student will assist his/her team until after the team signs out.<br />

Call at Queen’s:<br />

The student must take every call (q4) with his/her team throughout the inpatient rotation.<br />

Overnight Call - none<br />

Long Call – q4 days until 10 p.m. at the latest<br />

(May leave earlier depending on admissions and UL approval.)<br />

Non-Call Days –The student will assist his/her team with admitting patients and may<br />

leave after team is done with work, with UL approval.<br />

Call at Tripler:<br />

The student must take every call (q4) with his/her team throughout the inpatient rotation.<br />

Overnight Call - none<br />

Long Call: 0600 – 1900 q4 days<br />

Short Call: 0600 – 1600 all other days - The student will assist his/her team until sign<br />

out.<br />

4. The UL is responsible for assigning patients to the student (see Training Problems List).<br />

The student will admit 1 - 2 patients per call.<br />

The student should actively follow an average <strong>of</strong> 2 patients at all times (maximum 5).<br />

57


5. The student must interview and examine patients on his/her own. The student may<br />

observe the Intern and/or UL obtain the history and physical, but this observation does not<br />

qualify as the student’s history and physical.<br />

6. The student must pre-round and write daily Progress Notes on all his/her assigned<br />

patients before the Intern and/or UL write their notes. The UL should review the<br />

Progress Notes with the student, give constructive feedback and countersign the note.<br />

7. The UL is responsible for insuring proper supervision <strong>of</strong> the following parts <strong>of</strong> the<br />

physical exam performed by the student: female breast exams, pelvic exams, rectal<br />

exams and prostate exams. The supervision must be provided by a physician (such as<br />

Interns, ULs, Chief Residents or Attendings).<br />

8. The UL is responsible for insuring proper supervision <strong>of</strong> any procedure performed<br />

by the student. The supervision must be provided by a physician (such as Interns, ULs,<br />

Chief Residents or Attendings) who is certified or has expertise to competently perform<br />

the procedure in question. Note that there are no required procedures for students.<br />

9. The UL should assist the student in preparing case presentations at hospital rounds<br />

or conferences at a level that is appropriate for the student’s training. Whenever a<br />

student’s patient will be presented, the student is expected to be the one presenting the<br />

patient (unless it is an ICU patient at Kuakini – then it is up to the CMR and HSC).<br />

10. The student must have one (1) day <strong>of</strong>f per week, either a Saturday or a Sunday.<br />

At Kuakini, the day <strong>of</strong>f should be on Sunday - unless the student has call on Sunday, in<br />

which case the student will take Saturday <strong>of</strong>f.<br />

11. The student must inform the UL <strong>of</strong> the student’s activities and whereabouts at all<br />

times. Specifically, the student must notify the UL whenever leaving the hospital,<br />

including leaving to attend required 3 rd year or <strong>clerkship</strong> activities or to study, and should<br />

discuss if or when he/she needs to return.<br />

12. When the UL is absent or <strong>of</strong>f, the Intern should assume the UL’s role and<br />

responsibilities, including all those listed above, in supervising the student.<br />

13. The Intern and UL should discuss medical student issues and problems with the<br />

Chief Resident and/or Hospital Site Coordinator as soon as possible.<br />

1 st Upper Level Resident:<br />

Name _______________________ Signature ______________________ Date _________<br />

(Required by end <strong>of</strong> MS3’s 1 st week)<br />

2nd Upper Level Resident:<br />

Name _______________________ Signature ______________________ Date_________<br />

(Required by end <strong>of</strong> UL’s 1 st week)<br />

58


Third-Year Clerkship in Internal <strong>Medicine</strong><br />

INPATIENT WORK HOURS LOG<br />

Student _____________________________________<br />

Inpatient site (circle) KMC QMC TAMC<br />

Week ___ beginning ____/_____/2012<br />

On Call?<br />

Other Activities?<br />

# New Patients<br />

(admissions, transfers)<br />

# Old Patients<br />

TOTAL # PATIENTS<br />

For KMC,<br />

# ICU Patients<br />

Time In<br />

Time Out<br />

TOTAL # HOURS<br />

MONDAY TUESDAY WEDNESDAY THURSDAY FRIDAY SATURDAY SUNDAY<br />

TOTAL # HOURS FOR THE WEEK: _______<br />

o<br />

o<br />

o<br />

o<br />

o<br />

o<br />

o<br />

o<br />

o<br />

o<br />

o<br />

o<br />

Please log your hours daily – otherwise it’s difficult to remember.<br />

You should log only the hours that you are "working" which includes patient care and required 3 rd year, hospital and <strong>clerkship</strong> activities such as Colloquia, rounds, conferences,<br />

PBL Tutorials, Bedside Clinical Skills, Chief Rounds, CV PE, EBM, EKG, HIV <strong>Medicine</strong>, METS Sim Session, Neuro, etc.<br />

It's ok to include meals in the middle <strong>of</strong> your “work day” (as long as it's not a 1 hr lunch!) – it’s too much trouble to clock out for lunch and then clock in afterwards.<br />

You should not include meals at the hospital before/after your “work day” or reading at the hospital before/after your “work day.”<br />

You should not include writing your Comprehensive Write-ups and LIs – even if done at the hospital - since that is "home" work.<br />

You should work no more than 80 hours per week, averaged over the course <strong>of</strong> the entire <strong>clerkship</strong>.<br />

You should have 1 day <strong>of</strong>f per week, usually a Sat or Sun.<br />

You should follow the holiday schedule observed by your site, since this varies by site.<br />

On Call? If relevant, please specify Short, Long, or Overnight.<br />

Other Activities? Please specify <strong>of</strong>f-campus activities. Ex: Colloquia, HIV <strong>Medicine</strong>, METS Sim Session, etc.<br />

# Patients is the # <strong>of</strong> patients (counted at the end <strong>of</strong> each day) that you are actively following - that is, pre-rounding on, writing notes on and presenting.<br />

Ask your UL to sign the following Monday _____________________________________________Then turn completed Log in to your CMR/Hospital Site Coordinator.<br />

59


PHYSICIAN’S ORDERS<br />

DATE: / / TIME:<br />

IMPORTANT<br />

INSTRUCTIONS<br />

► 1. All orders must be written in metric system<br />

2. Write with ball point pen only<br />

SIGNATURE<br />

DATE: / / TIME:<br />

SIGNATURE<br />

DATE: / / TIME:<br />

SIGNATURE<br />

60


APPENDICES<br />

61


U.H. John A. Burns School <strong>of</strong> <strong>Medicine</strong><br />

Third-Year Clerkship in Internal <strong>Medicine</strong><br />

TRAINING PROBLEMS LIST<br />

You are required to see at least one patient with each <strong>of</strong> the 33 Training Problems during the course <strong>of</strong> this <strong>clerkship</strong>.<br />

This is the minimum requirement. Your goal, however, should be to see at least one inpatient patient and one<br />

ambulatory patient with each <strong>of</strong> the Training Problems. The Training Problem does not have to be the patient’s Chief<br />

Complaint. In fact, a patient may present with many Training Problems.<br />

Of course, the more patients you see with these Training Problems, the more you will learn.<br />

Keep track <strong>of</strong> your patient encounters in the chart below.<br />

Indicate which encounters involved Direct Patient Care (D) and which encounters did not involve direct patient care,<br />

referred to as “Special” (S), as in your PDA Patient Log.<br />

It is your responsibility to insure that you have seen patients with each <strong>of</strong> the 33 Training Problems by the end <strong>of</strong> the<br />

<strong>clerkship</strong>. You are advised to see your Upper Level Resident, Chief Medical Resident and/or Hospital Site Coordinator<br />

(when you are on inpatient medicine) or Ambulatory Preceptor (when you are on ambulatory medicine) as soon as<br />

possible to assist you in finding appropriate patients.<br />

Please refer to the Student Handbook for the Specific Learning Objectives for each Training Problem.<br />

Your study <strong>of</strong> Internal <strong>Medicine</strong> in this <strong>clerkship</strong> should be guided by these Training Problems and their Specific Learning<br />

Objectives.<br />

Inpatient<br />

Ambulatory<br />

-- -- D S 1. Healthy Patient:<br />

Health promotion, disease prevention and screening<br />

(i.e. annual or routine physical exam)<br />

Patients with a symptom, sign or<br />

lab abnormality (14):<br />

D S D S 2. Abdominal pain<br />

D S D S 3. Altered mental status<br />

D S D S 4. Anemia<br />

D S D S 5. Back pain<br />

D S D S 6. Chest pain<br />

D S D S 7. Cough<br />

D S D S 8. Dyspnea<br />

D S D S 9. Dysuria<br />

D S D S 10. Fever<br />

D S D S 11. Fluid, electrolyte & acid-base disorders<br />

D S D S 12. GI bleeding<br />

D S D S 13. Knee pain<br />

D S D S 14. Rash<br />

D S D S 15. Upper respiratory complaints<br />

62


TRAINING PROBLEMS LIST (continued)<br />

Inpatient<br />

Ambulatory<br />

Patients with a known condition<br />

(18):<br />

D S D S 16. Acute MI<br />

D S D S 17. Acute renal failure & Chronic kidney<br />

disease<br />

D S D S 18. Common cancers<br />

D S D S 19. COPD & Obstructive airways disease<br />

D S D S 20. Diabetes mellitus<br />

D S D S 21. Dyslipidemias<br />

D S D S 22. Heart failure<br />

D S D S 23. HIV infection<br />

D S D S 24. Hypertension<br />

D S D S 25. Liver disease<br />

D S D S 26. Major depression<br />

D S D S 27. Nosocomial infections<br />

D S D S 28. Obesity<br />

D S D S 29. Pneumonia<br />

D S D S 30. Rheumatologic problems<br />

D S D S 31. Smoking cessation<br />

D S D S 32. Substance abuse<br />

D S D S 33. Venous thromboembolism<br />

Complete the Inpatient half <strong>of</strong> this 2 page chart by your last day <strong>of</strong> inpatient medicine, including your Hospital Site<br />

Coordinator’s signature, and then turn in to Julieta Rajlevsky in the Dept <strong>of</strong> <strong>Medicine</strong> at UHT 7 th fl.<br />

Complete the Ambulatory half <strong>of</strong> this 2 page chart by your last day <strong>of</strong> ambulatory medicine, including your<br />

Ambulatory Preceptor’s signature, and then turn in to Julieta Rajlevsky in the Dept <strong>of</strong> <strong>Medicine</strong> at UHT 7 th fl.<br />

Student_____________________________<br />

Name<br />

____________________________ Date____________<br />

Signature<br />

Hospital Site Coordinator____________________ _____________________________ Date____________<br />

Name<br />

Signature<br />

Ambulatory Preceptor______________________<br />

Name<br />

_____________________________ Date ____________<br />

Signature<br />

63


TRAINING PROBLEM #1: THE HEALTHY PATIENT: HEALTH<br />

PROMOTION, DISEASE PREVENTION, AND SCREENING<br />

RATIONALE:<br />

The growing appreciation for the contributions <strong>of</strong> screening, prevention, and health promotion to<br />

health outcomes necessitates that basic clinical education incorporate advances made in this<br />

area. Especially important are those interventions that relate to prevention <strong>of</strong> cardiovascular<br />

disease, the early detection and treatment <strong>of</strong> potentially curable cancers, and to optimizing care<br />

for chronic diseases.<br />

PREREQUISITES:<br />

Prior knowledge, skills and attitudes acquired during the pre-<strong>clerkship</strong> experience should include:<br />

Ability to perform a complete medical history and physical exam.<br />

Ability to obtain a patient history that includes a family history and an assessment <strong>of</strong> risk<br />

factors.<br />

Knowledge <strong>of</strong> the warning signs <strong>of</strong> common cancers.<br />

Knowledge <strong>of</strong> basic criteria and principles <strong>of</strong> health screening.<br />

Knowledge <strong>of</strong> clinical epidemiologic concepts as they pertain to estimation <strong>of</strong> health risk and<br />

quantitative rationale for screening.<br />

SPECIFIC LEARNING OBJECTIVES:<br />

A. KNOWLEDGE: Students should be able to define, describe, and discuss:<br />

1. The epidemiology and definitions <strong>of</strong> hypertension, its contribution to cardiovascular risk, the<br />

impact <strong>of</strong> treatment on risk, and current. Recommendations for screening. (MK)<br />

2. The epidemiology <strong>of</strong> hyperlipidemia, its contribution to cardiovascular risk, the reliability <strong>of</strong><br />

testing modalities, the impact <strong>of</strong> treatment on cardiovascular risk, and current<br />

recommendations for screening. (MK)<br />

3. The epidemiology <strong>of</strong> common cancers, including:<br />

Breast cancer, including the efficacy <strong>of</strong> available screening modalities, impact <strong>of</strong> early<br />

treatment on survival, and current recommendations for screening. (MK)<br />

Common skin cancers, including the warning signs <strong>of</strong> melanoma and basal and squamous<br />

cell carcinoma. (MK)<br />

Cervical cancer, including the utility <strong>of</strong> the Pap smear, impact <strong>of</strong> early treatment on<br />

outcome, and current recommendations for screening.(MK)<br />

Colorectal cancer, including the utility <strong>of</strong> available screening methodologies, the impact <strong>of</strong><br />

early treatment on outcome, and current screening recommendations. (MK)<br />

Prostate cancer, including the utility <strong>of</strong> available screening modalities, impact <strong>of</strong> early<br />

treatment on outcome, and current screening recommendations. (MK)<br />

4. The risks, benefits, methods, and recommendations for immunizing adults against hepatitis B,<br />

influenza, pneumococcal infection, tetanus/diphtheria, and mumps/measles/rubella. (MK)<br />

5. Safe sexual practices and risks, benefits, and efficacy <strong>of</strong> common methods <strong>of</strong> contraception.<br />

(MK)<br />

6. Efficacy <strong>of</strong> seat belt use and proper belt application. (MK)<br />

7. Efficacy <strong>of</strong> exercise and weight loss in prevention <strong>of</strong> cardiovascular disease and recommended<br />

exercise programs. (MK)<br />

8. The clinical presentations <strong>of</strong> substance abuse and basic approaches to prevention and<br />

treatment. (MK)<br />

64


9. The impact <strong>of</strong> smoking on cardiovascular and cancer risk and basic approaches to smoking<br />

cessation. (MK)<br />

10. Daily caloric, fat, carbohydrate, protein, mineral, and vitamin requirements; adequacy <strong>of</strong> diets<br />

in providing such requirements; evidence <strong>of</strong> need for supplements (e.g. calcium, antioxidants).<br />

(MK)<br />

11. The functional status assessment in the geriatric patient and its impact on assuring the best<br />

possible functional state. (MK)<br />

12. Common environmental and occupational hazards. (MK)<br />

13. Controversies and differences that exist in the recommendations for preventive measures and<br />

screening. (MK)<br />

B. SKILLS: Students should demonstrate specific skills including:<br />

1. History-taking skills: Students should be able to obtain, document, and present an ageappropriate<br />

medical history, including:<br />

Dietary intake <strong>of</strong> fats and cholesterol. (PC, CS)<br />

Exercise and activity levels. (PC, CS)<br />

Substance use and its effects, including tobacco, alcohol, and elicit drugs. (PC, CS)<br />

Psychosocial stresses and environmental risks. (PC, CS)<br />

Specific cancer risks (e.g. family history, exposures, warning symptoms, preventive efforts).<br />

(PC, CS)<br />

Any high-risk sexual practices. (PC, CS)<br />

Immunization status appropriate for adults, including:<br />

o Diphtheria/tetanus for all adults. (PC, CS)<br />

o Influenza vaccine and pneumococcal vaccine for the elderly and those with underlying<br />

chronic disease. (PC, CS)<br />

o Rubella for sero-negative women <strong>of</strong> child-bearing age. (PC, CS)<br />

o Hepatitis B vaccine for medical personnel and other at-risk populations. (PC, CS)<br />

2. Physical exam skills: Students should be able to perform a physical exam with features<br />

depending on age/sex/race and medical history <strong>of</strong> an individual, including:<br />

Screening skin examination for signs <strong>of</strong> malignancy. (PC)<br />

Screening breast examination for a dominant nodule and secondary signs <strong>of</strong> malignancy.<br />

(PC)<br />

Participation in obtaining a Pap smear. (PC)<br />

Screening rectal examination that includes palpation <strong>of</strong> the prostate gland, identification <strong>of</strong><br />

any nodules, and performance <strong>of</strong> a stool test for occult blood. (PC)<br />

Performance <strong>of</strong> a functional status examination in the geriatric patient (PC)<br />

3. Differential diagnosis: Students should be able to generate a prioritized differential diagnosis<br />

using specific history, physical exam, and laboratory findings identified during the screening<br />

examination (PC, MK)<br />

4. Laboratory interpretation: Students should be able to recommend and interpret laboratory<br />

tests for screening purposes, including consideration <strong>of</strong> test cost and performance<br />

characteristics as well as patient preferences. Laboratory and other tests may include, when<br />

appropriate:<br />

Complete blood count. (PC, MK)<br />

Fasting lipid panel. (PC, MK)<br />

Fasting blood glucose. (PC, MK)<br />

Urinalysis. (PC, MK)<br />

65


Stool test for occult blood. (PC, MK)<br />

Prostate specific antigen. (PC, MK)<br />

Students should be able to define the indications for and interpret (with consultation) results<br />

<strong>of</strong>:<br />

Mammography. (PC, MK)<br />

Colonoscopy. (PC, MK)<br />

Pap smear. (PC, MK)<br />

Bone densitometry. (PC, MK)<br />

5. Communication skills: Students should be able to:<br />

Communicate results <strong>of</strong> the evaluation and counsel for disease prevention. (PC, CS)<br />

Elicit questions from the patient and his or her family about the plan. (PC, CS)<br />

6. Basic and advanced procedural skills: Students should be able to:<br />

Perform a urinalysis (dipstick and microscopic). (PC)<br />

Stool occult blood testing. (PC)<br />

Calculate a BMI. (PC)<br />

Perform a functional status examination for elderly patients. (PC)<br />

Administer intramuscular injection <strong>of</strong> a vaccine. (PC)<br />

Participate in obtaining a Pap smear. (PC)<br />

7. Management skills: Students should be able to develop an appropriate evaluation and<br />

treatment plan for healthy patients, including:<br />

Designing an appropriate work-up for any abnormalities noted on the screening exam. (PC,<br />

MK)<br />

Teaching breast self-examinations. (PC, CS)<br />

Counseling for:<br />

o Safe sexual practices. (PC, CS)<br />

o Seatbelt use. (PC, CS)<br />

o Healthy diet. (PC, CS)<br />

o Weight loss. (PC, CS)<br />

o Practical exercise program appropriate to the patient's age, and current physical<br />

condition. (PC, CS)<br />

o Stress management. (PC, CS)<br />

o Alcohol abstinence. (PC, CS)<br />

o Smoking cessation. (PC, CS)<br />

o Cancer screening. (PC, CS)<br />

o Limiting risks <strong>of</strong> occupational and environmental hazards. (PC, CS)<br />

Accessing and utilizing appropriate information systems and resources to help delineate<br />

issues related to healthy patients. (PC, PLI)<br />

Using a cost-effective approach based for screening. (PC, SBP)<br />

Incorporating patient preferences. (PC, P)<br />

Engaging the patient as an active participant in his/her health care. (PC,P)<br />

C. ATTITUDES AND PROFESSIONAL BEHAVIORS: Students should be able to:<br />

1. Recognize the importance <strong>of</strong> regularly screening all patients followed and <strong>of</strong> teaching all<br />

patients about preventive measures. (PC, P)<br />

2. Appreciate the necessity <strong>of</strong> keeping detailed records <strong>of</strong> screening and health maintenance<br />

measures. (PC, P)<br />

3. Understand that physicians and health care delivery organizations are frequently judged by<br />

their ability to deliver the highest quality screening and preventive measures. (PLI, P, SBP)<br />

66


4. Recognize the importance <strong>of</strong> addressing community sources <strong>of</strong> health risk. (PC, P)<br />

5. Respond appropriately to patients who are nonadherent preventive measures. (CS, P)<br />

6. Respect the patient’s right to refuse preventive measures and screening. (P)<br />

7. Demonstrate commitment to using risk-benefit, cost-benefit, and evidence-based<br />

considerations in the selection <strong>of</strong> screening tests. (PLI, P)<br />

8. Demonstrate ongoing commitment to self-directed learning regarding prevention and<br />

screening. (PLI, P)<br />

9. Recognize the importance and demonstrate a commitment to the utilization <strong>of</strong> other healthcare<br />

pr<strong>of</strong>essions in preventative medicine. (P, SBP)<br />

D. RESOURCES:<br />

484.<br />

USPSTF Recommendation: Screening for Breast Cancer<br />

www.ahcpr.gov/clinic/3rduspstf/breastcancer/brcanrr.htm<br />

USPSTF Recommendation: Screening for Cervical Cancer<br />

www.ahrq.gov/clinic/3rduspstf/cervcan/cervcanrr.htm<br />

USPSTF Recommendations Statement: Counseling to prevent tobacco use and tobaccocaused<br />

disease<br />

www.ahrq.gov/clinic/3rduspstf/tobacccoun/tobcounrs.htm<br />

Screening for Prostate Cancer. American College <strong>of</strong> Physicians. Ann Int Med 1997; 126: 480-<br />

Summary <strong>of</strong> Recommendations for Adult Immunization. Immunization Action Coalition Bulletin.<br />

Adapted from the recommendations <strong>of</strong> the Advisory Committee on Immunization Practices<br />

(ACIP), August 2005<br />

www.immunize.org/acip<br />

67


TRAINING PROBLEM #2: ABDOMINAL PAIN<br />

RATIONALE:<br />

Abdominal pain is a common symptom that can be attributed to a wide variety <strong>of</strong> acute and<br />

chronic disease processes, many <strong>of</strong> which may represent serious medical problems. Mastery <strong>of</strong><br />

the approach to patients with abdominal pain is important to third year medical students.<br />

PREREQUISITES:<br />

Prior knowledge, skills, and attitudes acquired during the pre-<strong>clerkship</strong> experience should include:<br />

Ability to perform a complete medical history and physical exam.<br />

Ability to communicate with patients <strong>of</strong> diverse backgrounds.<br />

Knowledge <strong>of</strong> gastrointestinal and gynecologic anatomy, physiology, and pathophysiology.<br />

SPECIFIC LEARNING OBJECTIVES:<br />

A. KNOWLEDGE: Students should be able to define, describe, and discuss:<br />

1. Three principal types <strong>of</strong> abdominal pain:<br />

Visceral pain: (MK)<br />

o Poorly localized but site roughly corresponds to dermatome that innervates the affected<br />

organ.<br />

o Characteristics may vary (dull, cramping, burning).<br />

o Frequently accompanied by secondary autonomic effects (nausea, vomiting, pallor,<br />

diaphoresis, restlessness).<br />

o Patient moves around in an attempt to alleviate discomfort.<br />

Somatoparietal or peritoneal pain: (MK)<br />

o More localized and more intense than visceral pain.<br />

o Arises from peritoneal irritation.<br />

o Aggravated by movement (patient attempts to lie still).<br />

Referred pain: (MK)<br />

o Usually well localized but felt in areas remote to affected organ.<br />

o May be felt in skin or in deeper tissues.<br />

o Results from convergence <strong>of</strong> visceral afferent neurons with somatic neurons from<br />

different anatomic regions.<br />

2. Relative likelihood <strong>of</strong> the common causes <strong>of</strong> abdominal pain based on the pain pattern and the<br />

quadrant in which the pain is located. (MK)<br />

3. Diagnostic discrimination between common causes <strong>of</strong> abdominal pain based on history,<br />

physical exam, laboratory testing, and imaging procedures. (MK)<br />

4. Symptoms and signs indicative <strong>of</strong> an acute/surgical abdomen. (MK)<br />

5. The influence <strong>of</strong> age, gender, menopausal status, and immunocompetency on the prevalence<br />

<strong>of</strong> different disease processes that may result in abdominal pain. (MK)<br />

B. SKILLS: Students should be able to demonstrate specific skills, including:<br />

1. History-taking skills: Students should be able to obtain, document, and present an<br />

appropriately complete medical history that differentiates among etiologies <strong>of</strong> disease,<br />

including:<br />

• Chronology. (PC, CS)<br />

68


• Location. (PC, CS)<br />

• Radiation. (PC, CS)<br />

• Character. (PC, CS)<br />

• Intensity. (PC, CS)<br />

• Duration. (PC, CS)<br />

• Aggravating or alleviating factors. (PC, CS)<br />

• Associated symptoms. (PC, CS)<br />

• Pertinent information about previous abdominal or pelvic surgeries, chronic medical<br />

conditions, sexual activity, medications, and family history. (PC, CS)<br />

2. Physical exam skills: Students should be able to perform a focused physical exam in<br />

patients who present with abdominal pain in order to:<br />

Establish a preliminary diagnosis <strong>of</strong> the cause. (PC)<br />

Assess the severity <strong>of</strong> the patient’s presenting symptoms and signs(PC)<br />

Determine the urgency <strong>of</strong> implementing diagnostic and treatment<br />

plans. (PC)<br />

The initial physical examination <strong>of</strong> the patient should include:<br />

A general assessment <strong>of</strong> the patient’s appearance, position, and degree <strong>of</strong> discomfort.<br />

(PC)<br />

Measurement <strong>of</strong> vital signs, including temperature, pulse, blood pressure, and, when<br />

indicated, orthostatic blood pressure and pulse. (PC)<br />

Correct order and technique for examining the abdomen. (PC)<br />

Inspection <strong>of</strong> the abdomen for surgical scars, distension, asymmetry or cutaneous<br />

abnormalities (dilated veins, ecchymoses, etc.). (PC)<br />

Auscultation <strong>of</strong> the abdomen for abnormal bowel sounds, bruits. (PC)<br />

Percussion <strong>of</strong> the abdomen for detection <strong>of</strong> hepatomegaly, splenomegaly, abdominal<br />

masses, or the presence <strong>of</strong> ascites. (PC)<br />

Palpation <strong>of</strong> the abdomen for areas <strong>of</strong> tenderness, signs <strong>of</strong> peritoneal inflammation,<br />

hepatomegaly, splenomegaly, abnormal masses, pulsations, or hernias. (PC)<br />

Performance <strong>of</strong> rectal and pelvic exams (under supervision). (PC)<br />

3. Differential diagnosis: Students should be able to generate a prioritized differential<br />

diagnosis <strong>of</strong> the most important and likely causes <strong>of</strong> a patient’s abdominal pain and<br />

recognize specific history, physical exam, and laboratory findings that distinguish<br />

between the following diagnoses or conditions:<br />

Appendicitis. (PC, MK)<br />

Cholecystitis (biliary colic). (PC, MK)<br />

Pancreatitis. (PC, MK)<br />

Diverticulitis. (PC, MK)<br />

Peptic ulcer disease including perforation. (PC, MK)<br />

Gastroenteritis. (PC, MK)<br />

Hepatitis. (PC, MK)<br />

Irritable bowel syndrome. (PC, MK)<br />

Small bowel obstruction. (PC, MK)<br />

Acute mesenteric ischemia. (MK, PC)<br />

Inflammatory bowel disease. (PC, MK)<br />

Ruptured abdominal aortic aneurysm. (PC, MK)<br />

Ureteral stones (renal colic). (PC, MK)<br />

Pelvic inflammatory disease. (PC, MK)<br />

69


Ruptured ectopic pregnancy. (PC, MK)<br />

Abdominal wall pain. (PC, MK)<br />

Referred pain. (PC, MK)<br />

4. Laboratory interpretation: Students should be able to interpret specific diagnostic tests<br />

and procedures that are commonly ordered to evaluate patients who present with<br />

abdominal pain. Test interpretation should take into account:<br />

Important differential diagnostic considerations including potential diagnostic<br />

emergencies. (PC, MK)<br />

Pre-test and post-test likelihood <strong>of</strong> disease (probabilistic reasoning). (PC, MK)<br />

Performance characteristics <strong>of</strong> individual tests (sensitivity, specificity, positive and<br />

negative predictive value, likelihood ratios). (PC, MK)<br />

Laboratory and diagnostic tests should include, when appropriate:<br />

• CBC with differential. (PC, MK)<br />

• UA. (PC, MK)<br />

• Pregnancy test. (PC, MK)<br />

• Stool for occult blood. (PC, MK)<br />

• Hepatic function panel. (PC, MK)<br />

• Amylase and lipase. (PC, MK)<br />

• Abdominal obstructive series. (PC, MK)<br />

Students should be able to define the indications for, and interpret (with<br />

consultation) the results <strong>of</strong>:<br />

Abdominal ultrasound. (PC, MK)<br />

Abdominal CT scan. (PC, MK)<br />

Paracentesis fluid studies. (PC, MK)<br />

Upper gastrointestinal endoscopy. (PC, MK)<br />

Sigmoidoscopy/colonoscopy. (PC, MK)<br />

Barium contrast studies. (PC, MK)<br />

Radionuclide scan <strong>of</strong> the hepatobiliary system. (PC, MK)<br />

5. Communication skills: Students should be able to:<br />

Communicate the diagnosis, treatment plan, and subsequent follow-up to the patient<br />

and his or her family. (PC, CS)<br />

Elicit questions from the patient and his or her family about the management plan. (PC,<br />

CS)<br />

Communicate in lay terms the indications, risk/benefits, and expected outcomes<br />

essential to obtaining informed consent for diagnostic and therapeutic procedures<br />

commonly used to evaluate and treat patients who present with abdominal pain. (PC,<br />

CS)<br />

6. Basic and advanced procedural skills: Students should be able to:<br />

Insert a nasogastric tube. (PC)<br />

Perform stool occult blood testing. (PC)<br />

Assist in performing a paracentesis after explaining the procedure to the patient. (PC,<br />

CS)<br />

7. Management skills: Students should be able to develop an appropriate<br />

evaluation and treatment plan for patients that includes:<br />

Recognizing the role <strong>of</strong> narcotic analgesics and empiric antibiotics in treating selected<br />

patients who present with acute abdominal pain. (PC, MK)<br />

Determining when to consult a gastroenterologist or a surgeon. (PC, SBP)<br />

Involving a surgeon as soon as possible when a patient is identified as having an acute<br />

abdomen. (PC, SBP)<br />

70


Selecting various tests and procedures commonly used to diagnose patients who<br />

present with symptoms <strong>of</strong> abdominal pain. ( PC, MK)<br />

Recommending basic initial management plans for the various causes <strong>of</strong> abdominal<br />

pain listed in the differential diagnosis. (PC, MK)<br />

Considering the potential value <strong>of</strong> addressing psychosocial issues in the management<br />

<strong>of</strong> chronic abdominal pain. (PC, MK)<br />

Accessing and utilizing appropriate information systems and resources to help<br />

delineate issues related to abdominal pain. (PC, PLI)<br />

Using a cost-effective approach based on the differential diagnosis. (PC, SBP)<br />

Incorporating patient preferences. (PC, P)<br />

C. ATTITUDES AND PROFESSIONAL BEHAVIORS: Students should be able to:<br />

1. Demonstrate commitment to using risk-benefit, cost-benefit, and evidence-based<br />

considerations in the selection <strong>of</strong> diagnostic and therapeutic interventions for abdominal<br />

pain. (PLI, P)<br />

2. Recognize the importance <strong>of</strong> patient needs and preferences when selecting among<br />

diagnostic and therapeutic options for abdominal pain. (P)<br />

3. Recognize the importance and demonstrate a commitment to the utilization <strong>of</strong> other<br />

healthcare pr<strong>of</strong>essions in the treatment <strong>of</strong> abdominal pain. (P, SBP)<br />

D. REFERENCES:<br />

Silen W. Cope’s Early Diagnosis <strong>of</strong> the Acute Abdomen. 20 th ed. New York: Oxford<br />

<strong>University</strong> Press; 2000.<br />

Wagner JM, McKinney WP, Carpenter JL. The rational clinical exam. Does this patient<br />

have appendicitis? JAMA. 1996;276:1589-94.<br />

Lederle F, Simel D. The rational clinical exam. Does this patient have an abdominal<br />

aortic aneurysm? JAMA. 1999;281:77-82.<br />

Trowbridge RL, Rutkowski NK, Shojania KG. The rational clinical exam. Does this<br />

patient have acute cholecystitis? JAMA. 2003;289: 80-86.<br />

71


TRAINING PROBLEM #3: ALTERED MENTAL STATUS<br />

RATIONALE:<br />

The diagnosis and management <strong>of</strong> altered mental status requires a working knowledge <strong>of</strong> all areas <strong>of</strong><br />

internal medicine, so varied are the etiologies and corresponding treatment strategies. Internists must<br />

master an approach to the problem as they are <strong>of</strong>ten the first physicians to see such patients.<br />

PREREQUISITES:<br />

Prior knowledge, skills, and attitudes acquired during the pre-clinical experience should include:<br />

Ability to perform a complete medical history and physical.<br />

Ability to communicate with patients <strong>of</strong> diverse backgrounds.<br />

Basic course work in physiology, pathophysiology, and neuroanatomy.<br />

SPECIFIC LEARNING OBJECTIVES:<br />

A. KNOWLEDGE: Students should be able to define, describe, and discuss:<br />

1. The differentiation <strong>of</strong> delirium, dementia, and depression. (MK)<br />

2. The pathophysiology, symptoms, and signs <strong>of</strong> the most common and most serious causes <strong>of</strong><br />

altered mental status, including:<br />

Metabolic causes (e.g. hyper/hyponatremia, hyper/hypoglycemia, hypercalcemia,<br />

hyper/hypothyroidism, hypoxia/hypercapnea, B12 deficiency, hepatic encephalopathy,<br />

uremic encephalopathy, drug/alcohol intoxication/withdrawal, and Wernicke’s<br />

encephalopathy). (MK)<br />

Structural lesions (e.g. primary or metastatic tumor, intracranial hemorrhage, subdural<br />

hematoma). (MK)<br />

Vascular (e.g. cerebrovascular accident, transient ischemic attack, cerebral vasculitis).<br />

(MK)<br />

Infectious etiologies (e.g. encephalitis, meningitis, urosepsis, endocarditis, pneumonia,<br />

cellulites). (MK)<br />

Seizures/ post-ictal state. (MK)<br />

Hypertensive encephalopathy. (MK)<br />

Low perfusion states (e.g. arrhythmias, MI, shock, acute blood loss, severe dehydration).<br />

(MK)<br />

Miscellaneous causes (e.g. fecal impaction, postoperative state, sleep deprivation, urinary<br />

retention). (MK)<br />

3. The importance <strong>of</strong> thoroughly reviewing prescription medications over-thecounter drugs, and<br />

supplements and inquiring about substance abuse. (MK)<br />

4. The risk factors for developing altered mental status, including:<br />

Dementia. (MK)<br />

Advanced age. (MK)<br />

Substance abuse. (MK)<br />

Comorbid physical problems such as sleep deprivation, immobility, dehydration, pain, and<br />

sensory impairment. (MK)<br />

ICU admission. (MK)<br />

5. The diagnostic evaluation <strong>of</strong> altered mental status. (MK)<br />

6. Indications, contraindications, and complications <strong>of</strong> lumbar puncture. (MK)<br />

72


7. Principles <strong>of</strong> management <strong>of</strong> the common causes <strong>of</strong> altered mental status. (MK)<br />

8. Nonpharmacologic measures to reduce agitation and aggression, including:<br />

Avoiding the use <strong>of</strong> physical restraints whenever possible. (MK)<br />

Using reorientation techniques. (MK)<br />

Assuring the patient has their devices to correct sensory deficits. (MK)<br />

Promoting normal sleep and day/night awareness. (MK)<br />

Preventing dehydration and electrolyte disturbances. (MK)<br />

Avoiding medications which may worsen delirium whenever possible<br />

(e.g. anticholinergics, benzodiazepines, etc.). (MK)<br />

9. The risks <strong>of</strong> using physical restraints. (MK)<br />

10. The risk and benefits <strong>of</strong> using low-dose high potency antipsychotics for delirium associated<br />

agitation and aggression. (MK)<br />

B. SKILLS: Students should be able to demonstrate specific skills, including:<br />

1 History-taking skills: Students should be able to obtain, document, and present an ageappropriate<br />

medical history that differentiates among etiologies <strong>of</strong> altered mental status including<br />

eliciting appropriate information from patients and their families regarding the onset, progression,<br />

associated symptoms, and level <strong>of</strong> physical and mental disability. (PC, CS)<br />

2. Physical exam skills: Students should be able to perform a physical exam to establish the<br />

diagnosis and severity <strong>of</strong> disease, including:<br />

Complete neurologic examination. (PC)<br />

Mental status examination. (PC)<br />

Fundoscopic examination. (PC)<br />

3. Differential diagnosis: Students should be able to generate a prioritized differential diagnosis<br />

recognizing specific history and physical exam findings that suggest a specific etiology for<br />

altered mental status. (PC, MK)<br />

4. Laboratory interpretation: Students should be able to recommend when to order diagnostic<br />

and laboratory tests and be able to interpret them, both prior to and after initiating treatment<br />

based on the differential diagnosis, including consideration <strong>of</strong> test cost and performance<br />

characteristics as well as patient preferences. Laboratory and diagnostic tests should include,<br />

when appropriate:<br />

CBC with differential. (PC, MK)<br />

Electrolytes, BUN/Cr, GLC, hepatic function panel, Ca. (PC, MK)<br />

ABG. (PC, MK)<br />

Toxicology screen. (PC, MK)<br />

VDRL. (PC, MK)<br />

Vitamin B12 and thiamine measurements. (PC, MK)<br />

Thyroid function tests. (PC, MK)<br />

Urinalysis and urine culture. (PC, MK)<br />

Blood cultures. (PC, MK)<br />

Cerebrospinal fluid analysis (color, opening pressure, chemistries, cell<br />

counts, staining, cultures, cytology, cryptococcal antigen, VDRL). (PC, MK)<br />

Students should be able to define the indications for and interpret (with consultation)<br />

the results <strong>of</strong>:<br />

Cranial CT. (PC, MK)<br />

Cranial MRI. (PC, MK)<br />

Electroencephalogram. (PC, MK)<br />

73


5. Communication skills: Students should be able to:<br />

Communicate the diagnosis, treatment plan, and subsequent follow-up to the patient and<br />

his or her family. (PC, CS)<br />

Elicit questions from the patient and his or her family about the management plan. (PC, CS)<br />

When the patient is unable to communicate, obtain a history from a collateral source such<br />

as a family member or other health care proxy. (PC, CS)<br />

6. Basic and advanced procedural skills: Students should be able to:<br />

Obtain an ABG. (PC)<br />

Assist in performing a lumbar puncture after explaining the procedure to the patient. (PC,<br />

CS)<br />

7. Management skills: Students should able to develop an appropriate evaluation and<br />

treatment plan for patients that includes:<br />

Recognizing that altered mental status in a older inpatient is a medical<br />

emergency and requires that the patient be evaluated immediately. (PC, MK)<br />

Writing appropriate fluid and replacement orders for patients with common electrolyte and<br />

metabolic disturbances. (PC, MK)<br />

Writing appropriate antibiotic orders for the treatment <strong>of</strong> common infectious etiologies. (PC,<br />

MK)<br />

Ordering appropriate nonpharmacologic and pharmacologic interventions for patients with<br />

acute altered mental status with accompanying agitation and aggression. (PC, MK)<br />

Determining when to obtain consultation from a neurologist or neurosurgeon. (PC, SBP)<br />

Utilizing hospital and community resources for patients with permanent or disabling<br />

conditions to help assist their transfer back to the community or rehabilitation facility. (PC,<br />

SBP)<br />

Using a cost-effective approach based on the differential diagnosis. (PC, SBP)<br />

Accessing and utilizing appropriate information systems and resources to help delineate<br />

issues related to altered mental status. (PC, PLI)<br />

Incorporating patient preferences. (PC, P)<br />

C. ATTITUDES AND PROFESSIONAL BEHAVIORS: Students should be able to:<br />

1. Appreciate the family’s concern and at times despair arising from a loved one’s<br />

development <strong>of</strong> altered mental status. (CS, P)<br />

2. Appreciate the patient’s distress and emotional response to that may accompany<br />

circumstances <strong>of</strong> altered mental status. (CS,P)<br />

3. Demonstrate commitment to using risk-benefit, cost-benefit, and evidence-based<br />

considerations in the selection diagnostic and therapeutic interventions for altered mental<br />

status. (PLI, P)<br />

4. Recognize the importance <strong>of</strong> patient preferences when selecting among diagnostic<br />

and therapeutic options for altered mental status. (P)<br />

5. Demonstrate ongoing commitment to self-directed learning regarding altered mental<br />

status. (PLI, P)<br />

6. Appreciate the impact altered mental status has on a patient’s quality <strong>of</strong> life, wellbeing,<br />

ability to work, and the family. (P)<br />

7. Recognize the importance <strong>of</strong> and demonstrate a commitment to the utilization <strong>of</strong><br />

other healthcare pr<strong>of</strong>essionals in the diagnosis and treatment <strong>of</strong> altered mental status. (P,<br />

SBP)<br />

D. REFERENCES:<br />

74


Ropper AH. (2005). Acute confusional states and coma. In Kasper DL, Braunwald EB,<br />

Fauci AS, Hauser SL, Longo DL, Jameson JL, eds. Harrison’s Principles <strong>of</strong> Internal <strong>Medicine</strong>.<br />

16 th ed. New York, NY: McGraw-Hill; 2005:1624-31.<br />

Gleason OC. Delirium. Am Fam Physician. 2003;67:1027-34.<br />

Brown TM, Boyle MF. Delirium. BMJ. 2002;325:644-7.<br />

Meagher DJ. Delirium: optimizing management. BMJ. 2001;322:144-9.<br />

75


TRAINING PROBLEM #4: ANEMIA<br />

RATIONALE:<br />

Anemia is a common finding, <strong>of</strong>ten identified incidentally in asymptomatic patients. It can be a<br />

manifestation <strong>of</strong> a serious underlying disease. Distinguishing among the many disorders that cause<br />

anemia, not all <strong>of</strong> which require treatment, is an important training problem for third year medical<br />

students.<br />

PREREQUISITES:<br />

Prior knowledge, skills and attitudes acquired during the pre-<strong>clerkship</strong> experience should<br />

include:<br />

Ability to perform a complete medical history and physical exam.<br />

Ability to communicate with patients <strong>of</strong> diverse backgrounds.<br />

Knowledge <strong>of</strong> pathogenesis and pathophysiology <strong>of</strong> anemia.<br />

Knowledge <strong>of</strong> the basic biochemistry and pathophysiology <strong>of</strong> the blood and bone marrow.<br />

Knowledge <strong>of</strong> the pharmacology <strong>of</strong> medications that can cause anemia as well as those used<br />

to treat it.<br />

SPECIFIC LEARNING OBJECTIVES:<br />

A. KNOWLEDGE: Students should be able to define, describe, and discuss:<br />

1. Classification <strong>of</strong> anemia based on red cell size:<br />

Microcytic:<br />

o Iron deficiency. (MK)<br />

o Thalassemic disorders. (MK)<br />

o Sideroblastic anemia. (MK)<br />

Normocytic:<br />

o Acute blood loss. (MK)<br />

o Hemolysis. (MK)<br />

o Anemia <strong>of</strong> chronic disease (e.g. infection, inflammation, malignancy). (MK)<br />

o Chronic renal insufficiency/erythropoietin deficiency. (MK)<br />

o Bone marrow suppression (e.g. bone marrow invasion, aplastic anemia).<br />

o Hypothyroidism. (MK)<br />

o Testosterone deficiency. (MK)<br />

o Early presentation <strong>of</strong> microcytic or macrocytic anemia (e.g. early iron deficiency<br />

anemia). (MK)<br />

o Combined presentation <strong>of</strong> microcytic and macrocytic anemias. (MK)<br />

Macrocytic:<br />

o Ethanol abuse. (MK)<br />

o B12 deficiency. (MK)<br />

o Folate deficiency. (MK)<br />

o Drug-induced. (MK)<br />

o Reticulcytosis. (MK)<br />

o Liver disease. (MK)<br />

o Myelodysplastic syndromes. (MK)<br />

o Hypothyroidism. (MK)<br />

10. Morphological characteristics, pathophysiology, and relative prevalence <strong>of</strong> each <strong>of</strong> the causes<br />

<strong>of</strong> anemia. (MK)<br />

76


11. The meaning and utility <strong>of</strong> various components <strong>of</strong> the hemogram (e.g. hemoglobin, hematocrit,<br />

mean corpuscular volume, and random distribution width). (MK)<br />

12. The classification <strong>of</strong> anemia into hypoproliferative and hyperproliferative categories and the<br />

utility <strong>of</strong> the reticulocyte count/index. (MK)<br />

13. The potential usefulness <strong>of</strong> the white blood cell count and red blood cell count when attempting<br />

to determine the cause <strong>of</strong> anemia. (MK)<br />

14. The diagnostic utility <strong>of</strong> the various tests for iron deficiency (e.g. serum iron, total iron binding<br />

capacity, transferrin saturation, ferritin). (MK)<br />

15. The genetic basis <strong>of</strong> some forms <strong>of</strong> anemia. (MK)<br />

16. Indications, contraindications, and complications <strong>of</strong> blood transfusion. (MK)<br />

B. SKILLS: Students should be able to demonstrate specific skills, including:<br />

1. History-taking skills: Students should be able to obtain, document, and present an ageappropriate<br />

medical history, that differentiates among etiologies <strong>of</strong> disease, including:<br />

Constitutional and systemic symptoms (e.g. fatigue, weight loss). (PC, CS)<br />

History <strong>of</strong> gastrointestinal bleeding or risk factors for it. (PC, CS)<br />

Abdominal pain. (PC, CS)<br />

Prior history <strong>of</strong> anemia or other blood diseases. (PC, CS)<br />

Medications. (PC, CS)<br />

Diet. (PC, CS)<br />

Alcohol use. (PC, CS)<br />

Menstrual history. (PC, CS)<br />

Family history <strong>of</strong> anemia or other blood diseases. (PC, CS)<br />

2. Physical exam skills: Students should be able to perform a physical exam to establish the<br />

diagnosis and severity <strong>of</strong> disease, including:<br />

Pallor (e.g. palms, conjunctiva, nail beds). (PC)<br />

Mouth (e.g. glossitis, cheilosis). (PC)<br />

Hyperdynamic precordium, systolic flow murmur. (PC)<br />

Lymph nodes. (PC)<br />

spleen. (PC)<br />

Obtain stool for occult blood testing. (PC)<br />

Nervous system. (PC)<br />

3. Differential diagnosis: Students should be able to generate a list <strong>of</strong> the most important and<br />

most common causes <strong>of</strong> anemia, recognizing specific history, physical exam, and laboratory<br />

findings that suggest a specific etiology. (PC, MK)<br />

4. Laboratory interpretation: Students should be able to recommend when to order diagnostic<br />

and laboratory tests and be able to interpret them, both prior to and after initiating treatment<br />

based on the differential diagnosis including consideration <strong>of</strong> test cost and performance<br />

characteristics as well as patient preferences. Laboratory and diagnostic tests should include,<br />

when appropriate:<br />

Hemoglobin and hematocrit. (PC, MK)<br />

Red cell indices (e.g. mean corpuscular volume and random distribution width). (PC, MK)<br />

White blood cell and platelet count. (PC, MK)<br />

Reticulocyte count. (PC, MK)<br />

Iron studies (serum iron, TIBC, ferritin, transferrin). (PC, MK)<br />

Serum B12 and folate. (PC, MK)<br />

Haptoglobin. (PC, MK)<br />

77


Lactic dehydrogenase. (LDH) (PC, MK)<br />

Hemoglobin electrophoresis. (PC, MK)<br />

Blood smear. (PC, MK)<br />

Students should be able to define the indications for and interpret (with consultation) results<br />

<strong>of</strong>:<br />

Bone marrow biopsy. (PC, MK)<br />

5. Communication skills: Students should be able to:<br />

Communicate the diagnosis, treatment plan, and subsequent follow-up to patients. (PC,<br />

CS)<br />

Elicit questions from the patient about the management plan. (PC, CS)<br />

Counsel with regard to (a) possible causes, (b) appropriate further evaluation to establish<br />

the diagnosis <strong>of</strong> an underlying disease, and (c) the impact on the family (genetic<br />

counseling). (PC, CS)<br />

6. Basic procedural skills: Students should be able to perform and interpret:<br />

Stool occult blood testing. (PC)<br />

7. Management skills: Students should be able to develop an appropriate evaluation and<br />

treatment plan for patients that includes:<br />

Evaluating for underlying disease processes, given that anemia is not a disease per se, but<br />

rather a common finding that requires further delineation in order to identify the underlying<br />

cause. (PC, MK)<br />

Prescribing indicated replacement therapy, including iron, vitamin B12, and folic acid. (PC,<br />

MK)<br />

Determining when to obtain consultation from a hematologist. (PC, SBP)<br />

Using a cost-effective approach based on the differential diagnosis. (PC, SBP)<br />

Accessing and utilizing appropriate information systems and resources to help delineate<br />

issues related to anemia. (PC, PLI)<br />

Incorporating patient preferences. (PC, P)<br />

C. ATTITUDES AND PROFESSIONAL BEHAVIORS: Students should be able to:<br />

1 Demonstrate commitment to using risk-benefit, cost-benefit, and evidence-based<br />

considerations in the selection <strong>of</strong> diagnostic and therapeutic interventions for anemia. (PLI, P)<br />

2 Respond appropriately to patients who are non-adherent to treatment for anemia. (CS, P)<br />

3 Demonstrate ongoing commitment to self-directed learning regarding anemia. (PLI, P)<br />

Appreciate the impact anemia has on a patient’s quality <strong>of</strong> life, well-being, ability to work, and<br />

the family. (P)<br />

4 Recognize the importance <strong>of</strong> and demonstrate a commitment to the utilization <strong>of</strong> other<br />

healthcare pr<strong>of</strong>essions in the treatment <strong>of</strong> anemia. (P, SBP)<br />

D. REFERENCES:<br />

Sheth TN. Choudhry NK. Bowes M. Detsky AS. The relation <strong>of</strong> conjunctival pallor to the<br />

presence <strong>of</strong> anemia. J Gen Intern Med. 1997;12:102-6.<br />

Guyatt, G H. Oxman, A D. Ali, M. Willan, A. McIlroy, W. Patterson, C. Laboratory<br />

diagnosis <strong>of</strong> iron-deficiency anemia: an overview. J Gen Intern Med. 1992;7:145-53.<br />

Kis AM. Carnes M. Detecting iron deficiency in anemic patients with concomitant<br />

medical problems. J Gen Intern Med. 1998; 13:455-61.<br />

Bain BJ. Diagnosis from the blood smear. N Engl Journal Med. 2005;353:498-507.<br />

Weiss G. Goodnough LT. Anemia <strong>of</strong> chronic disease. N Engl J Med. 2005;352:1011-23.<br />

78


H<strong>of</strong>fbrand V. Provan D. ABC <strong>of</strong> clinical haematology: macrocytic anaemias. BMJ.<br />

1997;314:430-3.<br />

79


TRAINING PROBLEM #5: BACK PAIN<br />

RATIONALE:<br />

Back pain is one <strong>of</strong> the most commonly encountered problems in the outpatient, primary care internal<br />

medicine setting. It has an important differential diagnosis, and the initial decision-making must be<br />

made on the basis <strong>of</strong> clinical findings. As such, it is an excellent training condition for teaching<br />

decision-making based on careful collection and interpretation <strong>of</strong> basic clinical data. There is<br />

emerging data on test utility, especially as regards expensive spinal imaging, which facilitates<br />

teaching rational, cost-effective test ordering. Moreover, its requirement for skillful management,<br />

patient education, and support facilitate the teaching <strong>of</strong> these competencies.<br />

PREREQUISITES:<br />

Prior knowledge, skills, and attitudes acquired during the pre-clinical experience should include:<br />

Ability to perform a complete medical history and physical.<br />

Ability to communicate with patients <strong>of</strong> diverse backgrounds.<br />

Anatomy and physiology <strong>of</strong> bony, s<strong>of</strong>t tissue, vascular, and <strong>of</strong> the spine.<br />

Pathogenesis and pathophysiology <strong>of</strong> muscular strain, osteoarthritis, spinal stenosis,<br />

osteoporosis, disc degeneration, and spinal metastases.<br />

Pharmacology <strong>of</strong> non-narcotic and narcotic analgesics, nonsteroidal anti-inflammatory drugs,<br />

muscle “relaxants.”<br />

SPECIFIC LEARNING OBJECTIVES:<br />

A. KNOWLEDGE: Students should be able to define, describe, and discuss:<br />

1. The symptoms, signs, and typical clinical course <strong>of</strong> the various causes <strong>of</strong> back pain including:<br />

Ligamentous/muscle strain (nonspecific musculoskeletal back pain). (MK)<br />

Degenerative arthritis (spondylosis). (MK)<br />

Disc herniation. (MK)<br />

Spinal stenosis. (MK)<br />

Vertebral compression fracture. (MK)<br />

Traumatic fracture. (MK)<br />

Sacroileitis. (MK)<br />

Spinal metastases. (MK)<br />

Spinal epidural abscess. (MK)<br />

Cauda equina syndrome. (MK)<br />

2. The role <strong>of</strong> diagnostic studies in the evaluation <strong>of</strong> the back pain there indications,<br />

limitations, cost:<br />

Plain radiography. (MK)<br />

CT. (MK)<br />

MRI. (MK)<br />

Myelogram. (MK)<br />

Electrodiagnosis (i.e. electromyography and nerve conduction studies). (MK)<br />

Bone densitometry. (MK)<br />

3. Response to therapy <strong>of</strong> the various etiologies, with understanding <strong>of</strong> the roles <strong>of</strong>:<br />

80


Bed rest. (MK)<br />

Exercise. (MK)<br />

Analgesia. (MK)<br />

NSAIDs. (MK)<br />

Heat/ice. (MK)<br />

Ultrasound. (MK)<br />

Spinal manipulation. (MK)<br />

Surgical interventions. (MK)<br />

4. Risk factor for and means <strong>of</strong> limiting disability and chronicity. (MK)<br />

5. Fear avoidance behaviors. (MK)<br />

6. Pain related behaviors with regard to chronic narcotic use. (MK)<br />

B. SKILLS: Students should be able to demonstrate specific skills including:<br />

1. History-taking skills: Students should be able to obtain, document, and present an<br />

age-appropriate medical history, that differentiates among etiologies <strong>of</strong> disease,<br />

including:<br />

Cancer history. (PC, CS)<br />

Weight loss. (PC, CS)<br />

Fever. (PC, CS)<br />

Recent infection. (PC, CS)<br />

Intravenous drug use. (PC, CS)<br />

Steroid use. (PC, CS)<br />

Trauma. (PC, CS)<br />

Rapidly progressive focal numbness and/or weakness. (PC, CS)<br />

Bowel/bladder dysfunction. (PC, CS)<br />

Saddle anesthesia. (PC, CS)<br />

Symptoms <strong>of</strong> systemic rheumatologic conditions. (PC, CS)<br />

Anatomic abnormalities (e.g. kyphosis, scoliosis). (PC, CS)<br />

2. Physical exam skills: Students should be able to perform a physical exam to establish the<br />

diagnosis and severity <strong>of</strong> disease, including:<br />

Examination <strong>of</strong> the spine. (PC)<br />

Neurologic examination <strong>of</strong> the lower extremities. (PC)<br />

Straight leg raising test. (PC)<br />

Testing for saddle anesthesia. (PC)<br />

Assessment <strong>of</strong> rectal tone. (PC)<br />

3. Differential diagnosis: Students should be able to generate a prioritized differential diagnosis<br />

recognizing specific history and physical exam findings that suggest a specific etiology for back<br />

pain (PC, MK)<br />

4. Laboratory interpretation: Students should be able to recommend when to order diagnostic<br />

and laboratory tests and be able to interpret them, both prior to and after initiating treatment<br />

based on the differential diagnosis, including consideration <strong>of</strong> test cost and performance<br />

characteristics as well as patient preferences.<br />

Laboratory and diagnostic tests should include, when appropriate:<br />

ESR. (PC, MK)<br />

CBC. (PC, MK)<br />

Serum Alk Phos. (PC, MK)<br />

Students should be able to define the indications for and interpret (with consultation) the<br />

81


esults <strong>of</strong>:<br />

Plain spinal radiography. (PC, MK)<br />

Spinal CT. (PC, MK)<br />

Spinal MRI. (PC, MK)<br />

Radionuclide bone scan. (PC, MK)<br />

Bone densitometry. (PC, MK)<br />

Electrodiagnostic tests. (PC, MK)<br />

5. Communication skills: Students should be able to:<br />

Communicate the diagnosis, treatment plan, and subsequent follow-up to patients. (PC,<br />

CS)<br />

Explain the importance <strong>of</strong> active participation in the treatment plan. (PC, CS)<br />

Elicit questions from the patient and their family about the management plan.<br />

(PC, CS)<br />

6. Management skills: Students should able to develop an appropriate evaluation and<br />

treatment plan for patients that includes:<br />

Patient education about the typical course <strong>of</strong> back pain. (PC, MK)<br />

Methods to prevent the development <strong>of</strong> chronic back pain. (PC, MK)<br />

Proper use <strong>of</strong> analgesics, NSAIDs, muscle relaxants, and local heat/ice. (PC, MK)<br />

Teaching back hygiene measures, exercises, and proper lifting and standing ergonomics.<br />

(PC, MK)<br />

Counseling patients about lifestyle modifications including weight loss. (PC, MK)<br />

The potential role <strong>of</strong> chiropractic, acupuncture, and massage (PC, MK)<br />

Determining when to obtain consultation from an appropriate back pain specialist. (PC,<br />

SBP)<br />

Using a cost-effective approach based on the differential diagnosis. (PC, SBP)<br />

Accessing and utilizing appropriate information systems and resources to help delineate<br />

issues related to back pain. (PC, PLI)<br />

Incorporating patient preferences. (PC, P)<br />

C. ATTITUDES AND PROFESSIONAL BEHAVIORS: Students should be able to:<br />

1 Demonstrate commitment to using risk-benefit, cost-benefit, and evidence-based<br />

considerations in the selection diagnostic and therapeutic interventions for back pain. (PLI, P)<br />

2 Recognize the importance <strong>of</strong> patient preferences when selecting among diagnostic and<br />

therapeutic options for back pain. (P)<br />

3 Appreciate the importance <strong>of</strong> active patient involvement in the treatment <strong>of</strong> back pain. (P)<br />

4 respond appropriately to patients who are nonadherent to treatment for back pain. (CS, P)<br />

5 respond appropriately to patients with chronic back pain (P)<br />

6 Demonstrate ongoing commitment to self-directed learning regarding back pain. (PLI, P)<br />

7 Appreciate the impact back pain has on a patient’s quality <strong>of</strong> life, well-being, ability to work,<br />

and the family. (P)<br />

8 Recognize the importance <strong>of</strong> and demonstrate a commitment to the utilization <strong>of</strong> other<br />

healthcare pr<strong>of</strong>essionals in the treatment <strong>of</strong> back pain. (P, SBP)<br />

D. REFERENCES:<br />

Carragee EJ. Persistent low back pain. N Engl J Med. 2005;352:1891-8.<br />

Deyo RA, Weinstein JN. Low back pain. N Engl J Med. 2001;344:363-70.<br />

Atlas SJ, Deyo RA. Evaluating and managing acute low back pain in the primary care<br />

82


setting. J Gen Intern Med. 2001;16:120-31.<br />

Deyo RA. Diagnostic evaluation <strong>of</strong> LBP: reaching a specific diagnosis is <strong>of</strong>ten<br />

impossible. Arch Intern Med. 2002;162:1444-7; discussion 1447-8.<br />

83


TRAINING PROBLEM #6: CHEST PAIN<br />

RATIONALE:<br />

Chest pain is a common and important presenting symptom for a variety <strong>of</strong> disorders, some <strong>of</strong> which<br />

may be life-threatening emergencies. The ability to distinguish chest pain caused by an acute<br />

coronary syndrome (unstable angina or acute myocardial infarction) from other cardiac,<br />

gastrointestinal, pulmonary, musculoskeletal or psychogenic etiologies is an important training<br />

problem for third-year medical students.<br />

PREREQUISITES:<br />

Prior knowledge, skills and attitudes acquired during the pre-<strong>clerkship</strong> experience should include:<br />

Ability to perform a complete medical history and physical exam.<br />

Ability to communicate appropriately with patients <strong>of</strong> diverse backgrounds, including the elderly<br />

patient.<br />

Knowledge <strong>of</strong> the anatomy <strong>of</strong> the heart, chest and abdomen.<br />

Understanding <strong>of</strong> the epidemiology <strong>of</strong> heart disease.<br />

Knowledge <strong>of</strong> the pathogenesis and pathophysiology <strong>of</strong> cardiovascular disease.<br />

Knowledge <strong>of</strong> the pharmacology <strong>of</strong> cardiovascular drugs.<br />

Ability to perform a cardiovascular risk assessment and understand issues related to primary<br />

and secondary prevention <strong>of</strong> cardiovascular disease.<br />

Ability to understand the impact <strong>of</strong> illness on individuals and their families.<br />

SPECIFIC LEARNING OBJECTIVES:<br />

A. KNOWLEDGE: Students should be able to define, describe and discuss:<br />

1 Symptoms and signs <strong>of</strong> chest pain that may be due to an acute coronary syndrome such as<br />

unstable angina or acute myocardial infarction. (MK)<br />

2 Symptoms and signs <strong>of</strong> chest pain that are characteristic <strong>of</strong> angina pectoris. (MK)<br />

3 Symptoms and signs <strong>of</strong> chest pain due to other cardiac causes such as:<br />

Atypical or variant angina (coronary vasospasm, Prinzmetal angina). (MK)<br />

Cocaine-induced chest pain. (MK)<br />

Pericarditis. (MK)<br />

Aortic dissection. (MK)<br />

Valvular heart disease (aortic stenosis, mitral valve prolapse). (MK)<br />

Non-ischemic cardiomyopathy. (MK)<br />

Syndrome X. (MK)<br />

4. Symptoms and signs <strong>of</strong> chest pain due to gastrointestinal disorders such as:<br />

Esophageal disease (GERD, esophagitis, esophageal dysmotility). (MK)<br />

Biliary disease (cholecystitis, cholangitis). (MK)<br />

Peptic ulcer disease. (MK)<br />

Pancreatitis. (MK)<br />

5. Symptoms and signs <strong>of</strong> chest pain due to pulmonary disorders such as:<br />

Pneumonia. (MK)<br />

Spontaneous pneumothorax. (MK)<br />

Pleurisy. (MK)<br />

Pulmonary embolism. (MK)<br />

84


Pulmonary hypertension/cor pulmonale. (MK)<br />

6. Symptoms and signs <strong>of</strong> chest pain due to musculoskeletal causes such as:<br />

Costochondritis. (MK)<br />

Rib fracture. (MK)<br />

My<strong>of</strong>ascial pain syndromes. (MK)<br />

Muscular strain. (MK)<br />

Herpes zoster. (MK)<br />

7. Symptoms and signs <strong>of</strong> chest pain due to psychogenic causes such as:<br />

Panic disorders. (MK)<br />

Hyperventilation. (MK)<br />

Somat<strong>of</strong>orm disorders. (MK)<br />

8. Factors that may be responsible for provoking or exacerbating symptoms <strong>of</strong> ischemic chest<br />

pain by:<br />

Increasing myocardial oxygen demand.<br />

o Tachycardia or tachyarrhythmia. (MK)<br />

o Hypertension. (MK)<br />

o Increased wall stress (aortic stenosis, cardiomyopathy). (MK)<br />

o Hyperthyroidism. (MK)<br />

Decreasing myocardial oxygen supply.<br />

o Anemia. (MK)<br />

o Hypoxemia. (MK)<br />

9. Risk factors for the development <strong>of</strong> coronary heart disease:<br />

Age and gender. (MK)<br />

Family history <strong>of</strong> sudden death or premature CAD. (MK)<br />

Personal history <strong>of</strong> peripheral vascular or cerebrovascular disease. (MK)<br />

Smoking. (MK)<br />

Lipid abnormalities (includes dietary history <strong>of</strong> saturated fat and cholesterol). (MK)<br />

Diabetes mellitus. (MK)<br />

Hypertension. (MK)<br />

Obesity. (MK)<br />

Sedentary lifestyle. (MK)<br />

Cocaine use. (MK)<br />

Estrogen use. (MK)<br />

Chronic inflammation. (MK)<br />

10. Physiologic basis and/or scientific evidence supporting each type <strong>of</strong> treatment, intervention or<br />

procedure commonly used in the management <strong>of</strong> patients who present with chest pain. (MK)<br />

11. Role <strong>of</strong> a critical pathway or practice guideline in delivering high quality, cost-effective care for<br />

patients presenting with symptoms <strong>of</strong> chest pain in the outpatient clinic, emergency room or<br />

hospital. (MK, PC, SBP)<br />

B. SKILLS: Students should be able to demonstrate specific skills, including:<br />

1. History-taking skills: Students should be able to obtain, document, and present an<br />

appropriately complete medical history that differentiates among the common etiologies <strong>of</strong><br />

chest pain.<br />

85


The initial medical history should allow students to categorize the patients’ symptoms as<br />

angina pectoris, atypical angina or non-cardiac chest pain. (PC, CS)<br />

Specifically, the medical history <strong>of</strong> a patient with chest pain should contain information<br />

about those clinical characteristics that are typical <strong>of</strong> angina pectoris:<br />

o Substernal location. (PC, CS)<br />

o Precipitated by exertion. (PC, CS)<br />

o Relieved by rest or nitroglycerin. (PC, CS)<br />

o Onset, duration, severity, radiation, presence or absence <strong>of</strong> associated symptoms (such<br />

as dyspnea, diaphoresis or lightheadedness). (PC, CS)<br />

The history <strong>of</strong> a patient with chest pain should also contain information about:<br />

o Risk factors for coronary heart disease. (PC, CS)<br />

o Previous history <strong>of</strong> ischemic heart disease or valvular heart disease (rheumatic fever,<br />

cardiac murmurs). (PC, CS)<br />

o Previous history <strong>of</strong> peripheral vascular disease or cerebrovascular disease. (PC, CS)<br />

Students should be able to use the medical history to assess the<br />

functional status <strong>of</strong> patients who present with ischemic chest pain. (PC, CS)<br />

2. Physical exam skills: Students should be able to perform a focused physical exam that<br />

includes the following elements:<br />

Accurate measurement <strong>of</strong> arterial blood pressure and recognition <strong>of</strong> the typical blood<br />

pressure findings that occur in patients with aortic stenosis, aortic insufficiency, and pulsus<br />

paradoxus. (PC)<br />

Assessment <strong>of</strong> major arterial pulses for abnormalities, including bruits. (PC)<br />

Assessment <strong>of</strong> the neck veins for jugular venous distention and, when necessary,<br />

evaluation for abdominal jugular reflux. (PC)<br />

Assessment <strong>of</strong> the conjunctiva and optic fundus. (PC)<br />

Assessment <strong>of</strong> the extremities to ascertain skin condition, including color, temperature and<br />

the presence <strong>of</strong> edema, xanthomas, cyanosis and clubbing. (PC)<br />

Assessment <strong>of</strong> the lungs for crackles, rhonchi, rubs and decreased breath sounds. (PC)<br />

Inspection and palpation <strong>of</strong> the anterior chest to identify right and left sided heaves, lifts,<br />

and thrills. (PC)<br />

Auscultation <strong>of</strong> the heart to determine rhythm, intensity <strong>of</strong> heart sounds, splitting <strong>of</strong> S2 and<br />

the presence <strong>of</strong> rubs, gallops (S3, S4, summation) or extra heart sounds (e.g. clicks). (PC)<br />

Auscultation <strong>of</strong> the heart to detect the presence <strong>of</strong> heart murmurs. When a heart murmur is<br />

present, students should be able to:<br />

o Identify timing (systolic vs. diastolic, holosystolic vs. ejection). (PC)<br />

o Describe pitch, location and pattern <strong>of</strong> radiation. (PC)<br />

o Gauge significance (innocent vs. pathologic, sclerosis vs. stenosis). (PC)<br />

Assessment <strong>of</strong> the abdomen to determine the presence <strong>of</strong> epigastric or right<br />

upper quadrant tenderness, hepatomegaly, abnormal pulsations or bruits. (PC)<br />

3. Differential diagnosis: Students should be able to generate a prioritized differential diagnosis<br />

and recognize specific history, physical exam, and laboratory findings that suggest a diagnosis<br />

<strong>of</strong> myocardial ischemia rather than a non-ischemic cause <strong>of</strong> chest pain (GI, pulmonary,<br />

musculoskeletal, psychogenic or undetermined). (PC, MK)<br />

4. Laboratory interpretation: Students should be able to recommend when to order diagnostic<br />

and laboratory tests and be able to interpret them, both prior to and after initiating treatment<br />

based on the differential diagnosis, including consideration <strong>of</strong> test cost and performance<br />

characteristics as well as patient preferences. Test interpretation should take into account:<br />

Important differential diagnostic considerations including the “must not miss” diagnoses.<br />

(PC, MK)<br />

86


Pre-test and post-test likelihood <strong>of</strong> disease (probabilistic reasoning). (PC, MK)<br />

Performance characteristics <strong>of</strong> individual tests (sensitivity, specificity,<br />

positive and negative predictive value, likelihood ratios). (PC, MK)<br />

Laboratory and diagnostic tests should include, when appropriate:<br />

Cardiac biomarkers indicative <strong>of</strong> myocardial necrosis. (PC, MK)<br />

12-lead ECG. (PC, MK)<br />

Chest radiograph. (PC, MK)<br />

ABG. (PC, MK)<br />

Students should be able to define the indications for, and interpret (with consultation) the<br />

results <strong>of</strong> the following diagnostic tests and procedures:<br />

Echocardiogram (transthoracic and transesophageal). (PC, MK)<br />

Exercise stress test. (PC, MK)<br />

Stress thallium (myocardial perfusion scan). (PC, MK)<br />

Dobutamine stress echocardiography. (PC, MK)<br />

Coronary angiography. (PC, MK)<br />

Electron beam CT scan (for coronary calcification). (PC, MK)<br />

Ventilation/perfusion lung (V/Q) scan. (PC, MK)<br />

Pulmonary embolism protocol CT scan. (PC, MK)<br />

Pulmonary angiography. (PC, MK)<br />

5. Communication skills: Students should be able to:<br />

Communicate the diagnosis, prognosis and treatment plan to patients and their families.<br />

(PC, CS)<br />

As appropriate for age and gender, educate patients about risk factors for cardiovascular<br />

disease. (PC, CS)<br />

Counsel patients or facilitate the provision <strong>of</strong> counseling related to:<br />

o<br />

Smoking cessation. (PC, CS)<br />

o<br />

Reduction <strong>of</strong> dietary saturated fats and cholesterol. (PC, CS)<br />

o<br />

Restriction <strong>of</strong> dietary sodium intake. (PC, CS)<br />

o<br />

Weight reduction. (PC, CS)<br />

o<br />

Increased physical activity. (PC, CS)<br />

6. Basic procedural skills: Students should be able to:<br />

• Perform a 12-lead ECG. (PC)<br />

7. Management skills: Students should be able to develop an appropriate evaluation and<br />

treatment plan for patients that includes:<br />

Identification <strong>of</strong> the indications, contraindications, mechanisms <strong>of</strong> action, adverse reactions,<br />

significant interactions, and relative costs <strong>of</strong> the following medications:<br />

o Anti-platelet agents (aspirin, clopidogrel). (PC, MK)<br />

o Nitroglycerin and long-acting nitrates. (PC, MK)<br />

o Beta-blockers. (PC, MK)<br />

o Angiotensin-converting enzyme inhibitors. (PC, MK)<br />

o Calcium channel blockers. (PC, MK)<br />

o Antithrombotic therapy (heparin, warfarin). (PC, MK)<br />

o Glycoprotein IIb/IIIa inhibitors. (PC, MK)<br />

o Lipid-lowering agents. (PC, MK)<br />

Identification <strong>of</strong> the indications, contraindications, complications, long-term outcomes and<br />

relative costs associated with the following treatment modalities for ischemic heart disease:<br />

o Thrombolytic therapy. (PC, MK)<br />

o Percutaneous coronary intervention (with or without stenting). (PC, MK)<br />

o Coronary artery bypass graft surgery (CABG). (PC, MK)<br />

87


Determining when to consult a cardiologist or other subspecialist in the management <strong>of</strong><br />

patients with chest pain. (PC, SBP)<br />

Description <strong>of</strong> how the diagnosis and treatment <strong>of</strong> chest pain in special populations may<br />

differ (e.g. very elderly, associated co-morbidities). (PC, MK)<br />

Accessing and utilizing appropriate information systems and resources to help delineate<br />

issues related to chest pain. (PC, PLI)<br />

Incorporating patient preferences. (PC, P)<br />

C. ATTITUDES AND PROFESSIONAL BEHAVIORS: Students should be able to:<br />

1. Understand the emotional impact <strong>of</strong> a diagnosis <strong>of</strong> coronary artery disease and its potential<br />

effect on lifestyle (work performance, sexual functioning, etc). (PC, P)<br />

2. Respond appropriately to patient who are nonadherent to lifestyle modifications.<br />

(CS, P)<br />

3. Recognize the importance <strong>of</strong> early detection and modification <strong>of</strong> risk factors that may<br />

contribute to the development <strong>of</strong> atherosclerosis. (PC, P)<br />

4. Demonstrate commitment to using risk-benefit, cost-benefit, and evidence-based<br />

considerations in the selection <strong>of</strong> diagnostic and therapeutic interventions for chest pain. (PLI,<br />

P)<br />

5. Demonstrate ongoing commitment to self-directed learning regarding chest pain. (PLI, P)<br />

6. Recognize the importance and demonstrate a commitment to the utilization <strong>of</strong> other healthcare<br />

pr<strong>of</strong>essions in the treatment <strong>of</strong> chest pain. (P, SBP)<br />

D. REFERENCES:<br />

ACC/AHA 2002 guideline update for the management <strong>of</strong> patients with chronic stable<br />

angina--summary article: a report <strong>of</strong> the American College <strong>of</strong> Cardiology/American<br />

Heart Association Task Force on practice guidelines (Committee on the Management<br />

<strong>of</strong> Patients With Chronic Stable Angina). J Am Coll Cardiol. 2003;41:159-68.<br />

http://www.acc.org/clinical/guidelines/stable/stable_clean.pdf<br />

Panju AA, Hemmeigarn BR, Guyatt GH, Simel DL. Is this patient having a myocardial<br />

infarction? JAMA. 1998;280:1256-63.<br />

Klompas M. Does this patient have an acute thoracic aortic dissection? JAMA.<br />

2002;287:2262-72.<br />

TRAINING PROBLEM #7: COUGH<br />

RATIONALE:<br />

Cough is one <strong>of</strong> the most common symptoms with which a patient will present in the outpatient<br />

setting. There are several common etiologies for cough <strong>of</strong> which a third year medical student<br />

should be aware, as well as more clinically concerning etiologies. A proper understanding <strong>of</strong> the<br />

pathophysiology, diagnosis, and treatment <strong>of</strong> cough is an important learning objective.<br />

PREREQUISITES:<br />

Prior knowledge, skills, and attitudes acquired during the pre-<strong>clerkship</strong> experience should include:<br />

Ability to perform a complete medical history and physical exam.<br />

88


Ability to communicate with patients <strong>of</strong> diverse backgrounds.<br />

Knowledge <strong>of</strong> respiratory anatomy, physiology and pathophysiology.<br />

SPECIFIC LEARNING OBJECTIVES:<br />

A. KNOWLEDGE: Students should be able to define, describe and discuss:<br />

1. The criteria used to classify a cough (e.g. acute vs. chronic, productive vs. non-productive).<br />

(MK)<br />

2. Symptoms, signs, pathophysiology, differential diagnosis, and typical clinical course <strong>of</strong> the<br />

most common causes cough:<br />

Acute cough:<br />

o<br />

Viral tracheitis. (MK)<br />

o<br />

o<br />

Acute bronchitis. (MK)<br />

Pneumonia. (MK)<br />

Chronic cough:<br />

o Gastroesophageal reflux. (MK)<br />

o Post-nasal drip. (MK)<br />

o Asthma/reactive airways disease. (MK)<br />

o Angiotensin converting enzyme inhibitors. (MK)<br />

o Post-infectious. (MK)<br />

o Infectious (pertussis, tuberculosis). (MK)<br />

o Chronic bronchitis. (MK)<br />

o Bronchiectasis. (MK)<br />

o Pleural effusion. (MK)<br />

o Lung cancer. (MK)<br />

o Congestive heart failure. (MK)<br />

B. SKILLS: Students should be able to demonstrate specific skills, including:<br />

1. History-taking skills: Students should be able to obtain, document, and present an ageappropriate<br />

medical history that differentiates among the etiologies <strong>of</strong> disease, including:<br />

Onset. (PC, CS)<br />

Duration. (PC, CS)<br />

Exacerbating/relieving factors. (PC, CS)<br />

Associated symptoms (fever, chills, weight loss). (PC, CS)<br />

Presence or absence <strong>of</strong> hemoptysis. (PC, CS)<br />

Tobacco history. (PC, CS)<br />

Relevant past medical history. (PC, CS)<br />

2. Physical exam skills: Students should be able to perform a physical exam to establish the<br />

diagnosis and severity <strong>of</strong> disease, including:<br />

Accurately determining respiratory rate and level <strong>of</strong> respiratory distress. (PC)<br />

Recognizing the pharyngeal signs <strong>of</strong> post nasal drip. (PC)<br />

Identifying rales, rhonchi, and wheezes. (PC)<br />

Recognizing signs <strong>of</strong> pulmonary consolidation. (PC)<br />

3. Differential diagnosis: Students should be able to generate a prioritorized differential<br />

diagnosis recognizing history, physical exam, and laboratory findings that suggest a specific<br />

etiology <strong>of</strong> cough. (PC, MK)<br />

4. Laboratory interpretations: Students should be able to recommend when to order diagnostic<br />

89


and laboratory tests and be able to interpret them, both prior to and after initiating treatment<br />

based on the differential diagnosis, including consideration <strong>of</strong> test cost and performance<br />

characteristics as well as patient preferences. Laboratory and diagnostic tests should include,<br />

when appropriate:<br />

Chest radiograph. (PC, MK)<br />

Pleural fluid cell count and chemistries. (PC, MK)<br />

PFTs. (PC, MK)<br />

Sputum Gram stain and sputum acid-fast stain. (PC, MK)<br />

Sputum culture and sensitivities. (PC, MK)<br />

Students should be able to define the indications for and interpret (with consultation) results<br />

<strong>of</strong>:<br />

Barium swallow. (PC, MK)<br />

Upper endoscopy. (PC, MK)<br />

Sputum cytology. (PC, MK)<br />

Chest CT scan. (PC, MK)<br />

5. Communication skills: Students should be able to:<br />

Counsel and educate patients about environmental contributors to their disease,<br />

pneumococcal and influenza immunizations, and smoking cessation. (PC, CS)<br />

Communicate the diagnosis, prognosis, and treatment plan, and subsequent follow-up to<br />

the patient and his or her family. (PC, CS)<br />

Elicit input and questions from the patient and his or her family about the management<br />

plan. (PC, CS)<br />

6. Management skills: Students should be able to develop an appropriate evaluation and<br />

treatment plan for patients that includes:<br />

Describing the indications, contraindications, mechanisms <strong>of</strong> action, adverse reactions,<br />

significant interactions, and relative costs <strong>of</strong> the various treatments, interventions, or<br />

procedures commonly used to diagnose and treat patients who present with symptoms <strong>of</strong><br />

cough. (PC, MK, SBP)<br />

Determining when to obtain consultation from a pulmonologist, allergist, otolaryngologist, or<br />

gastroenterologist. (PC, SBP)<br />

Using a cost-effective approach based on the differential diagnosis. (PC, SBP)<br />

Accessing and utilizing appropriate information systems and resources to help delineate<br />

issues related to patients with chronic cough. (PC, PLI)<br />

Incorporating patient needs and preferences. (PC, P)<br />

C. ATTITUDES AND PROFESSIONAL BEHAVIORS: Students should be able to:<br />

1. Demonstrate commitment to using risk-benefit, cost-benefit, and evidence-based<br />

considerations in the selection <strong>of</strong> diagnostic and therapeutic interventions for cough. (PLI, P)<br />

2. Respond appropriately to patients who are non-adherent to treatment for cough and smoking<br />

cessation. (CS, P)<br />

3. Demonstrate ongoing commitment to self-directed learning regarding diagnosis and<br />

management <strong>of</strong> cough. (PLI, P)<br />

4. Appreciate the impact that an acute or chronic cough has on a patient’s quality <strong>of</strong> life, wellbeing,<br />

ability to work, and the family. (P)<br />

5. Recognize the importance and demonstrate a commitment to the utilization <strong>of</strong> other healthcare<br />

pr<strong>of</strong>essions in the treatment <strong>of</strong> cough. (P, SBP)<br />

D. REFERENCES:<br />

90


Currie GP, Gray RD, McKay J. Chronic cough. BMJ. 2003;326:261.<br />

Irwin RS, Madison JM. The persistently troublesome cough. Am J Respir Crit Care Med.<br />

2002;165:1469-74.<br />

Jones HC, Chang SI. Clinical Inquires. What is the best approach to the evaluation and<br />

treatment <strong>of</strong> chronic cough? J Fam Pract. 2001;50:748-9.<br />

Irwin RS, Madison MJ. Primary care: the diagnosis and treatment <strong>of</strong> cough. N Engl J Med.<br />

2000;343:1715-21.<br />

Irwin RS, Boulet LP, Cloutier MM, et al. Managing cough as a defense mechanism and as a<br />

symptom. Consensus panel report <strong>of</strong> the American College <strong>of</strong> Chest Physicians. Chest.<br />

1998;114:133S-181S.<br />

91


TRAINING PROBLEM #8: DYSPNEA<br />

RATIONALE:<br />

Shortness <strong>of</strong> breath or dyspnea is one <strong>of</strong> the most common patient complaints encountered in<br />

internal medicine. It has a very large number <strong>of</strong> etiologic possibilities— some benign but many<br />

potentially life-threatening. Because <strong>of</strong> the latter, a systematic approach to dyspnea is crucial.<br />

PREREQUISITES:<br />

Prior knowledge, skills, and attitudes acquired during the pre-clinical experience should include:<br />

Ability to perform a complete medical history and physical.<br />

Ability to communicate with patients <strong>of</strong> diverse backgrounds.<br />

Anatomy, physiology, and pathophysiology <strong>of</strong> the pulmonary, cardiac, neurologic, and<br />

musculoskeletal systems.<br />

Physiology <strong>of</strong> acid-base homeostasis.<br />

SPECIFIC LEARNING OBJECTIVES:<br />

A. KNOWLEDGE: Students should be able to define, describe, and discuss:<br />

1. Major organ systems/pathologic states causing dyspnea and their<br />

pathophysiology, including:<br />

Cardiac. (MK)<br />

Pulmonary. (MK)<br />

Anemia/hypovolemia. (MK)<br />

Acid-base disorders and other metabolic derangements (MK)<br />

Neuromuscular weakness. (MK)<br />

Central neurologic derangements. (MK)<br />

2. The symptoms, signs, and laboratory values associated with respiratory failure and ventilatory<br />

failure. (MK)<br />

3. The alveolar-arterial oxygen gradient and the pathophysiologic states that can alter it. (MK)<br />

4. The potential risks <strong>of</strong> relying too heavily on pulse oximetry as the sole indicator <strong>of</strong> arterial<br />

oxygen content. (MK)<br />

5. The common causes <strong>of</strong> acute dyspnea, their pathophysiology, symptoms, and signs, including:<br />

Pulmonary edema. (MK)<br />

Pulmonary embolism. (MK)<br />

Pneumonia. (MK)<br />

Acute exacerbation <strong>of</strong> COPD. (MK)<br />

Asthma. (MK)<br />

Cardiac ischemia. (MK)<br />

Pneumothorax. (MK)<br />

Anxiety. (MK)<br />

6. The common causes <strong>of</strong> chronic dyspnea their pathophysiology, symptoms, and signs,<br />

including:<br />

Congestive heart failure. (MK)<br />

COPD. (MK)<br />

Pulmonary parenchymal disease. (MK)<br />

Pulmonary vascular disease. (MK)<br />

92


Anemia. (MK)<br />

Neuromuscular weakness. (MK)<br />

7. Basic treatment options for the common causes <strong>of</strong> acute and chronic dyspnea. (MK)<br />

8. The utility <strong>of</strong> supplemental oxygen therapy and the potential dangers <strong>of</strong> overly aggressive<br />

oxygen supplementation in some pathophysiologic states. (MK)<br />

B. SKILLS: Students should be able to demonstrate specific skills, including:<br />

1. History-taking skills: Students should be able to obtain, document, and present an<br />

age-appropriate medical history, that differentiates among etiologies <strong>of</strong> disease,<br />

including:<br />

Quantity, quality, severity, duration, ameliorating/exacerbating factors <strong>of</strong> the dyspnea. (PC,<br />

CS)<br />

Associated symptoms such as fevers, chills, sweats, orthopnea, paroxysmal nocturnal<br />

dyspnea, wheezing, edema, chest pain, cough, sputum production, hemoptysis,<br />

palpitations, nausea, anxiety, dizziness, orthostasis, weakness. (PC, CS)<br />

History <strong>of</strong> pulmonary, cardiac, neuromuscular/neurologic, renal, hepatic, and coagulopathic<br />

disorders. (PC, CS)<br />

Risk factors for deep vein thrombosis/pulmonary embolism. (PC, CS)<br />

Ingestion <strong>of</strong> drugs and toxic substances, administration <strong>of</strong> IV fluids. (PC, CS<br />

Smoking and environmental exposures. (PC, CS)<br />

2. Physical exam skills: Students should be able to perform a physical exam to establish the<br />

diagnosis and severity <strong>of</strong> disease, including:<br />

Accurately determining respiratory rate and level <strong>of</strong> respiratory distress. (PC)<br />

Assessing the use <strong>of</strong> accessory muscles for breathing. (PC)<br />

Accurately measuring pulsus paradox. (PC)<br />

Identifying bronchial breath sounds, rales, rhonchi, wheezes, and subcutaneous<br />

emphysema. (PC)<br />

Identifying signs <strong>of</strong> pulmonary consolidation and hyperresonance. (PC)<br />

Identifying signs <strong>of</strong> pleural effusion. (PC)<br />

Identifying signs <strong>of</strong> elevated central venous pressure. (PC)<br />

Identifying signs <strong>of</strong> hypovolemia. (PC)<br />

Identifying S3 gallop, edema, and pallor. (PC)<br />

Identifying signs <strong>of</strong> deep vein thrombosis. (PC)<br />

3. Differential diagnosis: Students should be able to generate a prioritized differential diagnosis<br />

recognizing specific history and physical exam findings that suggest a specific etiology <strong>of</strong><br />

dyspnea (PC, MK)<br />

4. Laboratory interpretation: Students should be able to recommend when to order diagnostic<br />

and laboratory tests and be able to interpret them, both prior to and after initiating treatment<br />

based on the differential diagnosis, including consideration <strong>of</strong> test cost and performance<br />

characteristics as well as patient preferences. Laboratory and diagnostic tests should include,<br />

when appropriate:<br />

CBC. (PC, MK)<br />

Electrolytes, BUN/Cr, GLC. (PC, MK)<br />

Pulse oximitry. (PC, MK)<br />

ABG. (PC, MK)<br />

Chest radiograph. (PC, MK)<br />

12-lead ECG. (PC, MK)<br />

Pulmonary function tests. (PC, MK)<br />

93


Students should be able to define the indications for and interpret (with consultation) the<br />

results <strong>of</strong>:<br />

Ventilation perfusion scintigraphy. (PC, MK)<br />

Chest CT. (PC, MK)<br />

Venous Doppler studies. (PC, MK)<br />

Cardiac stress test. (PC, MK)<br />

Echocardiography. (PC, MK)<br />

5. Communication skills: Students should be able to:<br />

Communicate the diagnosis, treatment plan, and subsequent follow-up to the patient and<br />

his or her family. (PC, CS)<br />

Elicit questions from the patient and his or her family about the management plan. (PC, CS)<br />

Counsel and educate patients about environmental contributors to their disease. (PC, CS)<br />

Counsel patients nonjudgmentally about smoking cessation. (PC, CS)<br />

6. Basic and advanced procedural skills: Students should be able to:<br />

Obtain an ABG. (PC)<br />

7. Management skills: Students should able to develop an appropriate evaluation and treatment<br />

plan for patients that includes:<br />

A rapid triage approach to the acutely dyspneic patient. (PC, MK)<br />

An appropriate assessment <strong>of</strong> the patient’s oxygenation status. (PC, MK)<br />

Appropriate oxygen supplementation as indicated. (PC, MK)<br />

Management plans for pulmonary edema/congestive heart failure, pneumonia, COPD,<br />

asthma, pulmonary embolism, cardiac ischemia, hypovolemia, anemia, and pneumothorax.<br />

(PC, MK)<br />

Determining when to obtain consultation from an appropriate specialist. (PC, SBP)<br />

Using a cost-effective approach based on the differential diagnosis. (PC, SBP)<br />

Accessing and utilizing appropriate information systems and resources to help delineate<br />

issues related to dyspnea. (PC, PLI)<br />

Incorporating patient preferences. (PC, P)<br />

C. ATTITUDES AND PROFESSIONAL BEHAVIORS: Students should be able to:<br />

1. Demonstrate commitment to using risk-benefit, cost-benefit, and evidence-based<br />

considerations in the selection diagnostic and therapeutic interventions for dyspnea. (PLI, P)<br />

2. Recognize the importance <strong>of</strong> patient preferences when selecting among diagnostic and<br />

therapeutic options for dyspnea. (P)<br />

3. Demonstrate ongoing commitment to self-directed learning regarding dyspnea. (PLI, P)<br />

4. Appreciate the impact dyspnea has/have on a patient’s quality <strong>of</strong> life, wellbeing, ability to work,<br />

and the family. (P)<br />

5. Recognize the importance <strong>of</strong> and demonstrate a commitment to the utilization<br />

<strong>of</strong> other healthcare pr<strong>of</strong>essionals in the diagnosis and treatment <strong>of</strong> dyspnea. (P, SBP)<br />

6. Show understanding for the difficulties patients face with smoking cessation. (P)<br />

D. REFERENCES:<br />

Ingram RH and Braunwald E. (2005). Dyspnea and pulmonary edema. In Kasper DL,<br />

Braunwald EB, Fauci AS, Hauser SL, Longo DL, Jameson JL, eds. Harrison’s Principles<br />

94


<strong>of</strong> Internal <strong>Medicine</strong>. 16 th ed. New York, NY: McGraw-Hill; 2005:201-5.<br />

Zoorob RJ. Campbell JS. Acute dyspnea in the <strong>of</strong>fice. Am Fam Physician.<br />

2003;68:1803-10.<br />

Fedullo PF. Tapson VF. Clinical practice. The evaluation <strong>of</strong> suspected pulmonary<br />

embolism. N Engl J Med. 2003;349:1247-56.<br />

Manning HL. Schwartzstein RM. Pathophysiology <strong>of</strong> dyspnea. N Engl J Med.<br />

1995;333:1547-53.<br />

95


TRAINING PROBLEM #9: DYSURIA<br />

RATIONALE:<br />

Dysuria is a very common presentation in the outpatient setting. Given the amount <strong>of</strong> health care<br />

dollars that are spent on antibiotic treatment <strong>of</strong> urinary tract infections as well as the emergence <strong>of</strong><br />

resistance, it is important for third year medical students to have a working knowledge <strong>of</strong> how to<br />

approach the patient with this complaint, and how to differentiate patients with cystitis from other<br />

common causes <strong>of</strong> dysuria.<br />

PREREQUISITES:<br />

Prior knowledge, skills, and attitudes acquired during the pre-<strong>clerkship</strong> experience should include:<br />

Ability to perform a complete medical history and physical exam.<br />

Ability to communicate with patients <strong>of</strong> diverse backgrounds.<br />

Knowledge <strong>of</strong> genitourinary anatomy, physiology and pathophysiology.<br />

SPECIFIC LEARNING OBJECTIVES:<br />

A. KNOWLEDGE: Students should be able to define, describe, and discuss:<br />

1. Presenting signs and symptoms <strong>of</strong> the common causes <strong>of</strong> dysuria, including:<br />

Cystitis. (MK)<br />

Urethritis, gonococcal and non-gonococcal (e.g. chlamydia, trichomonas, HSV). (MK)<br />

Pyelonephritis. (MK)<br />

Acute and chronic prostatitis. (MK)<br />

Epididymitis. (MK)<br />

Vaginitis (yeast, bacterial vaginosis, trichomonas, atrophic, irritant). (MK)<br />

Interstitial cystitis. (MK)<br />

2. Symptoms and signs <strong>of</strong> pyelonephritis and how to distinguish an upper from a lower UTI. (MK)<br />

3. Common bacteria that cause UTI. (MK)<br />

4. Aspects <strong>of</strong> pathogenesis that affect UTI, including gender, sexual activity, diabetes, anatomic<br />

anomalies, instrumentation, and use <strong>of</strong> an indwelling catheter. (MK)<br />

5. Indications for pursuing further work up for patients with UTI. (MK)<br />

B. SKILLS: Students should be able to demonstrate specific skills, including:<br />

1. History-taking skills: Students should be able to obtain, document, and present an ageappropriate<br />

history that differentiates among etiologies <strong>of</strong> dysuria, including:<br />

Timing, frequency, severity, and location <strong>of</strong> dysuria. (PC, CS)<br />

Fever, chills, sweats. (PC, CS)<br />

Frequency, urgency, hesitancy, incomplete voiding. (PC, CS)<br />

Back, abdominal, and groin pain. (PC, CS)<br />

History <strong>of</strong> nephrolithiasis. (PC, CS)<br />

Hematuria. (PC, CS)<br />

Vaginal or penile discharge. (PC, CS)<br />

Penile skin lesions. (PC, CS)<br />

Sexual activity. (PC, CS)<br />

History <strong>of</strong> sexual transmitted diseases. (PC, CS)<br />

96


Dyspareunia. (PC, CS)<br />

Scrotal, testicular, and perineal pain. (PC, CS)<br />

Use <strong>of</strong> topical hygiene products. (PC, CS)<br />

2. Physical exam skills: Students should be able to perform a physical exam to establish the<br />

diagnosis and severity <strong>of</strong> disease, including:<br />

Percussion and palpation <strong>of</strong> the bladder to accurately recognize distention and tenderness.<br />

(PC)<br />

Palpation over the kidneys to elicit flank tenderness. (PC)<br />

Palpation <strong>of</strong> the abdomen to elicit tenderness. (PC)<br />

Palpation and massage <strong>of</strong> the male prostate to obtain discharge. (PC)<br />

Accurate recognition <strong>of</strong> perineal or vaginal atrophy and inflammation. (PC)<br />

Techniques <strong>of</strong> the pelvic examination to assess for causes <strong>of</strong> vaginitis. (PC)<br />

3. Differential diagnosis: Students should be able to generate a differential diagnosis<br />

recognizing specific history, physical exam, and laboratory findings that suggest a specific<br />

etiology <strong>of</strong> dysuria. (PC, MK)<br />

4. Laboratory interpretation: Students should be able to recommend when to order diagnostic<br />

and laboratory tests and be able to interpret them, both prior to and after initiating treatment<br />

based on the differential diagnosis, including consideration <strong>of</strong> test cost and performance<br />

characteristics as well as patient preferences.<br />

Laboratory and diagnostic tests should include, when appropriate:<br />

Urinalysis interpretation including cells and casts, urine dipstick and Gram stain when<br />

appropriate. (PC, MK)<br />

Urine culture. (PC, MK)<br />

Gram stain and culture <strong>of</strong> urethral or cervical discharge. (PC, MK)<br />

KOH stain and normal saline wet prep <strong>of</strong> vaginal discharge. (PC, MK)<br />

Urinary or cervical PCR to test for gonorrhea and Chlamydia. (PC, MK)<br />

KUB radiograph. (PC, MK)<br />

5. Communication skills: Students should be able to:<br />

Communicate the diagnosis, treatment plan, and subsequent follow-up to the patient and<br />

his or her family. (PC, CS)<br />

Elicit input and questions from the patient and his or her family about the management<br />

plan. (PC, CS)<br />

Counsel patients about safe sexual activity. (PC, CS)<br />

Explain the risk <strong>of</strong> recurrent UTI and counsel regarding preventative measures. (PC, CS)<br />

6. Management skills: Students should be able to develop an appropriate evaluation and<br />

treatment plan for patients that includes:<br />

Selecting appropriate empiric antibiotic therapy for cystitis, pyelonephritis or urethritis prior<br />

to culture results. (PC, MK)<br />

Counseling patients on symptomatic therapies for acute cystitis. (PC, MK)<br />

Selecting the appropriate duration <strong>of</strong> therapy for cystitis and pyelonephritis. (PC, MK)<br />

Evaluating and managing patients with recurrent urinary tract infections including<br />

prophylaxis. (PC, MK)<br />

Choosing appropriate treatment for vaginitis depending on results <strong>of</strong> evaluation. (PC, MK)<br />

Understanding the treatment <strong>of</strong> prostatitis based on probable organisms and age. (PC, MK)<br />

Determining when to obtain consultation from a urologist or gynecologist. (PC, MK)<br />

Using a cost-effective approach based on the differential diagnosis. (PC, SBP)<br />

Accessing and utilizing appropriate information systems and resources to help delineate<br />

issues related to dysuria. (PC, PLI)<br />

97


Incorporating patient preferences. (PC, P)<br />

C. ATTITUDES AND PROFESSIONAL BEHAVIORS: Students should be able to:<br />

1. Demonstrate commitment to using risk-benefit, cost-benefit, and evidence-based<br />

considerations in the selection <strong>of</strong> diagnostic and therapeutic interventions for dysuria. (PLI, P)<br />

2. Recognize the importance <strong>of</strong> patient needs and preferences when selecting among diagnostic<br />

and therapeutic options for dysuria. (P)<br />

3. Demonstrate ongoing commitment to self-directed learning regarding dysuria. (PLI, P)<br />

4. Recognize the importance <strong>of</strong> and demonstrate a commitment to the utilization <strong>of</strong> other<br />

healthcare pr<strong>of</strong>essionals in the treatment <strong>of</strong> dysuria. (P, SBP)<br />

D. REFERENCES:<br />

32.<br />

Bremnor JD, Sadovsky R. Evaluation <strong>of</strong> dysuria in adults. Am Fam Physician.<br />

2002;65:1589-96.<br />

Bent S, Nallamothu BK, Simel DL, et al. Does this woman have an acute<br />

uncomplicated urinary tract infection? JAMA. 2002;287:2701-10.<br />

Owen MK, Clenney TL. Management <strong>of</strong> vaginitis. Am Fam Physician. 2004;70:2125-<br />

Fihn SD. Clinical practice. Acute uncomplicated urinary tract infection in<br />

women. N Engl J Med. 2003;349:259-66.<br />

98


TRAINING PROBLEM #10: FEVER<br />

RATIONALE:<br />

Because fever can have many infectious or noninfectious causes, patients with fever should be<br />

stratified by host susceptibility factors and evaluated in a systematic manner. A rational approach to<br />

patients with fever will help clinicians recognize presentations that need immediate attention, limit<br />

unnecessary diagnostic testing in less seriously ill patients, and help inform therapeutic decision<br />

making.<br />

PREREQUISITES:<br />

Prior knowledge, skills, and attitudes acquired during the pre-<strong>clerkship</strong> experience should include:<br />

Ability to perform a complete medical history and physical exam.<br />

Ability to communicate with patients <strong>of</strong> diverse backgrounds.<br />

Physiology and pathophysiology <strong>of</strong> thermoregulation and the immune response.<br />

Pharmacology <strong>of</strong> antipyretics.<br />

SPECIFIC LEARNING OBJECTIVES:<br />

A. KNOWLEDGE: Students should be able to define, describe, and discuss:<br />

1. Physiology <strong>of</strong> the acute febrile response, including the:<br />

Beneficial and detrimental effects <strong>of</strong> fever upon the host. (MK)<br />

The differences in clinical manifestations between immunocompetent and<br />

immunocompromised patients. (MK)<br />

2. Risk factors and co-morbidities that are important in determining the host response to infection<br />

(e.g. neutropenia, asplenia, cirrhosis, alcoholism, diabetes, corticosteroid use, malnutrition, T<br />

cell dysfunction) (MK)<br />

3. Etiology <strong>of</strong> fever in special populations, including patients with a history <strong>of</strong>:<br />

Neutropenia due to cancer-related myelosuppression. (MK)<br />

HIV disease. (MK)<br />

Intravenous drug abuse. (MK)<br />

Recent international travel or immigration. (MK)<br />

Concomitant skin rash and lymphadenopathy. (MK)<br />

4. Pathophysiology and clinical presentation <strong>of</strong> patients with sepsis syndromes. (MK)<br />

5. Common causes <strong>of</strong> prolonged fever without apparent source, including:<br />

FUO in a normal host. (MK)<br />

Nosocomial FUO. (MK)<br />

Neutropenic FUO. (MK)<br />

FUO associated with HIV disease. (MK)<br />

B. SKILLS: Students should be able to demonstrate specific skills, including:<br />

1. History-taking skills: Students should be able to obtain, document, and present an ageappropriate<br />

medical history that differentiates among etiologies <strong>of</strong> disease, including:<br />

Chronology, duration and pattern <strong>of</strong> fever. (PC, CS)<br />

Associated symptoms. (PC, CS)<br />

Immune status and baseline co-morbidities. (PC, CS)<br />

Immunization status. (PC, CS)<br />

99


Relevant history <strong>of</strong> exposures. (PC, CS)<br />

Occupational, travel, family, and sexual history. (PC, CS)<br />

Medication history, including use <strong>of</strong> over-the-counter and illicit drugs. (PC, CS)<br />

3. Physical exam skills: Students should be able to perform a complete physical exam to<br />

determine the severity <strong>of</strong> disease and establish a preliminary hypothesis about the cause <strong>of</strong><br />

fever. (PC)<br />

4. Differential diagnosis: Students should be able to generate a prioritized differential diagnosis<br />

recognizing specific history and physical exam findings that suggest a specific etiology:<br />

Infection. (PC, MK)<br />

Rheumatologic disease/inflammatory disorder. (PC, MK)<br />

Malignancy. (PC, MK)<br />

Drug reaction. (PC, MK)<br />

4. Laboratory interpretation: Students should be able to recommend when to order diagnostic<br />

and laboratory tests and be able to interpret them, both prior to and after initiating treatment<br />

based on the differential diagnosis, including consideration <strong>of</strong> test cost and performance<br />

characteristics as well as patient preferences. Laboratory and diagnostic tests should include,<br />

when appropriate:<br />

CBC with differential. (PC, MK)<br />

UA with exam <strong>of</strong> urinary sediment. (PC, MK)<br />

Chest radiography. (PC, MK)<br />

Blood cultures. (PC, MK)<br />

Urine cultures. (PC, MK)<br />

Sputum Gram stain and cultures. (PC, MK)<br />

Sputum AFB stain and culture. (PC, MK)<br />

ESR and/or specific rheumatologic tests. (PC, MK)<br />

PPD. (PC, MK)<br />

Cerebrospinal fluid analysis (color, opening pressure, chemistries, cell<br />

counts, staining, cultures, cytology, cryptococcal antigen, VDRL). (PC, MK)<br />

Chemistries, Gram stain, and culture <strong>of</strong> abnormal fluid collections<br />

(e.g. pleural effusion, ascites, abscesses). (PC, MK)<br />

Stool culture <strong>of</strong> enteric pathogens. (PC, MK)<br />

Stool Clostridium difficile toxin assay. (PC, MK)<br />

Stains and cultures from the throat, urethra, anus, cervix, vagina. (PC, MK)<br />

HIV ELISA and western blot. (PC, MK)<br />

Students should be able to define the indications for and interpret (with consultation) the<br />

results <strong>of</strong>:<br />

CT imaging. (PC, MK)<br />

Echocardiography. (PC, MK)<br />

Tissue biopsy. (PC, MK)<br />

5. Communication skills: Students should be able to:<br />

Communicate the diagnosis, treatment plan, and subsequent follow-up patients. (PC, CS)<br />

Elicit questions from the patient and their family about the management plan. (PC, CS)<br />

6. Basic and advanced procedural skills: Students should be able to:<br />

Obtain blood, wound, and throat cultures. (PC)<br />

Place and interpret a PPD. (PC)<br />

7. Management skills: Students should able to develop an appropriate evaluation and<br />

treatment plan for patients that includes:<br />

100


Developing an appropriate evaluation plan for patients with fever including ordering and<br />

interpreting appropriate laboratory and radiographic studies. (PC)<br />

Assessing the severity <strong>of</strong> presentation based on the history, host factors, physical exam<br />

and laboratory results and recognizing presentations that need immediate attention. (PC)<br />

Developing an appropriate treatment plan for patients with fever including the selection <strong>of</strong><br />

an initial, empiric treatment regimen for neutropenic patients with fever and/or patients with<br />

life threatening sepsis. (PC)<br />

Determining when to obtain consultation from an appropriate specialist. (PC, SBP)<br />

Using a cost-effective approach based on the differential diagnosis. (PC, SBP)<br />

Accessing and utilizing appropriate information systems and resources to help delineate<br />

issues related to fever. (PC, PLI)<br />

Incorporating patient preferences. (PC, P)<br />

C. ATTITUDES AND PROFESSIONAL BEHAVIORS: Students should be able to:<br />

3. Demonstrate commitment to using risk-benefit, cost-benefit, and evidence-based<br />

considerations in the selection diagnostic and therapeutic interventions for fever. (P, PLI)<br />

4. Appreciate the impact fever has on a patient’s quality <strong>of</strong> life, well-being, ability to work, and<br />

family; recognize the emotional impact <strong>of</strong> differential diagnosis. (P)<br />

5. Recognize the importance <strong>of</strong> and demonstrate a commitment to the utilization <strong>of</strong> other<br />

healthcare pr<strong>of</strong>essions in the diagnosis and treatment <strong>of</strong> fever. (P, SBP)<br />

D. REFERENCES:<br />

Dinarello CA, Gelfand JA. (2005). Fever and hyperthermia. In Kasper DL, Braunwald<br />

EB, Fauci AS, Hauser SL, Longo DL, Jameson JL, eds. Harrison’s Principles <strong>of</strong> Internal<br />

<strong>Medicine</strong>. 16 th ed. New York, NY: McGraw-Hill; 2005:104-8.<br />

Kaye ET, Kaye KM. (2005). Fever and rash. In Kasper DL, Braunwald EB, Fauci AS,<br />

Hauser SL, Longo DL, Jameson JL, eds. Harrison’s Principles <strong>of</strong> Internal <strong>Medicine</strong>. 16 th<br />

ed. New York, NY: McGraw-Hill; 2005:108-16.<br />

Gelfand JA, Callahan MV. (2005). Fever <strong>of</strong> unknown origin. In Kasper DL, Braunwald<br />

EB, Fauci AS, Hauser SL, Longo DL, Jameson JL, eds. Harrison’s Principles <strong>of</strong> Internal<br />

<strong>Medicine</strong>. 16 th ed. New York, NY: McGraw-Hill; 2005:116-21.<br />

Roth AR, Basello GM. Approach to the adult patient with fever <strong>of</strong> unknown origin. Am<br />

Fam Physician. 2003;68:2223-8.<br />

101


TRAINING PROBLEM #11: FLUID, ELECTROLYTE AND<br />

ACID-BASE DISORDERS<br />

RATIONALE:<br />

Many disease processes can cause serious disturbances in the fluid, electrolyte and acid-base<br />

status <strong>of</strong> patients. Clinicians must be prepared to identify and correct these disturbances as<br />

efficiently as possible, thus making it an important training problem for third year medical students.<br />

PREREQUISITES:<br />

Prior knowledge, skills and attitudes acquired during the pre-<strong>clerkship</strong> experience should include:<br />

Ability to perform a complete medical history and physical exam.<br />

Ability to communicate with patients <strong>of</strong> diverse backgrounds.<br />

Knowledge <strong>of</strong> pathogenesis and pathophysiology <strong>of</strong> fluid, electrolyte and acid-base disorders.<br />

Knowledge <strong>of</strong> medications that can cause alterations in fluid and electrolyte status as well as<br />

disturbance <strong>of</strong> acid-base status.<br />

SPECIFIC LEARNING OBJECTIVES:<br />

A. KNOWLEDGE: Students should be able to define, describe and discuss:<br />

1. The pathophysiology <strong>of</strong>:<br />

Hypo- and hypervolemia. (MK)<br />

Hypo- and hypernatremia. (MK)<br />

Hypo- and hyperkalemia. (MK)<br />

Hypo- and hypercalcemia. (MK)<br />

Simple and mixed acid-base disorders. (MK)<br />

Hypo- and hyperphosphatemia. (MK)<br />

Hypo- and hypermagnesemia. (MK)<br />

Respiratory acidosis and alkalosis. (MK)<br />

Metabolic acidosis and alkalosis. (MK)<br />

2. Presenting symptoms and signs <strong>of</strong> the above disorders. (MK)<br />

3. The importance <strong>of</strong> total body water and its distribution. (MK)<br />

4. The differential diagnosis <strong>of</strong> hypo- and hypernatremia in the setting <strong>of</strong> volume depletion,<br />

euvolemia, and hypervolemia. (MK)<br />

5. How to distinguish hyponatremia from pseudohyponatremia. (MK)<br />

6. How to identify spurious hyperkalemia or acidosis-related hyperkalemia. (MK)<br />

7. Risks <strong>of</strong> too rapid or delayed therapy for hyponatremia. (MK)<br />

8. The most common causes <strong>of</strong> respiratory acidosis, respiratory alkalosis, metabolic acidosis and<br />

metabolic alkalosis. (MK)<br />

9. How to calculate the anion gap and explain its relevance to determining the cause <strong>of</strong> a<br />

metabolic acidosis. (MK)<br />

10. Changes in total body water distribution that occur with aging. (MK)<br />

11. How altered mental status can contribute to electrolyte disorders. (MK)<br />

12. Tests to use in the evaluation <strong>of</strong> fluid, electrolyte, and acid-base disorders. (MK)<br />

13. Indications for obtaining an ABG. (MK)<br />

102


14. The types <strong>of</strong> fluid preparations to use in the treatment <strong>of</strong> fluid and electrolyte disorders. (MK)<br />

B. SKILLS: Students should demonstrate specific skills, including:<br />

1. History-taking skills: Students should be able to obtain, document, and present an ageappropriate<br />

medical history that differentiates among etiologies <strong>of</strong> disease, including:<br />

Eliciting appropriate information from patients with volume overload, including recent weight<br />

gain, edema or ascites, symptoms <strong>of</strong> heart failure, dietary sodium intake, changes in<br />

medications, noncompliance and intravenous fluid regimens. (PC, CS)<br />

Eliciting appropriate information from patients with volume depletion, including recent<br />

weight loss, thirst, gastrointestinal losses, urinary losses, oral intake, insensible losses, and<br />

intravenous fluid regimens. (PC, CS)<br />

Eliciting appropriate information from patients with electrolyte problems, including use <strong>of</strong><br />

diuretics and other medications, gastrointestinal losses, and history <strong>of</strong> relevant medical<br />

conditions (e.g., heart failure, liver disease, renal disease, pulmonary disease, central<br />

nervous system disease, and malignancy). (PC, CS)<br />

2. Physical exam skills: Students should be able to perform a physical exam to establish the<br />

diagnosis and severity <strong>of</strong> disease, including:<br />

Measurement <strong>of</strong> orthostatic vital signs. (PC)<br />

Identification <strong>of</strong> signs <strong>of</strong> volume overload including peripheral edema, pulmonary edema,<br />

ascites, edema. (PC)<br />

Identification <strong>of</strong> signs <strong>of</strong> volume depletion including tachycardia, orthostatic hypotension,<br />

dry mucous membranes, poor skin turgor. (PC)<br />

Identification <strong>of</strong> signs <strong>of</strong> sodium disorders including lethargy, weakness, encephalopathy,<br />

delirium, seizures. (PC)<br />

Identification <strong>of</strong> signs <strong>of</strong> potassium disorders including weakness, fatigue, constipation,<br />

ileus, cramping, tetany, hypo- or hyperreflexia. (PC)<br />

Identification <strong>of</strong> signs <strong>of</strong> calcium disorders including cramping, tetany, Chvostek’s and<br />

Trousseau’s sign, seizures, anorexia, constipation, polyuria, hypo- or hyperreflexia, stupor,<br />

coma. (PC)<br />

3. Differential diagnosis: Students should be able to generate a prioritized differential<br />

diagnosis recognizing specific history, physical exam, and laboratory findings that<br />

distinguish between:<br />

Hypo- and hypervolemia. (PC, MK)<br />

Hypo- and hypernatremia. (PC, MK)<br />

Hypo- and hyperkalemia. (PC, MK)<br />

Hypo- and hypercalcemia. (PC, MK)<br />

Hypo- and hyperphosphatemia. (PC, MK)<br />

Hypo- and hypermagnesemia. (PC, MK)<br />

Respiratory acidosis and alkalosis. (PC, MK)<br />

Metabolic acidosis and alkalosis. (PC, MK)<br />

4. Laboratory interpretation: Students should be able to recommend when to order diagnostic<br />

and laboratory tests and be able to interpret them, both prior to and after initiating treatment<br />

based on the differential diagnosis, including consideration <strong>of</strong> test cost and performance<br />

characteristics as well as patient preferences. Laboratory and diagnostic tests should include,<br />

when appropriate:<br />

Serum electrolytes, BUN/Cr. (PC, MK)<br />

Anion gap. (PC, MK)<br />

103


ABG. (PC, MK)<br />

Serum and urine osmolality. (PC, MK)<br />

Urinary sodium. (PC, MK)<br />

Fractional excretion <strong>of</strong> sodium. (PC, MK)<br />

ECG findings in hyper- and hypokalemia. (PC, MK)<br />

5. Communication skills: Students should be able to:<br />

Explain to a patient and his or her family why intravenous fluids are needed. (PC, CS)<br />

Communicate the diagnosis, treatment plan, and subsequent follow-up to the patient and<br />

his or her family. (PC, CS)<br />

Elicit input and questions from the patient and their family about the management plan.<br />

(PC, CS)<br />

6. Basic and advanced procedural skills: Students should be able to:<br />

Insert a peripheral intravenous catheter. (PC)<br />

Obtain an ABG. (PC)<br />

Assist in the insertion <strong>of</strong> a central venous catheter. (PC)<br />

7. Management skills: Students should be able to develop an appropriate evaluation and<br />

treatment plan for patients that includes:<br />

Writing appropriate fluid orders for the treatment <strong>of</strong> hypo- and hypervolemia, hypo- and<br />

hypernatremia, hypo- and hyperkalemia, hypo- and hypercalcemia. (PC, MK)<br />

Writing appropriate orders for replacing sodium, potassium, calcium, phosphates, and<br />

magnesium. (PC, MK)<br />

Writing appropriate orders for correcting hyperkalemia, hypercalcemia, hyperphosphatemia<br />

and hypermagnesemia. (PC, MK)<br />

Calculating the water deficit that needs to be corrected to treat hypernatremia. (PC, MK)<br />

Identifying indications for administration <strong>of</strong> bicarbonate. (PC, MK)<br />

Determining when to obtain consultation from a nephrologist. (PC, SBP)<br />

Using a cost-effective approach based on the differential diagnosis. (PC, SBP)<br />

Accessing and utilizing appropriate information systems and resources to help delineate<br />

issues related to fluid, electrolyte, and acid-base disorders. (PC SBP)<br />

Incorporating patient preferences. (PC, P)<br />

C. ATTITUDES AND PROFESSIONAL BEHAVIORS: Students should be able to:<br />

1. Demonstrate commitment to using risk-benefit, cost-benefit, and evidence-based<br />

considerations in the selection <strong>of</strong> diagnostic and therapeutic interventions for problems related<br />

to fluid, electrolyte and acid-base disorders. (PLI, P)<br />

3. Demonstrate ongoing commitment to self-directed learning regarding fluid, electrolyte and<br />

acid-based disorders. (PLI, P)<br />

4. Recognize the importance and demonstrate a commitment to the utilization <strong>of</strong> other healthcare<br />

pr<strong>of</strong>essions in the treatment <strong>of</strong> problems related to fluid, electrolyte and acid-base disorders.<br />

(P, SBP)<br />

D. REFERENCES:<br />

Singer GG, Brenner BM. Fluid and Electrolyte Disturbances. In Kasper DL, Braunwald<br />

EB, Fauci AS, Hauser SL, Longo DL, Jameson JL, eds. Harrison’s Principles <strong>of</strong> Internal<br />

<strong>Medicine</strong>. 16 th ed. New York, NY: McGraw-Hill; 2005:252-63.<br />

DuBose TD. (2005). Acidosis and Alkalosis. In Kasper DL, Braunwald EB, Fauci AS,<br />

104


Hauser SL, Longo DL, Jameson JL, eds. Harrison’s Principles <strong>of</strong> Internal <strong>Medicine</strong>. 16 th<br />

ed. New York, NY: McGraw-Hill; 2005:263-71.<br />

Preston RA. Acid-Base, Fluids, and Electrolytes Made Ridiculously Simple. Miami, FL:<br />

MedMaster Incorporated; 2002.<br />

Adrogue HJ. Madias NE. Hyponatremia. N Engl J Med. 2000;342:1581-9.<br />

Adrogue HJ. Madias NE. Hypernatremia. N Engl J Med. 2000;342:1493-9.<br />

Stewart AF. Clinical practice: Hypercalcemia associated with cancer. N Engl J Med.<br />

2005;352:373-9.<br />

Carroll MF. Schade DS. A practical approach to hypercalcemia. Am Fam Physician.<br />

2003;67:1959-66.<br />

Gennari FJ. Hypokalemia. N Engl J Med. 1998;339:451-8.<br />

105


TRAINING PROBLEM #12: GASTROINTESTINAL BLEEDING<br />

RATIONALE:<br />

Gastrointestinal bleeding is a common disorder which can be life-threatening if not properly<br />

diagnosed and treated. Knowledge <strong>of</strong> etiology, risk factors, approach, and management is<br />

integral to internal medicine training.<br />

Prerequisites:<br />

Prior knowledge, skills, and attitudes acquired during the pre-<strong>clerkship</strong> experience should include:<br />

Ability to perform a complete medical history and physical exam.<br />

Ability to communicate with patients <strong>of</strong> diverse backgrounds.<br />

Anatomy, physiology, and pathophysiology <strong>of</strong> the gastrointestinal tract.<br />

Pharmacology <strong>of</strong> non-steroidal anti-inflammatory medication (a major contributing factor in<br />

etiology <strong>of</strong> gastrointestinal bleeding) as well as proton pump inhibitors and other agents used<br />

in the acute setting for treatment <strong>of</strong> gastrointestinal bleeding.<br />

SPECIFIC LEARNING OBJECTIVES:<br />

A. KNOWLEDGE: Students should be able to define, describe, and discuss:<br />

1. The common causes for and symptoms <strong>of</strong> upper and lower gastrointestinal blood loss,<br />

including:<br />

Esophagitis/esophageal erosions. (MK)<br />

Mallory Weiss tear. (MK)<br />

Peptic and duodenal ulcer disease. (MK)<br />

Esophageal/gastric varices. (MK)<br />

Erosive gastritis. (MK)<br />

Arteriovenous malformations. (MK)<br />

Gastrointestinal tumors, benign and malignant. (MK)<br />

Diverticulosis. (MK)<br />

Ischemic colitis. (MK)<br />

Hemorrhoids. (MK)<br />

Anal fissures. (MK)<br />

2. The distinguishing features <strong>of</strong> upper versus lower GI bleeding (MK)<br />

3. The indications for inpatient versus outpatient evaluation and treatment (MK)<br />

4. The principles <strong>of</strong> stabilization and treatment <strong>of</strong> acute massive GI blood loss. (MK)<br />

5. The role <strong>of</strong> contributing factors in GI bleeding such as H. pylori infection; NSAIDs, alcohol,<br />

cigarette use, coagulopathies; and chronic liver disease. (MK)<br />

B. SKILLS: Students should demonstrate specific skills, including:<br />

1. History-taking skills: Students should be able to obtain, document, and present an age<br />

appropriate history that differentiates among etiologies <strong>of</strong> disease, including:<br />

Features that distinguish upper from lower GI bleeding. (PC, CS)<br />

Quantification <strong>of</strong> degree <strong>of</strong> blood loss. (PC, CS)<br />

Chronology and duration <strong>of</strong> bleeding. (PC, CS)<br />

Associated symptoms. (PC, CS)<br />

106


Relevant past medical history. (PC, CS)<br />

Medication history, including use <strong>of</strong> tobacco and alcohol. (PC, CS)<br />

2. Physical exam skills: Students should be able to perform a physical examination to<br />

establish the diagnosis and severity <strong>of</strong> disease, including:<br />

Postural blood pressure and pulse. (PC, MK)<br />

Abdominal palpation for organomegaly, masses, and tenderness. (PC, MK)<br />

Search for stigmata <strong>of</strong> chronic liver disease. (PC, CS)<br />

Anal and rectal examination. (PC, CS)<br />

3. Differential diagnosis: Students should be able to generate a differential diagnosis<br />

recognizing specific history and physical examination findings that suggest a specific etiology<br />

for GI bleeding. (PC, MK)<br />

4. Laboratory interpretation: Students should be able to recommend when to order<br />

diagnostic and laboratory tests and be able to interpret them, both prior to and after initiating<br />

treatment based on the differential diagnosis, including consideration <strong>of</strong> test cost and<br />

performance characteristics as well as patient preferences.<br />

Laboratory and diagnostic tests should include, when appropriate:<br />

Stool and gastric fluid tests for occult blood. (MK, PC)<br />

CBC. (MK, PC)<br />

PT/PTT. (MK, PC)<br />

Hepatic function panel. (MK, PC)<br />

Tests for Helicobacter pylori. (MK, PC)<br />

Students should be able to define the indications for and interpret (with consultation) results<br />

<strong>of</strong>:<br />

Esophagogastroduodenoscopy (EGD). (MK, PC)<br />

Colonoscopy. (MK, PC)<br />

Barium studies <strong>of</strong> the gastrointestinal tract. (MK, PC)<br />

5. Communication skills: Students should be able to:<br />

Communicate the diagnosis, treatment plan, and subsequent follow-up to patients. (PC,<br />

CS)<br />

Elicit questions from the patient and his or her family about the management plan. (PC, CS)<br />

6. Basic and advanced procedural skills: Students should be able to:<br />

Start an IV line using a large bore (i.e. 18 gauge) needle. (MK, PC)<br />

Perform a stool or emesis occult blood testing. (MK, PC)<br />

7. Management skills: Students should be able to develop an appropriate evaluation and<br />

treatment plan for patients that includes:<br />

Establishing adequate venous access. (PC, MK)<br />

Administering crystalloid fluid resuscitation. (PC, MK)<br />

Ordering blood and blood product transfusion. (PC, MK)<br />

Determining when to obtain consultation from a gastroenterologist or a general surgeon.<br />

(PC, MK)<br />

Using a cost-effective approach based on the differential diagnosis. (PC, SBP)<br />

Accessing and utilizing appropriate information systems and resources to help delineate<br />

issues related to gastrointestinal bleeding. (PC, PLI)<br />

Incorporating patient preferences. (PC,P)<br />

Outlining long-term management when appropriate (e.g. Helicobacter pylori eradication,<br />

antacid, H-2 blocker or proton pump inhibitor therapy, smoking /alcohol cessation, NSAID<br />

107


estriction, and dietary modification. (MK, CS)<br />

C. ATTITUDES AND PROFESSIONAL BEHAVIORS: Students should be able to:<br />

1. Demonstrate commitment to using risk-benefit, cost-benefit, and evidence-based<br />

considerations in the selection <strong>of</strong> diagnostic and therapeutic interventions for gastrointestinal<br />

bleeding. (PLI, P)<br />

2. Respond appropriately to patients who are nonadherent to treatment for gastrointestinal<br />

bleeding. (CS, P)<br />

3. Demonstrate ongoing commitment to self-directed learning regarding gastrointestinal bleeding.<br />

(PLI, P)<br />

4. Appreciate the impact gastrointestinal bleeding has on a patient’s quality <strong>of</strong> life, well-being,<br />

ability to work, and the family. (P)<br />

5. Recognize the importance and demonstrate a commitment to the utilization <strong>of</strong><br />

6. other health care pr<strong>of</strong>essions in the treatment <strong>of</strong> gastrointestinal bleeding. (P, SBP)<br />

D. RESOURCES:<br />

Fallah, MA, Prakash, C, Edmundowicz, S. Acute gastrointestinal bleeding. Med Clin<br />

North Am. 2000;84:1183-208.<br />

Laine L. (2005). Gastrointestinal bleeding. In Kasper DL, Braunwald EB, Fauci AS,<br />

Hauser SL, Longo DL, Jameson JL, eds. Harrison’s Principles <strong>of</strong> Internal <strong>Medicine</strong>. 16 th ed.<br />

New York, NY: McGraw-Hill; 2005:235-8.<br />

Del Valle J. (2005). Peptic ulcer disease and related disorders. In Kasper DL,<br />

Braunwald EB, Fauci AS, Hauser SL, Longo DL, Jameson JL, eds. Harrison’s Principles <strong>of</strong><br />

Internal <strong>Medicine</strong>. 16 th ed. New York, NY: McGraw-Hill; 2005:1746-62.<br />

Gearhart SL, Bulkley G. (2005). Common diseases <strong>of</strong> the colon and anorectum and<br />

mesenteric vascular insufficiency. In Kasper DL, Braunwald EB, Fauci AS, Hauser SL, Longo<br />

DL, Jameson JL, eds. Harrison’s Principles <strong>of</strong> Internal <strong>Medicine</strong>. 16 th ed. New York, NY:<br />

McGraw-Hill; 2005:1795-803.<br />

Mitchell SH, Schaefer DC. A new view <strong>of</strong> occult and obscure gastrointestinal<br />

bleeding. Am Fam Physician. 2004;69:875-81.<br />

108


TRAINING PROBLEM #13: KNEE PAIN<br />

RATIONALE:<br />

Musculoskeletal complaints are some <strong>of</strong> the most common problems for which patients seek medical<br />

attention, and the knee is the single most common joint pain. Many <strong>of</strong> these problems can be<br />

effectively tackled in the primary care setting without need for consultation. The principles presented<br />

in this training problem can be readily applied to other joint pains.<br />

PREREQUISITES:<br />

Prior knowledge, skills, and attitudes acquired during the pre-<strong>clerkship</strong> experience should include:<br />

Ability to perform a complete medical history and physical exam.<br />

Ability to communicate with patients <strong>of</strong> diverse backgrounds.<br />

Anatomy and physiology <strong>of</strong> the musculoskeletal system.<br />

Pharmacology <strong>of</strong> acetaminophen, nonsteroidal anti-inflammatory drugs (NSAIDs), topical<br />

medications (capsaicin and lidocaine) and glucocorticoids.<br />

Basic bone radiograph interpretation.<br />

SPECIFIC LEARNING OBJECTIVES:<br />

A. KNOWLEDGE: Students should be able to define, describe, and discuss:<br />

11. A systematic approach to joint pain based on an understanding <strong>of</strong> pathophysiology to classify<br />

potential causes. (MK)<br />

12. The effect <strong>of</strong> the time course <strong>of</strong> symptoms on the potential causes <strong>of</strong> joint pain (acute vs.<br />

subacute vs. chronic). (MK)<br />

13. The difference between and pathophysiology <strong>of</strong> arthralgia vs. arthritis and mechanical vs.<br />

inflammatory joint pain. (MK)<br />

14. The distinguishing features <strong>of</strong> intra-articular and periarticular complaints (joint pain vs. bursitis<br />

and tendonitis). (MK)<br />

15. The effect <strong>of</strong> the features <strong>of</strong> joint involvement on the potential causes <strong>of</strong> joint pain<br />

(monoarticular vs. oligoarticular vs. polyarticular, symmetric vs. asymmetric, axial and/or<br />

appendicular, small vs. large joints, additive vs. migratory vs. intermittent). (MK)<br />

16. Indications for performing an arthrocentesis and the results <strong>of</strong> synovial fluid analysis. (MK)<br />

17. The utility <strong>of</strong> describing the relative location <strong>of</strong> knee pain (anterior, medial, lateral, posterior).<br />

(MK)<br />

18. The relative frequency <strong>of</strong> the various causes <strong>of</strong> knee pain. (MK)<br />

19. The differential diagnosis, pathophysiology, and typical presentations <strong>of</strong> the common intraarticular<br />

causes <strong>of</strong> knee pain:<br />

Osteoarthritis. (MK)<br />

Inflammatory arthropathies. (MK)<br />

Crystalline arthropathies. (MK)<br />

Septic arthritis. (MK)<br />

Patell<strong>of</strong>emoral pain syndrome. (MK)<br />

Cruciate ligament tear. (MK)<br />

Meniscal damage. (MK)<br />

10. The differential diagnosis, pathophysiology, and typical presentations <strong>of</strong> the common<br />

periarticular causes <strong>of</strong> knee pain:<br />

109


Collateral ligament sprain/tear. (MK)<br />

Ileotibial band syndrome. (MK)<br />

Prepatellar bursitis. (MK)<br />

Popliteal (Baker) cyst. (MK)<br />

11. Basic symptomatic treatment for knee pain, including:<br />

Relative rest. (MK)<br />

Ice/heat. (MK)<br />

Compression. (MK)<br />

Elevation. (MK)<br />

Acetaminophen. (MK)<br />

Nonsteroidal anti-inflammatory drugs. (MK)<br />

Glucosamine and chondroitin sulfate. (MK)<br />

Physical therapy. (MK)<br />

Assistive devices. (MK)<br />

Topical “analgesics.” (MK)<br />

Corticosteroid injection. (MK)<br />

12. Indications for and efficacy <strong>of</strong> intra-articular corticosteroid injections. (MK)<br />

B. SKILLS: Students should be able to demonstrate specific skills, including:<br />

1. History-taking skills: Students should be able to obtain, document, and present an ageappropriate<br />

medical history that differentiates among etiologies <strong>of</strong> disease, including:<br />

Delineation <strong>of</strong> the specific features <strong>of</strong> the pain. (PC, CS)<br />

Presence <strong>of</strong> stiffness, swelling, warmth, redness. (PC, CS)<br />

Symptoms <strong>of</strong> instability, locking, clicking/popping, and weakness. (PC, CS)<br />

History <strong>of</strong> trauma, new activities, repetitive motion. (PC, CS)<br />

Impact on the patient’s ability to carry out activities <strong>of</strong> daily living. (PC, CS)<br />

2. Physical exam skills: Students should be able to perform a physical exam to establish the<br />

diagnosis and severity <strong>of</strong> disease, including:<br />

Examination <strong>of</strong> the knee, including:<br />

o Inspection. (PC)<br />

o Palpation. (PC)<br />

o Range <strong>of</strong> motion. (PC)<br />

o Gait assessment. (PC)<br />

o Evaluation for effusion. (PC)<br />

o Assessment <strong>of</strong> ligamentous and cartilaginous stability. (PC)<br />

3 Differential diagnosis: Students should be able to generate a prioritized differential diagnosis<br />

recognizing specific history and physical exam findings that suggest a specific etiology for<br />

knee pain. (PC, MK)<br />

4. Laboratory interpretation: Students should be able to recommend when to order diagnostic<br />

and laboratory tests and be able to interpret them, both prior to and after initiating treatment<br />

based on the differential diagnosis, including consideration <strong>of</strong> test cost and performance<br />

characteristics as well as patient preferences.<br />

Laboratory and diagnostic tests should include, when appropriate:<br />

Synovial fluid analysis. (PC, MK)<br />

Students should be able to define the indications for and interpret (with consultation) the<br />

results <strong>of</strong>:<br />

Plain radiographs <strong>of</strong> the knee. (PC, MK)<br />

CT and MRI <strong>of</strong> the knee. (PC, MK)<br />

110


5. Communication skills: Students should be able to:<br />

Communicate the diagnosis, treatment plan, and subsequent follow-up to patients. (PC,<br />

CS)<br />

Elicit questions from the patient and his or her family about the management plan. (PC, CS)<br />

6. Basic and advanced procedure skills: Students should be able to:<br />

Assist in the performance <strong>of</strong> an arthrocentesis and intra-articular corticosteroid injection.<br />

(PC)<br />

7. Management skills: Students should able to develop an appropriate evaluation and treatment<br />

plan for patients that includes:<br />

Determining when to perform an arthrocentesis. (PC, MK)<br />

Prescribing simple, nonmedicinal symptomatic measures such as rest, ice/heat,<br />

compression, and elevation. (PC, MK)<br />

Prescribing physical therapy and assistive devices (PC, MK)<br />

Prescribing exercise. (PC, MK)<br />

Counseling patients regarding weight loss. (PC, MK)<br />

Prescribing non-narcotic analgesics and anti-inflammatory agents. (PC, MK)<br />

Determining when to prescribe narcotic analgesics. (PC, MK)<br />

Determining when to prescribe intra-articular corticosteroid injection. (PC, MK)<br />

Determining when to obtain consultation from an orthopedic surgeon and rheumatologist.<br />

(PC, MK)<br />

Using a cost-effective approach based on the differential diagnosis. (PC, SBP)<br />

Accessing and utilizing appropriate information systems and resources to help delineate<br />

issues related to knee pain. (PC, PLI)<br />

Incorporating patient preferences. (PC, P)<br />

C. ATTITUDES AND PROFESSIONAL BEHAVIORS: Students should be able to:<br />

1. Demonstrate commitment to using risk-benefit, cost-benefit, and evidence-based<br />

considerations in the selection diagnostic and therapeutic interventions for knee pain. (PLI, P)<br />

2. Recognize the importance <strong>of</strong> patient preferences when selecting among diagnostic<br />

and therapeutic options for knee pain. (P)<br />

3. Respond appropriately to patients who are nonadherent to treatment for knee pain.<br />

(CS, P)<br />

4. Appreciate the impact chronic knee pain has on a patient’s quality <strong>of</strong> life,<br />

psychological well-being, ability to work, and the family. (P)<br />

5. Recognize the importance <strong>of</strong> and demonstrate a commitment to the utilization <strong>of</strong><br />

other health care pr<strong>of</strong>essions in the treatment <strong>of</strong> knee pain. (P, SBP)<br />

6. Appreciate the difficulty patients with limited mobility have in achieving weight loss.<br />

(P)<br />

7. Demonstrate an appropriate attitude in managing patients with chronic pain. (P)<br />

D. REFERENCES:<br />

Cush JJ, Lipsky PE. (2005). Approach to articular and musculoskeletal disorders. In<br />

Kasper DL, Braunwald EB, Fauci AS, Hauser SL, Longo DL, Jameson JL, eds.<br />

111


Harrison’s Principles <strong>of</strong> Internal <strong>Medicine</strong>. 16 th ed. New York, NY: McGraw-Hill;<br />

2005:2029-36.<br />

Brandt KD. (2005). Osteoarthritis. In Kasper DL, Braunwald EB, Fauci AS, Hauser SL,<br />

Longo DL, Jameson JL, eds. Harrison’s Principles <strong>of</strong> Internal <strong>Medicine</strong>. 16 th ed. New<br />

York, NY: McGraw-Hill; 2005:2036-45.<br />

Principles <strong>of</strong> diagnosis and management: Pattern recognition in arthritis. In Klippel JH,<br />

Dieppe PA, Ferri FF, eds. Primary Care Rheumatology. St. Louis, MO: Mosby; 1999:11-<br />

7.<br />

Regional pain and monoarticular disorders: Pain in the knee. (1999). In Klippel JH,<br />

Dieppe PA, Ferri FF, eds. Primary Care Rheumatology. St. Louis, MO: Mosby; 1999:99-<br />

107.<br />

DeHaven KE. Knee and lower leg. In Greene WB, ed. Essentials <strong>of</strong> Musculoskeletal<br />

Care. 2 nd ed. Rosemont, IL: American Academy <strong>of</strong> Orthopedic Surgeons; 2001:341-405.<br />

112


TRAINING PROBLEM #14: RASH<br />

RATIONALE:<br />

Rash is an extremely common complaint. It may be the manifestation <strong>of</strong> a primary cutaneous<br />

disorder or secondary to a systemic condition. Internists see many patients with both and,<br />

therefore, must be acquainted with the diagnosis and management.<br />

PREREQUISITES:<br />

Prior knowledge, skills, and attitudes acquired during the pre-<strong>clerkship</strong> experience should include:<br />

Ability to perform a complete medical history and physical exam.<br />

Ability to communicate with patients <strong>of</strong> diverse backgrounds.<br />

Anatomy, physiology, and pathophysiology <strong>of</strong> the skin.<br />

Pharmacology <strong>of</strong> glucocorticoids, antifungals, antibiotics, benzoyl peroxide, salicylic acid, and<br />

retinoids and derivatives.<br />

SPECIFIC LEARNING OBJECTIVES:<br />

A. KNOWLEDGE: Students should be able to define, describe, and discuss:<br />

3. The standard nomenclature used to describe rashes (macule, patch, papule, nodule, plaque,<br />

vesicle, pustule, bulla, cyst, wheal, telangiectasia, petechia, purpura, erosion, ulcer). (MK)<br />

4. The morphologic features used to describe potentially malignant skin lesions (Asymmetry,<br />

Border, Color, Diameter, Dynamic i.e. changing, Elevation, and Enlargement, “ABCDE”). (MK)<br />

5. The significance <strong>of</strong> focal, organ-based, and constitutional signs and symptoms in the context <strong>of</strong><br />

a rash (e.g. rash and fever, rash and arthritis, rash and renal failure). (MK)<br />

4. The differential diagnosis, pathophysiology, and typical presentations <strong>of</strong> the common causes <strong>of</strong><br />

eczematous dermatoses:<br />

Atopic dermatitis. (MK)<br />

Contact dermatitis. (MK)<br />

Stasis dermatitis. (MK)<br />

Seborrheic dermatitis. (MK)<br />

5. The differential diagnosis, pathophysiology, and typical presentations <strong>of</strong> the common causes <strong>of</strong><br />

maculopapular eruptions:<br />

viral exanthems. (MK)<br />

bacterial exanthems. (MK)<br />

erythema multiforme. (MK)<br />

6. The differential diagnosis, pathophysiology, and typical presentations <strong>of</strong> the common causes <strong>of</strong><br />

papulosquamous dermatoses:<br />

Psoriasis. (MK)<br />

Pityriasis rosea. (MK)<br />

7. The differential diagnosis, pathophysiology, and typical presentations <strong>of</strong> the common causes <strong>of</strong><br />

cutaneous infections:<br />

Impetigo. (MK)<br />

Cellulitis. (MK)<br />

Folliculitis. (MK)<br />

Dermatophytosis (tinea corporis, tinea capitis, tinea cruris, tinea pedis, onychomycosis).<br />

(MK)<br />

113


Tinea versicolor. (MK)<br />

Candidiasis. (MK)<br />

Condylomata. (MK)<br />

Herpes zoster. (MK)<br />

8. The prevention <strong>of</strong> community acquisition <strong>of</strong> Methicillin-resistant Staphylococcus aureus<br />

(MRSA), including good hygiene practices:<br />

Keeping hands clean by washing thoroughly with soap and water or using an alcohol-based<br />

sanitizer. (MK)<br />

Keeping cuts and scrapes clean and covered with a bandage until healed. (MK)<br />

Avoiding contact with other people’s wounds or bandages. (MK)<br />

Avoiding sharing personal items such as towels and razors. (MK)<br />

9. The differential diagnosis, pathophysiology, and typical presentations <strong>of</strong> the common causes <strong>of</strong><br />

pustular diseases:<br />

Acne. (MK)<br />

Rosacea. (MK)<br />

10. The differential diagnosis, pathophysiology, and typical presentations <strong>of</strong> the common causes <strong>of</strong><br />

cutaneous ulcers.<br />

Venous insufficiency. (MK)<br />

Peripheral arterial disease. (MK)<br />

Neuropathic. (MK)<br />

11. The significance <strong>of</strong> palpable purpura and other cutaneous findings <strong>of</strong> vasculitis. (MK)<br />

12. The differential diagnosis, pathophysiology, and typical presentations <strong>of</strong> the common causes <strong>of</strong><br />

urticaria and angioedema. (MK)<br />

13. The differential diagnosis, pathophysiology, and typical presentations <strong>of</strong> drug eruptions. (MK)<br />

14. The differential diagnosis, pathophysiology, and typical presentations <strong>of</strong> the common causes <strong>of</strong><br />

benign neoplasms and hyperplasias:<br />

Seborrheic keratosis. (MK)<br />

Epidermoid cyst. (MK)<br />

15. The differential diagnosis, pathophysiology, and typical presentations <strong>of</strong> the common causes <strong>of</strong><br />

premalignant lesions and malignancies:<br />

Actinic keratosis. (MK)<br />

Basal cell carcinoma. (MK)<br />

Squamous cell carcinoma. (MK)<br />

Malignant melanoma. (MK)<br />

16. The differential diagnosis, pathophysiology, and typical presentations <strong>of</strong> the cutaneous<br />

manifestations <strong>of</strong> sexually transmitted diseases.<br />

Syphilis. (MK)<br />

Disseminated gonorrhea infection. (MK)<br />

Human papilloma virus. (MK)<br />

Herpes simplex virus. (MK)<br />

17. The differential diagnosis, pathophysiology, and typical presentations <strong>of</strong> the cutaneous<br />

manifestations <strong>of</strong> internal/systemic diseases. (MK)<br />

18. The general indications for skin biopsy. (MK)<br />

B. SKILLS: Students should be able to demonstrate specific skills, including:<br />

1. History-taking skills: Students should be able to obtain, document, and present an ageappropriate<br />

medical history that differentiates among etiologies <strong>of</strong> disease, including:<br />

Evolution (site <strong>of</strong> onset, manner <strong>of</strong> spread, duration). (PC, CS)<br />

114


Symptoms associated with the rash (pruritis, pain, photosensitivity, malaise, fever,<br />

arthralgias). (PC, CS)<br />

Past medical history <strong>of</strong> systemic diseases known to have cutaneous manifestation. (PC,<br />

CS)<br />

Sexual history. (PC, CS)<br />

Medication usage and allergies. (PC, CS)<br />

Skin care product usage. (PC, CS)<br />

Chemical skin exposure. (PC, CS)<br />

Sun exposure. (PC, CS)<br />

Travel history. (PC, CS)<br />

2. Physical exam skills: Students should be able to perform a physical exam to establish the<br />

diagnosis and severity <strong>of</strong> disease including:<br />

Description <strong>of</strong> the type <strong>of</strong> primary skin lesion (macule, patch, papule, nodule, plaque,<br />

vesicle, pustule, bulla, cyst, wheal, telangiectasia, petechia, purpura, erosion, ulcer). (PC)<br />

Description <strong>of</strong> the shape, margination, color, arrangement, and distribution <strong>of</strong> the individual<br />

lesions. (PC)<br />

Describe potentially malignant lesions in terms <strong>of</strong> Asymmetry, Border, Color, Diameter,<br />

Elevation, and Enlargement (“ABCDE”). (PC)<br />

Presence <strong>of</strong> exudates: dry (crust) or wet (weeping) exudates. (PC)<br />

Presence <strong>of</strong> scale or lichenification. (PC)<br />

Palpation <strong>of</strong> lesions for consistency, alteration <strong>of</strong> temperature, mobility, and tenderness.<br />

(PC)<br />

3. Differential diagnosis: Students should be able to generate a prioritized differential<br />

diagnosis recognizing specific history and physical exam findings that suggest a specific<br />

etiology for a rash. (PC, MK)<br />

4. Laboratory interpretation: Students should be able to recommend when to order<br />

diagnostic and laboratory tests and be able to interpret them, both prior to and after initiating<br />

treatment based on the differential diagnosis, including consideration <strong>of</strong> test cost and<br />

performance characteristics as well as patient preferences.<br />

Laboratory and diagnostic tests should include, when appropriate:<br />

KOH preparation. (PC, MK)<br />

CBC with differential. (PC, MK)<br />

RPR and VDRL. (PC, MK)<br />

Bacterial culture. (PC, MK)<br />

Students should be able to define the indications for and interpret (with consultation) the<br />

significance <strong>of</strong> the results <strong>of</strong>:<br />

• Skin biopsy. (PC, MK)<br />

5. Communication skills: Students should be able to:<br />

Explain the dangers <strong>of</strong> excess sun exposure. (PC, CS)<br />

Communicate the diagnosis, treatment plan, and subsequent follow-up to the patient and<br />

his or her family. (PC, CS)<br />

Elicit questions from the patient and his or her family about the management plan. (PC, CS)<br />

Counsel patients regarding the prevention <strong>of</strong> community acquisition <strong>of</strong> MRSA. (PC, CS)<br />

6. Basic and advanced procedural skills: Students should be able to:<br />

Perform a skin scraping and KOH preparation. (PC)<br />

7. Management skills: Students should able to develop an appropriate evaluation and<br />

treatment plan for patients that includes:<br />

Determining when to perform a skin scraping and KOH preparation. (MK, PC)<br />

Determining when to obtain tests appropriate for the diagnosis <strong>of</strong> systemic medical<br />

115


conditions suspected as the cause <strong>of</strong> rash. (MK, PC)<br />

Prescribing a simple hypoallergenic skin care regimen. (MK, PC)<br />

Prescribing appropriate moisturizing/emollient treatment. (MK, PC)<br />

Discussing the importance <strong>of</strong> and prescribing sunscreen use. (PC, MK, CS)<br />

Prescribing appropriate treatment for eczematous dermatoses, mild psoriasis, common<br />

cutaneous skin infections, acne, rosacea, venous stasis dermatitis and ulcers, and common<br />

drug eruptions. (PC, MK)<br />

Determining when to obtain a consultation from a dermatologist. (PC)<br />

Using a cost-effective approach based on the differential diagnosis. (PC, SBP)<br />

Accessing and utilizing appropriate information systems and resources to help delineate<br />

issues related to common dermatologic complaints. (PC, PLI)<br />

Incorporating patient preferences. (PC, P)<br />

C. ATTITUDES AND PROFESSIONAL BEHAVIORS: Students should be able to:<br />

3. Demonstrate commitment to using risk-benefit, cost-benefit, and evidence-based<br />

considerations in the selection diagnostic and therapeutic interventions for rashes. (PLI, P)<br />

4. Recognize the importance <strong>of</strong> patient preferences when selecting among diagnostic and<br />

therapeutic options for rashes. (P)<br />

5. Appreciate the impact rashes have on a patient’s quality <strong>of</strong> life, well-being, ability to work,<br />

and the family. (P)<br />

D. REFERENCES:<br />

Drage LA. Life-threatening rashes: dermatologic signs <strong>of</strong> four infectious diseases. Mayo<br />

Clin Proc. 199;74:68-72.<br />

Williams HC. Clinical practice. Atopic dermatitis. N Engl J Med. 2005;352:2314-24.<br />

Abbasi NR. Shaw HM. Rigel DS. Friedman RJ. McCarthy WH. Osman I. Kopf AW.<br />

Polsky D. Early diagnosis <strong>of</strong> cutaneous melanoma: revisiting the ABCD criteria. JAMA.<br />

2004;292:2771-6.<br />

James WD. Clinical practice. Acne. N Engl J Med. 2005;352:1463-72.<br />

Wolff K, Johnson AJ, Suurmond R. Fitzpatrick’s Color Atlas & Synopsis <strong>of</strong> Clinical<br />

Dermatology. 4 th ed. New York, NY; 2005.<br />

American Academy <strong>of</strong> Dermatology<br />

Medical Student Core Curriculum<br />

www.aad.org/pr<strong>of</strong>essionals/Residents/MedStudCoreCurr/MedStudCoreC urr.htm<br />

Community-Associated MRSA<br />

Division <strong>of</strong> Healthcare Quality Promotion<br />

National Center for Infectious Diseases<br />

Centers for Disease Control and Prevention<br />

U.S. <strong>Department</strong> <strong>of</strong> Health and Human Services<br />

www.cdc.gov/ncidod/dhqp/ar_mrsa_ca.html<br />

116


TRAINING PROBLEM #15: UPPER RESPIRATORY COMPLAINTS<br />

RATIONALE:<br />

Upper respiratory tract infections (URIs) are some <strong>of</strong> the most common problems for which patients<br />

seek medical attention. Many patients inappropriately receive antibiotic therapy for these mostly<br />

viral infections.<br />

PREREQUISITES:<br />

Prior knowledge, skills, and attitudes acquired during the pre-<strong>clerkship</strong> experience should include:<br />

Ability to perform a complete medical history and physical exam.<br />

Ability to communicate with patients <strong>of</strong> diverse backgrounds.<br />

Anatomy and physiology <strong>of</strong> the upper airway, Eustachian tubes, and sinuses.<br />

Anatomy and physiology <strong>of</strong> the respiratory system.<br />

Pathogenesis and pathophysiology <strong>of</strong> upper respiratory tract diseases.<br />

Microbial pathogens associated with upper respiratory tract infections.<br />

Pharmacology <strong>of</strong> antibiotics.<br />

SPECIFIC LEARNING OBJECTIVES:<br />

A. KNOWLEDGE: Students should be able to define, describe, and discuss:<br />

1. A rational approach to the common URIs: nasal congestion, rhinorrhea, facial pain/tenderness,<br />

cough, sputum production, sore throat, and ear pain. (MK)<br />

2. Common constitutional symptoms that accompany URIs: generalized weakness, fatigue,<br />

malaise, headache, mild myalgias, and modest fever. (MK)<br />

3. The microbiology <strong>of</strong> URIs, highlighting the relative frequencies <strong>of</strong> viral and bacterial etiologies.<br />

(MK)<br />

4. The most common microbiologic agents that cause the common URIs. (MK)<br />

5. The pathophysiology and typical clinical presentation <strong>of</strong> the common URIs:<br />

Common cold. (MK)<br />

Acute bronchitis. (MK)<br />

Pharyngitis. (MK)<br />

Acute sinusitis. (MK)<br />

Otitis media. (MK)<br />

6. The pathophysiologic similarities between the common cold and acute sinusitis. (MK)<br />

7. The clinical features and microbiology <strong>of</strong> acute compared to chronic sinusitis. (MK)<br />

8. The pathophysiology and symptomatology <strong>of</strong> allergic rhinitis and the clinical features that may<br />

help differentiate it from the common cold and acute sinusitis. (MK)<br />

9. The clinical features that may help differentiate the common URIs from influenza. (MK)<br />

10. The pathophysiology and clinical features <strong>of</strong> acute compared to chronic bronchitis. (MK)<br />

11. The pathophysiology and clinical features <strong>of</strong> acute bronchitis compared to pneumonia. (MK)<br />

12. The pathophysiology and clinical features <strong>of</strong> otitis media and Eustachian tube malfunction.<br />

(MK)<br />

13. The signs and symptoms that may help distinguish viral from bacterial pharyngitis. (MK)<br />

14. Symptomatic treatment for URIs and the major side effects/contraindications for these<br />

treatments, including:<br />

Decongestants. (MK)<br />

Non-selective antihistamines. (MK)<br />

117


Mucolytics. (MK)<br />

Cough suppressants. (MK)<br />

Pain relievers/fever reducers. (MK)<br />

15. The general role <strong>of</strong> antibiotics in the treatment <strong>of</strong> URIs and specific evidence-based indications<br />

for them. (MK)<br />

16. The basic elements <strong>of</strong> the treatment <strong>of</strong> allergic rhinitis. (MK)<br />

17. The use <strong>of</strong> antiviral agents in the prophylaxis and treatment <strong>of</strong> influenza. (MK)<br />

B. SKILLS: Students should be able to demonstrate specific skills, including:<br />

1. History-taking skills: Students should be able to obtain, document, and present an ageappropriate<br />

medical history, that differentiates among etiologies <strong>of</strong> disease, including:<br />

The predominant symptom (nasal congestion/rhinorrhea, purulent nasal discharge with<br />

facial pain/tenderness, sore throat, cough with or without sputum, sore throat or ear pain).<br />

(PC, CS)<br />

Constitutional symptoms. (PC, CS)<br />

Symptoms <strong>of</strong> potential pneumonia. (PC, CS)<br />

History <strong>of</strong> or symptoms <strong>of</strong> serious cardiopulmonary diseases (e.g. asthma, chronic<br />

obstructive pulmonary disease, congestive heart failure) that may alter the treatment plan.<br />

(PC, CS)<br />

2. Physical exam skills: Students should be able to perform a physical exam to establish the<br />

diagnosis and severity <strong>of</strong> disease, including:<br />

Examination <strong>of</strong> the nasal cavity, pharynx, and sinuses. (PC)<br />

Otoscopic examination. (PC)<br />

Evaluation <strong>of</strong> the head and neck for lymphadenopathy. (PC)<br />

Auscultation <strong>of</strong> the lungs to distinguish pulmonary consolidation, pleural effusion,<br />

pulmonary congestion, and chronic obstructive pulmonary disease. (PC)<br />

3. Differential diagnosis: Students should be able to generate a prioritized differential diagnosis<br />

recognizing specific history and physical exam findings that suggest a specific etiology <strong>of</strong><br />

upper respiratory complaints:<br />

Common cold. (PC, MK)<br />

Acute sinusitis. (PC, MK)<br />

Chronic sinusitis. (PC, MK)<br />

Allergic rhinitis. (PC, MK)<br />

Pharyngitis. (PC, MK)<br />

Otitis media. (PC, MK)<br />

Otitis externa. (PC, MK)<br />

Acute bronchitis. (PC, MK)<br />

Chronic bronchitis. (PC, MK)<br />

Influenza. (PC, MK)<br />

Pneumonia. (PC, MK)<br />

Infectious mononucleosis. (PC, MK)<br />

4. Laboratory interpretation: Students should be able to recommend when to order diagnostic<br />

and laboratory tests and be able to interpret them, both prior to and after initiating treatment<br />

based on the differential diagnosis, including consideration <strong>of</strong> test cost and performance<br />

characteristics as well as patient preferences Laboratory and diagnostic tests should include,<br />

when appropriate:<br />

CBC with differential. (PC)<br />

Rapid strep test. (PC)<br />

118


Throat culture. (PC)<br />

Chest radiograph. (PC)<br />

PFTs. (PC)<br />

Monospot/heterophile antibody. (PC)<br />

5. Communication skills: Students should be able to:<br />

Communicate the diagnosis, treatment plan, and subsequent follow-up to the patient and<br />

his or her family. (CS)<br />

Elicit questions from the patient and his or her family about the management plan. (CS)<br />

Explain the microbiologic origin <strong>of</strong> most URIs and why antibiotics are generally ineffective.<br />

(CS)<br />

Explain the importance <strong>of</strong> antimicrobial resistance. (CS)<br />

6. Basic and advanced procedure skills:<br />

• Throat culture. (PC)<br />

7. Management skills: Students should able to develop an appropriate evaluation and<br />

treatment plan for patients that includes:<br />

Determining when to obtain a chest radiograph. (PC, MK)<br />

Determining when to prescribe antibiotics. (PC, MK)<br />

Selecting the most appropriate antibiotic for acute bacterial sinusitis, streptococcal<br />

pharyngitis, and bacterial otitis media. (PC, MK)<br />

Prescribing symptomatic treatments. (PC, MK)<br />

Determining when to obtain consultation from an allergist, otolaryngologist, or<br />

pulmonologist. (PC, SBP)<br />

Using a cost-effective approach based on the differential diagnosis. (PC, SBP)<br />

Accessing and utilizing appropriate information systems and resources to help delineate<br />

issues related to URIs. (PC, PLI)<br />

Incorporating patient preferences. (PC, P)<br />

C. ATTITUDES AND PROFESSIONAL BEHAVIORS: Students should be able to:<br />

1 Demonstrate commitment to using risk-benefit, cost-benefit, and evidence-based<br />

considerations in the selection diagnostic and therapeutic interventions for common URI complaints.<br />

(P, PLI)<br />

2 Appreciate the impact common URI complaints have on a patient’s quality <strong>of</strong> life, well-being,<br />

ability to work, and the family. (P)<br />

3 Discuss the patient’s perspective regarding the use <strong>of</strong> antibiotics for URIs. (CS, P)<br />

4 Discuss the role physicians play in the over-prescribing <strong>of</strong> antibiotics for URIs. (P)<br />

5 Discuss the importance <strong>of</strong> antimicrobial resistance from the point <strong>of</strong> view <strong>of</strong> the individual and<br />

society at large. (P)<br />

D. REFERENCES:<br />

Arroll B, Kenealy T. Antibiotics for the common cold and acute purulent rhinitis.<br />

Cochrane Database Syst Rev. 2005;(3):CD000247.<br />

Bisno AL. Acute pharyngitis. N Engl J Med. 2001;344:205-11.<br />

Del Mar CB, Glasziou PP, Spinks AB. Antibiotics for sore throat. Cochrane Database<br />

Syst Rev. 2004;(2):CD000023.<br />

Metlay JP, Kapoor WN, Fine MJ. Does this patient have community-acquired<br />

pneumonia? Diagnosing pneumonia by history and physical examination. JAMA.<br />

1997;278:1440-5.<br />

119


Smucny J, Fahey T, Becker L, et al. Antibiotics for acute bronchitis. Cochrane<br />

Database Syst Rev. 2004;(4):CD000245.<br />

Snow V, Mottur-Pilson C, Gonzales R, et al. Principles <strong>of</strong> appropriate antibiotic use for<br />

treatment <strong>of</strong> acute bronchitis in adults. Ann Intern Med. 2001;134:518-20.<br />

Guidelines for the Control <strong>of</strong> Pertussis Outbreaks<br />

National Immunization Program<br />

Centers for Disease Control and Prevention<br />

U.S. <strong>Department</strong> <strong>of</strong> Health and Human Services<br />

www.cdc.gov/nip/publications/pertussis/guide.htm<br />

Get Smart. Know When Antibiotics Work<br />

National Campaign for Appropriate Antibiotic Use<br />

Division <strong>of</strong> Bacterial and Mycotic Diseases<br />

National Center for Infectious Diseases<br />

Centers for Disease Control and Prevention<br />

U.S. <strong>Department</strong> <strong>of</strong> Health and Human Services<br />

www.cdc.gov/drugresistance/community/<br />

120


TRAINING PROBLEM #16: ACUTE MYOCARDIAL INFARCTION<br />

RATIONALE:<br />

Cardiovascular disease is the number one killer <strong>of</strong> Americans. Many associated risk factors are<br />

quite modifiable. Proper urgent management <strong>of</strong> acute myocardial infarctions significantly reduces<br />

mortality.<br />

PREREQUISITES:<br />

Prior knowledge, skills, and attitudes acquired during the pre-clinical experience should include:<br />

Ability to perform a complete medical history and physical.<br />

Ability to communicate with patients <strong>of</strong> diverse backgrounds.<br />

Anatomy and physiology <strong>of</strong> the heart and coronary vessels.<br />

Risk factors for and pathogenesis/pathophysiology <strong>of</strong> atherosclerosis.<br />

Pharmacology <strong>of</strong> aspirin, morphine, nitroglycerine, heparin, antiplatelet agents, thrombolytic<br />

agents, beta-blockers, angiotensin converting enzyme inhibitors (ACE-I), angiotensin II<br />

receptor blockers (ARB), and HMG-CoA reductase inhibitors.<br />

SPECIFIC LEARNING OBJECTIVES:<br />

A. KNOWLEDGE: Students should be able to define, describe, and discuss:<br />

1. The primary and secondary prevention <strong>of</strong> ischemic heart disease through the reduction <strong>of</strong><br />

cardiovascular risk factors (e.g. controlling hypertension and dyslipidemia, aggressive diabetes<br />

management, avoiding tobacco, and aspirin prophylaxis). (MK)<br />

2. The basic principles <strong>of</strong> the role <strong>of</strong> genetics in CAD. (MK)<br />

3. Pathogenesis, signs, and symptoms <strong>of</strong> the acute coronary syndromes:<br />

Unstable angina. (MK)<br />

Non-ST-elevation myocardial infarction (NSTEMI). (MK)<br />

ST-elevation myocardial infarction (STEMI). (MK)<br />

4. Atypical presentations <strong>of</strong> cardiac ischemia/infraction. (MK)<br />

5. The typical clinical course <strong>of</strong> the acute coronary syndromes. (MK)<br />

6. ECG findings and macromolecular markers (myoglobin, CK-MB, Troponin-I, Troponin-T) <strong>of</strong><br />

acute ischemia/MI. (MK)<br />

7. The utility <strong>of</strong> echocardiography in acute MI. (MK)<br />

8. The importance <strong>of</strong> monitoring for and immediate treatment <strong>of</strong> ventricular fibrillation in acute MI.<br />

(MK)<br />

9. Therapeutic options for acute MI and how they may differ for NSTEMI and STEMI, including:<br />

Aspirin. (MK)<br />

Morphine. (MK)<br />

Nitroglycerine. (MK)<br />

Oxygen. (MK)<br />

Heparin. (MK)<br />

Antiplatelet agents (glycoprotein IIb/IIIa inhibitors). (MK)<br />

Beta-blockers. (MK)<br />

ACE-I/ARB. (MK)<br />

HMG-CoA reductase inhibitors. (MK)<br />

Thrombolytic agents. (MK)<br />

121


Emergent cardiac catheterization with percutaneous coronary intervention. (MK)<br />

10. Pathogenesis, signs, and symptoms <strong>of</strong> the complications <strong>of</strong> acute MI, including arrhythmias,<br />

reduced ventricular function, cardiogenic shock, pericarditis, papillary muscle<br />

dysfunction/rupture, acute valvular dysfunction, and cardiac free wall rupture. (MK)<br />

11. The general approach to the evaluation and treatment <strong>of</strong> ventricular tachycardia and fibrillation.<br />

(MK)<br />

12. The importance <strong>of</strong> post-MI risk stratification, including the burden <strong>of</strong> residual coronary disease<br />

and assessment <strong>of</strong> left ventricular function. (MK)<br />

13. Basic principles <strong>of</strong> cardiac rehabilitation. (MK)<br />

14. Indications for coronary artery bypass grafting (CABG). (MK)<br />

15. The Centers for Medicare & Medicaid Services (CMS) and the Joint Commission on the<br />

Accreditation <strong>of</strong> Healthcare Organizations (JCAHO) quality measures for acute MI treatment.<br />

(MK, PLI, SBP)<br />

B. SKILLS: Students should be able to demonstrate specific skills, including:<br />

1. History-taking skills: Students should be able to obtain, document, and present an ageappropriate<br />

medical history that differentiates among etiologies <strong>of</strong> disease, including:<br />

Cardiac risk factors. (PC, CS)<br />

Location, duration, intensity, exacerbating/ameliorating factors, radiation <strong>of</strong> chest pain. (PC,<br />

CS)<br />

Symptoms associated with chest pain (e.g. nausea, emesis, dyspnea, diaphoresis,<br />

palpitations, dizziness, syncope, heartburn belching, etc.). (PC, CS)<br />

2. Physical exam skills: Students should be able to perform a physical exam to establish the<br />

diagnosis and severity <strong>of</strong> disease including:<br />

Recognition <strong>of</strong> dyspnea and anxiety. (PC)<br />

Accurate measurement <strong>of</strong> vital signs. (PC)<br />

Examination <strong>of</strong> the heart and vascular system. (PC)<br />

Examination <strong>of</strong> the lungs. (PC)<br />

3. Differential diagnosis: Students should be able to generate a prioritized differential<br />

diagnosis recognizing specific history and physical exam findings that suggest a specific<br />

etiology <strong>of</strong> chest pain:<br />

Stable angina. (PC, MK)<br />

Coronary vasospasm. (PC, MK)<br />

Unstable angina. (PC, MK)<br />

Acute MI. (PC, MK)<br />

Pericarditis. (PC, MK)<br />

Aortic dissection. (PC, MK)<br />

Pulmonary embolism. (PC, MK)<br />

Other noncardiac causes <strong>of</strong> chest pain. (PC, MK)<br />

4. Laboratory interpretation: Students should be able to recommend when to order diagnostic<br />

and laboratory tests and be able to interpret them, both prior to and after initiating treatment<br />

based on the differential diagnosis, including consideration <strong>of</strong> test cost and performance<br />

characteristics as well as patient preferences<br />

Laboratory and diagnostic tests should include, when appropriate:<br />

ECG. (PC, MK)<br />

Chest radiograph. (PC, MK)<br />

Macromolecular markers (myoglobin, CK-MB, Troponin-I, Troponin- T). (PC, MK)<br />

Students should be able to define the indications for and interpret (with consultation) the<br />

122


esults <strong>of</strong>:<br />

Echocardiogram. (PC, MK)<br />

Cardiac stress testing. (PC, MK)<br />

Coronary angiography. (PC, MK)<br />

5. Communication skills: Students should be able to:<br />

Communicate the diagnosis, treatment plan, and subsequent follow-up to patients. (PC,<br />

CS)<br />

Elicit questions from the patient and his or her family about the diagnostic and management<br />

plan. (PC, CS)<br />

Educate patients about modifying cardiac risk factors. (PC, CS)<br />

6. Management skills: Students should able to develop an appropriate evaluation and<br />

treatment plan for patients that includes:<br />

Medical management <strong>of</strong> acute MI. (PC, MK)<br />

CCU monitoring. (PC, MK)<br />

Indications for and complications <strong>of</strong> thrombolytic therapy, cardiac catheterization with<br />

percutaneous coronary intervention, and CABG. (PC, MK)<br />

Proper pre-discharge risk stratification. (PC, MK)<br />

Secondary risk factor modification. (PC, MK)<br />

Determining when to obtain consultation from a cardiologist and cardiothoracic surgeon.<br />

(PC, SBP)<br />

Using a cost-effective approach based on the differential diagnosis. (PC, SBP)<br />

Accessing and utilizing appropriate information systems and resources to help delineate<br />

issues related to acute MI. (PC, PLI)<br />

Incorporating patient preferences. (PC, P)<br />

C. ATTITUDES AND PROFESSIONAL BEHAVIORS: Students should be able to:<br />

1. Demonstrate a commitment to meeting national quality standards for the care <strong>of</strong><br />

patient with acute MI. (P, PLI, SBP)<br />

2. Demonstrate commitment to using risk-benefit, cost-benefit, and evidence-based<br />

considerations in the selection diagnostic and therapeutic interventions for acute MI. (PLI, P)<br />

3. Recognize the importance <strong>of</strong> patient preferences when selecting among diagnostic<br />

and therapeutic options for acute MI. (P)<br />

4. Demonstrate ongoing commitment to self-directed learning regarding acute MI. (PLI,<br />

P)<br />

5. Appreciate the impact acute MI has on a patient’s quality <strong>of</strong> life, well-being, ability to<br />

work, and the family. (P)<br />

6. Recognize the importance <strong>of</strong> and demonstrate a commitment to the utilization <strong>of</strong><br />

other healthcare pr<strong>of</strong>essionals in the treatment <strong>of</strong> acute MI. (P, SBP)<br />

D. REFERENCES:<br />

ACC/AHA 2002 guideline update for the management <strong>of</strong> patients with chronic stable<br />

angina--summary article: a report <strong>of</strong> the American College <strong>of</strong> Cardiology/American<br />

Heart Association Task Force on practice guidelines (Committee on the Management<br />

<strong>of</strong> Patients with Chronic Stable Angina). J Am Coll Cardiol 2003; 41:159-68.<br />

www.acc.org/clinical/topic/topic.htm#guidelines<br />

ACC/AHA 2002 guideline update for the management <strong>of</strong> patients with unstable<br />

angina and non-ST-segment elevation myocardial infarction-summary article: a report<br />

123


<strong>of</strong> the American College <strong>of</strong> Cardiology/American Heart Association task force on<br />

practice guidelines (Committee on the Management <strong>of</strong> Patients With Unstable<br />

Angina). J Am Coll Cardiol. 2002;40:1366-74.<br />

www.acc.org/clinical/topic/topic.htm#guidelines<br />

ACC/AHA guidelines for the management <strong>of</strong> patients with ST-elevation myocardial<br />

infarction--executive summary. A report <strong>of</strong> the American College <strong>of</strong><br />

Cardiology/American Heart Association Task Force on Practice Guidelines (Writing<br />

Committee to revise the 1999 guidelines for the management <strong>of</strong> patients with acute<br />

myocardial infarction). J Am Coll Cardiol. 2004;44:671-719.<br />

Erratum in: J Am Coll Cardiol. 2005;45(8):1376.<br />

www.acc.org/clinical/topic/topic.htm#guidelines<br />

AHA/ACC Scientific Statement: AHA/ACC guidelines for preventing heart attack and<br />

death in patients with atherosclerotic cardiovascular disease: 2001 update: A<br />

statement for healthcare pr<strong>of</strong>essionals from the American Heart Association and the<br />

American College <strong>of</strong> Cardiology. Circulation. 2001;25:1577-9.<br />

www.acc.org/clinical/topic/topic.htm#guidelines<br />

124


TRAINING PROBLEM #17: ACUTE RENAL FAILURE AND<br />

CHRONIC KIDNEY DISEASE<br />

RATIONALE:<br />

Renal disease is a common problem in internal medicine and may manifest with symptoms<br />

referable to the kidney as well as other systems. Patients who go on to end-stage renal disease<br />

have high morbidity and mortality, despite advances in dialysis treatment. Thus, an understanding<br />

<strong>of</strong> chronic kidney disease is useful to all physicians. A rational approach to patients with suspected<br />

or known acute renal failure allows students and clinicians to quickly assess the etiology and initiate<br />

treatment without unnecessary delay in an effort to prevent the development <strong>of</strong> chronic kidney<br />

disease.<br />

PREREQUISITES:<br />

Prior knowledge, skills and attitudes acquired during the pre-<strong>clerkship</strong> experience should include:<br />

Ability to perform a complete medical history and physical exam.<br />

Ability to communicate with patients <strong>of</strong> diverse backgrounds.<br />

Knowledge <strong>of</strong> pathogenesis and pathophysiology <strong>of</strong> acute renal failure and the development <strong>of</strong><br />

chronic kidney disease.<br />

Understanding <strong>of</strong> drugs that can have adverse effect on renal function.<br />

SPECIFIC LEARNING OBJECTIVES:<br />

A. KNOWLEDGE: Students should be able to define, describe and discuss:<br />

1. The distinction between the three major pathophysiologic etiologies for acute renal failure<br />

(ARF):<br />

Decreased renal perfusion (prerenal). (MK)<br />

Intrinsic renal disease (renal). (MK)<br />

Acute renal obstruction (postrenal). (MK)<br />

2. The pathophysiology <strong>of</strong> the major etiologies <strong>of</strong> “prerenal” ARF, including:<br />

Hypovolemia. (MK)<br />

Decreased cardiac output. (MK)<br />

Systemic vasodilation. (MK)<br />

Renal vasoconstriction. (MK)<br />

3. The pathophysiology <strong>of</strong> the major etiologies <strong>of</strong> intrinsic “renal” ARF, including:<br />

Vascular lesions. (MK)<br />

Glomerular lesions. (MK)<br />

interstitial nephritis. (MK)<br />

Intra-tubule deposition/obstruction. (MK)<br />

Acute tubular necrosis (ATN). (MK)<br />

4. The pathophysiology <strong>of</strong> the major etiologies <strong>of</strong> “postrenal” ARF, including:<br />

Urethral (e.g. tumors, calculi, clot, sloughed papillae, retroperitoneal fibrosis,<br />

lymphadenopathy). (MK)<br />

Bladder neck (e.g. tumors, calculi, prostatic hypertrophy or carcinoma, neurogenic). (MK)<br />

Urethral (e.g. stricture, tumors, obstructed indwelling catheters). (MK)<br />

125


5. The pathophysiology and clinical findings <strong>of</strong> uremia. (MK)<br />

6. The natural history, initial evaluation and treatment, and complications <strong>of</strong> ARF. (MK)<br />

7. The most common etiologies <strong>of</strong> chronic kidney disease (CKD):<br />

DM. (MK)<br />

Hypertension. (MK)<br />

Glomerulonephritis. (MK)<br />

Polycystic kidney disease. (MK)<br />

Autoimmune diseases (e.g. systemic lupus erythematosus). (MK)<br />

The staging scheme for CKD. (MK)<br />

8. The significance for proteinuria in CKD. (MK)<br />

9. The use <strong>of</strong> ACE-Is and ARBs in the management <strong>of</strong> CKD. (MK)<br />

10. The importance <strong>of</strong> secondary hyperparathyroidism in CKD. (MK)<br />

11. The pathophysiology <strong>of</strong> anemia in CKD. (MK)<br />

12. The value <strong>of</strong> glycemic and hypertension control in limiting the progression <strong>of</strong> CKD.<br />

(MK)<br />

13. The value <strong>of</strong> CAD risk factor modification in patients with CKD, particularly those<br />

treated with dialysis. (MK)<br />

14. The basic principles <strong>of</strong> renal replacement therapy (e.g., hemodialysis and peritoneal<br />

dialysis) as well as the complications. (MK)<br />

B. SKILLS: Students should be able to demonstrate specific skills, including:<br />

1 History-taking skills: Students should be able to obtain, document, and present an ageappropriate<br />

history that distinguishes among the three major reasons for ARF (pre-renal, renal,<br />

post-renal), including the predisposing conditions, nephrotoxic drugs or agents, and systemic<br />

disease and the major causes <strong>of</strong> CKD. (PC, CS)<br />

2. Physical exam skills: Students should be able to perform a physical examination to establish<br />

the diagnosis and severity <strong>of</strong> disease, including:<br />

The determination <strong>of</strong> a patient’s volume status through estimation <strong>of</strong> the central venous<br />

pressure using the height <strong>of</strong> jugular venous distention and measurement <strong>of</strong> pulse and blood<br />

pressure in the lying/standing position. (PC)<br />

Palpation and percussion <strong>of</strong> the bladder to recognize bladder distention. (PC)<br />

Palpation <strong>of</strong> the prostate. (PC)<br />

Determination <strong>of</strong> the presence <strong>of</strong> pulmonary edema, peripheral edema, ascites, and signs<br />

<strong>of</strong> heart failure. (PC)<br />

Findings consistent with uremia. (PC)<br />

Examination for evidence <strong>of</strong> systemic disease, including but not limited to: skin, joints, and<br />

nails. (PC)<br />

3. Differential diagnosis: Students should be able to generate a differential diagnosis for a<br />

patient with ARF or CKD recognizing specific history, physical exam, and laboratory findings<br />

that suggest a specific etiology. (PC, MK)<br />

4. Laboratory interpretation: Students should be able to recommend when to order diagnostic<br />

and laboratory tests and be able to interpret them, both prior to and after initiating treatment<br />

based on the differential diagnosis, including consideration <strong>of</strong> test cost and performance<br />

characteristics as well as patient preferences. Laboratory and diagnostic tests should include,<br />

when appropriate:<br />

Serum electrolytes, BUN/Cr, calcium, phosphorus. (PC, MK)<br />

Urine sodium. (PC, MK)<br />

Serum and urine osmolality. (PC, MK)<br />

126


Anion gap. (PC, MK)<br />

ABG (PC, MK)<br />

Serum BUN to Cr ratio. (PC, MK)<br />

CBC, ferritin. (PC, MK)<br />

Performing and interpreting a urinalysis, including microscopic examination for casts, red<br />

blood cells, white blood cells, and crystals. (PC, MK)<br />

Calculating fractional excretion <strong>of</strong> sodium and appreciate its usefulness in distinguishing<br />

between pre-renal and intrinsic renal disease. (PC, MK)<br />

Calculating creatinine clearance using the Cockcr<strong>of</strong>t-Gault or MDRD (“modification <strong>of</strong> diet<br />

in renal disease study”) equations. (PC, MK)<br />

Serum parathyroid hormone level. (PC, MK)<br />

ECG findings in hyperkalemia. (PC, MK)<br />

Students should be able to define the indications for and interpret (with consultation) results<br />

<strong>of</strong>:<br />

Renal ultrasonography. (PC, MK)<br />

5. Communication skills: Students should be able to:<br />

Communicate the diagnosis, treatment plan, and subsequent follow-up to the patient and<br />

his or her family. (PC, CS)<br />

Elicit questions from the patient and his or her family about the management plan. (PC, CS)<br />

Counsel patients regarding a renal diet. (PC, CS)<br />

6. Basic and advanced procedure skills: Students should be able to:<br />

Insert a peripheral intravenous catheter. (PC)<br />

Place a urinary catheter. (PC)<br />

Obtain an ABG. (PC)<br />

7. Management skills: Students should be able to develop an appropriate evaluation and<br />

treatment plan for patients, including:<br />

Designing an appropriate management plan for initial management <strong>of</strong> ARF, including<br />

volume management, dietary recommendations, drug dosage alterations, electrolyte<br />

monitoring, and indications for dialysis. (PC, MK)<br />

Developing a management plan to effectively treat HTN and DM. (PC, MK)<br />

Recommending treatment with phosphate binders, calcium replacement, and vitamin D<br />

replacement. (PC, MK)<br />

Recommending treatment for dyslipidemia. (PC, MK)<br />

Recommending treatment for anemia secondary to CKD. (PC, MK)<br />

Recommending acute treatment for hyperkalemia. (PC, MK)<br />

Determining when to obtain consultation from a nephrologist. (PC, MK)<br />

Using a cost-effective approach based on the differential diagnosis. (PC, SBP)<br />

Accessing and utilizing appropriate information systems and resources to help delineate<br />

issues related to renal failure. (PC, PLI)<br />

Incorporating patient preferences. (PC, P)<br />

C. ATTITUDES AND PROFESSIONAL BEHAVIORS: Students should be able to:<br />

3. Demonstrate commitment to using risk-benefit, cost-benefit, and evidence-based considerations in<br />

the selection <strong>of</strong> diagnostic and therapeutic interventions for ARF and CKD. (PLI, P)<br />

4. Respond appropriately to patients who are nonadherent to treatment for renal failure. (CS, P)<br />

5. Demonstrate ongoing commitment to self-directed learning regarding renal failure. (PLI, P)<br />

4. Appreciate the impact renal failure has on a patient’s quality <strong>of</strong> life, wellbeing, ability to work,<br />

127


and the family. (P)<br />

5. Recognize the importance and demonstrate a commitment to the utilization <strong>of</strong> other healthcare<br />

pr<strong>of</strong>essions in the treatment <strong>of</strong> renal failure. (P, SBP)<br />

D. REFERENCES:<br />

11.<br />

Lamiere N, Van Breson W, Vanholder R. Acute renal failure. Lancet. 2005;365:417-30.<br />

Singri N, Ahya SN, Levin ML. Acute renal failure. JAMA. 2003;289:747-51.<br />

Schrier RW, Wang W, Poole B, Mitra A. Acute renal failure: definitions, diagnosis,<br />

pathogenesis, and therapy. J Clin Invest. 2004;114:5-14.<br />

Esson ML, Schrier RW. Diagnosis and treatment <strong>of</strong> acute tubular necrosis. Ann Intern<br />

Med. 2002;137:744-52.<br />

Levey AS. Clinical practice. Nondiabetic kidney disease. N Engl J Med. 2002;347:1505-<br />

Palmer BF. Renal dysfunction complicating the treatment <strong>of</strong> hypertension. N Engl J<br />

Med. 2002;347:1256-61.<br />

128


TRAINING PROBLEM #18: COMMON CANCERS<br />

RATIONALE:<br />

A skillful initial workup for suspected cancer is an essential part <strong>of</strong> effective primary care practice.<br />

Developing a logical and practical diagnostic approach to the more common cancers (e.g. skin,<br />

colorectal, lung, breast, cervical, and prostate) is an excellent means <strong>of</strong> honing basic history-taking,<br />

physical examination, and communication skills and learning how to use diagnostic studies in a cost<br />

effective manner. Encountering patients in whom cancer is a diagnostic possibility will stimulate<br />

learning <strong>of</strong> the important clinical presentations and natural histories <strong>of</strong> these life-threatening<br />

conditions. Focusing on cancer diagnosis helps to concentrate the student’s learning and avoids<br />

premature immersion in the <strong>of</strong>ten very technical and specialized issues <strong>of</strong> cancer treatment.<br />

PREREQUISITES:<br />

Prior knowledge, skills, and attitudes acquired during the pre-<strong>clerkship</strong> experience should include:<br />

Ability to perform a complete medical history and physical exam.<br />

Ability to communicate with patients <strong>of</strong> diverse backgrounds.<br />

Anatomy, physiology, and pathophysiology <strong>of</strong> common cancers.<br />

Basic knowledge <strong>of</strong> the common symptoms and signs <strong>of</strong> the most common cancers.<br />

Knowledge <strong>of</strong> basic concepts <strong>of</strong> clinical epidemiology pertinent to test selection and<br />

interpretation (e.g. sensitivity, specificity, positive predictive value, negative predictive value).<br />

SPECIFIC LEARNING OBJECTIVES:<br />

A. KNOWLEDGE: Students should be able to define, describe, and discuss:<br />

3. Primary prevention measures for common cancers. (MK)<br />

4. Current screening recommendations for skin, colorectal, lung, breast, cervical, and<br />

prostate cancer. (MK)<br />

3. Principle clinical presentations, clinical courses, complications, and causes <strong>of</strong> death for the<br />

most common cancers (e.g. skin, colorectal, lung, breast, cervical, and prostate). (MK)<br />

4. Basic methods <strong>of</strong> initial evaluation, including the sensitivity and specificity <strong>of</strong> basic diagnostic<br />

studies and indication for their use, including:<br />

Indications for skin biopsy in a patient with a suspicious skin lesion. (MK)<br />

Indications for colonoscopy in individuals a risk for colon cancer. (MK)<br />

Indications for breast biopsy in a patient with a breast nodule or abnormal screening<br />

mammogram. (MK) Indications for a lymph node biopsy in a patent with suspicious<br />

lymphadenopathy. (MK)<br />

Initial cost-effective workups for: isolated pleural effusion, pulmonary nodule, liver nodule,<br />

prostate nodule, elevated prostate-specific antigen, testicular nodule, stool test positive for<br />

occult blood, abnormal<br />

Pap smear, and other findings suggestive <strong>of</strong> gastrointestinal and urogenital cancers.<br />

(MK)<br />

5. Genetic considerations <strong>of</strong> selected cancers (e.g. hereditary nonpolyposis colon cancer, familial<br />

adenomatous polyposis, BRCA1/BRCA2, HER2, Philadelphia chromosome/BRC-ABL). (MK)<br />

6. The role <strong>of</strong> human papilloma virus in cervical cancer. (MK)<br />

7. The similarities and differences between curative and palliative cancer care. (MK)<br />

8. The principles <strong>of</strong> palliative care and hospice care. (MK)<br />

9. Symptoms sometimes seen during end-<strong>of</strong>-life care and the basic principles <strong>of</strong> their<br />

129


management (e.g., pain, dyspnea, nausea and vomiting, anorexia, fatigue, depression,<br />

delirium, constipation). (MK)<br />

B. SKILLS: Students should be able to demonstrate specific skills, including:<br />

1. History-taking skills: Students should be able to obtain, document, and present an<br />

age-appropriate medical history, that differentiates among etiologies <strong>of</strong> disease,<br />

including:<br />

Unintentional weight loss, fever, bone pain. (PC, CS)<br />

Sun exposure history, abnormal skin lesions. (PC, CS)<br />

Blood in the stool, alterations in bowel movements, abdominal pain, abdominal mass. (PC,<br />

CS)<br />

Smoking, cough, hemoptysis, chest pain, dyspnea. (PC, CS)<br />

Breast nodules and secondary signs <strong>of</strong> breast cancer. (PC, CS)<br />

Abnormal vaginal bleeding. (PC, CS)<br />

Abnormal urinary symptoms. (PC, CS)<br />

Lymphadenopathy. (PC, CS)<br />

2. Physical exam skills: Students should be able to perform a physical exam to establish the<br />

diagnosis and severity <strong>of</strong> disease, including:<br />

Skin examination. (PC)<br />

Digital rectal examination. (PC)<br />

Breast examination. (PC)<br />

Lymph node examination. (PC)<br />

Male genital examination and prostate examination. (PC)<br />

Pelvic examination and Pap smear. (PC)<br />

3. Differential diagnosis: Students should be able to generate a prioritized differential<br />

diagnosis recognizing specific history and physical exam findings that suggest a specific<br />

etiology for:<br />

Unintentional weight loss. (PC, MK)<br />

Fever. (PC, MK)<br />

Abnormal skin lesions. (PC, MK)<br />

Occult blood positive stool. (PC, MK)<br />

Colorectal mass. (PC, MK)<br />

Chronic cough, hemoptysis, pulmonary nodule, and pleural effusion. (PC, MK)<br />

Breast mass. (PC, MK)<br />

Abnormal Pap smear. (PC, MK)<br />

Abdominal or pelvic mass. (PC, MK)<br />

Prostate nodule and elevated prostate specific antigen. (PC, MK)<br />

Lymphadenopathy. (PC, MK)<br />

5. Laboratory interpretation: Students should be able to recommend when to order<br />

diagnostic and laboratory tests and be able to interpret them, both prior to and after<br />

initiating treatment based on the differential diagnosis, including consideration <strong>of</strong> test<br />

cost and performance characteristics as well as patient preferences.<br />

Laboratory and diagnostic tests should include, when appropriate:<br />

CBC. (PC)<br />

Electrolytes, BUN/Cr, Ca, hepatic function panel. (PC)<br />

Stool occult blood testing. (PC)<br />

PSA. (PC)<br />

130


Students should be able to define the indications for and interpret (with consultation) the<br />

significance <strong>of</strong> the results <strong>of</strong>:<br />

Skin biopsy. (PC)<br />

Mammogram. (PC)<br />

Breast biopsy. (PC)<br />

Colon/rectal biopsy. (PC)<br />

Lung biopsy. (PC)<br />

Pap smear. (PC)<br />

Prostate biopsy. (PC)<br />

Lymph node biopsy. (PC)<br />

5. Communication skills: Students should be able to:<br />

Communicate the diagnostic plan and subsequent follow-up to patients. (PC, CS)<br />

Elicit questions from the patient and his or her family about the management plan. (PC, CS)<br />

With guidance and direct supervision, participate in breaking bad news to patients. (PC,<br />

CS)<br />

With guidance and direct supervision, participate in discussing basic issues regarding<br />

advance directives with the patient and his or her family. (PC, CS)<br />

With guidance and direct supervision participate in discussing basic end-<strong>of</strong>-life issues with<br />

the patient and his or her family. (PC, CS)<br />

6. Basic and advanced procedure skills: Students should be able to:<br />

Cervical Pap smear. (PC)<br />

Stool occult blood testing. (PC)<br />

7. Management skills: Students should able to develop an appropriate evaluation and<br />

treatment plan for patients that includes:<br />

Initial work-up <strong>of</strong> the symptom, sign, or abnormal laboratory value suspected to be due to<br />

cancer. (PC)<br />

Provision <strong>of</strong> support and information for the patient. (PC)<br />

Coordination <strong>of</strong> care for workup. (PC, SBP)<br />

Determining when to obtain consultation from appropriate specialists. (PC, SBP)<br />

A cost-effective approach based on the differential diagnosis. (PC, SBP)<br />

Accessing and utilizing appropriate information systems and resources to help delineate<br />

issues related to common cancers. (PC, PLI)<br />

Incorporating patient needs and preferences. (PC, P)<br />

Appropriately assessing and treating pain when necessary with nonnarctoic and narcotic<br />

analgesics. (PC)<br />

Anticipating and treating narcotic side effects if necessary. (PC)<br />

Adjusting the therapeutic plan when goals <strong>of</strong> care change (e.g., a shift toward palliative<br />

care). (PC)<br />

Alleviation <strong>of</strong> symptoms sometimes seen during end <strong>of</strong> life care (e.g., pain, dyspnea,<br />

nausea and vomiting, anorexia, fatigue, depression, delirium, constipation). (PC)<br />

Utilizing supportive care or hospice service when appropriate. (PC, SBP)<br />

C. ATTITUDES AND PROFESSIONAL BEHAVIORS: Students should be able to:<br />

1. Appreciate the uncertainty and fear patients experience when cancer is a significant diagnostic<br />

possibility. (P)<br />

2. Respect the patient’s right to refuse cancer screening. (P)<br />

3. Demonstrate commitment to using risk-benefit, cost-benefit, and evidence-based<br />

131


considerations in the selection diagnostic and therapeutic interventions for common cancers. (PLI, P)<br />

4. Recognize the importance <strong>of</strong> patient preferences when selecting among diagnostic options for<br />

common cancers. (P)<br />

5. Demonstrate ongoing commitment to self-directed learning regarding common cancers. (PLI,<br />

P)<br />

6. Appreciate the impact common cancers have on a patient’s quality <strong>of</strong> life, well-being, ability to<br />

work, and the family. (P)<br />

7. Recognize the importance <strong>of</strong> and demonstrate a commitment to the utilization <strong>of</strong> other<br />

healthcare pr<strong>of</strong>essions in the workup and treatment <strong>of</strong> common cancers. (P, SBP)<br />

D. REFERENCES:<br />

National Cancer Institute<br />

National Institutes <strong>of</strong> Health<br />

www.cancer.gov<br />

Guide to Clinical Preventive Services<br />

U.S. Preventative Services Task Force (USPSTF)<br />

Agency for Healthcare Research and Quality<br />

U.S. <strong>Department</strong> <strong>of</strong> Health and Human Services<br />

www.ahrq.gov/clinic/cps3dix.htm#cancer<br />

American Cancer Society<br />

www.cancer.org<br />

NCCN Clinical Practice Guidelines in Oncology National Comprehensive<br />

Cancer Network<br />

www.nccn.org/pr<strong>of</strong>essionals/physician_gls/default.asp<br />

132


TRAINING PROBLEM #19: COPD/OBSTRUCTIVE AIRWAYS<br />

DISEASE<br />

RATIONALE:<br />

The chronic obstructive pulmonary diseases (chronic bronchitis and emphysema) are important<br />

causes <strong>of</strong> morbidity and mortality and are a major cause <strong>of</strong> total disability, second only to coronary<br />

artery disease. Cigarette smoking plays a major role in the progression <strong>of</strong> the disease, with survival<br />

rates lower among patients who continue to smoke cigarettes. The severity and debilitation <strong>of</strong> these<br />

disorders make them an important training problem for all third year medical students. The number<br />

<strong>of</strong> new cases <strong>of</strong> asthma is dramatically increasing. Most cases with appropriate treatment can have<br />

minimal symptoms.<br />

PREREQUISITES:<br />

Prior knowledge, skills, and attitudes acquired during the pre-clinical experience should include:<br />

Ability to perform a complete medical history and physical.<br />

Ability to communicate with patients <strong>of</strong> diverse backgrounds.<br />

Normal structure and function <strong>of</strong> the heart and lungs and how these are altered in respiratory<br />

system diseases.<br />

Pathogenesis and pathophysiology <strong>of</strong> pulmonary diseases.<br />

Pharmacology <strong>of</strong> bronchodilators, corticosteroids, and antibiotics.<br />

SPECIFIC LEARNING OBJECTIVES:<br />

A. KNOWLEDGE: Students should be able to define, describe, and discuss:<br />

1. The epidemiology, risk factors, symptoms, signs, and typical clinical course <strong>of</strong> the common<br />

forms <strong>of</strong> COPD, including chronic bronchitis and emphysema. (MK)<br />

2. Common causes <strong>of</strong> acute exacerbations <strong>of</strong> COPD (AECOPD), including:<br />

Acute infectious bronchitis. (MK)<br />

Pneumonia. (MK)<br />

Pulmonary edema. (MK)<br />

Poor air quality (e.g. ozone, pollutants, tobacco smoke). (MK)<br />

Occupational exposures. (MK)<br />

Medical noncompliance. (MK)<br />

3. The etiology, pathogenesis, evaluation, and management <strong>of</strong> hypoxemia and hypercapnia.<br />

(MK)<br />

4. The genetics and role <strong>of</strong> alpha-1 antitrypsin deficiency in some patients with emphysema. (MK)<br />

5. The epidemiology, risk factors, symptoms, signs, and typical clinical course <strong>of</strong> asthma. (MK)<br />

6. Allergic and non-allergic factors that may precipitate bronchospasm and exacerbate asthma,<br />

including:<br />

Grass and tree pollen. (MK)<br />

Animal dander. (MK)<br />

Cockroaches. (MK)<br />

Dust mites. (MK)<br />

Allergic rhinitis/post-nasal drip. (MK)<br />

133


Acute/chronic infectious sinusitis. (MK)<br />

Acute infectious bronchitis. (MK)<br />

Pneumonia. (MK)<br />

Pulmonary edema. (MK)<br />

Exercise. (MK)<br />

Anxiety/stress. (MK)<br />

Poor air quality (e.g. ozone, pollutants, tobacco smoke). (MK)<br />

Occupational exposures. (MK)<br />

Medical noncompliance. (MK)<br />

7. Therapies for COPD and asthma, including:<br />

Beta-agonist bronchodilators. (MK)<br />

Anticholinergic bronchodilators. (MK)<br />

Leukotriene inhibitors. (MK)<br />

Mast cell stabilizers. (MK)<br />

Theophylline. (MK)<br />

Inhaled corticosteroids. (MK)<br />

Systemic corticosteroids. (MK)<br />

Antimicrobial agents. (MK)<br />

Supplemental oxygen. (MK)<br />

Immunotherapy. (MK)<br />

8. The indications for and the efficacy <strong>of</strong> influenza and pneumococcal vaccines.<br />

(MK)<br />

B. SKILLS: Students should be able to demonstrate specific skills, including:<br />

1. History-taking skills: Students should be able to obtain, document, and present an ageappropriate<br />

medical history, that differentiates among etiologies <strong>of</strong> disease including:<br />

Existence, duration, and severity <strong>of</strong> dyspnea, orthopnea, paroxysmal nocturnal dyspnea,<br />

cough, sputum production, wheezing, fever, chills, sweats, chest pain, hemoptysis. (PC,<br />

CS)<br />

Smoking history and passive exposure to tobacco smoke. (PC, CS)<br />

Occupational history. (PC, CS)<br />

Family history <strong>of</strong> pulmonary problems. (PC, CS)<br />

2. Physical exam skills: Students should be able to perform a physical exam to establish the<br />

diagnosis and severity <strong>of</strong> disease including:<br />

Accurately determining respiratory rate and level <strong>of</strong> respiratory distress. (PC)<br />

Assessing the use <strong>of</strong> accessory muscles for breathing. (PC)<br />

Identifying bronchial breath sounds, rales, rhonchi, and wheezes. (PC)<br />

Identifying signs <strong>of</strong> pulmonary consolidation, pleural effusion, and pneumothorax. (PC)<br />

Identifying the signs <strong>of</strong> pulmonary hyperresonance/hyperexpansion. (PC)<br />

3. Differential diagnosis: Students should be able to generate a prioritized differential diagnosis<br />

recognizing specific history and physical exam findings that suggest a diagnosis <strong>of</strong> chronic<br />

bronchitis, emphysema, asthma, or other conditions with similar findings.<br />

4. Laboratory interpretation: Students should be able to recommend when to order diagnostic<br />

and laboratory tests and be able to interpret them, both prior to and after initiating treatment<br />

based on the differential diagnosis, including consideration <strong>of</strong> test cost and performance<br />

characteristics as well as patient preferences.<br />

Laboratory and diagnostic tests should include, when appropriate:<br />

134


Pulse oximitry. (PC, MK)<br />

ABG. (PC, MK)<br />

Chest radiograph. (PC, MK)<br />

Pulmonary function tests. (PC, MK)<br />

5. Communication skills: Students should be able to:<br />

Communicate the diagnosis, treatment plan, and subsequent follow-up to the patient and<br />

his or her family. (PC, CS)<br />

Elicit questions from the patient and his or her family about the management plan. (PC, CS)<br />

Counsel patients about smoking cessation. (PC, CS)<br />

Counsel patients about the performance <strong>of</strong> home peak flow monitoring. (PC, CS)<br />

Counsel patients about environmental controls. (PC, CS)<br />

Encourage asthma patients to be involved in their own disease management and counsel<br />

them about an “asthma action plan.” (PC, CS)<br />

6. Basic and advanced procedure skills: Students should be able to:<br />

Obtain an ABG. (PC)<br />

7. Management skills: Students should able to develop an appropriate evaluation and treatment<br />

plan for patients that includes:<br />

The use <strong>of</strong> bronchodilators and inhaled corticosteroids. (PC, MK)<br />

The key components <strong>of</strong> the care <strong>of</strong> patients admitted with acute exacerbations <strong>of</strong> COPD<br />

and asthma. (PC, MK)<br />

Using systemic corticosteroids appropriately. (PC, MK)<br />

Judicious use <strong>of</strong> antimicrobial agents. (PC, MK)<br />

The principles <strong>of</strong> oxygen therapy. (PC, MK)<br />

Determining when to obtain consultation from a pulmonologist or allergist/immunologist.<br />

(PC, SBP)<br />

Smoking cessation strategies. (PC)<br />

Using a cost-effective approach based on the differential diagnosis. (PC, SBP)<br />

Accessing and utilizing appropriate information systems and resources to help delineate<br />

issues related to COPD and asthma. (PC, PLI)<br />

Incorporating patient preferences. (PC, P)<br />

C. ATTITUDES AND PROFESSIONAL BEHAVIORS: Students should be able to:<br />

1 Demonstrate commitment to using risk-benefit, cost-benefit, and evidence-based<br />

considerations in the selection diagnostic and therapeutic interventions for COPD and asthma.<br />

(PLI, P)<br />

2 Recognize the importance <strong>of</strong> patient preferences when selecting among diagnostic and<br />

therapeutic options for COPD and asthma (P)<br />

3 Respond appropriately to patients who are nonadherent to treatment for COPD and asthma.<br />

(CS, P)<br />

4 Appreciate the impact <strong>of</strong> working, living, and environmental conditions on the development and<br />

progression <strong>of</strong> respiratory tract disease; demonstrate understanding that patients are <strong>of</strong>ten<br />

unable to change these factors on their own. (P)<br />

5 Demonstrate ongoing commitment to self-directed learning regarding COPD and asthma. (PLI,<br />

P)<br />

6 Appreciate the impact COPD and asthma have on a patient’s quality <strong>of</strong> life, well-being, ability<br />

to work, and the family. (P)<br />

7 Recognize the importance <strong>of</strong> and demonstrate a commitment to the utilization <strong>of</strong> other<br />

healthcare pr<strong>of</strong>essionals in the diagnosis and treatment <strong>of</strong> COPD and asthma. (P, SBP)<br />

135


8 Appreciate the importance <strong>of</strong> antimicrobial resistance. (P)<br />

9. Show understanding for the difficulties patients face with smoking cessation. (P)<br />

D. REFERENCES:<br />

National Heart Lung and Blood Institute/World Health Organization Global Initiative for<br />

Chronic Obstructive Lung Disease Diagnosis, management, and prevention <strong>of</strong> chronic<br />

obstructive pulmonary disease<br />

www.goldcopd.com<br />

National Institutes <strong>of</strong> Health, National Heart, Lung, and Blood Institute National Asthma<br />

Education and Prevention Program Practical Guide for the Diagnosis and Management<br />

<strong>of</strong> Asthma NIH Publication 97-4074. Bethesda, MD. 2003.<br />

http://www.nhlbi.nih.gov/health/indexpro.htm<br />

Celli BR, MacNee W. Standards for the diagnosis and treatment <strong>of</strong> patients with<br />

COPD: a summary <strong>of</strong> the ATS/ERS position paper. Eur Respir J 2004;23:932-46.<br />

136


TRAINING PROBLEM #20: DIABETES MELLITUS<br />

RATIONALE:<br />

Diabetes mellitus is an increasingly prevalent illness in the United States. It is estimated that five to<br />

nine percent <strong>of</strong> American adults are diabetic with the illness appearing at earlier ages in some<br />

populations. It is a leading cause <strong>of</strong> disability and death. Over 130 billion health care dollars are<br />

spent on diabetes annually. All internists must identify those at risk and institute appropriate<br />

management to ameliorate the potentially fatal complications <strong>of</strong> this illness.<br />

PREREQUISITES:<br />

Prior knowledge, skills, and attitudes acquired during the pre-clinical experience should include:<br />

Ability to perform a complete medical history and physical.<br />

Ability to communicate with patients <strong>of</strong> diverse backgrounds.<br />

Pathogenesis and pathophysiology <strong>of</strong> type I and II diabetes mellitus, diabetic ketoacidosis,<br />

nonketotic hyperglycemia.<br />

Effects <strong>of</strong> insulin on glucose and fat metabolism.<br />

Pharmacology <strong>of</strong> insulin, sulfonylureas, metformin, thiazolidinediones, and glucose absorption<br />

inhibitors.<br />

SPECIFIC LEARNING OBJECTIVES:<br />

A. KNOWLEDGE: Students should be able to define, describe, and discuss:<br />

1. Diagnostic criteria for impaired fasting glucose and impaired glucose tolerance. (MK)<br />

2. Diagnostic criteria for type I and type II diabetes mellitus, based on a history, physical<br />

examination, and laboratory testing. (MK)<br />

3. Pathophysiology, risk factors, and epidemiology <strong>of</strong> type I and type II diabetes mellitus. (MK)<br />

4. The basic principles <strong>of</strong> the role <strong>of</strong> genetics in diabetes mellitus. (MK)<br />

5. Presenting symptoms and signs <strong>of</strong> type I and type II diabetes mellitus. (MK)<br />

6. Presenting symptoms and signs <strong>of</strong> diabetic ketoacidosis (DKA) and nonketotic hyperglycemic<br />

(NKH). (MK)<br />

7. Pathophysiology for the abnormal laboratory values in DKA and NKH including plasma<br />

sodium, potassium, and bicarbonate. (MK)<br />

8. Precipitants <strong>of</strong> DKA and NKH. (MK)<br />

9. Major causes <strong>of</strong> morbidity and mortality in diabetes mellitus (coronary artery disease,<br />

peripheral vascular disease, hypoglycemia, DKA, NKH coma, retinopathy, neuropathy—<br />

peripheral and autonomic, nephropathy, foot disorders, infections). (MK)<br />

10. Laboratory tests needed to screen, diagnose, and follow diabetic patients including: glucose,<br />

electrolytes, blood urea nitrogen/creatinine, fasting lipid pr<strong>of</strong>ile, HgA1c, urine<br />

microalbumin/creatinine ratio, urine dipstick for protein. (MK)<br />

11. Non-pharmacologic and pharmacologic (drugs and side effects) treatment <strong>of</strong> diabetes mellitus<br />

to maintain acceptable levels <strong>of</strong> glycemic control, prevent target organ disease, and other<br />

associated complications. (MK)<br />

12. The specific components <strong>of</strong> the American Diabetes Association (ADA) dietary recommendations<br />

for type I and type II diabetes mellitus. (MK)<br />

13. Basic management <strong>of</strong> diabetic ketoacidosis and nonketotic hyperglycemic states, including the<br />

similarities and differences in fluid and electrolyte replacement. (MK)<br />

14. Basic management <strong>of</strong> blood glucoses in the hospitalized patient. (MK)<br />

137


15. The Somogyi effect and the Dawn phenomenon and the implications <strong>of</strong> each in diabetes<br />

pharmacologic management. (MK)<br />

16. The fundamental aspects <strong>of</strong> the American Diabetes Association (ADA) clinical practice<br />

recommendations and how they encourage high quality diabetes care. (MK, PLI, SBP)<br />

17. Basic management <strong>of</strong> hypertension and hyperlipidemia in the diabetic patient. (MK)<br />

B. SKILLS: Students should be able to demonstrate specific skills including:<br />

1. History-taking skills: Students should be able to obtain, document, and present an<br />

age-appropriate medical history, that differentiates among etiologies <strong>of</strong> disease,<br />

including:<br />

Weight changes. (PC, CS)<br />

Hypo- or hyperglycemic symptoms. (PC, CS)<br />

Medication history (adherence, side effects, other medications). (PC, CS)<br />

Home glucose monitoring results. (PC, CS)<br />

Target organ disease complications (cardiovascular, foot, gastrointestinal, infectious,<br />

neurological, sexual, skin, urinary, or vision symptoms). (PC, CS)<br />

Diet history (total caloric intake, intake <strong>of</strong> sugar-containing foods, intake <strong>of</strong> saturated fat and<br />

cholesterol, physical activity level, timing <strong>of</strong> meals). (PC, CS)<br />

2. Physical exam skills: Students should be able to perform a physical exam to establish the<br />

diagnosis and severity <strong>of</strong> disease, including:<br />

Skin examination for diabetic dermopathy, furuncles/carbuncles, candidiasis, necrobiosis<br />

lipoidica diabeticorum, dermatophytosis, and acanthosis nigricans. (PC)<br />

Fundoscopic exam. (PC)<br />

Arterial pulses. (PC)<br />

Peripheral nerves (e.g. mon<strong>of</strong>ilament testing). (PC)<br />

Examination <strong>of</strong> the feet for corns, calluses, and ulcerations. (PC)<br />

In patients with DKA or NKH evaluate for mental status alterations, Kussmaul’s<br />

respirations, fruity breath, and signs <strong>of</strong> volume depletion. (PC)<br />

3. Differential diagnosis: Students should be able to generate a prioritized differential diagnosis<br />

recognizing specific history and physical exam findings that suggest a specific etiology for:<br />

Hyperglycemia. (PC, MK)<br />

Hypoglycemia. (PC, MK)<br />

Anion gap acidosis. (PC, MK)<br />

Ketosis. (PC, MK)<br />

Hyperosmolality. (PC, MK)<br />

4. Laboratory interpretation: Students should be able to recommend when to order diagnostic<br />

and laboratory tests and be able to interpret them, both prior to and after initiating treatment<br />

based on the differential diagnosis, including consideration <strong>of</strong> test cost and performance<br />

characteristics as well as patient preferences Laboratory and diagnostic tests should include,<br />

when appropriate:<br />

Fasting serum GLC. (PC, MK)<br />

Electrolytes, BUN/Cr. (PC, MK)<br />

Serum and urine ketones. (PC, MK)<br />

Serum and urine osmolality. (PC, MK)<br />

HbA1c. (PC, MK)<br />

Fasting lipid pr<strong>of</strong>ile. (PC, MK)<br />

UA. (PC, MK)<br />

Urine microalbumin/creatinine ratio. (PC, MK)<br />

138


24-hour urine for protein and creatinine clearance. (PC, MK)<br />

5. Communication skills: Students should be able to:<br />

Communicate the diagnosis, treatment plan, and subsequent follow-up to patients. (PC,<br />

CS)<br />

Elicit questions from the patient and their family about the management plan. (PC, CS)<br />

Counsel patients appropriately on dietary measures, exercise, medication adherence,<br />

proper foot care, and prevention <strong>of</strong> other target organ disease. (PC, CS)<br />

6. Basic and advanced procedural skills: Students should be able to:<br />

Finger-stick capillary blood glucose determination. (PC)<br />

Obtain an ABG. (PC)<br />

7. Management skills: Students should able to develop an appropriate evaluation and<br />

treatment plan for patients that includes:<br />

Writing appropriate fluid and insulin orders and outline critical steps for the treatment <strong>of</strong><br />

DKA and DKH. (PC, MK)<br />

Counseling patients regarding basic features <strong>of</strong> ADA diabetic diet recommendations. (PC,<br />

CS)<br />

Instructing patients in home blood glucose monitoring. (PC, CS)<br />

Counseling patients on behavior changes (smoking cessation, medication adherence, poor<br />

glycemic control, obesity, hypertension, dyslipidemia, and infection) to avoid the<br />

complications <strong>of</strong> diabetes. (PC, CS)<br />

Counseling patients regarding basic foot care. (PC, CS)<br />

Determining when to institute diet therapy, oral hypoglycemic agents, and insulin therapy.<br />

(PC, MK)<br />

Calculating an appropriate insulin dose for a diabetic patient. (PC, MK)<br />

Using community resources (ADA, hospital and community-based education programs) to<br />

aid the patient in understanding and managing his or her illness. (PC, SBP)<br />

Determining when to obtain consultation from an endocrinologist, nephrologist,<br />

ophthalmologist, podiatrist, and dietician. (PC, SBP)<br />

Accessing and utilizing appropriate information systems and resources to help delineate<br />

issues related to diabetes mellitus. (PC, PLI)<br />

Incorporating patient preferences. (PC)<br />

C. ATTITUDES AND PROFESSIONAL BEHAVIORS: Students should be able to:<br />

1. Demonstrate a commitment to meeting ADA clinical practice<br />

recommendations to insure quality diabetes care. (PLI, P, SBP)<br />

2. Demonstrate commitment to using risk-benefit, cost-benefit, and evidence-based considerations in<br />

the selection diagnostic and therapeutic interventions for diabetes mellitus. (PLI, P)<br />

3. Recognize the importance <strong>of</strong> patient preferences when selecting among diagnostic and<br />

therapeutic options for diabetes mellitus. (P)<br />

4. Respond appropriately to patients who are nonadherent to treatment for diabetes mellitus. (CS, P)<br />

5. Demonstrate ongoing commitment to self-directed learning regarding diabetes mellitus. (PLI, P)<br />

6. Appreciate the impact diabetes mellitus has on a patient’s quality <strong>of</strong> life, wellbeing, ability to work,<br />

and the family. (P)<br />

7. Recognize the importance <strong>of</strong> and demonstrate a commitment to the utilization <strong>of</strong> other healthcare<br />

pr<strong>of</strong>essionals in the treatment <strong>of</strong> diabetes mellitus. (P, SBP)<br />

D. REFERENCES:<br />

139


The Diabetes Control and Complication Trail Research Group. The effect <strong>of</strong> intensive<br />

treatment <strong>of</strong> diabetes on the development and progression <strong>of</strong> long-term complications in<br />

insulin-dependent diabetes mellitus. N Engl JMed. 1993;329:977-86.<br />

UK Prospective Diabetes Study (UKPDS) Group. Intensive blood-glucose control with<br />

sulphonylureas or insulin compared with conventional treatment and risk <strong>of</strong><br />

complications in patients with type 2 diabetes (UKPDS 33). Lancet. 1998;352:837-53.<br />

Clinical Practice Recommendations American Diabetes Association<br />

www.diabetes.org/for-health-pr<strong>of</strong>essionals-and-scientists/cpr.jsp<br />

Kitabchi AE. Wall BM. Management <strong>of</strong> diabetic ketoacidosis. Am Fam Physician.<br />

1999;60:455-64.<br />

140


TRAINING PROBLEM #21: DYSLIPIDEMIA<br />

RATIONALE:<br />

Dyslipidemia is a common, important, and treatable cardiovascular risk factor. Its pathophysiology is<br />

increasingly understood, diagnostic tests are readily available, and treatment modalities range from<br />

diet and exercise to a multitude <strong>of</strong> pharmacotherapies. Competency in the evaluation and<br />

management <strong>of</strong> this problem helps develop skills in rational test selection, patient education, and<br />

design <strong>of</strong> cost-effective treatment strategies. It also draws attention to the importance <strong>of</strong> community<br />

health education and nutrition.<br />

PREREQUISITES:<br />

Prior knowledge, skills, and attitudes acquired during the pre-clinical experience should include:<br />

Ability to perform a complete medical history and physical.<br />

Ability to communicate with patients <strong>of</strong> diverse backgrounds.<br />

Anatomy and physiology <strong>of</strong> the vascular system.<br />

Basic cholesterol and lipoprotein metabolism.<br />

Pathogenesis and pathophysiology <strong>of</strong> atherosclerosis.<br />

Pharmacology <strong>of</strong> bile acid sequestrants (resins), nicotinic acid, fibric acid derivatives, HMG-<br />

CoA reductase inhibitors (statins), and cholesterol absorption inhibitors (ezetimibe).<br />

SPECIFIC LEARNING OBJECTIVES:<br />

A. KNOWLEDGE: Students should be able to define, describe, and discuss:<br />

1 The contribution <strong>of</strong> lipoproteins to atherogenesis and CAD risk, including the importance <strong>of</strong><br />

elevations in total cholesterol, LDL cholesterol, ratio <strong>of</strong> total to HDL cholesterol, and<br />

Lipoprotein a. (MK)<br />

2 The classification and etiologies <strong>of</strong> primary dyslipidemias. (MK)<br />

3 Etiologies and underlying pathophysiology <strong>of</strong> secondary dyslipidemias. (MK)<br />

4. The basic principles <strong>of</strong> the role <strong>of</strong> genetics in dyslipidemia, particularly familial combined<br />

hyperlipidemia. (MK)<br />

5. Screening recommendations for dyslipidemias in American adults. (MK)<br />

6. The importance <strong>of</strong> identifying and treating asymptomatic patients at high risk for CAD as<br />

aggressively as those with symptomatic disease. (MK)<br />

7. The available diagnostic studies and their use, particularly determinations <strong>of</strong> HDL, LDL, and total<br />

cholesterol, as well as the need to test for other cardiovascular risk factors. (MK)<br />

8. The current National Cholesterol Education Program (NCEP, ATP III) guidelines for risk factor<br />

assessment, diagnosis and management <strong>of</strong> dyslipidemias, including goal LDL cholesterol, goal<br />

non-HDL cholesterol, and the concept <strong>of</strong> coronary artery disease equivalent based on risk<br />

factors for coronary artery disease. (MK, PLI, SBP)<br />

9. Basic management <strong>of</strong> the common dyslipidemias, including diet, fiber, exercise, and<br />

risk/benefits/cost <strong>of</strong> drug therapy (statins, fibrates, ezetimide, nicotinic acid, resins). (MK)<br />

10. Diagnosis and implications <strong>of</strong> the “metabolic syndrome.” (MK)<br />

B. SKILLS: Students should be able to demonstrate specific skills including:<br />

1. History-taking skills: Students should be able to obtain, document, and present an age-<br />

141


appropriate medical history, that differentiates among etiologies <strong>of</strong> disease including:<br />

Prior patient or family history <strong>of</strong> dyslipidemia. (PC, CS)<br />

Other coronary risk factors. (PC, CS)<br />

Family history <strong>of</strong> early cardiovascular disease. (PC, CS)<br />

Dietary fat, saturated fat, fiber, cholesterol, and refined carbohydrate intake. (PC, CS)<br />

Exercise habits. (PC, CS)<br />

Alcohol use. (PC, CS)<br />

Past history <strong>of</strong> established CAD, cerebral vascular disease, and other vascular disease.<br />

(PC, CS)<br />

Presence <strong>of</strong> symptoms <strong>of</strong> angina and peripheral vascular disease. (PC, CS)<br />

History <strong>of</strong> renal, hepatic, or myopathic disease. (PC, CS)<br />

2. Physical exam skills: Students should be able to perform a physical exam to establish the<br />

diagnosis and severity <strong>of</strong> disease, including:<br />

Blood pressure elevation. (PC)<br />

Xanthomata. (PC)<br />

Atherosclerotic fundoscopic changes. (PC)<br />

Carotid or femoral bruits. (PC, CS)<br />

Diminished peripheral pulses. (PC)<br />

3. Differential diagnosis: Students should be able to generate a prioritized differential diagnosis<br />

recognizing specific history and physical exam findings that suggest primary or secondary<br />

causes <strong>of</strong> dyslipidemia. (PC, CS)<br />

4. Laboratory interpretation: Students should be able to recommend when to order diagnostic<br />

and laboratory tests and be able to interpret them, both prior to and after initiating treatment<br />

based on the differential diagnosis, including consideration <strong>of</strong> test cost and performance<br />

characteristics as well as patient preferences.<br />

Laboratory and diagnostic tests should include, when appropriate:<br />

Fasting lipid pr<strong>of</strong>ile. (PC, MK)<br />

TSH (PC, MK)<br />

Fasting GLC, electrolytes, BUN/Cr. (PC, MK)<br />

Hepatic function panel. (PC, MK)<br />

CK. (PC, MK)<br />

5. Communication skills: Students should be able to:<br />

Communicate the diagnosis, treatment plan, and subsequent follow-up to patients. (PC,<br />

CS)<br />

Elicit questions from the patient and his or her family about the management plan. (PC, CS)<br />

Counsel patients about dietary measures to reduce cholesterol and saturated fats. (PC,<br />

CS)<br />

Counsel patients about ways to increase exercise. (PC, CS)<br />

Counsel patients about other modifiable cardiovascular risk factors. (PC, CS)<br />

6. Management skills: Students should able to develop an appropriate evaluation and<br />

treatment plan for patients that includes:<br />

An individual treatment plan that follows the NCEP ATP III guidelines. (PC, MK)<br />

Lifestyle modification (diet, exercise). (PC, MK)<br />

Appropriate pharmacologic interventions, including bile acid sequestrants (resins), nicotinic<br />

acid, fibric acid derivatives, HMG-CoA reductase inhibitors (statins), and cholesterol<br />

absorption inhibitors (ezetimibe). (PC, MK)<br />

Monitoring for adherence and side effects due to pharmacologic management. (PC, MK)<br />

142


Laboratory response to therapy. (PC, MK)<br />

Identifying barriers that prevent patients from adhering to recommended dietary, exercise,<br />

and pharmacologic plans. (PC, MK)<br />

Determining when to obtain consultation from an endocrinologist, or dietician. (PC, SBP)<br />

Using a cost-effective approach based on the differential diagnosis. (PC, SBP)<br />

Accessing and utilizing appropriate information systems and resources to help delineate<br />

issues related to dyslipidemia. (PC, PLI)<br />

Incorporating patient preferences. (PC)<br />

C. ATTITUDES AND PROFESSIONAL BEHAVIORS: Students should be able to:<br />

1 Demonstrate a commitment to meeting NCEP ATP III guidelines to insure quality care <strong>of</strong><br />

patients with dyslipidemia. (PLI, P, SBP)<br />

2 Appreciate the importance <strong>of</strong> encouraging patients to assume responsibility for modifying their<br />

diet and increasing their exercise level. (P, CS)<br />

3 Appreciate the difficulties and frustrations that patients and health care providers face with<br />

recommended dietary changes. (P)<br />

4 Demonstrate commitment to using risk-benefit, cost-benefit, and evidence-based<br />

considerations in the selection diagnostic and therapeutic interventions for dyslipidemia. (PLI,<br />

P)<br />

5 Recognize the importance <strong>of</strong> patient preferences when selecting among diagnostic and<br />

therapeutic options for dyslipidemia. (P)<br />

6 Respond appropriately to patients who are nonadherent to treatment for dyslipidemia. (CS, P)<br />

7 Demonstrate ongoing commitment to self-directed learning regarding dyslipidemia. (PLI, P)<br />

8 Recognize the importance <strong>of</strong> and demonstrate a commitment to the utilization <strong>of</strong> other<br />

healthcare pr<strong>of</strong>essionals in the treatment <strong>of</strong> dyslipidemia. (P, SBP)<br />

D. REFERENCES:<br />

National Institutes <strong>of</strong> Health, National Heart, Lung, and Blood Institute National<br />

Cholesterol Education Program Clinical Practice Guidelines for Cholesterol<br />

Management in Adults (ATP III)<br />

www.nhlbi.nih.gov/about/ncep<br />

Grundy SM, Cleeman JI, Merz CN, et al; National Heart, Lung, and Blood Institute;<br />

American College <strong>of</strong> Cardiology Foundation; American Heart Association. Implications<br />

<strong>of</strong> recent clinical trials for the National Cholesterol Education Program Adult Treatment<br />

Panel III guidelines. Circulation. 2004;110:227-39.<br />

Ashen MD, Blumenthal RS. Clinical practice. Low HDL cholesterol levels. N Engl J<br />

Med. 2005;353:1252-60.<br />

Hansson GK. Inflammation, atherosclerosis, and coronary artery disease. N Engl J<br />

Med. 2005;352:1685-95.<br />

143


TRAINING PROBLEM #22: HEART FAILURE<br />

RATIONALE:<br />

Chronic heart failure (HF) is one <strong>of</strong> the most common cardiac problems encountered in clinical<br />

practice. Identification and correction <strong>of</strong> treatable underlying causes, control <strong>of</strong> precipitating factors<br />

and judicious use <strong>of</strong> multi-drug regimens for individuals with HF are important issues for third-year<br />

medical students.<br />

PREREQUISITES:<br />

Prior knowledge, skills and attitudes acquired during the pre-<strong>clerkship</strong> years should include:<br />

Knowledge <strong>of</strong> the structure and function <strong>of</strong> the heart and lungs.<br />

Understanding <strong>of</strong> the epidemiology <strong>of</strong> heart disease.<br />

Knowledge <strong>of</strong> the atherogenesis and pathophysiology <strong>of</strong> cardiovascular disease.<br />

Knowledge <strong>of</strong> the pharmacology <strong>of</strong> cardiovascular drugs.<br />

Ability to communicate appropriately with all types <strong>of</strong> patients including the elderly and those<br />

with diverse backgrounds.<br />

Ability to perform a complete medical history and physical exam.<br />

Ability to perform a cardiovascular risk assessment and understand issues related to primary<br />

and secondary prevention <strong>of</strong> cardiovascular disease.<br />

Ability to understand the impact <strong>of</strong> illness on individuals and their families and, when<br />

appropriate, to address issues related to end-<strong>of</strong>-life care.<br />

SPECIFIC LEARNING OBJECTIVES:<br />

A. KNOWLEDGE: Students should be able to define, describe, and discuss:<br />

1. Types <strong>of</strong> processes and most common disease entities that cause HF (i.e. ischemic, valvular,<br />

hypertrophic, infiltrative, inflammatory, etc.). (MK)<br />

2. The basic role <strong>of</strong> genetics in certain forms <strong>of</strong> cardiomyopathy. (MK)<br />

3. Staging system for heart failure:<br />

Stage A: high risk for HF but no structural heart disease is present. (MK)<br />

Stage B: structural heart disease is present but never any symptoms. (MK)<br />

Stage C: past or current symptoms associated with structural heart disease. (MK)<br />

Stage D: end-stage disease with requirements for specialized treatment. (MK)<br />

4. Types <strong>of</strong> processes that cause systolic vs. diastolic dysfunction. (MK)<br />

5. Symptoms and signs <strong>of</strong> left-sided vs. right-sided heart failure. (MK)<br />

6. Compensatory mechanisms <strong>of</strong> heart failure including cardiac remodeling and activation <strong>of</strong><br />

endogenous neurohormonal systems. (MK)<br />

7. Factors leading to symptomatic exacerbation <strong>of</strong> HF, including ischemia, arrhythmias,<br />

hypoxemia, anemia, fever, hypertension, thyroid disorders, non-compliance with medications<br />

and dietary restrictions and use <strong>of</strong> nonsteroidal anti-inflammatory drugs. (MK)<br />

8. Importance <strong>of</strong> age, gender and ethnicity on the prevalence and<br />

prognosis <strong>of</strong> HF. (MK)<br />

9. Physiological basis and scientific evidence supporting each type<br />

<strong>of</strong> treatment, intervention, or procedure commonly used in the management <strong>of</strong> patients who<br />

present with HF. (MK)<br />

10. The general approach to the evaluation and treatment <strong>of</strong> atrial<br />

144


fibrillation (MK)<br />

11. Role <strong>of</strong> critical pathways or practice guidelines in delivering<br />

high-quality, cost effective care for patients presenting with new or recurrent heart failure. (PC,<br />

SBP)<br />

12. The Centers for Medicare & Medicaid Services (CMS) and the<br />

Joint Commission on the Accreditation <strong>of</strong> Healthcare Organizations (JCAHO) quality measures<br />

for HF treatment. (MK, PLI, SBP)<br />

B. SKILLS: Students should be able to demonstrate specific skills, including:<br />

1. History-taking skills: Students should be able to obtain, document, and present an<br />

age-appropriate medical history, including:<br />

Differentiating between various etiologies <strong>of</strong> heart failure (answers the question: Why is the<br />

patient in heart failure?). (PC, CS)<br />

Identifying clinical factors responsible for symptomatic exacerbation (answers the question:<br />

Why is the patient worse now?). (PC, CS)<br />

Exercise intolerance (fatigue, dyspnea on exertion). (PC, CS)<br />

Fluid retention (peripheral edema, dyspnea). (PC, CS)<br />

Changes in sleep pattern (orthopnea, paroxysmal nocturnal dyspnea [PND], nocturia). (PC,<br />

CS)<br />

Assessing the functional capacity <strong>of</strong> patients with HF (walking distance, New York Heart<br />

classification). (PC, CS)<br />

Cardiac risk factors. (PC, CS)<br />

2. Physical exam skills: Students should be able to perform a focused physical exam to<br />

help establish the diagnosis <strong>of</strong> HF and estimate its severity:<br />

Measurement <strong>of</strong> vital signs including weight and respiratory rate/pattern. (PC)<br />

Accurate measurement <strong>of</strong> arterial blood pressure and recognition <strong>of</strong> the typical blood<br />

pressure findings that occur in patients with aortic stenosis, aortic insufficiency and pulsus<br />

paradoxus. (PC)<br />

Assessment <strong>of</strong> major arterial pulses for abnormalities, including bruits. (PC)<br />

Assessment <strong>of</strong> the neck veins for jugular venous distention and, when necessary,<br />

evaluation for abdominal jugular reflux. (PC)<br />

Assessment <strong>of</strong> the conjunctiva and optic fundus. (PC)<br />

Assessment <strong>of</strong> the extremities to ascertain for skin conditions, including color, temperature<br />

and the presence <strong>of</strong> edema, cyanosis or clubbing. (PC)<br />

Assessment <strong>of</strong> the lungs for crackles, rhonchi and decreased breath sounds. (PC)<br />

Inspection and palpation <strong>of</strong> the anterior chest to identify right and left sided heaves, lifts and<br />

thrills. (PC)<br />

Auscultation <strong>of</strong> the heart to determine rhythm, intensity <strong>of</strong> heart sounds, splitting <strong>of</strong> S2 and<br />

the presence <strong>of</strong> rubs, gallops (S3, S4, summation) or extra heart sounds (e.g. clicks). (PC)<br />

Auscultation <strong>of</strong> the heart to detect the presence <strong>of</strong> heart murmurs; when a murmur is<br />

present, students should be able to:<br />

o Identify timing (systolic vs. diastolic, holosystolic vs. ejection). (PC)<br />

o Describe pitch, location and pattern <strong>of</strong> radiation. (PC)<br />

o Gauge significance (innocent vs. pathologic, sclerosis vs. stenosis). (PC)<br />

Assessment <strong>of</strong> the abdomen to determine the presence <strong>of</strong> hepatomegaly,<br />

ascites, abnormal pulsations and bruits. (PC)<br />

3. Differential diagnosis: Students should be able to generate a prioritized differential<br />

145


diagnosis and recognize specific history, physical exam and/or laboratory findings that:<br />

Help support or refute a clinical diagnosis <strong>of</strong> heart failure. (PC, MK)<br />

Distinguish between various underlying etiologies <strong>of</strong> HF, including disease processes that<br />

primarily affect:<br />

o Pericardium (constrictive pericarditis, pericardial tamponade). (PC, MK)<br />

o Endocardium (valvular [congenital, acquired], endocarditis). (PC, MK)<br />

o Myocardium (hypertrophic, restrictive, congestive). (PC, MK)<br />

4. Laboratory interpretation: Students should be able interpret specific diagnostic tests and<br />

procedures that are commonly ordered to evaluate patients who present with heart failure.<br />

Test interpretation should take into account: Laboratory and diagnostic tests should include,<br />

when appropriate:<br />

12-lead ECG. (PC, MK)<br />

Chest radiograph. (PC, MK)<br />

B-type natriuretic peptide. (PC, MK)<br />

Students should be able to define the indications for, and interpret (with consultation) the<br />

results <strong>of</strong> the following diagnostic tests and procedures:<br />

Echocardiography. (PC, MK)<br />

Treadmill and nuclear exercise testing. (PC, MK)<br />

Radionuclide ventriculogram. (PC, MK)<br />

Cardiac. (PC, MK)<br />

Coronary angiography. (PC, MK)<br />

5. Communication skills: Students should be able to:<br />

Communicate the diagnosis, prognosis and treatment plan to the patient and his or her<br />

family. (PC, CS)<br />

Elicit questions from the patient and his or her family about the management plan. (PC, CS)<br />

Educate patients about cardiovascular risk factors. (PC, CS)<br />

Council patients regarding a sodium-restricted diet. (PC, CS)<br />

Address palliative care and end-<strong>of</strong>-life issues with patients who have<br />

intractable symptoms associated with end-stage heart failure. (PC, CS, P)<br />

6. Basic and advanced procedural skills: students should be able to:<br />

Perform a 12-lead ECG. (PC)<br />

Obtain an ABG. (PC)<br />

7. Management skills: Students should be able to develop an appropriate evaluation and<br />

treatment plan for patients that includes:<br />

Recognize the importance <strong>of</strong> early detection and treatment <strong>of</strong> risk factors that may lead to<br />

the development <strong>of</strong> heart failure. (PC)<br />

Identifying the indications, contraindications, mechanisms <strong>of</strong> action, adverse reactions,<br />

significant interactions, and relative costs <strong>of</strong> the following treatments/interventions:<br />

o Non-pharmacological management. (PC, MK)<br />

-Sodium restriction. (PC, MK)<br />

-Physical activity and limitations. (PC, MK)<br />

o Pharmacological management (recommended for routine use). (PC, MK)<br />

-Diuretics. (PC, MK)<br />

-ACE-I/ARB. (PC, MK)<br />

-Beta-blockers. (PC, MK)<br />

-Aldosterone antagonists (spironolactone, eplerenone). (PC, MK)<br />

-digoxin. (PC, MK)<br />

o Interventions considered for use in selected patients. (PC, MK)<br />

-Hydralazine and isosorbide dinitrate. (PC,MK)<br />

146


-Angoitensin receptor blockers. (PC, MK)<br />

-Calcium channel blockers. (PC, MK)<br />

-Anti-arrhythmic agents. (PC, MK)<br />

-Anticoagulants/anti a thrombotic agents. (PC, MK)<br />

o other modalities(PC, MK)<br />

-Coronary revascularization. (PC, MK)<br />

-Synchronized biventricular pacing. (PC, MK)<br />

-Implantable cardiac defibrillators. (PC, MK)<br />

Developing a timely and appropriate evaluation and treatment plan for patients with heart<br />

failure due to diastolic dysfunction, including:<br />

o Control <strong>of</strong> physiologic factors (blood pressure, heart rate). (PC, MK)<br />

o Reduction in central blood volume by judicious use <strong>of</strong> diuretics. (PC, MK)<br />

o Alleviation <strong>of</strong> myocardial ischemia. (PC, MK)<br />

o Use <strong>of</strong> calcium channel blockers. (PC, MK)<br />

Describing use <strong>of</strong> other agents and interventions that may be useful in treating patients with<br />

refractory, end-stage heart failure:<br />

o Intravenous vasodilators. (PC, MK)<br />

o Intravenous positive inotropic agents. (PC, MK)<br />

o Infusion <strong>of</strong> B-type natriuretic peptide (nesiritide). (PC, MK)<br />

o Ventricular assist devices. (PC, MK)<br />

o Heart transplantation. (PC, MK)<br />

Defining and describing how the diagnosis and treatment <strong>of</strong> HF in special<br />

populations may differ (e.g. very elderly, associated co-morbidities). (PC, MK)<br />

Demonstrating how critical pathways or practice guidelines in ambulatory or hospitalized<br />

patients with HF can be used to guide diagnostic test ordering and medical decision<br />

making. (PC, PLI, SBP)<br />

Determining when to consult a cardiologist. (PC, SBP)<br />

Identifying when palliative care may be appropriate for patients with refractory symptoms<br />

associated with end-stage disease. (PC)<br />

Using a cost-effective approach based on the differential diagnosis. (PC, SBP)<br />

Accessing and utilizing appropriate information systems and resources to help delineate<br />

issues related to HF. (PC, PLI)<br />

Incorporating patient preferences. (PC)<br />

C. ATTITUDES AND PROFESSIONAL BEHAVIORS: Students should be able to:<br />

1. Demonstrate commitment to using risk-benefit, cost-benefit, and evidence-based<br />

considerations in the selection diagnostic and therapeutic interventions for HF. (PLI, P)<br />

2. Recognize the significant morbidity and mortality associated with HF (P)<br />

3. Recognize the impact <strong>of</strong> lifestyle limitations caused by HF. (P)<br />

4. Respond appropriately to patients who are non-adherent to treatment for HF. (CS, P)<br />

5. Demonstrate ongoing commitment to self-directed learning regarding heart failure. (PLI, P)<br />

6. Recognize the importance and demonstrate a commitment to the utilization <strong>of</strong> other healthcare<br />

pr<strong>of</strong>essions in the treatment <strong>of</strong> heart failure. (P, SBP)<br />

7. Appreciate the importance <strong>of</strong> and demonstrate a commitment to meeting national health care<br />

quality measures for the treatment <strong>of</strong> HF. (PLI, P, SBP)<br />

147


D. REFERENCES:<br />

ACC/AHA 2005 guideline update for the diagnosis and management <strong>of</strong> chronic<br />

heart failure in the adult: a report <strong>of</strong> the American College <strong>of</strong> Cardiology/American<br />

Heart Association Task Force on Practice Guidelines (Writing Committee to<br />

Update the 2001 Guidelines for the Evaluation and Management <strong>of</strong> Heart Failure).<br />

J Am Coll Cardiol. 2005;46:1116-43.<br />

http://www.acc.org/clinical/guidelines/failure/update/index.pdf<br />

Cook DJ, Simel DL. Does this patient have abnormal central venous pressure? JAMA.<br />

1996; 275: 630-634.<br />

Wang CS. FitzGerald JM. Schulzer M. Mak E. Ayas NT. Does this dyspneic patient in<br />

the emergency department have congestive heart failure? JAMA. 2005;294:1944-56.<br />

Doust JA. Glasziou PP. Pietrzak E. Dobson AJ. A systematic review <strong>of</strong> the diagnostic<br />

accuracy <strong>of</strong> natriuretic peptides for heart failure. Arch Intern Med. 2004;164:1978-84.<br />

Yan AT. Yan RT. Liu PP. Narrative review: pharmacotherapy for chronic heart failure:<br />

evidence from recent clinical trials. Ann Intern Med. 2005;142(2):132-45.<br />

Aurigemma GP. Gaasch WH. Clinical practice. Diastolic heart failure. N Engl J Med.<br />

2004;351:1097-105.<br />

148


TRAINING PROBLEM #23: HIV INFECTION<br />

RATIONALE:<br />

HIV infection and AIDS represent one <strong>of</strong> the most difficult challenges in clinical medicine today. An<br />

HIV specialist (usually an infectious diseases physician) cares for the vast majority <strong>of</strong> patients with<br />

HIV infection and AIDS. Given that there is no proven cure, this remains an important training<br />

problem for third year medical students. The enormous and continuously evolving complexities <strong>of</strong><br />

antiretroviral treatment are generally beyond the level <strong>of</strong> the third year medical student and for that<br />

matter most general internists. Rather, an approach to HIV infection, AIDS, and its most common<br />

and serious complications are stressed.<br />

PREREQUISITES:<br />

Prior knowledge, skills, and attitudes acquired during the pre-clinical experience should include:<br />

Ability to perform a complete medical history and physical.<br />

Ability to communicate with patients <strong>of</strong> diverse backgrounds.<br />

Knowledge <strong>of</strong> the worldwide epidemiology, biology, and immunology <strong>of</strong> HIV.<br />

Microbiology <strong>of</strong> common opportunistic organisms.<br />

Pharmacology <strong>of</strong> antimicrobial agents and antiretrovirals.<br />

Understanding <strong>of</strong> universal precautions.<br />

SPECIFIC LEARNING OBJECTIVES:<br />

A. KNOWLEDGE: Students should be able to define, describe, and discuss:<br />

1. Symptoms and signs <strong>of</strong> acute HIV seroconversion. (MK)<br />

2. CDC AIDS case definition. (MK)<br />

3. Specific tests for HIV (e.g. HIV ELISA, confirmatory western blot, quantitative PCR) and their<br />

operating characteristics. (MK)<br />

4. Relationship <strong>of</strong> CD4 lymphocyte count to opportunistic infections as well as relationship between<br />

CD4 lymphocyte count and viral load to overall disease progression. (MK)<br />

5. The basic principles <strong>of</strong> highly active antiretroviral therapy (HAART), including the different classes<br />

<strong>of</strong> antiviral medications and their use, as well as common side effects and drug-drug<br />

interactions. (MK)<br />

6. Basics <strong>of</strong> post-exposure prophylaxis. (MK)<br />

7. The marked importance <strong>of</strong> antiretroviral medication adherence and the potential consequences <strong>of</strong><br />

erratic or poor adherence. (MK)<br />

8. Vaccination recommendation for patients infected with HIV. (MK)<br />

9. Indications for and utility and risks <strong>of</strong> prophylaxis <strong>of</strong> HIV-related opportunistic infections. (MK)<br />

10. Pathogenesis, symptoms, signs, typical clinical course, and management <strong>of</strong> HIV-related<br />

opportunistic infections with a recognition <strong>of</strong> which are most common:<br />

Pneumocystis jiroveci. (MK)<br />

Candidiasis (oral, esophageal, vaginal). (MK)<br />

Cryptococcus ne<strong>of</strong>ormans. (MK)<br />

Cryptosporidium parvum. (MK)<br />

Cytomegalovirus infection (gastrointestinal, neurologic, retinal). (MK)<br />

Varicella-zoster virus. (MK)<br />

Isospora belli. (MK)<br />

149


Microsporidiosis. (MK)<br />

Mycobacterium avium complex. (MK)<br />

Mycobacterium tuberculosis. (MK)<br />

Toxoplasma gondii. (MK)<br />

11. Symptoms and signs <strong>of</strong> the following HIV-related malignancies:<br />

Kaposi’s sarcoma. (MK)<br />

Non-Hodgkin’s lymphoma. (MK)<br />

Cervical carcinoma. (MK)<br />

12. Common skin and oral manifestations <strong>of</strong> HIV infection and AIDS:<br />

Molluscum contagiosum. (MK)<br />

Cryptococcus ne<strong>of</strong>ormans. (MK)<br />

Viral warts. (MK)<br />

Lipodystrophy. (MK)<br />

Herpes zoster. (MK)<br />

Seborrhoeic dermatitis. (MK)<br />

Buccal candidiasis. (MK)<br />

Oral hairy leukoplakia. (MK)<br />

13. “Safe sex” practices (MK)<br />

14. The importance <strong>of</strong> proper ongoing dental care. (MK)<br />

B. SKILLS: Students should be able to demonstrate specific skills, including:<br />

1. History-taking skills: Students should be able to obtain, document, and present an ageappropriate<br />

medical history that differentiates among etiologies <strong>of</strong> disease, including:<br />

HIV infection risk factors. (PC, CS)<br />

Sexual contacts. (PC, CS)<br />

Parenteral exposure to infected blood by needle sharing or transfusion. (PC, CS)<br />

Occupational exposures. (PC, CS)<br />

Other sexually transmitted diseases. (PC, CS)<br />

Tuberculosis exposure. (PC, CS)<br />

Prior HIV serology results, CD4 lymphocyte count and viral load. (PC, CS)<br />

Prior HIV-related opportunistic infections. (PC, CS)<br />

Current/prior antiretroviral medications and their side effects. (PC, CS)<br />

Fever, sweats, anorexia, unintentional weight loss, rash/skin lesions, lymphadenopathy.<br />

(PC, CS)<br />

Cough, sputum production, dyspnea, chest pain. (PC, CS)<br />

Headache, altered mental status, psychiatric complaints. (PC, CS)<br />

Odynophagia, dysphagia. (PC, CS)<br />

Vaginal discharge, history <strong>of</strong> cervical dysplasia or neoplasia. (PC, CS)<br />

Diarrhea. (PC, CS)<br />

Visual changes. (PC, CS)<br />

A dietary history to determine caloric intake. (PC, CS)<br />

2. Physical exam skills: Students should be able to perform a physical exam to establish the<br />

diagnosis and severity <strong>of</strong> disease, including:<br />

General appearance regarding atrophy/wasting/cachexia. (PC)<br />

Complete neurologic examination. (PC)<br />

150


Mental status examination. (PC)<br />

Fundoscopic examination. (PC)<br />

Lymph node examination. (PC)<br />

Skin and oral examination. (PC)<br />

Pelvic and male genital examination. (PC)<br />

3. Differential diagnosis: Students should be able to generate a prioritized differential diagnosis<br />

recognizing specific history and physical exam findings that suggest a specific etiology in an<br />

potentially or known HIV-infected patient for the following:<br />

Fever. (PC, MK)<br />

Unintentional weight loss/wasting/cachexia. (PC, MK)<br />

Lymphadenopathy. (PC, MK)<br />

Rash and skin lesions. (PC, MK)<br />

Cough, sputum production, dyspnea, abnormal chest radiography. (PC, MK)<br />

Diarrhea, odynophagia, dysphagia. (PC, MK)<br />

Altered mental status and psychiatric changes. (PC, MK)<br />

Headache. (PC, MK)<br />

Oral lesions. (PC, MK)<br />

Visual/retinal abnormalities. (PC, MK)<br />

4. Laboratory interpretation: Students should be able to recommend when to order diagnostic<br />

and laboratory tests and be able to interpret them, both prior to and after initiating treatment<br />

based on the differential diagnosis, including consideration <strong>of</strong> test cost and performance<br />

characteristics as well as patient preferences.<br />

Laboratory and diagnostic tests should include, when appropriate:<br />

Specific tests for HIV (e.g. HIV ELISA, confirmatory western blot, quantitative PCR). (PC,<br />

MK)<br />

CD4 lymphocyte count. (PC, MK)<br />

CBC with differential. (PC, MK)<br />

Sputum staining and cultures. (PC, MK)<br />

Blood cultures. (PC, MK)<br />

Cerebrospinal fluid analysis (color, opening pressure, chemistries, cell counts, staining,<br />

cultures, cytology, cryptococcal antigen, VDRL,<br />

Ebstein Barr virus, cytomegalovirus, toxoplasmosis, JC virus). (PC, MK)<br />

Stool for ova and parasites, cryptosporium, isospora, microsporidia, cytomegalovirus<br />

antigen. (PC, MK)<br />

Chest radiograph. (PC, MK)<br />

Students should be able to define the indications for and interpret (with consultation) the<br />

results <strong>of</strong>:<br />

Chest CT. (PC, MK)<br />

Cranial CT. (PC, MK)<br />

Cranial MRI. (PC, MK)<br />

5. Communication skills: Students should be able to:<br />

Communicate the diagnosis, treatment plan, and subsequent follow-up to the patient and<br />

his or her family. (PC, CS)<br />

Elicit input and questions from the patient and his or her family about the management<br />

plan. (PC, CS)<br />

counsel and educate patients about HIV exposure prevention (PC, CS)<br />

Counsel an exposed patient about seroconversion rates and, in appropriate situations, the<br />

availability <strong>of</strong> post-exposure prophylaxis. (PC, CS)<br />

151


Counsel and educate patients about complications <strong>of</strong> HIV drug therapy, drug-drug<br />

interactions, and the marked importance <strong>of</strong> adherence. (PC, CS)<br />

6. Basic and advanced procedural skills: Students should be able to:<br />

Obtain blood cultures. (PC)<br />

Obtain an ABG. (PC)<br />

Place and interpret a PPD. (PC)<br />

Assist in performing a lumbar puncture after explaining the procedure to the patient. (PC)<br />

7. Management skills: Students should able to develop an appropriate evaluation and<br />

treatment plan for patients that includes:<br />

Ordering appropriate laboratory tests. (PC, MK)<br />

Advising patients regarding HIV transmission prevention. (PC, MK)<br />

Insuring antiretroviral adherence. (PC, MK)<br />

Following parameters <strong>of</strong> disease progression/activity. (e.g. CD4 lymphocyte count, viral<br />

load). (PC, MK)<br />

Monitoring for the development <strong>of</strong> side effects from antiretroviral treatment and drug-drug<br />

interactions. (PC, MK)<br />

Insuring the administration <strong>of</strong> appropriate vaccinations. (PC, MK)<br />

Assessing PPD status. (PC, MK)<br />

Prescribing and monitoring appropriate opportunist infection prophylaxis. (PC, MK)<br />

Ordering nutritional supplements to manage and prevent malnutrition. (PC, MK)<br />

Assisting in the procurement <strong>of</strong> proper and ongoing dental care. (PC, MK)<br />

Identifying and recommending community health care resources available for the care <strong>of</strong><br />

AIDS patients. (PC, SBP)<br />

Determining when to obtain consultation from an infectious diseases specialist. (PC, SBP)<br />

Using a cost-effective approach based on the differential diagnosis. (PC, SBP)<br />

Accessing and utilizing appropriate information systems and resources to help delineate<br />

issues related to HIV infection and AIDS. (PC, PLI)<br />

Incorporating patient need and preferences. (PC)<br />

C. ATTITUDES AND PROFESSIONAL BEHAVIORS: Students should be able to:<br />

1. Appreciate the bioethical, social, and legal issues concerning patient confidentiality <strong>of</strong> HIV<br />

infection. (PC, CS)<br />

2. Demonstrate a nonjudgmental attitude regarding the mode <strong>of</strong> HIV acquisition. (P)<br />

3 Appreciate the sometimes severe social stigma <strong>of</strong> HIV infection and AIDS. (P)<br />

4 Show respect <strong>of</strong> “alternative lifestyles.” (P)<br />

5 Demonstrate commitment to using risk-benefit, cost-benefit, and evidence-based<br />

considerations in the selection <strong>of</strong> diagnostic and therapeutic interventions for HIV infection and<br />

AIDS. (PLI, P)<br />

6 Recognize the importance <strong>of</strong> patient needs and preferences when selecting among diagnostic<br />

and therapeutic options for patients with HIV infection or AIDS. (P)<br />

7 Respond appropriately to patients who are nonadherent to antiretroviral treatment. (CS, P)<br />

8 Demonstrate ongoing commitment to self-directed learning regarding HIV infection and AIDS.<br />

(PLI, P)<br />

9 Appreciate the impact HIV infection and AIDS have on a patient’s quality <strong>of</strong> life, well-being,<br />

ability to work, and the family. (P)<br />

10 Recognize the importance <strong>of</strong> and demonstrate a commitment to the utilization <strong>of</strong> other health<br />

care pr<strong>of</strong>essionals in the diagnosis and treatment <strong>of</strong> HIV infection and AIDS. (P, SBP)<br />

152


D. REFERENCES:<br />

Guidelines for preventing Opportunistic Infections Among HIV-Infected Persons-2002.<br />

Recommendations <strong>of</strong> the U.S. Public Health Service and the Infectious Diseases Society <strong>of</strong><br />

America. MMWR 51(RR-08);1-60. Centers for Disease Control and Prevention<br />

U.S. <strong>Department</strong> <strong>of</strong> Health and Human Services<br />

www.cdc.gov/mmwr/PDF/RR/RR5108.pdf<br />

Guidelines for Using Antiretroviral Agents Among HIV-Infected Adults and Adolescents-<br />

2002. Recommendations <strong>of</strong> the Panel on Clinical Practices for Treatment <strong>of</strong> HIV. MMWR<br />

51(RR-07);1-64.<br />

Centers for Disease Control and Prevention<br />

U.S. <strong>Department</strong> <strong>of</strong> Health and Human Services<br />

www.cdc.gov/mmwr/PDF/RR/RR5107<br />

Aberg JA, Gallant JE, Anderson J, Oleske JM, Libman H, Currier JS, Stone VE, Kaplan,<br />

JE. Primary care guideline for the management <strong>of</strong> persons infected with human<br />

immunodeficiency virus: recommendation <strong>of</strong> the HIV <strong>Medicine</strong> Association <strong>of</strong> the Infectious<br />

Diseases Society <strong>of</strong> America. Clin Infect Dis. 2004;39:609-29.<br />

www.journals.uchicago.edu/CID/journal/issues/v39n5/34135/34135.web.p df<br />

www.guideline.gov/summary/summary.aspx?view_id=1&doc_id=5625<br />

AIDSinfo National Institutes <strong>of</strong> Health<br />

U.S. <strong>Department</strong> <strong>of</strong> Health and Human Services<br />

www.aidsinfo.nih.gov<br />

Divisions <strong>of</strong> HIV/AIDS Prevention National Center for HIV, STD and TB Prevention Centers<br />

for Disease Control and Prevention U.S. <strong>Department</strong> <strong>of</strong> Health and Human Services<br />

www.cdc.gov/hiv/dhap.htm<br />

Hammer SM. Management <strong>of</strong> newly diagnosed HIV infection. N Engl J Med.<br />

2005;353:1702-10.<br />

153


TRAINING PROBLEM #24: HYPERTENSION<br />

RATIONALE:<br />

As many as 50 million Americans have elevated blood pressure (systolic pressure 140 mmHg or<br />

greater and/or diastolic blood pressure 90 mmHg or greater) or are taking antihypertensive<br />

medication. Nonfatal and fatal cardiovascular disease (CVD)— including coronary heart disease<br />

(CHD), peripheral vascular disease, stroke and renal disease—all increase progressively with higher<br />

levels <strong>of</strong> both systolic (SBP) and diastolic (DBP) blood pressure levels. These relationships are<br />

strong, continuous, independent, predictive and etiologically significant, and indicate that reduction <strong>of</strong><br />

blood pressure reduces these risks.<br />

PREREQUISITES:<br />

Prior knowledge, skills and attitudes acquired during the pre-clinical years should include:<br />

Ability to perform a complete medical history and physical exam.<br />

Ability to communicate with patients <strong>of</strong> diverse backgrounds.<br />

Knowledge <strong>of</strong> the pathogenesis and pathophysiology <strong>of</strong> hypertension.<br />

Knowledge <strong>of</strong> the epidemiology and risk factors for hypertension.<br />

Understanding <strong>of</strong> the pharmacologic management <strong>of</strong> acute and chronic hypertension.<br />

Understanding the behavioral issues by sex, race, culture, and age that relate to the<br />

management and treatment <strong>of</strong> hypertension.<br />

SPECIFIC LEARNING OBJECTIVES:<br />

A. KNOWLEDGE: Students should be able to define, describe and discuss:<br />

1. The etiologies and relative prevalence <strong>of</strong> primary and secondary hypertension. (MK)<br />

2. The basic principles <strong>of</strong> the role <strong>of</strong> genetics in hypertension. (MK)<br />

3. The definition <strong>of</strong> hypertensive urgency and emergency, citing examples <strong>of</strong> both. (MK)<br />

4. The difference between essential (primary) and secondary hypertension. (MK)<br />

5. Symptoms and signs <strong>of</strong> the following disorders associated with secondary hypertension:<br />

Renovascular hypertension. (MK)<br />

Renal failure. (MK)<br />

Polycystic kidney disease. (MK)<br />

Cushing’s disease or syndrome. (MK)<br />

Hyperaldosteronism. (MK)<br />

Hyperthyroidism. (MK)<br />

Hypercalcemia. (MK)<br />

Medication, alcohol, and illicit drug use. (MK)<br />

Coarctation <strong>of</strong> the aorta. (MK)<br />

Sleep apnea. (MK)<br />

6. The manifestations <strong>of</strong> target-organ disease due to hypertension. (MK)<br />

7. Classification <strong>of</strong> blood pressure (SBP and DBP for all age 18 or older). (MK)<br />

8. Basic approaches to the pharmacological management <strong>of</strong> acute and chronic hypertension,<br />

including the physiologic basis and scientific evidence supporting these approaches, and<br />

causes for lack <strong>of</strong> responsiveness to therapy. (MK)<br />

9. Prevention strategies for reducing hypertension (including lifestyle factors, such as dietary<br />

intake <strong>of</strong> sodium, weight, and exercise level), and explain the physiologic basis and/or<br />

154


scientific evidence supporting each strategy. (MK)<br />

10. Steps in management <strong>of</strong> patients with a hypertensive emergency. (MK)<br />

11. Factors that contribute to non-adherence with antihypertensive medications. (MK)<br />

B. SKILLS: Students should demonstrate specific skills including:<br />

1. History-taking skills: Students should be able to obtain, document, and present an ageappropriate<br />

medical history that differentiates among etiologies <strong>of</strong> disease, including:<br />

Duration and levels <strong>of</strong> elevated blood pressure. (PC, CS)<br />

History <strong>of</strong> symptoms <strong>of</strong> cardiovascular, cerebrovascular, peripheral vascular or renal<br />

disease; diabetes; dyslipidemia; or gout. (PC, CS)<br />

History <strong>of</strong> symptoms suggesting secondary hypertension. (PC, CS)<br />

History <strong>of</strong> weight gain, leisure-time physical activities, and smoking or other tobacco use.<br />

(PC, CS)<br />

Family history <strong>of</strong> high blood pressure, premature CHD, stroke, CVD, diabetes mellitus and<br />

dyslipidemia. (PC, CS)<br />

Psychosocial and environmental factors that may elevate blood pressure (family situation,<br />

employment status, working conditions, education level). (PC, CS)<br />

Dietary assessment, including sodium intake and intake <strong>of</strong> saturated fat and cholesterol.<br />

(PC, CS)<br />

Results and side effects <strong>of</strong> previous antihypertensive therapy. (PC, CS)<br />

Use <strong>of</strong> commonly prescribed, over-the -counter, and illicit medications that may raise blood<br />

pressure or interfere with the effectiveness <strong>of</strong> antihypertensive medications. (PC, CS)<br />

Alcohol intake. (PC, CS)<br />

2. Physical exam skills: Students should be able to perform a physical exam to establish the<br />

diagnosis and severity <strong>of</strong> disease, including:<br />

Blood pressure measurements to detect and confirm the presence <strong>of</strong> high blood pressure.<br />

(PC)<br />

Examination <strong>of</strong> the fundus for arteriolar narrowing, arteriovenous nicking, hemorrhages,<br />

exudates, or papilledema. (PC)<br />

Neck for carotid bruits, distended veins, or an enlarged thyroid gland. ((PC)<br />

Heart for increased rate, increased size, precordial heave, clicks, murmurs, arrhythmias,<br />

and third (S3) and fourth (S4) sounds. (PC)<br />

Abdomen for bruits, enlarged kidneys, masses, and abnormal aortic pulsation. (PC)<br />

Extremities for diminished, delayed, or absent peripheral arterial pulsations, bruits, and<br />

edema. (PC)<br />

Peripheral pulses specifically femoral arterial pulses. (PC)<br />

Body habitus, looking for changes associated with secondary hypertension. (PC)<br />

Peripheral and central nervous system for ischemic changes. (PC)<br />

3. Differential diagnosis: Students should be able to generate a prioritized differential<br />

diagnosis recognizing specific history, physical exam, and laboratory findings that suggest a<br />

specific etiology <strong>of</strong> hypertension. (PC, MK)<br />

4. Laboratory interpretation: Students should be able to recommend and interpret<br />

diagnostic and laboratory tests, both prior to and after initiating treatment based on the<br />

differential diagnosis, including consideration <strong>of</strong> test cost and performance characteristics as<br />

well as patient preferences. Laboratory and diagnostic tests should include, when appropriate:<br />

UA. (PC, MK)<br />

CBC. (PC, MK)<br />

Blood glucose (fasting if possible). (PC, MK)<br />

155


Electrolytes, BUN/Cr. (PC, MK)<br />

Uric acid. (PC, MK)<br />

Fasting lipid pr<strong>of</strong>ile. (PC, MK)<br />

ECG. (PC, MK)<br />

5. Communication skills: Students should be able to:<br />

Communicate the diagnosis, treatment plan and prognosis <strong>of</strong> the disease to the patient and<br />

his or her family, taking into account the patient’s knowledge <strong>of</strong> hypertension and his or her<br />

preferences regarding treatment options. (PC, CS)<br />

Elicit questions from the patient and his or her family about the management plan. (PC, CS)<br />

Educate patients about hypertension risk factors, taking into account:<br />

o Demographics. (PC, CS)<br />

o Concomitant diseases and therapies. (PC, CS)<br />

o Quality <strong>of</strong> life. (PC, CS)<br />

o Physiologic and biochemical measurements. (PC, CS)<br />

o Economic considerations. (PC, CS)<br />

6. Basic and advanced procedural skills: Students should be able to perform:<br />

UA (dipstick and microscopic). (PC)<br />

12-lead ECG. (PC)<br />

7. Management skills: Students should be able to develop an appropriate evaluation and<br />

treatment plan for patients that includes:<br />

Treating acute and chronic hypertension. (PC, MK)<br />

Treating primary (essential) hypertension versus secondary hypertension. (PC, MK)<br />

Using a cost-effective approach based on the differential diagnosis. (PC, SBP)<br />

Prescribing preventative strategies to diminish hypertension, including:<br />

o Weight reduction. (PC, MK)<br />

o Moderation <strong>of</strong> alcohol intake. (PC, MK)<br />

o Regular physical activity. (PC, MK)<br />

o Reduction <strong>of</strong> sodium intake. (PC, MK)<br />

o Increase in potassium intake. (PC, MK)<br />

o Smoking cessation. (PC, MK)<br />

Accessing and utilizing appropriate information systems and resources to help delineate<br />

issues related to hypertension. (PC, PLI)<br />

Incorporating patient preferences. (PC)<br />

C. ATTITUDES AND PROFESSIONAL BEHAVIORS: Students should be able to:<br />

1. Appreciate the importance <strong>of</strong> patient preferences and adherence with management plans for<br />

those with hypertension. (P)<br />

2. Recognize the responsibility <strong>of</strong> the physician with regard to non-adherence. (P)<br />

3. Respond appropriately to patients who are non-adherent to treatment for hypertension. (CS, P)<br />

4. Appreciate how preventative strategies may diminish need for medications. (P)<br />

5. Appreciate the importance <strong>of</strong> side effects <strong>of</strong> medications and their impact on quality <strong>of</strong> life and<br />

adherence (including those side effects to which the geriatric population may be more prone)<br />

and demonstrate a commitment to limiting the whenever possible. (P)<br />

6. Demonstrate commitment to using risk-benefit, cost-benefit, and evidence-based<br />

considerations in the selection <strong>of</strong> diagnostic and therapeutic interventions for hypertension.<br />

(PLI, P)<br />

7. Demonstrate ongoing commitment to self-directed learning regarding hypertension. (PLI, P)<br />

8. Appreciate the impact hypertension has on a patient’s quality <strong>of</strong> life, wellbeing, ability to work,<br />

156


and the family. (P)<br />

9. Recognize the importance and demonstrate a commitment to the utilization <strong>of</strong> other healthcare<br />

pr<strong>of</strong>essions in the treatment <strong>of</strong> hypertension. (P, SBP)<br />

D. REFERENCES:<br />

Chobanian, AV, Bakris, GL, Black, HR, et al. The seventh report <strong>of</strong> the joint national<br />

committee on prevention, detection, evaluation, and treatment <strong>of</strong> high blood pressure:<br />

the JNC 7 report. JAMA. 2003; 289:2560-72.<br />

www.nhlbi.nih.gov/guidelines/hypertension/jncintro.htm<br />

Major cardiovascular events in hypertensive patients randomized to doxazosin vs<br />

chlorthalidone: the antihypertensive and lipid-lowering treatment to prevent<br />

heart attack trial (ALLHAT). ALLHAT Collaborative Research Group. JAMA.<br />

2000;283:1967-75.<br />

ALLHAT Officers and Coordinators for the ALLHAT Collaborative Research Group. The<br />

Antihypertensive and Lipid-Lowering Treatment to Prevent Heart Attack Trial. Major<br />

outcomes in high-risk hypertensive patients randomized to angiotensin-converting<br />

enzyme inhibitor or calcium channel blocker vs diuretic: The Antihypertensive and Lipid-<br />

Lowering Treatment to Prevent Heart Attack Trial (ALLHAT). JAMA. 2002;288:2981-97.<br />

Davis BR, Furberg CD, Wright JT Jr, Cutler JA, Whelton P. ALLHAT Collaborative<br />

Research Group. ALLHAT: setting the record straight. Ann Intern Med. 2004;141:39-<br />

46.<br />

Eisenberg MJ, Brox A, Bestawros AN. Calcium channel blockers: an update. Am J<br />

Med. 2004;116:35-43.<br />

Wing LM, Reid CM, Ryan P, et al. Second Australian National Blood Pressure Study<br />

Group. A comparison <strong>of</strong> outcomes with angiotensin-converting--enzyme inhibitors and<br />

diuretics for hypertension in the elderly. N Engl J Med. 2003;348:583-92.<br />

Agodoa LY, Appel L, Bakris GL, et al. African American Study <strong>of</strong> Kidney Disease and<br />

Hypertension (AASK) Study Group. Effect <strong>of</strong> ramipril vs amlodipine on renal outcomes<br />

in hypertensive nephrosclerosis: a randomized controlled trial. JAMA. 2001;285:2719-<br />

28.<br />

Brenner BM, Cooper ME, de Zeeuw D, et al. RENAAL Study Investigators. Effects <strong>of</strong><br />

losartan on renal and cardiovascular outcomes in patients with type 2 diabetes and<br />

nephropathy. N Engl J Med. 2001;345:861-9.<br />

Lewis EJ, Hunsicker LG, Clarke WR, et al. Collaborative Study Group. Renoprotective<br />

effect <strong>of</strong> the angiotensin-receptor antagonist irbesartan in patients with nephropathy due<br />

to type 2 diabetes. N Engl J Med. 2001;345:851-60.<br />

Tuomilehto J, Rastenyte D, Birkenhager WH, et al. Effects <strong>of</strong> calcium-channel blockade<br />

in older patients with diabetes and systolic hypertension. Systolic Hypertension in<br />

Europe Trial Investigators. N Engl J Med. 1999;340:677-84.<br />

Anonymous. Tight blood pressure control and risk <strong>of</strong> macrovascular and microvascular<br />

complications in type 2 diabetes: UKPDS 38. UK Prospective Diabetes Study Group.<br />

BMJ. 1998;317:703-13.<br />

Yusuf S, Sleight P, Pogue J, Bosch J, Davies R, Dagenais G. Effects <strong>of</strong> an angiotensinconverting-enzyme<br />

inhibitor, ramipril, on cardiovascular events in high-risk patients. The<br />

Heart Outcomes Prevention Evaluation Study Investigators. N Engl J Med.<br />

2000;342:145-53.<br />

Dahl<strong>of</strong> B, Devereux RB, Kjeldsen SE, et al. LIFE Study Group. Cardiovascular<br />

morbidity and mortality in the Losartan Intervention For Endpoint reduction in<br />

hypertension study (LIFE): a randomised trial against atenolol. Lancet. 2002;359:995-<br />

157


1003.<br />

158


TRAINING PROBLEM #25: LIVER DISEASE<br />

RATIONALE:<br />

The causes <strong>of</strong> hepatobiliary disease are many and can be quite overwhelming to the internal<br />

medicine clerk. A thorough understanding <strong>of</strong> a systematic approach to hyperbilirubinemia/jaundice is<br />

by far preferable to random knowledge <strong>of</strong> highly specific etiologies. The liver responds pathologically<br />

to injury in characteristic ways and knowledge <strong>of</strong> these patterns can also be very useful in differential<br />

diagnosis. Several etiologies <strong>of</strong> liver disease such as acute/chronic viral hepatitis and alcoholinduced<br />

liver disease are sufficiently common as to require specific attention. In addition, many liver<br />

diseases can result in cirrhosis and its complications and, therefore, understanding this end-stage<br />

development is important.<br />

PREREQUISITES:<br />

Prior knowledge, skills, and attitudes acquired during the pre-clinical experience should include:<br />

Ability to perform a complete medical history and physical.<br />

Ability to communicate with patients <strong>of</strong> diverse backgrounds.<br />

Anatomy, physiology, and pathophysiology <strong>of</strong> the hepatobiliary system.<br />

SPECIFIC LEARNING OBJECTIVES:<br />

A. KNOWLEDGE: Students should be able to define, describe, and discuss:<br />

1. The biochemical/physiologic/mechanistic approach to hyperbilirubinemia, including:<br />

Increased production. (MK)<br />

Decreased hepatocyte uptake. (MK)<br />

Decreased conjugation. (MK)<br />

Decreased excretion from the hepatocyte. (MK)<br />

Decreased small duct transport (intrahepatic cholestasis). (MK)<br />

Decreased large duct transport (extrahepatic cholestasis, obstructive jaundice). (MK)<br />

2. The biochemistry and common causes <strong>of</strong> unconjugated and conjugated hyperbilirubinemia.<br />

(MK)<br />

3. The use <strong>of</strong> serum markers <strong>of</strong> liver injury (e.g. AST, ALT, GGT, Alk Phos) and function (e.g.<br />

bilirubin, ALB, PT/INR) in the diagnostic evaluation <strong>of</strong> hepatobiliary disease. (MK)<br />

4. The clinical significance <strong>of</strong> asymptomatic, isolated elevation <strong>of</strong> AST, ALT, GGT, and/or Alk<br />

Phos. (MK)<br />

5. The common pathologic patterns <strong>of</strong> liver disease and their common causes, including:<br />

Steatosis (fatty liver). (MK)<br />

Hepatitis. (MK)<br />

Cirrhosis. (MK)<br />

Infiltrative. (MK)<br />

Intrahepatic cholestasis. (MK)<br />

Extrahepatic cholestasis (obstructive jaundice). (MK)<br />

6. The epidemiology, symptoms, signs, typical clinical course, and prevention <strong>of</strong> viral hepatitis.<br />

(MK)<br />

7. The distinctions between acute and chronic hepatitis. (MK)<br />

8. The indications for and efficacy <strong>of</strong> hepatitis A and B vaccinations. (MK)<br />

9. The common causes and clinical significance <strong>of</strong> hepatic steatosis and steatohepatis. (MK)<br />

159


10. The epidemiology, symptoms, signs, and typical clinical course <strong>of</strong> autoimmune liver diseases<br />

such as autoimmune hepatitis, primary biliary cirrhosis, and primary sclerosing cholangitis.<br />

(MK)<br />

11. The epidemiology, symptoms, signs, and typical clinical course <strong>of</strong> cirrhosis. (MK)<br />

12. The pathophysiologic manifestations, symptoms, signs, and complications <strong>of</strong> alcohol-induced<br />

liver disease. (MK)<br />

13. The symptoms, signs, and complications <strong>of</strong> portal hypertension. (MK)<br />

14. The pathophysiology and common causes <strong>of</strong> ascites. (MK)<br />

15. The pathophysiologic manifestations, symptoms, and signs <strong>of</strong> spontaneous bacterial<br />

peritonitis. (MK)<br />

16. The basic pathophysiology, symptoms, signs, typical clinical course, and precipitants <strong>of</strong><br />

hepatic encephalopathy. (MK)<br />

17. The basic pathophysiology, symptoms, signs, and typical clinical course <strong>of</strong> the hepatorenal<br />

syndrome. (MK)<br />

18. The analysis <strong>of</strong> ascitic fluid and its use in the diagnostic evaluation <strong>of</strong> liver disease. (MK)<br />

19. Common causes <strong>of</strong> drug-induced liver injury. (MK)<br />

20. Genetic considerations in liver disease (i.e. hemochromatosis, Wilson’s disease, alpha-1<br />

antitrypsin deficiency, Gilbert’s syndrome). (MK)<br />

21. The epidemiology, pathophysiology, symptoms, signs, and typical clinical course <strong>of</strong><br />

cholelithiasis and cholecystitis. (MK)<br />

22. The clinical syndrome <strong>of</strong> “ascending cholangitis” including its common causes and typical<br />

clinical course. (MK)<br />

23. The indications for and risks <strong>of</strong> paracentesis and liver biopsy. (MK)<br />

24. The indications for and utility <strong>of</strong> hepatobiliary imaging studies, including:<br />

Ultrasound. (MK)<br />

Nuclear medicine studies. (MK)<br />

CT. (MK)<br />

MRI. (MK)<br />

Magnetic resonance cholangiopancreatography (MRCP). (MK)<br />

Endoscopic retrograde cholangiopancreatography (ERCP). (MK)<br />

B. SKILLS: Students should be able to demonstrate specific skills, including:<br />

1. History-taking skills: Students should be able to obtain, document, and present an<br />

age-appropriate medical history, that differentiates among etiologies <strong>of</strong> disease,<br />

including:<br />

Jaundice, discolored urine, pruritis, light-colored stool, unintentional weight loss, fever,<br />

nausea, emesis, diarrhea, altered mental status, abdominal pain, increased abdominal<br />

girth, edema, rectal bleeding, hematemesis. (PC, CS)<br />

DM. (PC, CS)<br />

Alcohol use. (PC, CS)<br />

Prescription, over-the-counter, and illicit drug use. (PC, CS)<br />

Transfusions and other sources <strong>of</strong> potential blood-born pathogen exposure. (PC, CS)<br />

Consumption <strong>of</strong> uncooked shellfish and other food items potentially contaminated with fecal<br />

matter. (PC, CS)<br />

Sexual history. (PC, CS)<br />

Vaccination history. (PC, CS)<br />

Family history <strong>of</strong> liver diseases. (PC, CS)<br />

2. Physical exam skills: Students should be able to perform a physical exam to establish the<br />

160


diagnosis and severity <strong>of</strong> disease, including:<br />

Jaundice. (PC)<br />

Complete abdominal examination including findings consistent with ascites (e.g. bulging<br />

flanks, shifting dullness, fluid wave). (PC)<br />

Findings compatible with chronic alcohol use and portal hypertension (e.g. palmar<br />

erythema, spider angiomas, gynecomastia, testicular atrophy, Dupuytren’s contracture,<br />

muscle wasting, splenomegaly, ascites, edema, caput medusa, hemorrhoids). (PC)<br />

Findings compatible with hepatic (portosystemic) encephalopathy (e.g. disturbances <strong>of</strong><br />

consciousness and behavior, fluctuating neurologic signs, asterixis). (PC)<br />

3. Differential diagnosis: Students should be able to generate a prioritized differential diagnosis<br />

recognizing specific history and physical exam findings that suggest a specific etiology <strong>of</strong> liver<br />

disease. (PC, MK)<br />

4. Laboratory interpretation: Students should be able to recommend when to order diagnostic<br />

and laboratory tests and be able to interpret them, both prior to and after initiating treatment<br />

based on the differential diagnosis, including consideration <strong>of</strong> test cost and performance<br />

characteristics as well as patient preferences. Laboratory and diagnostic tests should include,<br />

when appropriate:<br />

CBC. (PC, MK)<br />

Electrolytes, BUN/Cr, GLC. (PC, MK)<br />

ALB, TP, total bilirubin, direct bilirubin, PT/INR, AST, ALT, Alk Phos. (PC, MK)<br />

Hepatitis serology. (PC, MK)<br />

Ascitic fluid ALB, amylase, cell counts, staining, cultures, and the<br />

serum-ascites albumin gradient (SAAG). (PC, MK)<br />

Students should be able to define the indications for and interpret (with consultation) the<br />

results <strong>of</strong>:<br />

Ultrasound. (PC, MK)<br />

Nuclear medicine studies. (PC, MK)<br />

CT. (PC, MK)<br />

MRI. (PC, MK)<br />

Magnetic resonance cholangiopancreatography (MRCP). (PC, MK)<br />

Endoscopic retrograde cholangiopancreatography (ERCP). (PC, MK)<br />

5. Communication skills: Students should be able to:<br />

Communicate the diagnosis, treatment plan, and subsequent follow-up to the patient and<br />

his or her family. (PC, CS)<br />

Elicit input and questions from the patient and his or her family about the management<br />

plan. (PC, CS)<br />

Discuss the avoidance <strong>of</strong> known hepatotoxins. (PC, CS)<br />

Counsel patients regarding alcohol abstinence. (PC, CS)<br />

Discuss the importance <strong>of</strong> hepatitis A and B vaccinations for nonimmune patients. (PC, CS)<br />

6. Basic and advanced procedural skills: Students should be able to:<br />

Assist in performing a paracentesis after explaining the procedure to the patient. (PC)<br />

7. Management skills: Students should able to develop an appropriate evaluation and<br />

treatment plan for patients that includes:<br />

The diagnostic evaluation <strong>of</strong> asymptomatic, isolated elevation <strong>of</strong> the transaminases and/or<br />

Alk Phos. (PC, MK)<br />

The diagnostic evaluation <strong>of</strong> patients with jaundice and unconjugated or conjugated<br />

hyperbilirubinemia. (PC, MK)<br />

The basic management <strong>of</strong> steatosis, hepatitis, cirrhosis, intra- and extra hepatic<br />

cholestasis, acute cholecystitis, ascites, portal hypertension, spontaneous bacterial<br />

161


peritonitis, and hepatic encephalopathy. (PC, MK)<br />

Determining when to obtain consultation from a gastroenterologist, hepatologist, or biliary<br />

surgeon. (PC, SBP)<br />

Using a cost-effective approach based on the differential diagnosis. (PC, SBP)<br />

Accessing and utilizing appropriate information systems and resources to help delineate<br />

issues related to liver disease. (PC, PLI)<br />

Incorporating patient preferences. (PC)<br />

C. ATTITUDES AND PROFESSIONAL BEHAVIORS: Students should be able to:<br />

1. Demonstrate commitment to using risk-benefit, cost-benefit, and evidence-based<br />

considerations in the selection <strong>of</strong> diagnostic and therapeutic interventions for liver disease. (PLI, P)<br />

2. Recognize the importance <strong>of</strong> patient needs and preferences when selecting among diagnostic<br />

and therapeutic options for liver disease. (P)<br />

3. Respond appropriately to patients who are nonadherent to treatment for liver disease. (CS, P)<br />

4. Demonstrate ongoing commitment to self-directed learning regarding liver disease. (PLI, P)<br />

5. Appreciate the impact liver disease has on a patient’s quality <strong>of</strong> life, wellbeing, ability to work,<br />

and the family. (P)<br />

6. Recognize the importance <strong>of</strong> and demonstrate a commitment to the utilization <strong>of</strong> other<br />

healthcare pr<strong>of</strong>essionals in the diagnosis and treatment <strong>of</strong> liver disease. (P, SBP)<br />

7. Discuss the public health role physicians play in the prevention <strong>of</strong> viral hepatitis. (P, SBP)<br />

8. Appreciate the difficulties patient face with alcohol abstinence. (P)<br />

D. REFERENCES:<br />

Viral Hepatitis<br />

National Center for Infectious Diseases<br />

Center for Disease Control and Prevention<br />

U.S. <strong>Department</strong> <strong>of</strong> Health and Human Services<br />

www.cdc.gov/ncidod/diseases/hepatits/index.htm<br />

Practice Guidelines<br />

American Association for the Study <strong>of</strong> Liver Diseases<br />

www.aasld.org<br />

National Institute on Alcohol Abuse and Alcoholism<br />

National Institutes <strong>of</strong> Health<br />

U.S. <strong>Department</strong> <strong>of</strong> Health and Human Services<br />

www.niaaa.nih.gov<br />

Pratt DS, Kaplan MM. Evaluation <strong>of</strong> abnormal liver-enzyme results in asymptomatic<br />

patients. N Engl J Med. 2000;342:1266-71.<br />

Angulo P. Nonalcoholic fatty liver disease. N Engl J Med. 2002;346:1221-31.<br />

Trowbridge RL, Rutkowski NK, Shojania. Does this patient have acute cholecystitis?<br />

JAMA. 2003;289:80-6.<br />

Indar AA, Beckingham IJ. Acute cholecystitis. BMJ. 2002;325:639-43.<br />

Krige JE, Bechingham IJ. ABC <strong>of</strong> diseases <strong>of</strong> liver, pancreas, and biliary system.<br />

Portal hypertension-1: varices.<br />

Krige JE, Bechingham IJ. ABC <strong>of</strong> diseases <strong>of</strong> liver, pancreas, and biliary system.<br />

Portal hypertension-2. Ascites, encephalopathy, and other conditions. BMJ.<br />

2001;322:416-8.<br />

162


TRAINING PROBLEM #26: MAJOR DEPRESSION<br />

RATIONALE:<br />

Major depression is a very common problem in adults, resulting in significant morbidity and mortality.<br />

Most <strong>of</strong>ten the primary care provider is the first health care pr<strong>of</strong>essional to see a depressed patient.<br />

Frequently, the initial presentation is associated with somatic complaints that bring the patient to the<br />

physician. Major depression is also a relatively common accompaniment to serious medical<br />

conditions. There is significant evidence that primary care physicians commonly fail to diagnose<br />

major depression. With relatively recent improvements in available treatment, it is even more<br />

important for internists to screen for major depression and to know the common presenting<br />

symptoms. The internist should also be familiar with available therapeutic options and be prepared to<br />

treat selected patients, including those who decline consultation with a mental health pr<strong>of</strong>essional.<br />

PREREQUISITES:<br />

Prior knowledge, skills, and attitudes acquired during the pre-clinical experience should include:<br />

Ability to perform a complete medical history and physical.<br />

Ability to communicate with patients <strong>of</strong> diverse backgrounds.<br />

Neurochemistry <strong>of</strong> major depression.<br />

Pharmacology <strong>of</strong> the major classes <strong>of</strong> antidepressants.<br />

Basic understanding <strong>of</strong> the efficacy <strong>of</strong> psychotherapy, antidepressants, and electroconvulsive<br />

therapy.<br />

SPECIFIC LEARNING OBJECTIVES:<br />

A. KNOWLEDGE: Students should be able to define, describe, and discuss:<br />

1. The epidemiology <strong>of</strong> major depression in the general population and the impact <strong>of</strong> major illness<br />

on the prevalence <strong>of</strong> major depression (e.g. stroke, heart disease, DM, cancer, Parkinson’s<br />

disease, HIV/AIDS). (MK)<br />

2. The impact <strong>of</strong> major depression on the outcome <strong>of</strong> medical illness. (MK)<br />

3. The American Psychiatric Associations’ Diagnostic and Statistical Manual 4 th edition (DSM-IV)<br />

diagnostic criteria for major depression. (MK)<br />

4. Common psychological symptoms and signs <strong>of</strong> major depression (e.g. low mood/affect,<br />

anxiety, irritability/anger, disinterest, anhedonia, decreased libido, guilt, poor self-esteem, poor<br />

concentration, rumination, helplessness, hopelessness, thoughts <strong>of</strong> death and suicide, somatic<br />

complaints). (MK)<br />

5. Common neurovegetative symptoms and signs <strong>of</strong> major depression (e.g. appetite disturbance,<br />

decreased energy, psychomotor retardation or agitation, sleep disturbance). (MK)<br />

6. Common somatic complaints that accompany depressive disorders and the potential for the<br />

occurrence <strong>of</strong> these symptoms without obvious psychological symptoms (e.g. fatigue,<br />

weakness, myalgias, arthralgias, headache, nausea, dyspnea, palpitations, chest<br />

pain/discomfort, lightheadedness/dizziness, bowel movement alterations). (MK)<br />

7. The distinguishing features <strong>of</strong> major depression with psychotic features, bipolar disorder,<br />

dementia, and delirium. (MK)<br />

8. The differential diagnosis <strong>of</strong> major depression, including:<br />

Other psychiatric disorders. (MK)<br />

Drug-induced (e.g. corticosteroids, cimetidine, metoclopramide, clonidine, etc.). (MK)<br />

163


Drug withdrawal (e.g. amphetamine, cocaine). (MK)<br />

Infection (e.g. tertiary syphilis). (MK)<br />

Endocrine/metabolic (e.g. hypo/hyperthyroidism, Cushing’s, Addison’s). (MK)<br />

Collagen vascular diseases (e.g. lupus, fibromyalgia). (MK)<br />

Neurologic (e.g. stoke, multiple sclerosis, Parkinson’s disease, head trauma, complex<br />

partial seizures). (MK)<br />

Nutritional (e.g. B12, folate, niacin, thiamine deficiencies). (MK)<br />

Neoplastic (e.g. pancreatic cancer, disseminated carcinomatosis). (MK)<br />

9. US Preventive Services Task Force (USPSTF) depression screening recommendations. (MK)<br />

10. The risks <strong>of</strong> untreated major depression. (MK)<br />

11. Assessment <strong>of</strong> the risk <strong>of</strong> suicide. (MK)<br />

12. The demographics and risk factors for completed suicide. (MK)<br />

13. The potential link between major depression and substance abuse. (MK)<br />

14. The potential role <strong>of</strong> genetics in depression.(MK)<br />

15. Indications and efficacy <strong>of</strong> the basic therapeutic options for major depression, including:<br />

Psychotherapy (cognitive behavioral therapy or interpersonal psychotherapy). (MK)<br />

Pharmacotherapy. (MK)<br />

Electroconvulsive therapy. (MK)<br />

16. The side effects <strong>of</strong> the major classes <strong>of</strong> antidepressants and common interaction with other<br />

medications. (MK)<br />

B. SKILLS: Students should be able to demonstrate specific skills including:<br />

1. History-taking skills: Students should be able to obtain, document, and present an ageappropriate<br />

medical history, that differentiates among etiologies <strong>of</strong> disease including:<br />

Eliciting the symptoms <strong>of</strong> major depression. (PC, CS)<br />

Determining the presence or absence <strong>of</strong> underlying dementia, anxiety disorders, adverse<br />

drug effects, and grief in any patient suspected <strong>of</strong> having major depression. (PC, CS)<br />

Obtaining a complete drug history (including illicit drugs). (PC, CS)<br />

Identifying chronic diseases that are associated with increased risk <strong>of</strong> major depression.<br />

(PC, CS)<br />

2. Physical exam skills: Students should be able to perform a physical exam to establish the<br />

diagnosis and severity <strong>of</strong> disease, including:<br />

A complete neurologic examination. (PC)<br />

A complete mental status exam. (PC)<br />

5. Differential diagnosis: Students should be able to generate a prioritized differential diagnosis<br />

recognizing specific history and physical exam findings that suggest a specific etiology for<br />

major depression (psychiatric and nonpsychiatric). (PC, MK)<br />

6. Laboratory interpretation: Students should be able to recommend when to order diagnostic<br />

and laboratory tests and be able to interpret them, both prior to and after initiating treatment<br />

based on the differential diagnosis, including consideration <strong>of</strong> test cost and performance<br />

characteristics as well as patient preferences.<br />

Laboratory and diagnostic tests should include, when appropriate:<br />

Blood and urine drug screening. (PC, MK)<br />

Thyroid function tests. (PC, MK)<br />

Serum RPR and VDRL. (PC, MK)<br />

B12, folate, and thiamine levels. (PC, MK)<br />

Students should be able to define the indications for and interpret (with consultation) the<br />

results <strong>of</strong>:<br />

164


Cranial CT. (PC, MK)<br />

Cranial MRI. (PC, MK)<br />

7. Communication skills: Students should be able to:<br />

Communicate the diagnosis, treatment plan, and subsequent follow-up to the patient and<br />

his or her family. (PC, CS)<br />

Elicit input and questions from the patient and his or her family about the management<br />

plan. (PC, CS)<br />

Demonstrate effective listening skills and empathy. (PC, CS)<br />

Advise the patient <strong>of</strong> the delay in therapeutic benefit from antidepressant medications. (PC,<br />

CS)<br />

6. Management skills: Students should able to develop an appropriate evaluation and treatment<br />

plan for patients that includes:<br />

An appreciation <strong>of</strong> the fact that major depression is not generally a “diagnosis <strong>of</strong> exclusion”<br />

and that ruling out all other possible medical causes is typically not necessary. (PC, MK)<br />

Making an accurate diagnosis <strong>of</strong> major depression. (PC, MK)<br />

Assessing for the risk <strong>of</strong> suicide. (PC, MK)<br />

Recommending psychotherapy (cognitive behavioral therapy or interpersonal<br />

psychotherapy). (PC, MK)<br />

Selecting appropriate initial pharmacologic therapy considering efficacy, side effects, and<br />

potential drug-drug interactions. (PC, MK, SBP)<br />

Identifying barriers to major depression treatment. (PC, SBP)<br />

Anticipating potential resistance to seeing a psychiatrist and antidepressant treatment. (PC)<br />

Planning appropriate follow-up. (PC, MK)<br />

Recognizing success or failure <strong>of</strong> initial treatment and making appropriate adjustments.<br />

(PC, MK)<br />

Determining when to obtain consultation from a psychiatrist, psychologist, or other mental<br />

health pr<strong>of</strong>essional. (PC, SBP)<br />

Using a cost-effective approach to treatment. (PC, SBP)<br />

Accessing and utilizing appropriate information systems and resources to help delineate<br />

issues related to major depression. (PC, PLI)<br />

Incorporating patient needs and preferences. (PC)<br />

C. ATTITUDES AND PROFESSIONAL BEHAVIORS: Students should be able to:<br />

1. Recognize major depression as an important and potentially life-threatening disease. (P)<br />

2. Appreciate the social stigma <strong>of</strong> psychiatric diagnoses and the ways non-psychiatric physicians<br />

may inadvertently contribute to this. (P)<br />

3. Appreciated the reluctance <strong>of</strong> some patients to see a psychiatrist. (P)<br />

4. Demonstrate commitment to using risk-benefit, cost-benefit, and evidence-based<br />

considerations in the selection <strong>of</strong> diagnostic and therapeutic interventions for major depression. (PLI,<br />

P)<br />

5. Recognize the importance <strong>of</strong> patient needs and preferences when selecting among diagnostic<br />

and therapeutic options for major depression. (P)<br />

6. Respond appropriately to patients who are nonadherent to treatment for major depression.<br />

(CS, P)<br />

7. Demonstrate ongoing commitment to self-directed learning regarding major depression. (PLI,<br />

P)<br />

8. Appreciate the impact major depression has on a patient’s quality <strong>of</strong> life, wellbeing, ability to<br />

work, and the family. (P)<br />

165


9. Recognize the importance <strong>of</strong> and demonstrate a commitment to the utilization <strong>of</strong> other health<br />

care pr<strong>of</strong>essionals in the diagnosis and treatment <strong>of</strong> major depression. (P, SBP)<br />

D. REFERENCES:<br />

Depression<br />

National Institute <strong>of</strong> Mental Health<br />

National Institutes <strong>of</strong> Health<br />

U.S. <strong>Department</strong> <strong>of</strong> Health and Human Services<br />

www.nimh.nih.gov/publicat//defresssion.cfm<br />

Practice Guidelines<br />

American Psychiatric Association<br />

www.psych.org/psych_pract/treatg/pg/prac_guide.cfm<br />

The National Association on Mental Illness<br />

www.nami.org/Template.cfm?Section=By_Illness&Template=/TaggedPa<br />

ge/TaggedPageDisplay.cfm&TPLID=54&ContentID=26414<br />

Mann JJ. The medical management <strong>of</strong> depression. N Engl J Med. 2005;353:1819-34.<br />

166


TRAINING PROBLEM #27: NOSOCOMIAL INFECTIONS<br />

RATIONALE:<br />

Nosocomial infections have been occurring since the inception <strong>of</strong> the hospital. Despite many<br />

advances the incidence is still roughly five percent <strong>of</strong> all acute care hospitalizations or about two<br />

million cases a year. Nosocomial infections are estimated to approximately double the morbidity and<br />

mortality rates <strong>of</strong> any person admitted to the hospital. Directly attributable deaths can total up to<br />

88,000 per year with the expenditure <strong>of</strong> many millions <strong>of</strong> excess health care dollars. Preventing<br />

nosocomial infections is the responsibility <strong>of</strong> every heath care worker, including physicians, house<br />

<strong>of</strong>ficers, medical students, nurses, technicians, administrators, etc. Also considered here are<br />

occupational exposures for which health care workers are at risk.<br />

PREREQUISITES:<br />

Prior knowledge, skills, and attitudes acquired during the pre-clinical experience should include:<br />

Ability to perform a complete medical history and physical.<br />

Ability to communicate with patients <strong>of</strong> diverse backgrounds.<br />

Basic training in body substance isolation procedures.<br />

Microbiology and pathophysiology <strong>of</strong> the common nosocomial organisms, including<br />

Staphylococcus aureus (methicillin sensitive and resistant), Staphylococcus epidermidis,<br />

Enterococcus species (vancomycin sensitive and resistant), Pseudomonas aeruginosa and<br />

other nosocomial gram-negative bacilli, Clostridium difficile, and Candida species.<br />

The pharmacology <strong>of</strong> antimicrobial agents.<br />

SPECIFIC LEARNING OBJECTIVES:<br />

A. KNOWLEDGE: Students should be able to define, describe, and discuss:<br />

1. The epidemiology and significance <strong>of</strong> nosocomial infections in the United States. (MK)<br />

2. The general clinical risk factors for nosocomial infection. including:<br />

Immunocompromise. (MK)<br />

Immunosuppressive drugs. (MK)<br />

Extremes <strong>of</strong> age. (MK)<br />

Compromise <strong>of</strong> the skin and mucosal surfaces secondary to:<br />

o Drugs. (MK)<br />

o Irradiation. (MK)<br />

o Trauma. (MK)<br />

o Invasive diagnostic and therapeutic procedures. (MK)<br />

o Invasive indwelling devises (e.g. intravenous catheter, bladder catheter, endotracheal<br />

tube, etc.). (MK)<br />

3. The major routes <strong>of</strong> nosocomial infection transmission, including:<br />

Contact. (MK)<br />

Droplet. (MK)<br />

Airborne. (MK)<br />

Common vehicle. (MK)<br />

4. The epidemiology, pathophysiology, microbiology, symptoms, signs, typical clinical course,<br />

and preventive strategies for the most common nosocomial infections, including:<br />

Urinary tract infection. (MK)<br />

167


Pneumonia. (MK)<br />

Surgical site infection. (MK)<br />

Intravascular devised-related bloodstream infections. (MK)<br />

Skin infections. (MK)<br />

Health care associated diarrhea. (MK)<br />

5. Empiric antibiotic therapy for the most common nosocomial infections. (MK)<br />

6. The epidemiology, pathophysiology, microbiology, symptoms, signs, typical clinical course,<br />

and preventive strategies for colonization or infection with the following organisms:<br />

Vancomycin-resistant enterococci. (MK)<br />

Clostridium difficile. (MK)<br />

Methicillin-resistant Staphylococcus aureus. (MRSA) (MK)<br />

Multidrug-resistant Gram-negative bacteria. (MK)<br />

7. The crucial importance <strong>of</strong> judicious antibiotic use. (MK)<br />

8. The effect <strong>of</strong> widespread use <strong>of</strong> broad spectrum anti-microbial agents on endogenous body<br />

flora and the hospital microbial flora. (MK)<br />

9. The types <strong>of</strong> isolation procedures and their indications:<br />

Standard. (MK)<br />

Airborne. (MK)<br />

Contact. (MK)<br />

Droplet. (MK)<br />

10. The Centers for Disease Control and Prevention (CDC) guidelines for hand hygiene. (MK)<br />

11. Preventive strategies for needlestick and sharps injuries intended to reduce the transmission <strong>of</strong><br />

bloodborne pathogens (hepatitis B, hepatitis C, and HIV). (MK)<br />

12. Local hospital post-exposure (i.e. after an eye/mucous membrane splash, needlestick or other<br />

sharps injury) protocols for prompt reporting, evaluation, counseling, treatment, and follow-up.<br />

(MK, SBP)<br />

13. The indications, efficacy, and side effects <strong>of</strong> post-exposure prophylaxis for hepatitis B and<br />

HIV/AIDS. (MK)<br />

14. negative-pressure ventilation isolation for known or suspected tuberculosis patients (MK)<br />

15. National Institute for Occupational Safety and Health (NIOSH) approved personal respiratory<br />

protective equipment (i.e. N95 respirator) use for the prevention <strong>of</strong> transmission <strong>of</strong><br />

Mycobacterium tuberculosis to health care workers. (MK)<br />

B. SKILLS: Students should be able to demonstrate specific skills, including:<br />

3. History-taking skills: Students should be able to obtain, document, and present an ageappropriate<br />

medical history, that differentiates among etiologies <strong>of</strong> disease in the organ systems<br />

likely to be involved with nosocomial infection. (PC, CS)<br />

4. Physical exam skills: Students should be able to perform a physical examination <strong>of</strong> skin,<br />

vascular access sites, lungs, abdomen, wounds, and catheter and drain sites and recognize signs<br />

<strong>of</strong> local or systemic infection (PC)<br />

3. Differential diagnosis: Students should be able to generate a prioritized differential diagnosis<br />

<strong>of</strong> the likely sites and organisms involved, recognizing specific history and physical exam<br />

findings that suggest a specific etiology. (PC, MK)<br />

4. Laboratory interpretation: Students should be able to recommend when to order diagnostic<br />

and laboratory tests and be able to interpret them, both prior to and after initiating treatment<br />

based o the differential diagnosis, including consideration <strong>of</strong> test cost and performance<br />

168


characteristics as well as patient preferences.<br />

Laboratory and diagnostic tests should include, when appropriate:<br />

Urinalysis and culture and sensitivities. (PC, MK)<br />

Sputum Gram stain and culture and sensitivities. (PC, MK)<br />

Chest radiograph. (PC, MK)<br />

Wound cultures and sensitivities. (PC, MK)<br />

Clostridium difficile toxin assay. (PC, MK)<br />

Hepatitis serologies. (PC, MK)<br />

HIV ELISA and western blot. (PC, MK)<br />

Sputum AFB staining and culture. (PC, MK)<br />

5. Communication skills: Students should be able to:<br />

Communicate the diagnosis, treatment plan, and subsequent follow-up to the patient and<br />

his or her family. (PC, CS)<br />

Elicit input and questions from the patient and his or her family about the management<br />

plan. (PC, CS)<br />

Explain the necessity for isolation procedures. (PC, CS)<br />

Counsel patients about the need for judicious antibiotic usage and the potential patientspecific<br />

and public health risks <strong>of</strong> not doing so. (PC, CS)<br />

6. Basic and advanced procedural skills: Students should be able to:<br />

Obtain blood cultures. (PC)<br />

Place and interpret a PPD. (PC)<br />

Demonstrate proper sterile technique for invasive procedures. (PC)<br />

7. Management skills: Students should able to develop an appropriate evaluation and<br />

treatment plan for patients that includes:<br />

Assessing a hospitalized patient who develops a new fever 48 or more hours after<br />

admission. (PC, MK)<br />

Developing a plan for the evaluation and treatment <strong>of</strong> hospital acquired infection. (PC, MK)<br />

Demonstrating appropriate choice <strong>of</strong> antimicrobial drugs which considers mechanisms <strong>of</strong><br />

action, spectrum <strong>of</strong> activity, pharmacokinetics, drug interactions, and adverse reactions.<br />

(PC, MK)<br />

Recognizing when indwelling intravascular and urinary collection devices should be<br />

removed. (PC, MK)<br />

Requesting appropriate isolation measures to protect other patients and health care<br />

workers. (PC, SBP)<br />

Determining when to obtain consultation from an infectious diseases specialist. (PC, SBP)<br />

Contacting hospital infection control experts when appropriate. (SBP)<br />

Using a cost-effective approach based on the differential diagnosis. (PC, SBP)<br />

Accessing and utilizing appropriate information systems and resources to help delineate<br />

issues related to nosocomial infections. (PC, PLI)<br />

Incorporating patient needs and preferences. (PC)<br />

C. ATTITUDES AND PROFESSIONAL BEHAVIORS: Students should be able to:<br />

1. Serve as a role model to all other health care providers by strictly following all infection control<br />

measures including hand hygiene and all isolation procedures. (P, SBP)<br />

2. Appreciate the role physicians play in the inappropriate prescribing <strong>of</strong> antimicrobial agents and<br />

the public health ramifications. (P, SBP)<br />

3. Demonstrate commitment to using risk-benefit, cost-benefit, and evidence-based<br />

considerations in the selection <strong>of</strong> diagnostic and therapeutic interventions for nosocomial infections.<br />

169


(PLI, P)<br />

4. Recognize the importance <strong>of</strong> patient needs and preferences when selecting among diagnostic<br />

and therapeutic options for nosocomial infections. (P)<br />

5. Demonstrate ongoing commitment to self-directed learning regarding nosocomial infections.<br />

(PLI, P)<br />

6. Appreciate the impact nosocomial infections have on a patient’s quality <strong>of</strong> life, well-being,<br />

ability to work, and the family. (P)<br />

7. Recognize the importance <strong>of</strong> and demonstrate a commitment to the utilization <strong>of</strong> other health<br />

care pr<strong>of</strong>essionals in the diagnosis, treatment, and prevention <strong>of</strong> nosocomial infections. (P, SBP)<br />

D. REFERENCES:<br />

Infection Control Guidelines<br />

Division <strong>of</strong> Healthcare Quality Promotion<br />

National Center for Infectious Diseases<br />

Centers for Disease Control and Prevention<br />

U.S. <strong>Department</strong> <strong>of</strong> Health and Human Services<br />

www.cdc.gov/ncidod/hip/default.htm<br />

Vancomycin-Resistant Enterococci Division <strong>of</strong> Healthcare<br />

Quality Promotion National Center for Infectious Diseases<br />

Centers for Disease Control and Prevention<br />

U.S. <strong>Department</strong> <strong>of</strong> Health and Human Services<br />

www.cdc.gov/ncidod/hip/ARESIST/vre.htm<br />

Pautanen SM, Simor AE. Clostridium difficile associated diarrhea in adults. Can Med<br />

Assoc J. 2004;171:51-8.<br />

www.cmaj.ca/cgi/reprint/171/1/51<br />

Healthcare-Associated MRSA Division <strong>of</strong> Healthcare Quality Promotion National Center<br />

for Infectious Diseases Centers for Disease Control and Prevention<br />

U.S. <strong>Department</strong> <strong>of</strong> Health and Human Services<br />

www.cdc.gov/ncidod/dhqp/ar_mrsa.html<br />

170


TRAINING PROBLEM #28: OBESITY<br />

RATIONALE:<br />

Obesity and overweight are recognized as ever growing epidemics in the United States. These<br />

conditions have been correlated with the development <strong>of</strong> medical conditions such as diabetes,<br />

hypertension, heart disease, and osteoarthritis. Mastery <strong>of</strong> the approach to patients who are not at<br />

an ideal body weight is important to general internists because they <strong>of</strong>ten deal with the sequelae <strong>of</strong><br />

the comorbid illnesses.<br />

PREREQUISITES:<br />

Prior knowledge, skills, and attitudes acquired during the pre-<strong>clerkship</strong> experience should include:<br />

Ability to perform a complete medical history and physical exam.<br />

Ability to communicate with patients <strong>of</strong> diverse backgrounds.<br />

Psychology associated with addictive behavior.<br />

Anatomy, physiology, and pathophysiology <strong>of</strong> the gastrointestinal tract and digestion.<br />

Pharmacology <strong>of</strong> the available drugs used to treat obesity.<br />

Nutrition and caloric requirements.<br />

SPECIFIC LEARNING OBJECTIVES:<br />

A. KNOWLEDGE: Students should be able to define, describe, and discuss:<br />

1. The etiology <strong>of</strong> obesity including excessive caloric intake, insufficient energy expenditure<br />

leading to low resting metabolic rate, genetic predisposition, environmental factors affecting weight<br />

gain, psychologic stressors, and lower socioeconomic status. (MK)<br />

2. The definition and classification <strong>of</strong> overweight and obese using BMI. (MK)<br />

3. The health implications that being overweight or obese may have on the patient. (MK)<br />

4. How daily caloric requirements are calculated and the caloric deficit required to achieve a five<br />

to 10 percent weight reduction in six to 12 months. (MK)<br />

5. The principles <strong>of</strong> behavior modification. (MK)<br />

6. How to develop an exercise program and assist the patient in setting goals for weight loss.<br />

(MK)<br />

7. Treatment options, including nonpharmacologic and pharmacologic treatment, behavioral<br />

therapy and surgical intervention. (MK)<br />

B. SKILLS: Students should be able to demonstrate specific skills including:<br />

1. History-taking skills: Students should be able to obtain, document, and present an<br />

age-appropriate medical history, including:<br />

Reviewing the patient’s weight history from childhood. (PC, CS)<br />

Assessing the risk factors for obesity related conditions. (PC, CS)<br />

Assessing the patient’s motivation for losing weight. (PC, CS)<br />

Reviewing the patient’s past experience with losing weight and determining barriers<br />

encountered in prior attempts. (PC, CS)<br />

Reviewing the patient’s activity level and diet. (PC, CS)<br />

Obtaining an assessment <strong>of</strong> tobacco and drug use especially noting if the patient is in the<br />

171


process <strong>of</strong> stopping either. (PC, CS)<br />

Obtaining a family history focusing on weight related issues and comorbid illnesses<br />

associated with obesity. (PC, CS)<br />

Obtaining a focused review <strong>of</strong> systems including signs and symptoms <strong>of</strong> secondary causes<br />

<strong>of</strong> obesity such as Cushing’s syndrome, hypothyroidism, and hypogonadism. (PC, CS)<br />

2. Physical exam skills: Students should be able to perform a physical exam to establish the<br />

diagnosis and severity <strong>of</strong> disease, including:<br />

Calculation <strong>of</strong> degree <strong>of</strong> obesity from the patient’s height and weight by calculating BMI.<br />

(PC)<br />

Noting the presence <strong>of</strong> abdominal obesity based on waist-to-hip circumference. (PC)<br />

Assessing the signs <strong>of</strong> vascular disease including hypertension, carotid bruits, abdominal<br />

aortic size, blood pressure and peripheral pulses. (PC)<br />

Assessing for signs <strong>of</strong> endocrine abnormalities, including: striae, peripheral neuropathy,<br />

depressed tendon reflexes, bruising, and signs <strong>of</strong> dyslipidemia (e.g. xanthomas and<br />

xanthalasma). (PC)<br />

3. Differential diagnosis: Students should be able to generate a prioritized differential diagnosis<br />

recognizing specific history and physical exam findings that suggest a specific etiology <strong>of</strong><br />

primary and secondary obesity. (MK, PC)<br />

4. Laboratory interpretation: Students should be able to recommend when to order diagnostic<br />

and laboratory tests and be able to interpret them, both prior to and after initiating treatment<br />

based on the differential diagnosis, including consideration <strong>of</strong> test cost and performance<br />

characteristics as well as patient preferences.<br />

Laboratory and diagnostic tests should include, when appropriate:<br />

Serum GLC. (PC, MK)<br />

TSH. (PC, MK)<br />

Lipid pr<strong>of</strong>ile. (PC, MK)<br />

HbA1c. (PC, MK)<br />

BUN/Cr. (PC, MK)<br />

Urine microalbumin. (PC, MK)<br />

ECG. (PC, MK)<br />

24-hour urinary cortisol (PC, MK)<br />

5. Communication skills: Students should be able to:<br />

Communicate the diagnosis, treatment plan, and subsequent follow-up to patients. (PC,<br />

CS)<br />

Elicit questions from the patient and his or her family about the management plan. (PC, CS)<br />

Adapt to the patient’s life-style and preferences, with emphasis on the patient’s role in<br />

treatment and maximizing compliance. (PC, CS)<br />

Assist the patient in understanding that attainment <strong>of</strong> ideal body weight may not necessarily<br />

be a realistic goal and that health benefits may be achieved with losses <strong>of</strong> five to 10<br />

percent body weight. (PC, CS)<br />

6. Management skills: Students should able to develop an appropriate evaluation and treatment<br />

plan for patients that includes:<br />

Determining when to obtain consultation from an endocrinologist, dietician, or obesity<br />

management specialist. (PC, SBP)<br />

Developing reasonable weight loss goals with the patient. (PC, MK)<br />

Developing a dietary plan. (PC, MK)<br />

Developing a prescription for physical activity. (PC, MK)<br />

Identifying indications for pharmacotherapy. (PC, MK)<br />

Identifying indications for bariatric surgery. (PC, MK)<br />

172


Accessing and utilizing appropriate information systems and resources to help delineate<br />

issues related to obesity. (PC, PLI)<br />

Incorporating patient preferences in the treatment plan. (PC)<br />

C. ATTITUDES AND PROFESSIONAL BEHAVIORS: Students should be able to:<br />

1. Demonstrate commitment to using risk-benefit, cost-benefit, and evidence-based<br />

considerations in the selection diagnostic and therapeutic interventions for obesity. (PLI, P)<br />

2. Respond appropriately to patients who are nonadherent to treatment for obesity.<br />

(CS, P)<br />

3. Demonstrate ongoing commitment to self-directed learning regarding obesity. (PLI,<br />

P)<br />

4. Appreciate the impact obesity has on a patient’s quality <strong>of</strong> life, well-being, ability to<br />

work, and family. (P)<br />

5. Recognize the importance <strong>of</strong> and demonstrate a commitment to the utilization <strong>of</strong><br />

other healthcare pr<strong>of</strong>essions in the treatment <strong>of</strong> obesity. (P, SBP)<br />

D. REFERENCES:<br />

Overweight and Obesity<br />

National Center for Chronic Disease Prevention and Health Promotion Center for Disease<br />

Control and Prevention<br />

U.S. <strong>Department</strong> <strong>of</strong> Health and Human Services<br />

www.cdc.gov/nccdphp/dnpa/obesity<br />

Aim for a Healthy Weight National Heart, Lung, and Blood Institute Obesity Education<br />

Initiative National Institutes <strong>of</strong> Health<br />

U.S. <strong>Department</strong> <strong>of</strong> Health and Human Services<br />

www.nhlbi.nih.gov/health/public/heart/obesity/lose_wt/index.htm<br />

Screening for Obesity in Adults<br />

Agency for Healthcare Research and Quality<br />

U.S. <strong>Department</strong> <strong>of</strong> Health and Human Services<br />

www.ahrq.gov/clinic/uspstf/uspsobes.htm<br />

Tsai AG, Wadden TA. Systematic review: an evaluation <strong>of</strong> major commercial weight loss<br />

programs in the United States. Ann Intern Med. 2005;152-56-66.<br />

Buchwald H, Avidor Y, Braunwald E, et al. Bariatric surgery: a systematic review and<br />

meta-analysis. JAMA. 2004;292:1724-37.<br />

Li Z, Maglione M, Tu W, et al. Meta-analysis: pharmacologic treatment <strong>of</strong> obesity. Ann<br />

Intern Med. 2005;142:532-46. \<br />

173


TRAINING PROBLEM #29: PNEUMONIA<br />

RATIONALE:<br />

Pneumonia continues to be a major public health issue, a leading reason for hospitalization, and<br />

a significant cause <strong>of</strong> mortality. Not only that, it is an important complication <strong>of</strong> admission for<br />

other causes. Many different specialties encounter pneumonia in the course <strong>of</strong> practice, the<br />

internist most particularly.<br />

PREREQUISITES:<br />

Prior knowledge, skills, and attitudes acquired during the pre-clinical experience should include:<br />

Ability to perform a complete medical history and physical.<br />

Ability to communicate with patients <strong>of</strong> diverse backgrounds.<br />

Anatomy and physiology <strong>of</strong> the pulmonary system.<br />

Pathogenesis and pathophysiology <strong>of</strong> pneumonia.<br />

Microbiology <strong>of</strong> the common pneumonia pathogens.<br />

Pharmacology <strong>of</strong> antimicrobial agents.<br />

SPECIFIC LEARNING OBJECTIVES:<br />

A. KNOWLEDGE: Students should be able to define, describe, and discuss:<br />

1. The epidemiology, pathophysiology, symptoms, signs, and typical clinical course <strong>of</strong><br />

community-acquired, nosocomial, and aspiration pneumonia and pneumonia in the<br />

immunocompromised host. (MK)<br />

2. The conceptualization <strong>of</strong> “typical” and “atypical” pneumonia and its limitations. (MK)<br />

3. Common pneumonia pathogens (viral, bacterial, mycobacterial, and fungal) in<br />

immunocompetent and immunocompromised hosts). (MK)<br />

4. Identify patients who are at risk for impaired immunity. (MK)<br />

5. Indications for hospitalization and ICU admission <strong>of</strong> patient with pneumonia. (MK)<br />

6. The radiographic findings <strong>of</strong> the various types <strong>of</strong> pneumonia. (MK)<br />

7. The antimicrobial treatments (e.g. antiviral, antibacterial, antimycobacterial, and antifungal)<br />

for community-acquired, nosocomial, and aspiration pneumonia, and pneumonia in the<br />

immunocompromised host. (MK)<br />

8. The implications <strong>of</strong> antimicrobial resistance. (MK)<br />

9. The pathogenesis, symptoms, and signs <strong>of</strong> the complications <strong>of</strong> acute bacterial pneumonia<br />

including: bacteremia, sepsis, parapneumonic effusion, empyema, meningitis, and<br />

metastatic microabscesses. (MK)<br />

10. The indications for and complications <strong>of</strong> chest tube placement. (MK)<br />

11. The indications for and efficacy <strong>of</strong> influenza and pneumococcal vaccinations. (MK)<br />

12. The indications and procedures for respiratory isolation. (MK)<br />

13. The Centers for Medicare & Medicaid Services (CMS) and the Joint Commission on the<br />

Accreditation <strong>of</strong> Healthcare Organizations (JCAHO) quality measures for communityacquired<br />

pneumonia treatment. (MK, PLI, SBP)<br />

B. SKILLS: Students should be able to demonstrate specific skills including:<br />

1. History-taking skills: Students should be able to obtain, document, and present an ageappropriate<br />

medical history that differentiates among etiologies <strong>of</strong> disease, including:<br />

174


The presence and quantification <strong>of</strong> fever, chills, sweats, cough, sputum, hemoptysis,<br />

dyspnea, and chest pain. (PC, CS)<br />

Historical features consistent with potential immunocompromise. (PC, CS)<br />

potential tuberculosis exposure (PC, CS)<br />

2. Physical exam skills: Students should be able to perform a physical exam to establish the<br />

diagnosis and severity <strong>of</strong> disease, including:<br />

Accurately determining respiratory rate and level <strong>of</strong> respiratory distress. (PC)<br />

Identifying bronchial breath sounds, rales, rhonchi, and wheezes. (PC)<br />

Identifying signs <strong>of</strong> pulmonary consolidation. (PC)<br />

Identifying signs <strong>of</strong> pleural effusion. (PC)<br />

Identifying signs <strong>of</strong> the complications <strong>of</strong> pneumonia. (PC)<br />

3. Differential diagnosis: Students should be able to generate a prioritized differential<br />

diagnosis recognizing specific history and physical exam findings that suggest a specific<br />

etiology <strong>of</strong> pneumonia and other possible diagnoses, including:<br />

Common cold. (PC, MK)<br />

Acute bronchitis. (PC, MK)<br />

Influenza. (PC, MK)<br />

Acute exacerbation <strong>of</strong> COPD. (PC, MK)<br />

Asthma exacerbation. (PC, MK)<br />

CHF. (PC, MK)<br />

Pulmonary embolism. (PC, MK)<br />

Aspiration. (PC, MK)<br />

4. Laboratory interpretation: Students should be able to recommend when to order diagnostic<br />

and laboratory tests and be able to interpret them, both prior to and after initiating treatment<br />

based on the differential diagnosis, including consideration <strong>of</strong> test cost and performance<br />

characteristics as well as patient preferences. Laboratory and diagnostic tests should include,<br />

when appropriate:<br />

CBC. (PC, MK)<br />

Blood cultures. (PC, MK)<br />

ABG. (PC, MK)<br />

Pleural fluid chemistry, cell counts, staining, and culture. (PC, MK)<br />

Chest radiograph. (PC, MK)<br />

Students should be able to define the indications for and interpret (with consultation)<br />

the results <strong>of</strong>:<br />

Chest CT. (PC, MK)<br />

5. Communication skills: Students should be able to:<br />

Communicate the diagnosis, treatment plan, prognosis, and subsequent follow-up to the<br />

patient and his or her family. (PC, CS)<br />

Elicit questions from the patient and his or her family about the management plan. (PC, CS)<br />

Educate the patient about pneumococcal and influenza immunizations. (PC, CS)<br />

Educate the patient about the importance <strong>of</strong> smoking cessation. (PC, CS)<br />

6. Basic and advanced procedural skills: Students should be able to:<br />

Place and interpret a tuberculin skin test (PPD). (PC)<br />

Obtain blood cultures. (PC)<br />

Obtain an ABG. (PC)<br />

7. Management skills: Students should able to develop an appropriate evaluation and<br />

treatment plan for patients that includes:<br />

175


Selecting an appropriate empiric antibiotic regimen for community-acquired, nosocomial,<br />

immunocompromised-host, and aspiration pneumonia, taking into account pertinent patient<br />

features. (PC, MK)<br />

Adjusting antimicrobial treatment according to the sputum staining and culture results. (PC,<br />

MK)<br />

Recognizing the complications <strong>of</strong> pneumonia. (PC, MK)<br />

Determining when to obtain consultation from a pulmonologist or infectious diseases<br />

specialist. (PC, SBP)<br />

Using a cost-effective approach based on the differential diagnosis. (PC, SBP)<br />

Accessing and utilizing appropriate information systems and resources to help delineate<br />

issues related to pneumonia. (PC, PLI)<br />

Incorporating patient preferences. (PC)<br />

C. ATTITUDES AND PROFESSIONAL BEHAVIORS: Students should be able to:<br />

1. Demonstrate commitment to using risk-benefit, cost-benefit, and evidence-based<br />

considerations in the selection diagnostic and therapeutic interventions for the various types <strong>of</strong><br />

pneumonia. (PLI, P)<br />

2. Recognize the importance <strong>of</strong> patient preferences when selecting among diagnostic and<br />

therapeutic options for pneumonia. (P)<br />

3. Demonstrate ongoing commitment to self-directed learning regarding pneumonia. (PLI, P)<br />

4. Appreciate the impact pneumonia has on a patient’s quality <strong>of</strong> life, well-being, ability to work,<br />

and the family. (P)<br />

5. Recognize the importance <strong>of</strong> and demonstrate a commitment to the utilization <strong>of</strong> other<br />

healthcare pr<strong>of</strong>essionals in the treatment <strong>of</strong> pneumonia. (P, SBP)<br />

6. Appreciate the importance <strong>of</strong> antimicrobial resistance. (P)<br />

7. Appreciate the public health role <strong>of</strong> the physician when treating certain types <strong>of</strong> pneumonia<br />

(e.g. tuberculosis). (P)<br />

8. Appreciate the importance <strong>of</strong> and demonstrate a commitment to meeting national health care<br />

quality measures for the treatment <strong>of</strong> acute MI. (P, SBP, PLI)<br />

D. REFERENCES:<br />

Improving Treatment Decisions for Patients with Community-Acquired Pneumonia Agency<br />

for Healthcare Research and Quality<br />

U.S. <strong>Department</strong> <strong>of</strong> Health and Human Services<br />

www.ahrq.gov/clinic/pneumonia/pneumonria.htm<br />

Ramsdell J, Narsavage GL, Fink JB. Management <strong>of</strong> community-acquired pneumonia<br />

in the home: an American College <strong>of</strong> Chest Physicians clinical position statement.<br />

Chest. 2005;127:1752-63.<br />

www.guideline.gov/summary/summary.aspx?ss=15&doc_id=7325&nbr=4 348<br />

Influenza<br />

Centers for Disease Control and Prevention<br />

U.S. <strong>Department</strong> <strong>of</strong> Health and Human Services<br />

www.cdc.gov/flu/<br />

Prevention <strong>of</strong> Pneumococcal Disease: Recommendations <strong>of</strong> the Advisory<br />

Committee on Immunization Practices (ACIP). MMWR 46(RR-08);1-24. Centers for<br />

Disease Control and Prevention<br />

U.S. <strong>Department</strong> <strong>of</strong> Health and Human Services<br />

176


www.cdc.gov/mmwr/PDF/RR/RR4608.pdf<br />

Prevention and Control <strong>of</strong> Influenza: Recommendations <strong>of</strong> the Advisory Committee<br />

on Immunization Practices (ACIP). MMWR 50(RR-04);1-46. Centers for Disease<br />

Control and Prevention<br />

U.S. <strong>Department</strong> <strong>of</strong> Health and Human Services<br />

www.cdc.gov/mmwr/PDF/RR/RR5004<br />

177


TRAINING PROBLEM #30: RHEUMATOLOGIC PROBLEMS<br />

RATIONALE:<br />

Rheumatologic diseases are an important part <strong>of</strong> the practice <strong>of</strong> internal medicine. This includes<br />

problems referring to specific joints as well as patients with systemic symptoms that are sometimes<br />

difficult to unify into a single diagnosis.<br />

PREREQUISITES:<br />

Prior knowledge, skills, and attitudes acquired during the pre-<strong>clerkship</strong> experience should include:<br />

Ability to perform a complete medical history and physical exam.<br />

Ability to communicate with patients <strong>of</strong> diverse backgrounds.<br />

Anatomy, physiology, and pathophysiology <strong>of</strong> the musculoskeletal system.<br />

Basic course work in immunology.<br />

Pharmacology <strong>of</strong> acetaminophen, nonsteroidal anti-inflammatory drugs (NSAIDs),<br />

glucocorticoids, disease-modifying antirheumatic drugs (DMARDs), drugs use in the treatment<br />

<strong>of</strong> gout.<br />

Basic bone radiograph interpretation.<br />

SPECIFIC LEARNING OBJECTIVES:<br />

A. KNOWLEDGE: Students should be able to define, describe, and discuss:<br />

1. A systematic approach to joint pain based on an understanding <strong>of</strong> pathophysiology to classify<br />

potential causes. (MK)<br />

2. The effect <strong>of</strong> the time course <strong>of</strong> symptoms on the potential causes <strong>of</strong> joint pain (acute vs.<br />

subacute vs. chronic). (MK)<br />

3. The difference between and pathophysiology <strong>of</strong> arthralgia vs. arthritis and mechanical vs.<br />

inflammatory joint pain. (MK)<br />

4. The distinguishing features <strong>of</strong> intra-articular and periarticular complaints (joint pain vs. bursitis<br />

and tendonitis). (MK)<br />

5. The effect <strong>of</strong> the features <strong>of</strong> joint involvement on the potential causes <strong>of</strong> joint pain<br />

(monoarticular vs. oligoarticular vs. polyarticular, symmetric vs. asymmetric, axial and/or<br />

appendicular, small vs. large joints, additive vs. migratory vs. intermittent). (MK)<br />

6. Indications for performing an arthrocentesis and the results <strong>of</strong> synovial fluid analysis. (MK)<br />

7. The pathophysiology and common signs and symptoms <strong>of</strong>:<br />

Osteoarthritis. (MK)<br />

Crystalline arthropathies. (MK)<br />

Septic arthritis. (MK)<br />

8. Indications for and effectiveness <strong>of</strong> intra-articular steroid injections. (MK)<br />

9. Treatment options for gout (e.g. colchicine, NSAIDs, steroids, uricosurics, xanthine oxidase<br />

inhibitors). (MK)<br />

10. The pathophysiology and common signs and symptoms <strong>of</strong> common periarticular disorders:<br />

Sprain/stain. (MK)<br />

Tendonitis. (MK)<br />

Bursitis. (MK)<br />

11. The basic pathophysiology <strong>of</strong> autoimmunity and autoimmune diseases. (MK)<br />

12. The basic role <strong>of</strong> genetics in autoimmune disorders. (MK)<br />

178


13. Typical clinical scenarios when systemic rheumatologic disorders should be considered:<br />

Diffuse aches and pains. (MK)<br />

Generalized weakness/fatigue. (MK)<br />

Myalgias with or without weakness. (MK)<br />

Arthritis with systemic signs (e.g. fever, weight loss). (MK)<br />

Arthritis with disorders <strong>of</strong> other systems (e.g. rash, cardiopulmonary symptoms,<br />

gastrointestinal symptoms, eye disease, renal disease, neurologic symptoms). (MK)<br />

14. The common signs and symptoms <strong>of</strong> and diagnostic approach to:<br />

Rheumatoid arthritis. (MK)<br />

Spondyloarthropathies (reactive arthritis/Reiter’s syndrome, ankylosing spondylitis,<br />

psoriatic arthritis). (MK)<br />

Systemic lupus erythematosus. (MK)<br />

Systemic sclerosis. (MK)<br />

Raynaud’s syndrome/phenomenon. (MK)<br />

Sjögren’s syndrome. (MK)<br />

Temporal arteritis and polymyalgia rheumatica. (MK)<br />

Other systemic vasculitides. (MK)<br />

Polymyositis and dermatomyositis. (MK)<br />

Fibromyalgia. (MK)<br />

B. SKILLS: Students should be able to demonstrate specific skills, including:<br />

1. History-taking skills: Students should be able to obtain, document, and present an ageappropriate<br />

medical history that differentiates among etiologies <strong>of</strong> disease, including:<br />

Eliciting features <strong>of</strong> joint complaints:<br />

o Pain. (PC, CS)<br />

o Stiffness. (PC, CS)<br />

o Location. (PC, CS)<br />

o Mode <strong>of</strong> onset. (PC, CS)<br />

o Duration. (PC, CS)<br />

o Severity. (PC, CS)<br />

o Exacerbating and alleviating factors. (PC, CS)<br />

o Warmth, redness, and tenderness. (PC, CS)<br />

o Associated nonarticular symptoms. (PC, CS)<br />

Determining when in the course <strong>of</strong> acute arthritis it is necessary to obtain a sexual history.<br />

(PC, CS)<br />

Determining the impact <strong>of</strong> rheumatologic complaints on a patient's activities <strong>of</strong> daily living.<br />

(PC, CS)<br />

2. Physical exam skills: Students should be able to perform a physical exam to establish the<br />

diagnosis and severity <strong>of</strong> disease, including:<br />

A systematic examination <strong>of</strong> all joints identifying the following abnormal findings:<br />

o Erythema, warmth, tenderness, and swelling. (PC)<br />

o Effusion. (PC)<br />

o Crepitus. (PC)<br />

o Altered range <strong>of</strong> motion. (PC)<br />

o Ulnar deviation. (PC)<br />

o Synovial thickening. (PC)<br />

o Joint alignment deformities (e.g. varus and valgus). (PC)<br />

o Podagra. (PC)<br />

179


Muscular bulk, strength, and tenderness. (PC)<br />

Examination <strong>of</strong> the skin identifying the following abnormal findings:<br />

o Rheumatoid and tophaceous nodules. (PC)<br />

o Alopecia. (PC)<br />

o Malar rash. (PC)<br />

o Sclerodactyly. (PC)<br />

o Telangiectasias. (PC)<br />

o Raynaud’s phenomenon. (PC)<br />

o Psoriasis. (PC)<br />

o Cutaneous manifestations <strong>of</strong> vasculitis (e.g. palpable purpura). (PC)<br />

3. Differential diagnosis: Students should be able to generate a prioritized differential diagnosis<br />

recognizing specific history and physical exam findings that suggest a specific etiology:<br />

Osteoarthritis. (PC, MK)<br />

Crystalline arthropathies. (PC, MK)<br />

Septic arthritis. (PC, MK)<br />

Rheumatoid arthritis. (PC, MK)<br />

Spondyloarthropathies (reactive arthritis/Reiter’s syndrome, ankylosing spondylitis,<br />

psoriatic arthritis). (PC, MK)<br />

Systemic lupus erythematosus. (PC, MK)<br />

Systemic sclerosis. (PC, MK)<br />

Raynaud’s syndrome/phenomenon. (PC, MK)<br />

Sjörgren’s syndrome. (PC, MK)<br />

Temporal arteritis and polymyalgia rheumatica. (PC, MK)<br />

Other systemic vasculitides. (PC, MK)<br />

Polymyositis and dermatomyositis. (PC, MK)<br />

Fibromyalgia. (PC, MK)<br />

5. Laboratory interpretation: Students should be able to recommend when to order diagnostic and<br />

laboratory tests and be able to interpret them, both priorto and after initiating treatment based<br />

on the differential diagnosis, including consideration <strong>of</strong> test cost and performance<br />

characteristics as well as patient preferences.<br />

Laboratory and diagnostic tests should include, when appropriate:<br />

CBC with differential. (PC, MK)<br />

Synovial fluid analysis (Gram stain, culture, crystal exam, cell count with differential, and<br />

glucose). (PC, MK)<br />

Uric acid. (PC, MK)<br />

ESR. (PC, MK)<br />

Rheumatoid factor (RF). (PC, MK)<br />

Antinuclear antibody test (ANA) and anti-DNA test. (PC, MK)<br />

Students should be able to define the indications for and interpret (with consultation)<br />

the results <strong>of</strong>:<br />

Plain radiographs <strong>of</strong> the shoulder, elbow, wrist, hand, hip, knee, ankle, and foot. (PC, MK)<br />

5. Communication skills: Students should be able to:<br />

Communicate the diagnosis, treatment plan, and subsequent follow-up to patients. (PC,<br />

CS)<br />

Elicit questions from the patient about the management plan. (PC, CS)<br />

6. Basic and advanced procedure skills: Students should be able to:<br />

Assist in the performance <strong>of</strong> an arthrocentesis and intra-articular corticosteroid<br />

injection. (PC)<br />

7. Management skills: Students should able to develop an appropriate evaluation and<br />

180


treatment plan for patients that includes:<br />

Selecting appropriate medications for the relief <strong>of</strong> joint pain. (PC, MK)<br />

Prescribing acute and preventative treatment for crystalline arthropathies. (PC, MK)<br />

Prescribing basic treatment options for septic arthritis. (PC, MK)<br />

Prescribing basic treatment options for systemic rheumatologic conditions. (PC, MK)<br />

Determining when to obtain consultation from a rheumatologist and orthopedic surgeon.<br />

(PC, SBP)<br />

Using a cost-effective approach based on the differential diagnosis. (PC, SBP)<br />

Accessing and utilizing appropriate information systems and resources to help delineate<br />

issues related to rheumatologic problems. (PC, PLI)<br />

Incorporating patient preferences. (PC)<br />

C. ATTITUDES AND PROFESSIONAL BEHAVIORS: Students should be able to:<br />

1. Demonstrate commitment to using risk-benefit, cost-benefit, and evidence-based<br />

considerations in the selection diagnostic and therapeutic interventions for rheumatologic<br />

problems. (PLI, P)<br />

2. Recognize the importance <strong>of</strong> patient preferences when selecting among diagnostic and<br />

therapeutic options for rheumatologic problems. (P)<br />

3. Respond appropriately to patients who are nonadherent to treatment for rheumatologic<br />

problems. (CS, P)<br />

4. Demonstrate ongoing commitment to self-directed learning regarding rheumatologic problems.<br />

(PLI, P)<br />

5. Appreciate the impact rheumatologic problems have on a patient’s quality <strong>of</strong> life, well-being,<br />

ability to work, and the family. (P)<br />

6. Recognize the importance <strong>of</strong> and demonstrate a commitment to the utilization<br />

<strong>of</strong> other healthcare pr<strong>of</strong>essions in the treatment <strong>of</strong> rheumatologic problems. (P, SBP)<br />

D. REFERENCES:<br />

Evaluation <strong>of</strong> the patient – history and physical examination, laboratory assessment,<br />

arthrocentesis and synovial fluid analysis. In Schumacher HR, Klippel JH, Koopman<br />

WJ, eds. Primer on the Rheumatic Diseases. 12 th Ed. Atlanta, GA: Arthritis Foundation;<br />

2001.<br />

Management Guidelines<br />

American College <strong>of</strong> Rheumatology<br />

www.rheumatology.org/publications/guidelines/index.asp<br />

Arthritis Foundation<br />

www.arthritis.org<br />

National Institute <strong>of</strong> Arthritis and Musculoskeletal and Skin Diseases<br />

National Institutes <strong>of</strong> Health<br />

U.S. <strong>Department</strong> <strong>of</strong> Health and Human Services<br />

www.niams.nih.gov<br />

181


TRAINING PROBLEM #31: SMOKING CESSATION<br />

RATIONALE:<br />

Smoking is a major public health issue because it causes or aggravates many serious illnesses.<br />

Effective intervention strategies for chronic smokers have been developed using principals <strong>of</strong><br />

behavioral counseling. These principals are applicable to other risky health behaviors. Health<br />

behavior risk assessment and intervention is now expected <strong>of</strong> physicians as part <strong>of</strong> the<br />

comprehensive care <strong>of</strong> adults. Selecting and performing an appropriate smoking cessation<br />

intervention is an important training problem for the third year medical student.<br />

PREREQUISITES:<br />

Prior knowledge, skills, and attitudes acquired during the pre-<strong>clerkship</strong> experience should include:<br />

Ability to perform a complete medical history and physical exam.<br />

Ability to communicate with patients <strong>of</strong> diverse backgrounds.<br />

Knowledge <strong>of</strong> the anatomy, physiology, and pathophysiology <strong>of</strong> the cardiopulmonary system.<br />

Knowledge <strong>of</strong> the pharmacology <strong>of</strong> addictive drugs.<br />

Knowledge <strong>of</strong> the risks <strong>of</strong> smoking, passive smoke, and smokeless tobacco.<br />

Appreciation <strong>of</strong> the reasons for or against discontinuing smoking.<br />

SPECIFIC LEARNING OBJECTIVES:<br />

A. KNOWLEDGE: Students should be able to define, describe, and discuss:<br />

1. The pharmacologic effects <strong>of</strong> nicotine. (MK)<br />

2. Nicotine withdrawal symptoms. (MK)<br />

3. Intervention strategies physicians can use for those patients willing and not willing to quit. (MK)<br />

4. The stages <strong>of</strong> change, including:<br />

Precontemplation. (MK)<br />

Contemplation. (MK)<br />

Preparation. (MK)<br />

Action. (MK)<br />

Maintenance. (MK)<br />

5. The “five A’s” <strong>of</strong> smoking cessation:<br />

Ask. (MK)<br />

Advise. (MK)<br />

Assess. (MK)<br />

Assist. (MK)<br />

Arrange. (MK)<br />

6. The “five R’s” <strong>of</strong> smoking cessation:<br />

Relevance. (MK)<br />

Risks. (MK)<br />

Rewards. (MK)<br />

Roadblocks. (MK)<br />

Repetition. (MK)<br />

7. The common barriers preventing patients from undertaking smoking cessation. (MK)<br />

8. The principles <strong>of</strong> at least one theory <strong>of</strong> behavior modification. (MK)<br />

9. Common medical diseases associated with chronic smoking and the effects <strong>of</strong> stopping on<br />

182


future risk. (MK)<br />

10. The indications for nicotine replacement therapy, pharmacotherapy (i.e. bupropion) or both.<br />

(MK)<br />

11. The association between smoking cessation and weight gain. (MK)<br />

12. The fact that tobacco dependence is considered a chronic relapsing disorder. (MK)<br />

13. The Centers for Medicare & Medicaid Services (CMS) and the Joint Commission on the<br />

Accreditation <strong>of</strong> Healthcare Organizations (JCAHO) quality measures for smoking cessation<br />

advice (i.e. all smoking patients admitted with pneumonia, HF, or an acute MI are given<br />

smoking cessation advice or counseling during hospital stay). (MK, PLI, SBP)<br />

B. SKILLS: Students should demonstrate specific skills, including:<br />

1. History-taking skills: Students should be able to obtain, document, and present an ageappropriate<br />

medical history, including:<br />

Ask the patient if he or she uses tobacco. (PC, CS)<br />

Determine the length and magnitude <strong>of</strong> tobacco use. (PC, CS)<br />

Ask if the patient is interested in stopping.<br />

Ask about the patient’s past experiences with smoking cessation. (PC, CS)<br />

Ask relevant questions regarding the symptoms <strong>of</strong> diseases associated with long-term<br />

smoking (e.g. CAD, COPD, PVD, CVA, lung cancer). (PC, CS)<br />

2. Physical exam skills: Students should be able to perform a physical exam to establish the<br />

diagnosis and severity <strong>of</strong> disease, including:<br />

Identification <strong>of</strong> nicotine stains. (PC)<br />

Identification <strong>of</strong> lesions with malignant potential on the lips and in the oral cavity. (PC)<br />

Identification <strong>of</strong> chest findings consistent with chronic obstructive lung disease and lung<br />

cancer. (PC)<br />

Examination <strong>of</strong> the heart and vascular system. (PC)<br />

3. Laboratory interpretation: Students should be able to recommend when to order diagnostic<br />

and laboratory tests and be able to interpret them, both prior to and after initiating treatment<br />

based on the differential diagnosis, including consideration <strong>of</strong> test cost and performance<br />

characteristics as well as patient preferences. Laboratory and diagnostic tests should include,<br />

when appropriate:<br />

Complete blood count to detect erythrocytosis. (PC, MK)<br />

Lipid pr<strong>of</strong>ile to aid in cardiovascular stratification. (PC, MK)<br />

4. Communication skills: Students should be able to:<br />

1. Ask every patient if he or she uses tobacco. (PC, CS)<br />

2. Advise every patient who smokes to stop in a nonjudgmental manner. (PC, CS)<br />

3. Assess the patient’s willingness to make attempt to quit. (PC, CS)<br />

4. Assist those who are willing to make a quit attempt through counseling. (PC, CS)<br />

5. Respond positively and non-judgmentally to the patient’s excuses or concerns about<br />

cessation. (PC, CS)<br />

6. Get the patient to commit to a specific action plan that can lead to complete<br />

cessation. (PC, CS)<br />

7. For those unwilling to quit, use <strong>of</strong> “5 R’s” to motivate the patient:<br />

o Relevance. (PC, CS)<br />

o Risks. (PC, CS)<br />

o Rewards. (PC, CS)<br />

183


o Roadblocks. (PC, CS)<br />

o Repetition. (PC, CS)<br />

5. Management skills: Students should be able to develop an appropriate evaluation and<br />

treatment plan for patient, including:<br />

Designing an intervention that matches the stage <strong>of</strong> behavior change demonstrated by the<br />

patient. (PC, CS)<br />

Explaining how to use nicotine patch, nasal spray or inhaler, and/or bupropion therapy.<br />

(PC, CS)<br />

Negotiating a follow-up plan with the patient. (PC, CS)<br />

Encouraging the patient to increase physical activity to lessen weight gain, if medically<br />

appropriate. (PC, CS)<br />

Accessing and utilizing appropriate information systems and resources to help delineate<br />

issues/resources related to aiding smoking cessation. (PC, PLI)<br />

Incorporating patient preferences. (PC)<br />

C. ATTITUDES AND PROFESSIONAL BEHAVIORS: Student should be able to:<br />

1. Demonstrate a commitment to meeting national quality standards for smoking cessation. (P,<br />

PLI, SBP)<br />

2. Maintain a non-judgmental attitude at all times regarding smoking cessation. (P)<br />

3. Demonstrate a commitment to deliver a non-judgmental "stop smoking" message to every<br />

patient who smokes. (P)<br />

4. Promote problem-solving by the patient. (P)<br />

5. Demonstrate commitment to using risk-benefit, cost-benefit, and evidence-based<br />

considerations in the selection <strong>of</strong> diagnostic and therapeutic interventions for smoking<br />

cessation. (PLI, P)<br />

6. Respond appropriately to patients who are non-adherent to treatment for smoking cessation.<br />

(P)<br />

7. Demonstrate ongoing commitment to self-directed learning regarding smoking cessation. (PLI,<br />

P)<br />

8. Appreciate the impact smoking cessation has on a patient’s quality <strong>of</strong> life, well-being, ability to<br />

work, and the famRecognize the importance and demonstrate a commitment to the utilization<br />

<strong>of</strong> other healthcare pr<strong>of</strong>essions in the treatment <strong>of</strong> smoking cessation. (P, SBP)<br />

D. RESOURCES:<br />

Public Health Service<br />

<strong>Department</strong> <strong>of</strong> Health and Human Services<br />

Tobacco Cessation Guideline<br />

www.surgeongeneral.gov/tobacco/default.htm<br />

Silagy C, Lancaster T, Stead L, Mant D, Fowler G. Nicotine replacement therapy for<br />

smoking cessation. Cochrane Database <strong>of</strong> Syst Rev. 2004;(3):CD000146.<br />

Schroeder SA. What to do with a patient who smokes. JAMA. 2005;294:482-7.<br />

Talwar A, Jain M, Vijayan VK. Pharmacotherapy <strong>of</strong> tobacco dependence. Med Clin<br />

North Am. 2004;88:1517-34.<br />

184


TRAINING PROBLEM #32: SUBSTANCE ABUSE<br />

RATIONALE:<br />

Substance abuse is a prevalent problem that intersects with patient care on a variety <strong>of</strong> different<br />

levels and in patients from every socio-economic status. Being able to recognize it, counsel<br />

patients appropriately, and devise an appropriate treatment plan is integral to the practice <strong>of</strong><br />

internal medicine.<br />

PREREQUISITES:<br />

Prior knowledge, skills, and attitudes acquired during the pre-<strong>clerkship</strong> experiences should<br />

include:<br />

Ability to perform a complete medical history and physical exam.<br />

Ability to communicate with patients <strong>of</strong> diverse backgrounds.<br />

Knowledge <strong>of</strong> drug and alcohol metabolism and physiology.<br />

SPECIFIC LEARNING OBJECTIVES:<br />

A. KNOWLEDGE: Students should be able to define, describe, and discuss:<br />

1. Presenting signs and symptoms <strong>of</strong> abuse <strong>of</strong> the following substances:<br />

Alcohol. (MK)<br />

Opioids. (MK)<br />

Cocaine. (MK)<br />

Amphetamines.(MK)<br />

Hallucinogens. (MK)<br />

Barbiturates. (MK)<br />

Marijuana. (MK)<br />

Anabolic steroids. (MK)<br />

Benzodiazepines. (MK)<br />

2. Signs, symptoms, risk factors for, and major causes <strong>of</strong> morbidity and mortality secondary to<br />

alcohol and drug abuse, intoxication, overdose, and withdrawal. (MK)<br />

3, Diagnostic criteria for substance abuse, dependency and addiction. (MK)<br />

4. Questions in the CAGE questionnaire:<br />

Cut down. (MK)<br />

Annoyed/angry. (MK)<br />

Guilty. (MK)<br />

Eye opener. (MK)<br />

5. Health benefits <strong>of</strong> substance abuse cessation. (MK)<br />

6. The potential role <strong>of</strong> genetics in substance abuse vulnerability. (MK)<br />

B. SKILLS: Students should demonstrate specific skills, including:<br />

1. History-taking skills: Students should be able to obtain, document, and present an<br />

age-appropriate medical history, that differentiates among etiologies <strong>of</strong> disease,<br />

including:<br />

Social history that is elicited in a nonjudgmental, supportive manner, using appropriate<br />

questioning (e.g. CAGE questions, etc.). (PC, CS)<br />

185


Use <strong>of</strong> injection drugs and shared needles. (PC, CS)<br />

Relevant medication history. (PC, CS)<br />

Immune status. (PC, CS)<br />

Family history <strong>of</strong> substance abuse. (PC, CS)<br />

Lifestyle factors that will influence patient’s access to illicit substances and interfere with<br />

ability to enable effective treatment. (PC, CS)<br />

Screening for depression and other psychiatric disease. (PC, CS)<br />

2. Physical exam skills: Students should be able to perform a physical exam to establish the<br />

diagnosis and severity <strong>of</strong> disease, including:<br />

Accurate recognition <strong>of</strong> signs that may indicate intoxication or withdrawal (e.g. behavioral or<br />

speech changes, changes in pupil size, conjunctival or nasal injection, tachycardia,<br />

sweating, piloerection, yawning, unsteady gait, etc.). (PC, MK)<br />

Examination <strong>of</strong> the nose for septal perforation as complication <strong>of</strong> cocaine use. (PC, MK)<br />

Examination <strong>of</strong> the skin for track marks or signs <strong>of</strong> needle use. (PC, MK)<br />

Identification <strong>of</strong> stigmata <strong>of</strong> secondary disease states (e.g. cirrhosis – splenomegaly,<br />

gynecomastia, telangiectasias, caput medusa, etc.) (PC, MK)<br />

Assessing for signs <strong>of</strong> endocarditis (e.g., fever, murmur, rash, etc). (PC, MK)<br />

Obtaining full mental status examination. (PC, MK)<br />

3. Differential diagnosis: Students should be able to generate a differential diagnosis<br />

recognizing history, physical exam and/or laboratory findings to determine the diagnosis <strong>of</strong><br />

abuse <strong>of</strong> drugs or alcohol and their sequelae. (PC, MK)<br />

4. Laboratory interpretation: Students should be able to recommend when to order diagnostic<br />

and laboratory tests and be able to interpret them, both prior to and after initiating treatment<br />

based on the differential diagnosis, including consideration <strong>of</strong> test cost and performance<br />

characteristics as well as patient preferences.<br />

Laboratory and diagnostic tests should include, when appropriate:<br />

Blood alcohol level. (PC, MK)<br />

Urine and serum toxicology screens. (PC, MK)<br />

Hepatic function panel. (PC, MK)<br />

Amylase and lipase levels. (PC, MK)<br />

Tests for HIV, hepatitis B and hepatitis C. (PC, MK)<br />

CBC. (PC, MK)<br />

Blood cultures. (PC, MK)<br />

5. Communication skills: Students should be able to:<br />

Communicate the evaluation, treatment plan, and subsequent follow up to the patient and<br />

his or her family in a non-judgmental manner. (PC, CS)<br />

Elicit questions from the patient and his or her family about the disease process and<br />

management plan. (PC, CS)<br />

Counsel patients regarding cessation and available community referral resources. (PC, CS,<br />

SBP)<br />

6. Management skills: Students should be able to develop an appropriate evaluation and<br />

treatment plan that includes:<br />

Assessing the patient’s motivation for achieving sobriety/abstinence. (PC, MK)<br />

Understanding the principles <strong>of</strong> acute management <strong>of</strong> drug/alcohol intoxication and<br />

withdrawal versus long-term treatment planning. (MK, PC)<br />

Using Clinical Institute Withdrawal Assessment for Alcohol. (CIWA-Ar) scale in acute<br />

alcohol withdrawal to prevent seizures or delirium tremens (MK, PC)<br />

Recommending appropriate use <strong>of</strong> benzodiazepines for alcohol withdrawal. (MK, PC)<br />

186


Determining when to obtain consultation from a psychiatrist. (PC, SBP)<br />

Accessing and utilizing appropriate information systems and resources to help delineate<br />

issues related to substance abuse. (PC, PLI)<br />

Incorporating patient preferences and understanding limitations <strong>of</strong> treatment. (PC)<br />

C. ATTITUDES AND PROFESSIONAL BEHAVIORS: Students should be able to:<br />

1. Demonstrate commitment to using risk-benefit, cost-benefit, and evidence-based<br />

considerations in the selection <strong>of</strong> diagnostic and therapeutic interventions for substance abuse. (PLI,<br />

P)<br />

2. Respond appropriately to patients who are non-adherent to treatment for substance abuse.<br />

(CS, P)<br />

3. Demonstrate ongoing commitment to self-directed learning regarding substance abuse. (PLI,<br />

P)<br />

4. Appreciate the impact substance abuse has on a patient’s as well as a family’s quality <strong>of</strong> life,<br />

well-being, and ability to work. (P)<br />

5. Recognize the importance and demonstrate a commitment to the utilization <strong>of</strong> other healthcare<br />

pr<strong>of</strong>essions in the treatment <strong>of</strong> substance abuse. (P, SBP)<br />

D. REFERENCES:<br />

U.S. Preventive Services Task Force. Screening and behavioral counseling<br />

interventions in primary care to reduce alcohol misuse recommendation statement. Ann<br />

Intern Med. 2004;140:554-6.<br />

Kosten TR, O'Connor PG. Management <strong>of</strong> drug and alcohol withdrawal. N Engl J Med.<br />

2003;348:1786-95.<br />

Mersy DJ. Recognition <strong>of</strong> alcohol and substance abuse. Am Fam Physician.<br />

2003;67:1529-32.<br />

187


TRAINING PROBLEM #33: VENOUS THROMBOEMBOLISM<br />

RATIONALE:<br />

Venous thromboembolic disease (DVT and PE) is a very common problem in internal medicine and<br />

one that can have devastating consequences if not appropriately diagnosed and treated. Diagnosis <strong>of</strong><br />

DVT and PE can be especially challenging. Prophylactic measures are very effective but do not<br />

eliminate the risk.<br />

PREREQUISITES:<br />

Prior knowledge, skills and attitudes acquired during the pre-<strong>clerkship</strong> experience should include:<br />

Ability to perform a complete medical history and physical exam (with particular attention to the<br />

cardiac, pulmonary, and venous systems).<br />

Ability to communicate with patients <strong>of</strong> diverse backgrounds.<br />

Knowledge <strong>of</strong> the anatomy, physiology, and pathophysiology <strong>of</strong> the cardiac, pulmonary, and<br />

venous systems.<br />

Physiology and pathophysiology <strong>of</strong> the hemostatic system.<br />

Pharmacology <strong>of</strong> antithrombotic agents.<br />

SPECIFIC LEARNING OBJECTIVES:<br />

A. KNOWLEDGE: Students should be able to define, describe and discuss:<br />

1. Risk factors for developing DVT, including:<br />

Prior history <strong>of</strong> DVT/PE. (MK)<br />

Immobility/hospitalization. (MK)<br />

Increasing age. (MK)<br />

Obesity. (MK)<br />

Trauma. (MK)<br />

Smoking. (MK)<br />

Surgery. (MK)<br />

Cancer. (MK)<br />

Acute MI. (MK)<br />

Stroke and neurologic trauma. (MK)<br />

Coagulopathy. (MK)<br />

Pregnancy. (MK)<br />

Oral estrogens. (MK)<br />

2. Genetic considerations predisposing to venous thrombosis. (MK)<br />

3. The symptoms and signs <strong>of</strong> DVT and PE. (MK)<br />

4. The differential diagnosis <strong>of</strong> DVT including the many causes <strong>of</strong> unilateral leg pain and swelling:<br />

Venous stasis and the postphlebitic syndrome. (MK)<br />

Lymphedema. (MK)<br />

Cellulitis. (MK)<br />

Superficial thrombophlebitis. (MK)<br />

Ruptured popliteal cyst. (MK)<br />

Musculoskeletal injury. (MK)<br />

Arterial occlusive disorders. (MK)<br />

5. The differential diagnosis <strong>of</strong> PE including the many causes <strong>of</strong> chest pain and dyspnea:<br />

188


MI/unstable angina. (MK)<br />

Congestive heart failure. (MK)<br />

Pericarditis. (MK)<br />

Pneumonia/bronchitis/COPD exacerbation. (MK)<br />

Asthma. (MK)<br />

Pulmonary hypertension. (MK)<br />

Pneumothorax. (MK)<br />

Musculoskeletal pain (e.g. rib fracture, costochondritis). (MK)<br />

6. Treatment modalities for DVT/PE, including:<br />

Unfractionated heparin. (MK)<br />

Low-molecular-weight heparin. (MK)<br />

Warfarin. (MK)<br />

Thrombolytics. (MK)<br />

7. The risks, benefits, and indications for inferior vena cava filters. (MK)<br />

8. The long-term sequelae <strong>of</strong> DVT and PE. (MK)<br />

9. Methods <strong>of</strong> DVT/PE prophylaxis, their indications and efficacy, including:<br />

Ambulation. (MK)<br />

Graded compression stockings. (MK)<br />

Pneumatic compression devices. (MK)<br />

Unfractionated heparin. (MK)<br />

Low-molecular-weight heparin. (MK)<br />

Warfarin. (MK)<br />

B. SKILLS: Students should demonstrate specific skills, including:<br />

1. History-taking skills: Students should be able to obtain, document and present an ageappropriate<br />

medical history that suggests the diagnosis <strong>of</strong> DVT or PE, including:<br />

The presence or absence <strong>of</strong> known risk factors. (PC, CS)<br />

Presence or absence <strong>of</strong> leg pain, swelling, warmth, discoloration, and palpable cord. (PC,<br />

CS)<br />

The presence or absence <strong>of</strong> dyspnea, chest pain, palpitations, cough, hemoptysis. (PC,<br />

CS)<br />

2. Physical exam skills: Students should be able to perform a physical examination to<br />

establish the diagnosis and severity <strong>of</strong> disease, including:<br />

Assessment <strong>of</strong> vital signs (i.e. hypotension, tachycardia, tachypnea, fever) and general<br />

appearance (i.e. degree <strong>of</strong> respiratory distress, anxiety). (PC)<br />

Accurate identification <strong>of</strong> leg swelling, erythema, warmth, and tenderness. (PC)<br />

Inspection for signs <strong>of</strong> lower extremity trauma, arthritis, or joint effusion. (PC)<br />

Identification <strong>of</strong> pleural friction rubs, wheezes, rales, rhonchi, and signs <strong>of</strong> pneumothorax.<br />

(PC)<br />

3. Differential diagnosis: Students should be able to generate a differential diagnosis<br />

for a patient suspected <strong>of</strong> having DVT/PE, recognizing specific history, physical examination<br />

and laboratory findings which suggest DVT/PE, including the disease states noted above. (PC,<br />

MK)<br />

4. Laboratory interpretation: Students should be able to recommend when to order diagnostic<br />

and laboratory tests and be able to interpret them, both prior to and after initiating treatment<br />

based on<br />

189


the differential diagnosis, including consideration <strong>of</strong> test cost and performance characteristics<br />

as well as patient preferences.<br />

Laboratory and diagnostic tests should include, where appropriate:<br />

Pulse oximetry. (PC, MK)<br />

12-lead ECG. (PC, MK)<br />

Chest radiograph. (PC, MK)<br />

ABG. (PC, MK)<br />

D-dimer. (PC, MK)<br />

Students should be able to define the indications for and interpret (with consultation)<br />

the results <strong>of</strong>:<br />

Duplex venous ultrasonography. (PC, MK)<br />

Ventilation perfusion (V/Q) scan. (PC, MK)<br />

CT angiography. (PC, MK)<br />

Pulmonary angiography. (PC, MK)<br />

Echocardiography. (PC, MK)<br />

5. Communication skills: Students should be able to:<br />

Communicate the diagnosis, treatment plan, and subsequent follow-up to the patient and<br />

his or her family. (PC, CS)<br />

Elicit questions from the patient and his or her family about the management plan. (PC, CS)<br />

6. Basic and advanced procedural skills: Students should be able to:<br />

Perform a 12-lead ECG. (PC)<br />

Obtain an ABG. (PC)<br />

7. Management skills: Students should be able to develop an appropriate evaluation and<br />

treatment plan for patients that includes:<br />

Outlining the acute and long-term treatment <strong>of</strong> isolated calf vein phlebitis, superficial<br />

thrombophlebitis, DVT, and thromboembolism, including appropriate use and monitoring <strong>of</strong><br />

heparin and warfarin. (MK,<br />

PC)<br />

Understanding the indications for placement <strong>of</strong> inferior vena cava filter, indications and<br />

complications <strong>of</strong> thrombolytic therapy, as well as indications for performing a<br />

hypercoaguability work-up. (PC, MK)<br />

Determining when to obtain consultation from a pulmonologist or interventional radiologist.<br />

(PC, MK)<br />

Using a cost-effective approach based on the differential diagnosis. (PC, SBP)<br />

Accessing and utilizing appropriate information systems and resources to help delineate<br />

issues related to venous thromboembolism. (PC, PLI)<br />

Incorporating patient preferences. (PC)<br />

C. ATTITUDES AND PROFESSIONAL BEHAVIORS: Students should be able to:<br />

1. Demonstrate commitment to using risk-benefit, cost-benefit, and evidence-based<br />

considerations in the selection <strong>of</strong> diagnostic and therapeutic interventions for venous<br />

thromboembolic disease. (PLI, P)<br />

2. Respond appropriately to patients who are non-adherent to treatment for venous<br />

thromboembolic disease. (CS, P)<br />

3. Demonstrate ongoing commitment to self-directed learning regarding venous thromboembolic<br />

disease. (PLI, P)<br />

4. Appreciate the impact venous thromboembolic disease has on a patient’s quality <strong>of</strong> life, well-<br />

190


eing, ability to work, and the family. (P)<br />

5. Recognize the importance and demonstrate a commitment to the utilization <strong>of</strong> other healthcare<br />

pr<strong>of</strong>essions in the treatment <strong>of</strong> venous thromboembolic disease. (P, SBP)<br />

D. REFERENCES:<br />

American College <strong>of</strong> Chest Physicians. The seventh ACCP conference on<br />

antithrombotic and thrombolytic therapy: evidence-based guidelines. Chest. 2004;126<br />

(Number 3 Supplement)<br />

www.chestjournal.org/content/vol126/3_suppl<br />

Spyropoulos AC. Emerging strategies in the prevention <strong>of</strong> venous thromboembolism in<br />

hospitalized medical patients. Chest. 2005;128:958-69.<br />

Stein PD, Hull RD, Patel KC, Olson RE, Ghali WA, Brant R, Biel RK, Bharadia V, Kalra<br />

NK. D-dimer for the exclusion <strong>of</strong> acute venous thrombosis and pulmonary embolism: a<br />

systematic review. Ann Intern Med. 2004;140:589-602.<br />

Bates SM, Ginsberg JS. Clinical practice. Treatment <strong>of</strong> deep-vein thrombosis. N Engl J<br />

Med. 2004;351:268-77.<br />

Fedullo PF. Tapson VF. Clinical practice. The evaluation <strong>of</strong> suspected pulmonary<br />

embolism. N Engl J Med. 2003;349:1247-56.<br />

191


GENERAL CLINICAL CORE COMPETENCIES IN INTERNAL MEDICINE<br />

#1 DIAGNOSTIC DECISION MAKING<br />

RATIONALE:<br />

Physicians are responsible for directing and conducting the diagnostic evaluation <strong>of</strong> a broad range <strong>of</strong><br />

patients, including patients seeking advice on prevention <strong>of</strong> and screening for disease and patients<br />

with acute and chronic illnesses. In a time <strong>of</strong> rapidly proliferating tests, medical students must learn<br />

how to design safe, expeditious, and cost-effective diagnostic evaluations. This requires welldeveloped<br />

diagnostic decision-making skills that incorporate probability-based thinking.<br />

PREREQUISITES:<br />

Prior knowledge, skills, and attitudes acquired during the pre-<strong>clerkship</strong> experience should include:<br />

Required course in pathophysiology.<br />

Required course in clinical epidemiology and biostatistics.<br />

Ability to perform a complete medical history and physical exam.<br />

Ability to communicate with patients <strong>of</strong> diverse backgrounds.<br />

SPECIFIC LEARNING OBJECTIVES:<br />

A. KNOWLEDGE: Students should be able to define, describe, and discuss:<br />

1. Key history and physical examination findings pertinent to the differential diagnosis. (MK)<br />

2. Information resources for determining diagnostic options for patients with common and<br />

uncommon medical problems. (MK, PLI)<br />

3. Key factors to consider when selecting from among diagnostic tests, including pretest<br />

probabilities, performance characteristics <strong>of</strong> tests (sensitivity, specificity, likelihood ratios), cost,<br />

risk, and patient preferences. (MK, P)<br />

4. The basics <strong>of</strong> the potential role <strong>of</strong> genetic information in diagnostic decision making. (MK)<br />

5. Relative cost <strong>of</strong> diagnostic tests. (MK)<br />

6. How critical pathways or practice guidelines can be used to guide diagnostic test ordering.<br />

(MK)<br />

7. The methods <strong>of</strong> deductive reasoning, forward thinking, and pattern recognition in clinical<br />

decision making. (MK)<br />

B. SKILLS: Students should demonstrate specific skills, including:<br />

1. Identifying problems with which a patient presents, appropriately synthesizing these into logical<br />

clinical syndromes. (PC)<br />

2. Identifying which problems are <strong>of</strong> highest priority. (PC)<br />

3. Formulating a differential diagnosis based on the findings from the history and physical<br />

examination. (PC)<br />

4. Using probability-based thinking and pattern recognition to identify the most likely diagnoses.<br />

(PC)<br />

5. Using the differential diagnosis to help guide diagnostic test ordering and sequencing. (PC)<br />

6. Using pretest probabilities and scientific evidence about performance characteristics <strong>of</strong> tests<br />

(sensitivity, specificity, likelihood ratios) to determine post-test probabilities according to the<br />

predictive value paradigm. (PC)<br />

7. Participating in selecting the diagnostic studies with the greatest likelihood <strong>of</strong> providing useful<br />

results at a reasonable cost. (PC)<br />

192


8. Communicating the prioritized differential diagnosis to the patient and his or her family. (CS)<br />

C. ATTITUDES AND PROFESSIONAL BEHAVIORS: Students should be able to:<br />

1. Incorporate the patient’s perspective into diagnostic decision making. (P)<br />

2. Recognize the importance <strong>of</strong> patient preferences when selecting among diagnostic tests. (P)<br />

3. Demonstrate commitment to using risk-benefit, cost-benefit, and evidence-based<br />

considerations in the selection <strong>of</strong> diagnostic tests. (PLI, P)<br />

4. Seek feedback regularly regarding diagnostic decision making and respond appropriately and<br />

productively. (P)<br />

5. Limit the chances <strong>of</strong> false positive/false negative results by demonstrating thoughtful test<br />

selection. (P)<br />

6. Appreciate the element <strong>of</strong> uncertainty in diagnostic testing, including the occurrence and<br />

causes <strong>of</strong> false positive and false negative results. (P)<br />

7. Appreciate the impact uncertainty may have on the patient. (P)<br />

8. Recognize the importance <strong>of</strong> and demonstrate a commitment to the utilization <strong>of</strong> other health<br />

care pr<strong>of</strong>essionals in diagnostic decision making. (P, SBP)<br />

D. REFERENCES:<br />

Mark, DB. Decision-making in clinical medicine. In Kasper DL, Braunwald EB, Fauci AS, Hauser<br />

SL, Longo DL, Jameson JL, eds. Harrison’s Principles <strong>of</strong> Internal <strong>Medicine</strong>. 16th ed. New York, NY:<br />

McGraw-Hill; 2005:6-13.<br />

Strauss SE, Richardson WS, Glasziou P, Haynes RB. Evidence Based <strong>Medicine</strong>: How to Practice<br />

and Teach EBM. 3rd ed. New York, NY: Churchill Livingstone; 2005.<br />

Primer to the Internal <strong>Medicine</strong> Clerkship A Guide Produced by the Clerkship Directors in Internal<br />

<strong>Medicine</strong> Clerkship Directors in Internal <strong>Medicine</strong><br />

www.im.org/CDIM/primer.htm<br />

Ferri FF. Differential diagnosis. In Ferri FF, ed. Practical Guide to the Care <strong>of</strong> the Medical Patient.<br />

6th ed. St. Louis, MO: Mosby; 2004:39-113.<br />

193


#2 CASE PRESENTATION<br />

GENERAL CLINICAL CORE COMPETENCIES<br />

RATIONALE:<br />

Communicating patient care information to colleagues and other health care pr<strong>of</strong>essionals is an<br />

essential skill regardless <strong>of</strong> specialty. Internists have traditionally given special attention to case<br />

presentation skills because <strong>of</strong> the comprehensive nature <strong>of</strong> patient evaluations and the various<br />

settings in which internal medicine is practiced. Students should develop facility with different types<br />

<strong>of</strong> case presentations: written and oral, new patient and follow-up, inpatient and outpatient.<br />

PREREQUISITES:<br />

Prior knowledge, skills, and attitudes acquired during the pre-<strong>clerkship</strong> experience should include:<br />

Ability to perform a complete medical history and physical exam.<br />

Ability to communicate with patients <strong>of</strong> diverse backgrounds.<br />

SPECIFIC LEARNING OBJECTIVES:<br />

A. KNOWLEDGE: Students should be able to define, describe, and discuss:<br />

1. Components <strong>of</strong> comprehensive and abbreviated case presentations (oral and written)<br />

and settings appropriate for each. (MK)<br />

B. SKILLS: Students should be able to demonstrate specific skills, including:<br />

1. Prepare legible, comprehensive, and focused new patient workups that include the<br />

following features as clinically appropriate:<br />

Chief complaint. (PC, CS)<br />

Identifying data. (PC, CS)<br />

Concise history <strong>of</strong> the present illness organized chronologically, with minimal repetition,<br />

omission, or extraneous information, and including pertinent positives and negatives.<br />

(PC, CS)<br />

Past medical history, including relevant details. (PC, CS)<br />

Medications with dosages and frequencies, including herbals, supplements, and overthe-counter<br />

medications. (PC, CS)<br />

Allergies with specific details <strong>of</strong> the reaction. (PC, CS)<br />

Substance use, including tobacco, alcohol, and illicit drugs. (PC, CS)<br />

Family history. (PC, CS)<br />

Social history. (PC, CS)<br />

Review <strong>of</strong> symptoms. (PC, CS)<br />

A comprehensive physical examination with detail pertinent to the patient’s problem.<br />

(PC, CS)<br />

A succinct, prioritized, and where appropriate complete list <strong>of</strong> all problems identified by<br />

the history and physical examination. (PC, CS)<br />

A differential diagnosis (appropriate for the student’s level <strong>of</strong> training) for each problem<br />

that is neither over-inclusive or under-inclusive, addresses all reasonable possibilities,<br />

pays special attention to diagnoses that are potentially the most serious or lifethreatening,<br />

and is supported by the use <strong>of</strong> pertinent positives and negatives. (PC, CS)<br />

194


A diagnostic and treatment plan for each problem (appropriate for the student’s level <strong>of</strong><br />

training). (PC, CS)<br />

2. Orally present a new inpatient’s or outpatient’s case in a manner that includes the<br />

following characteristics:<br />

Logically and chronologically develops the history <strong>of</strong> the present illness and tells the<br />

patient’s “story.” (PC, CS)<br />

Summarizes the pertinent positives and negatives. (PC, CS)<br />

Succinctly presents past medical history, family history, social history, and review <strong>of</strong><br />

symptoms. (PC, CS)<br />

Includes a logical, organized, and prioritized differential diagnosis. (PC, CS)<br />

Includes diagnostic and therapeutic plans. (PC, CS)<br />

Can be made briefer when necessary. (PC, CS)<br />

Is presented as much from memory as possible with minimal reference to memory<br />

aids with the exception <strong>of</strong> highly important dates, diagnostic tests, laboratory values.<br />

(PC, CS)<br />

3. Orally present a follow-up inpatient’s or outpatient’s case in a manner that includes the<br />

following characteristics:<br />

Focused and very concise. (PC, CS)<br />

Problem-based. (PC, CS)<br />

Emphasizes pertinent new findings. (PC, CS)<br />

Includes diagnostic and therapeutic plans. (PC, CS)<br />

Can be made briefer when necessary. (PC, CS)<br />

Is presented as much from memory as possible with minimal reference to memory<br />

aids with the exception <strong>of</strong> highly important dates, diagnostic tests, laboratory values.<br />

(PC, CS)<br />

4. Produce inpatient or outpatient progress notes in a manner that includes the following<br />

characteristics:<br />

Is appropriately titled. (PC, CS)<br />

Includes a brief subjective that addresses new or changed patient symptoms. (PC,<br />

CS)<br />

Provides an accurate and succinct accounting <strong>of</strong> the objective data<br />

(e.g. vital signs, in/outs, telemetry monitoring, focused physical examination,<br />

laboratory results, and diagnostic tests). (PC, CS)<br />

Includes a prioritized problem list with a concise assessment and plan for each. (PC,<br />

CS)<br />

5. Select the mode <strong>of</strong> presentation that is most appropriate to the clinical situation (e.g.<br />

written vs. oral, long vs. short, etc.). (PC, CS)<br />

C. ATTITUDES AND PROFESSIONAL BEHAVIORS: Students should be able to:<br />

1. Demonstrate ongoing commitment to self-directed learning regarding case presentation<br />

skills by regularly seeking feedback on presentations. (PLI, P)<br />

2. Respond appropriately and productively to feedback regarding performance.<br />

(P)<br />

3. Accurately and objectively record and present all data. (P)<br />

195


4. Demonstrate respect for the patient’s privacy when dealing with protected health<br />

information and follow Health Information Portability and Accountability Act (HIPAA)<br />

standards. (P)<br />

D. REFERENCES:<br />

Primer to the Internal <strong>Medicine</strong> Clerkship A Guide Produced by the Clerkship Directors in<br />

Internal <strong>Medicine</strong> Clerkship Directors in Internal <strong>Medicine</strong><br />

www.im.org/CDIM/primer.htm<br />

Sobel RK. MSL – medicine as a second language. N Engl J Med. 2005;352:1945<br />

196


GENERAL CLINICAL CORE COMPETENCIES<br />

#3 HISTORY TAKING AND PHYSICAL EXAMINATION<br />

RATIONALE:<br />

The ability to obtain an accurate medical history and carefully perform a physical examination is<br />

fundamental to providing comprehensive care to adult patients. In particular, the internist must be<br />

thorough and efficient in obtaining a history and performing a physical examination with a wide variety<br />

<strong>of</strong> patients, including healthy adults (both young and old), adults with acute and chronic medical<br />

problems, adults with complex life-threatening diseases, and adults from diverse socioeconomic and<br />

cultural backgrounds. The optimal selection <strong>of</strong> diagnostic tests, choice <strong>of</strong> treatment, and use <strong>of</strong><br />

subspecialists, as well as the physician’s relationship and rapport with patients, all depend on welldeveloped<br />

history-taking and physical diagnosis skills. These skills, which are fundamental to<br />

effective patient care, should be a primary focus <strong>of</strong> the student’s work during the core <strong>clerkship</strong> in<br />

internal medicine.<br />

PREREQUISITES:<br />

Prior knowledge, skills, and attitudes acquired during the pre-<strong>clerkship</strong> experience<br />

should include:<br />

Required pre-clinical courses in physical examination and physician-patient<br />

communication (should include instruction in breast, pelvic, rectal, and male genital exams).<br />

Ability to perform a complete medical history and physical exam on a wide variety <strong>of</strong><br />

patients including adolescents and older adults.<br />

Ability to effectively communicate with patients <strong>of</strong> diverse backgrounds.<br />

Basic skills for obtaining a history related to substance abuse, sexual, occupational,<br />

and mental health.<br />

SPECIFIC LEARNING OBJECTIVES:<br />

A. KNOWLEDGE: Students should be able to define, describe, and discuss:<br />

1. The significant attributes <strong>of</strong> a symptom, including: location and radiation, intensity, quality,<br />

temporal sequence (onset, duration, frequency), alleviating factors, aggravating factors,<br />

setting, associated symptoms, functional impairment, and patient’s interpretation <strong>of</strong> symptom.<br />

(MK)<br />

2. The four methods <strong>of</strong> physical examination (inspection, palpation, percussion, and<br />

auscultation), including where and when to use them, their purposes, and the findings they<br />

elicit. (MK)<br />

3. The physiologic mechanisms that explain key findings in the history and physical exam. (MK)<br />

4. The diagnostic value <strong>of</strong> the history and physical examination. (MK)<br />

B. SKILLS: Students should be able to demonstrate specific skills, including:<br />

1. Using language appropriate for each patient. (PC, CS)<br />

2. Using non-verbal techniques to facilitate communication and pursue relevant inquiry. (PC, CS)<br />

3. Eliciting the patient’s chief complaint as well as a complete list <strong>of</strong> the patient’s concerns.<br />

(PC,CS)<br />

4. Obtaining a patient’s history in a logical, organized, and thorough manner, covering the<br />

following:<br />

197


History <strong>of</strong> present illness. (PC, CS)<br />

past medical history (including usual source <strong>of</strong> and access to health care, childhood and<br />

adult illnesses, injuries, surgical procedures, obstetrical history, psychiatric problems,<br />

sexual history, and hospitalizations). (PC, CS)<br />

Preventive health measures. (PC, CS)<br />

Medications with dosages and frequencies, including herbals, supplements, and overthe-counter<br />

medications. (PC, CS)<br />

Allergies with specific details <strong>of</strong> the reaction. (PC, CS)<br />

Substance use including tobacco, alcohol, and illicit drugs. (PC, CS)<br />

Family history. (PC, CS)<br />

Social history. (PC, CS)<br />

Occupational history. (PC, CS)<br />

Review <strong>of</strong> symptoms. (PC, CS)<br />

5. Obtaining, whenever necessary, supplemental historical information from collateral sources,<br />

such as significant others or previous physicians. (PC, CS)<br />

6. Demonstrating proper hygienic practices whenever examining a patient. (PC)<br />

7. Positioning the patient and self properly for each part <strong>of</strong> the physical examination. (PC)<br />

8. Performing a physical examination for a patient in a logical, organized, respectful, and<br />

thorough manner, including:<br />

The patient’s general appearance. (PC)<br />

Vital signs. (PC)<br />

Pertinent body regions/organ systems. (PC)<br />

When appropriate breast, pelvic, rectal, male genital exams. (PC)<br />

When appropriate fundoscopic exam. (PC)<br />

When appropriate full neurologic exam. (PC)<br />

9. Adapting the scope and focus <strong>of</strong> the history and physical exam appropriately to the medical<br />

situation and the time available. (PC)<br />

10. Being observant <strong>of</strong> the patient’s modesty as much as possible. (PC, P)<br />

C. ATTITUDES AND PROFESSIONAL BEHAVIORS: Students should be able to:<br />

1. Appreciate the essential contribution <strong>of</strong> a pertinent and history and physical examination<br />

to patient care. (P)<br />

2. Demonstrate ongoing commitment to self-directed learning regarding history taking and<br />

physical examination skills. (PLI, P)<br />

3. Seek feedback regularly regarding history and physical examination skills and respond<br />

appropriately and productively. (P)<br />

4. Recognize the importance <strong>of</strong> and demonstrate a commitment to the utilization <strong>of</strong> other<br />

health care pr<strong>of</strong>essions in obtaining a history and physical examination (e.g. interpreter<br />

services, advanced practice nurses, etc.). (P, SBP)<br />

5. Establish a habit <strong>of</strong> updating historical information and repeating important parts <strong>of</strong> the<br />

physical examination during follow-up visits. (P)<br />

6. Demonstrate consideration for the patient’s modesty, feelings, limitations, and<br />

sociocultural background whenever taking a history and performing a physical<br />

examination. (P)<br />

7. Appreciate that some patients will be very anxious about the physical examination,<br />

particularly the breast, pelvic, rectal, and male genital exams. (P)<br />

D. REFERENCES:<br />

198


McGee SR. Evidence-Based Physical Diagnosis. Philadelphia, PA: W. B. Saunders Company;<br />

2001.<br />

Bickley LS, Szilagyi PG. Bates’ Guide to Physical Examination and History Taking. 8th Edition.<br />

Philadelphia, PA: Lipincott Williams and Wilkens; 2002.<br />

The Auscultation Assistant<br />

www.med.ucla.edu/wilkes/intro.html<br />

Criley JM. The Physiologic Origin <strong>of</strong> Heart Sounds and Murmurs: The Unique Interactive Guide<br />

to Cardiac Diagnosis. Philadelphia, PA: Lippincott Williams and Wilkins; 1997.<br />

Heart Sounds and Cardiac Arrhythmias<br />

Medical Multimedia Laboratories<br />

www.blaufuss.org<br />

199


GENERAL CLINICAL CORE COMPETENCIES<br />

#4 COMMUNICATION AND RELATIONSHIPS WITH PATIENTS AND COLLEAGUES<br />

RATIONALE:<br />

The physician-patient relationship forms the core <strong>of</strong> the practice <strong>of</strong> internal medicine. Many<br />

physicians view it as the most satisfying aspect <strong>of</strong> their work. The medical interview and the<br />

relationship between physician and patient are important diagnostic and therapeutic tools. Effective<br />

communication skills are needed for a physician to serve as an effective patient advocate.<br />

Communication skills also are needed to address patient concerns and requests. Pr<strong>of</strong>iciency in<br />

communicating with patients results in increased patient and physician satisfaction, increased<br />

adherence to therapy, and reduced risk <strong>of</strong> malpractice claims. The student on the internal medicine<br />

<strong>clerkship</strong> interacts with a diverse array <strong>of</strong> patients, physicians, and other health team members,<br />

necessitating pr<strong>of</strong>iciency in communication and interpersonal skills. Students also witness how<br />

diversities <strong>of</strong> age, gender, race, culture, socioeconomic class, personality, and intellect require a<br />

sensitive and flexible approach. The result <strong>of</strong> pr<strong>of</strong>iciency in communication and interpersonal skills is<br />

increased satisfaction for both doctor and patient.<br />

PREREQUISITES:<br />

Prior knowledge, skills, and attitudes acquired during the pre-<strong>clerkship</strong> experience should<br />

include:<br />

Required pre-clinical courses in physician-patient communication.<br />

Ability to perform a complete medical history on a wide variety <strong>of</strong> patients, including adolescents<br />

and older adults.<br />

Ability to communicate with patients <strong>of</strong> diverse backgrounds.<br />

Basic skills for obtaining a history related to substance abuse and sexual, occupational, and<br />

mental health.<br />

Basic skills for discussing issues relating to advance directives.<br />

Basic skills for breaking bad news.<br />

SPECIFIC LEARNING OBJECTIVES:<br />

A. KNOWLEDGE: Students should be able to define, describe, and discuss:<br />

1. How patients’ and physicians’ perceptions, preferences, and actions are affected by<br />

cultural and psychosocial factors and how these factors affect the doctor-patient<br />

relationship. (MK, P)<br />

2. The role and contribution <strong>of</strong> each team member to the care <strong>of</strong> the patient. (MK, SBP)<br />

3. The role <strong>of</strong> psychosocial factors in team interactions. (MK)<br />

4. The role <strong>of</strong> the physician as patient advocate. (MK)<br />

5. Strategies for establishing positive patient-doctor relationships. (MK)<br />

6. Patient, physician, and system barriers to successfully negotiated treatment plans and<br />

patient adherence; strategies that may be used to overcome these barriers. (MK, SBP)<br />

6. Useful strategies when a communicating with patients via an interpreter. (MK)<br />

7. Basic techniques for breaking bad news. (MK)<br />

8. Basic tenants <strong>of</strong> genetic counseling. (MK)<br />

B. SKILLS: Students should be able to demonstrate specific skills, including:<br />

200


1. Demonstrating appropriate listening skills, including verbal and non-verbal techniques<br />

(e.g., restating, probing, clarifying, silence, eye contact, posture, touch) to communicate<br />

empathy and help educate the patient. (CS)<br />

2. Demonstrating effective verbal skills including appropriate use <strong>of</strong> open- and closedended<br />

questions, repetition, facilitation, explanation, and interpretation. (CS)<br />

3. Determining the information a patient has independently obtained about his or her<br />

problems. (CS)<br />

4. Identifying patients’ emotional needs. (CS)<br />

5. Respond to empathic opportunities by naming the emotions or feelings expressed. (CS)<br />

6. Eliciting the patient’s point <strong>of</strong> view and concerns about his or her illness and the medical<br />

care he or she is receiving. (CS)<br />

7. Discussing how the health problem affects the patient’s life. (CS)<br />

8. Determining the extent to which a patient wants to be involved in making decisions<br />

about his or her care. (CS)<br />

9. Providing basic information and an explanation <strong>of</strong> the diagnosis, prognosis, and<br />

treatment plan. (CS)<br />

10. Responding to patients’ concerns and expectations. (CS)<br />

11. With guidance and direct supervision, participating in breaking bad news to patients.<br />

(CS)<br />

12. With guidance and direct supervision, participating in discussing basic issues regarding<br />

advance directives with patients and their families. (CS)<br />

13. With guidance and direct supervision, participating in discussing basic end-<strong>of</strong> life issues<br />

with patients and their families. (CS)<br />

14. Assessing patient commitment and adherence to a treatment plan taking into account<br />

personal and economic circumstances. (CS)<br />

15. Working with a variety <strong>of</strong> patients, including multi-problem patients, angry patients,<br />

somatizing patients, and substance abuse patients. (CS)<br />

16. Working as an effective member <strong>of</strong> the patient care team, incorporating skills in interpr<strong>of</strong>essional<br />

communication and collaboration. (CS, SBP)<br />

17. Giving and receiving constructive feedback. (CS)<br />

18. Orally presenting a new inpatient’s or outpatient’s case in a manner that includes the<br />

following characteristics:<br />

Logically and chronologically develops the history <strong>of</strong> the present illness and tells<br />

the patient’s “story.” (PC, CS)<br />

Summarizes the pertinent positives and negatives. (PC, CS)<br />

Succinctly presents past medical history, family history, social history, and review<br />

<strong>of</strong> symptoms. (PC, CS)<br />

includes a logical, organized, and prioritized differential diagnosis (PC, CS)<br />

Includes diagnostic and therapeutic plans. (PC, CS)<br />

Can be made briefer when necessary. (PC, CS)<br />

Is presented as much from memory as possible with minimal reference to<br />

memory aids with the exception <strong>of</strong> highly important dates, diagnostic tests,<br />

laboratory values. (PC, CS)<br />

19. Orally presenting a follow-up inpatient’s or outpatient’s case in a manner that includes<br />

the following characteristics:<br />

Is focused, very concise, and problem-based. (PC, CS)<br />

Emphasizes pertinent new findings. (PC, CS)<br />

Includes diagnostic and therapeutic plans. (PC, CS)<br />

201


Can be made briefer when necessary. (PC, CS)<br />

Is presented as much from memory as possible with minimal reference to<br />

memory aids with the exception <strong>of</strong> highly important dates, diagnostic tests, and<br />

laboratory values. (PC, CS)<br />

20. Demonstrating the ability to make clear and concise presentations about topics<br />

assigned to research. (CS)<br />

21. Demonstrating basic strategies for conflict management and resolution. (CS)<br />

22. Demonstrating basic techniques <strong>of</strong> communication with non-English speaking patient<br />

via an interpreter. (PC, CS)<br />

C. ATTITUDES AND PROFESSIONAL BEHAVIORS: Students should be able to:<br />

1. Demonstrate ongoing commitment to self-directed learning regarding effective doctorpatient<br />

communication skills. (PLI, P)<br />

2. Seek feedback regularly regarding communication skills and respond appropriately and<br />

productively. (P)<br />

3. take into consideration in each case the patient’s psychosocial status (P)<br />

4. Demonstrate respect for patients. (P)<br />

5. Involve the patient actively in his or her health care whenever possible. (P)<br />

6. Demonstrate teamwork and respect toward all members <strong>of</strong> the health care team, as<br />

manifested by reliability, responsibility, honesty, helpfulness, selflessness, and initiative<br />

in working with the team. (SBP, P)<br />

7. Attend to or advocate for the patient’s interests and needs in a manner appropriate to<br />

the student’s role. (P)<br />

8. Maintain confidentiality when dealing with protected health information and follow Health<br />

Information Portability and Accountability Act (HIPAA) guidelines. (P, SBP)<br />

D. REFERENCES:<br />

Clinician-Patient Communication to Enhance Health Outcomes. Institute for Health Care<br />

communication, Inc., West Haven, Connecticut, 1998.<br />

www.healthcarecomm.org<br />

Contemporary Issues in <strong>Medicine</strong>: Communication in <strong>Medicine</strong> Medical School Objectives<br />

Project, October 1999 American Association <strong>of</strong> Medical Colleges<br />

www.aamc.org/meded/msop/msop3.pdf<br />

Kurtz S, Silverman J, Draper J. Teaching and Learning Communication Skills in <strong>Medicine</strong>.<br />

Oxford: Radcliffe Medical Press Ltd; 1998.<br />

Kurtz S, Silverman J, Benson J, Draper J. Marrying content and process in clinical method<br />

teaching: enhancing the Calgary-Cambridge guides. Acad Med. 2003;78:802-9.<br />

Makoul G. The SEGUE Framework for teaching and assessing<br />

communication skills. Patient Educ Couns. 2001;45:23-34.<br />

Makoul G. Essential elements <strong>of</strong> communication in medical encounters: the Kalamazoo<br />

consensus statement. Acad Med. 2001;76:390-3.<br />

von Gunten CF, Ferris FD, Emanuel LL. Ensuring competency in end-<strong>of</strong>-life care:<br />

communication and relational skills. JAMA. 2000;284:3051-7.<br />

202


GENERAL CLINICAL CORE COMPETENCIES<br />

#5 INTERPRETATION OF CLINICAL INFORMATION<br />

RATIONALE:<br />

In the routine course <strong>of</strong> clinical practice, most physicians are required to order and interpret a wide<br />

variety <strong>of</strong> diagnostic tests and procedures. Determining how these test results will influence clinical<br />

decision making and communicating this information to patients in a timely and effective manner are<br />

core clinical skills that third-year medical students should possess.<br />

PREREQUISITES:<br />

Prior knowledge, skills and attitudes acquired during the pre-<strong>clerkship</strong> experience should include:<br />

Introductory course in clinical pathology and laboratory medicine.<br />

Introductory course in epidemiology and biostatistics.<br />

SPECIFIC LEARNING OBJECTIVES:<br />

A. KNOWLEDGE: Students should be able to:<br />

1. Interpret specific diagnostic tests and procedures that are ordered to evaluate patients who<br />

present with common symptoms and diagnoses encountered in the practice <strong>of</strong> internal<br />

medicine. (PC, MK)<br />

2. Take into account:<br />

Important differential diagnostic considerations, including potential diagnostic<br />

emergencies. (PC, MK)<br />

Pre-test and post-test likelihood <strong>of</strong> disease (probabilistic reasoning). (PC, MK)<br />

Performance characteristics <strong>of</strong> individual tests. (sensitivity, specificity, positive and<br />

negative predictive value, likelihood ratios). (PC, MK)<br />

3. Define and describe for the tests and procedures listed:<br />

Indications for testing. (PC, MK)<br />

Range <strong>of</strong> normal variation. (PC, MK)<br />

Critical values that require immediate attention. (PC, MK)<br />

Pathophysiologic implications <strong>of</strong> abnormal results. (PC, MK)<br />

Relative cost. (MK, SBP)<br />

4. Independently interpret the results <strong>of</strong> the following laboratory tests:<br />

CBC with diff and blood smear. (PC, MK)<br />

UA. (PC, MK)<br />

Electrolytes. (PC, MK)<br />

BUN/Cr. (PC, MK)<br />

GLC. (PC, MK)<br />

Hepatic function panel. (PC, MK)<br />

Hepatitis serologies. (PC, MK)<br />

Cardiac biomarkers (e.g. myoglobin, CK-MB, and Troponin I/T). (PC, MK)<br />

Routine coagulation tests (e.g. PT/PTT and INR). (PC, MK)<br />

Thyroid function tests (e.g. T3, T4, and TSH). (PC, MK)<br />

ABG.(PC, MK)<br />

Body fluid cell counts and chemistries. (PC, MK)<br />

5. Independently interpret the results <strong>of</strong> the following diagnostic procedures:<br />

203


12-lead ECG. (PC, MK)<br />

Chest radiograph. (PC, MK)<br />

Plain abdominal films (e.g. obstructive series, KUB). (PC, MK)<br />

Pulmonary function tests. (PC, MK)<br />

6. Describe the basic electrophysiologic events that produce the surface ECG. (MK)<br />

7. Describe how errors in test interpretation can affect clinical outcomes and costs. (PC, MK)<br />

8. Describe the concept <strong>of</strong> a threshold as it relates to testing and treatment decisions. (PC, MK)<br />

9. Describe the basic principles <strong>of</strong> using genetic information in clinical decision making. (PC, MK)<br />

B. SKILLS: Students should be able to demonstrate specific skills, including:<br />

1. Interpreting a blood smear, Gram stain, and UA. (PC)<br />

2. Approaching ECG interpretation in a systematic and logical fashion analyzing the following:<br />

rate, rhythm, P wave morphology, PR interval, QRS width, axis, voltage, QT interval, ST<br />

segment morphology, and T wave morphology. (PC)<br />

3. Recognizing the following on ECG:<br />

Sinus tachycardia, sinus bradycardia, sinus arrhythmia. (PC)<br />

Premature atrial beats, ectopic atrial rhythm/tachycardia, narrow complex supraventricular<br />

tachycardia. (PC)<br />

Multifocal atrial tachycardia, atrial flutter, atrial fibrillation (PC)<br />

First degree, second degree (Mobitz type I and II), and third degree (complete) heart<br />

block. (PC)<br />

Junctional rhythm. (PC)<br />

Premature ventricular beats. (PC)<br />

Typical ventricular tachycardia, ventricular fibrillation. (PC)<br />

Left and right atrial enlargement. (PC)<br />

Left ventricular hypertrophy. (PC)<br />

Left and right bundle branch block, left anterior and posterior fascicular block. (PC)<br />

The characteristic features <strong>of</strong> a properly functioning ventricular or dual chamber<br />

pacemaker. (PC)<br />

The delta wave in Wolf-Parkinson-White Syndrome. (PC)<br />

The classic features <strong>of</strong> myocardial ischemia and infarction and be able to localize the<br />

findings (i.e. inferior, anterior, lateral, posterior, right ventricular) and identify the probable<br />

culprit vessel. (PC)<br />

The classic features <strong>of</strong> pulmonary embolism. (PC)<br />

The characteristic effects <strong>of</strong> hypo- and hyperkalemia. (PC)<br />

4. Approaching chest radiography interpretation in a systematic and logical fashion analyzing the<br />

following: technique (e.g. view, rotation, exposure), visible abdomen, s<strong>of</strong>t tissues and bones <strong>of</strong><br />

the thorax, mediastinum/hila, and lungs. (PC)<br />

5. Recognizing the following on chest radiograph:<br />

Rib fracture. (PC)<br />

Cardiomegaly. (PC)<br />

Lobar pneumonia. (PC)<br />

Pleural effusion. (PC)<br />

Pneumothorax. (PC)<br />

Pulmonary nodule. (PC)<br />

204


Pulmonary edema/“congestive heart failure” (e.g. cardiomegaly, pulmonary vascular<br />

redistribution, Kerley’s B Lines, interstitial/alveolar edema). (PC)<br />

Hilar lymphadenopathy. (PC)<br />

Mediastinal widening. (PC)<br />

6. Recording the results <strong>of</strong> laboratory tests in an organized manner, using flow sheets when<br />

appropriate. (PC)<br />

7. Estimating the pre-test likelihood <strong>of</strong> a disease or condition. (PC, MK)<br />

8. Estimating the post-test probability <strong>of</strong> disease and stating the clinical significance <strong>of</strong> the results<br />

<strong>of</strong> laboratory tests and diagnostic procedures. (PC, MK)<br />

C. ATTITUDES AND PROFESSIONAL BEHAVIORS: Students should be able to:<br />

1. Demonstrate commitment to using risk-benefit, cost-benefit, and evidence-based<br />

considerations in the selection <strong>of</strong> diagnostic and therapeutic interventions for acute MI. (PLI, P,<br />

SBP)<br />

2. Regularly seek feedback regarding interpretation <strong>of</strong> clinical information and respond<br />

appropriately and productively. (P)<br />

3. Recognize the importance <strong>of</strong> patient preferences when selecting among diagnostic testing<br />

options. (P)<br />

4. Demonstrate ongoing commitment to self-directed learning regarding test interpretation. (PLI,<br />

P)<br />

5. Appreciate the implications <strong>of</strong> test results before ordering tests. (P)<br />

6. Appreciate the importance <strong>of</strong> follow-up on all diagnostic tests and procedures and timely<br />

communication <strong>of</strong> information to patients and appropriate team members. (P)<br />

7. Demonstrate a commitment to excellence by personally reviewing radiographs, ECGs, Gram<br />

stains, blood smears, etc. to assess the accuracy and significance <strong>of</strong> the results. (P)<br />

D. REFERENCES:<br />

Jaeschke R, Guyatt G, Sackett DL. Users’ guides to the medical literature.<br />

III. How to use an article about a diagnostic test. A. Are the results <strong>of</strong> the study valid? JAMA.<br />

1994; 271:389-91.<br />

Jaeschke R, Guyatt G, Sackett DL. Users’ guides to the medical literature.<br />

III. How to use an article about a diagnostic test. B. What are the results and will they help me<br />

in caring for my patients? JAMA. 1994; 271: 703-7.<br />

Dubin D. Rapid Interpretation <strong>of</strong> EKG's. 5th ed. Tampa, FL: Cover Publishing Company; 2000.<br />

Goodman LR. Felson’s Principles <strong>of</strong> Chest Roentgenology: A Programmed Text. 2nd ed.<br />

Philadelphia, PA: W. B. Saunders; 1999.<br />

Novelline RA. Squire's Fundamentals <strong>of</strong> Radiology. 6th ed. Cambridge, MA: <strong>University</strong> Press;<br />

2004.<br />

Mark, DB. Decision-making in clinical medicine. In Kasper DL, Braunwald EB, Fauci AS,<br />

Hauser SL, Longo DL, Jameson JL, eds. Harrison’s Principles <strong>of</strong> Internal <strong>Medicine</strong>. 16th ed.<br />

New York: McGraw-Hill; 2005:6-13.<br />

Lab Test Online<br />

www.labtestsonline.org/<br />

RadQuiz: Your Gateway to Radiology Resources<br />

www.radquiz.com<br />

Introduction to Chest Radiology<br />

205


<strong>Department</strong> <strong>of</strong> Radiology<br />

<strong>University</strong> <strong>of</strong> Virginia Health Sciences Center<br />

www.med-ed.virginia.edu/courses/rad/cxr/index.html<br />

Ferri FF. Laboratory values and interpretation <strong>of</strong> results. In Ferri FF, ed. Practical Guide to<br />

the Care <strong>of</strong> the Medical Patient. 6th ed. St. Louis, MO: Mosby; 2004:935-976.<br />

206


#6 THERAPEUTIC DECISION MAKING<br />

GENERAL CLINICAL CORE COMPETENCIES<br />

RATIONALE:<br />

Internists are responsible for directing and coordinating the therapeutic management <strong>of</strong> patients with<br />

a wide variety <strong>of</strong> problems, including critically ill patients with complex medical problems and the<br />

chronically ill. To manage patients effectively, physicians need basic therapeutic decision-making<br />

skills that incorporate both pathophysiologic reasoning and evidence-based knowledge.<br />

PREREQUISITES:<br />

Introductory coursework in clinical epidemiology and biostatistics.<br />

Introductory coursework in physiology and pathology.<br />

Introductory coursework in pharmacology.<br />

SPECIFIC LEARNING OBJECTIVES:<br />

A. KNOWLEDGE: Students should be able to define, describe, and discuss:<br />

1. Information resources for determining medical and surgical treatment options for patients<br />

with common and uncommon medical problems. (MK)<br />

2. Key factors to consider in choosing among treatment options, including risk, cost, evidence<br />

about efficacy, and consistency with pathophysiologic reasoning (MK)<br />

3. How to use critical pathways and clinical practice guidelines to help guide therapeutic<br />

decision making. (MK)<br />

4. Factors that frequently alter the effects <strong>of</strong> medications, including drug interactions and<br />

compliance problems. (MK)<br />

5. Factors to consider in selecting a medication from within a class <strong>of</strong> medications. (MK)<br />

6. Factors to consider in monitoring a patient’s response to treatment, including potential<br />

adverse effects. (MK)<br />

7. Various ways that evidence about clinical effectiveness is presented to clinicians and the<br />

potential biases <strong>of</strong> using absolute or relative risk or number <strong>of</strong> patients needed to treat. (MK)<br />

8. Methods <strong>of</strong> monitoring therapy and how to communicate them in both written and oral form.<br />

(MK)<br />

9. The basics <strong>of</strong> the potential role <strong>of</strong> genetic information in therapeutic decision making. (MK)<br />

B. SKILLS: Students should be able to demonstrate specific skills, including:<br />

1. Formulating an initial therapeutic plan. (PC)<br />

2. Changing the therapeutic plan when goals <strong>of</strong> care change (e.g. a shift toward palliative<br />

care). (PC)<br />

3. Accessing and utilizing, when appropriate, information resources to help develop an<br />

appropriate and timely therapeutic plan. (PC, PLI)<br />

4. Explaining the extent to which the therapeutic plan is based on pathophysiologic reasoning<br />

and scientific evidence <strong>of</strong> effectiveness. (PC)<br />

5. Beginning to estimate the probability that a therapeutic plan will produce the desired<br />

outcome. (PC)<br />

6. Writing prescriptions and inpatient orders safely and accurately. (PC)<br />

7. Counseling patients about how to take their medications and what to expect<br />

when doing so, including beneficial outcomes and potential adverse effects. (PC, CS)<br />

207


8. Monitoring response to therapy. (PC)<br />

9. Recognizing when to seek consultation for additional diagnostic and therapeutic<br />

recommendations. (PC, SBP)<br />

10. Recognizing when to screen for certain conditions based on age and risk factors and what<br />

to do with the results <strong>of</strong> the screening tests. (PC)<br />

C. ATTITUDES AND PROFESSIONAL BEHAVIORS: Students should be able to:<br />

1. Demonstrate commitment to using risk-benefit, cost-benefit, and evidence-based<br />

consideration in the selection <strong>of</strong> therapeutic interventions. (PLI, P)<br />

2. Demonstrate ongoing commitment to self-directed learning regarding therapeutic<br />

interventions. (PLI, P)<br />

3. Seek feedback regularly regarding therapeutic decision making and respond appropriately<br />

and productively. (P)<br />

4. Appreciate the impact therapeutic decisions have on a patient’s quality <strong>of</strong> life (P)<br />

5. Incorporate the patient in therapeutic decision making, explaining the risks and benefits <strong>of</strong><br />

treatment. (CS, P)<br />

6. Respect patients’ autonomy and informed choices, including the right to refuse treatment.<br />

(P)<br />

7. Demonstrate an understanding <strong>of</strong> the importance <strong>of</strong> close follow-up <strong>of</strong> patients under active<br />

care. (P)<br />

8. Recognize the importance <strong>of</strong> and demonstrate a commitment to the utilization <strong>of</strong> other<br />

health care pr<strong>of</strong>essionals in therapeutic decision making. (P, SBP)<br />

D. REFERENCES:<br />

Mark, DB. Decision-making in clinical medicine. In Kasper DL, Braunwald EB, Fauci AS,<br />

Hauser SL, Longo DL, Jameson JL, eds. Harrison’s Principles <strong>of</strong> Internal <strong>Medicine</strong>. 16th ed.<br />

New York: McGraw-Hill; 2005:6-13.<br />

Roden DM. Principles <strong>of</strong> clinical pharmacology. In Kasper DL, Braunwald EB, Fauci AS,<br />

Hauser SL, Longo DL, Jameson JL, eds. Harrison’s Principles <strong>of</strong> Internal <strong>Medicine</strong>. 16th ed.<br />

New York: McGraw-Hill; 2005:13-25.<br />

Users' guides to the medical literature. II. How to use an article about therapy or prevention. A.<br />

Are the results <strong>of</strong> the study valid? Evidence-Based <strong>Medicine</strong> Working Group. JAMA.<br />

1993;270:2598-601.<br />

Users' guides to the medical literature. II. How to use an article about therapy or prevention. B.<br />

What were the results and will they help me in caring for my patients? Evidence-Based<br />

<strong>Medicine</strong> Working Group. JAMA. 1994;271:59-63.<br />

#7 BIOETHICS OF CARE<br />

GENERAL CLINICAL CORE COMPETENCIES<br />

RATIONALE:<br />

A basic understanding <strong>of</strong> ethical principles and their application to patient care is essential for all<br />

physicians. During the internal medicine core <strong>clerkship</strong>, the student can put into practice some <strong>of</strong> the<br />

ethical principles learned in the preclinical years, especially by participating in discussions <strong>of</strong> informed<br />

208


consent and advance directives. Additionally, the student learns to recognize ethical dilemmas and<br />

respect different perceptions <strong>of</strong> health, illness, and health care held by patients <strong>of</strong> various religious<br />

and cultural backgrounds.<br />

PREREQUISITES:<br />

Introductory course on medical ethics providing a basic understanding <strong>of</strong> ethical principles and<br />

fiduciary relationships and their application in clinical medicine:<br />

Autonomy.<br />

Beneficence.<br />

Nonmaleficence.<br />

Truth-telling.<br />

Confidentiality.<br />

Respect for autonomy (informed choice).<br />

SPECIFIC LEARNING OBJECTIVES:<br />

A. KNOWLEDGE: Students should be able to define, describe, and discuss:<br />

1. Basic ethical principles (autonomy, beneficence, nonmaleficence, truth-telling, confidentiality,<br />

and autonomy). (MK)<br />

2. The patient’s right to refuse care. (MK)<br />

3. The unique nature <strong>of</strong> a fiduciary relationship. (MK)<br />

4. Basic elements <strong>of</strong> informed consent. (MK)<br />

5. Circumstances under which informed consent is necessary and unnecessary (MK)<br />

6. Basic concepts <strong>of</strong> treatment efficacy, quality <strong>of</strong> life, and societal demands. (MK)<br />

7. Potential conflicts between individual patient preferences and societal demands (MK)<br />

8. The role <strong>of</strong> the physician in making decisions about the use <strong>of</strong> expensive or controversial tests<br />

and treatments. (MK)<br />

9. Bioethical concerns regarding genetic information, privacy issues in particular. (MK)<br />

10. The unique bioethical concerns regarding end-<strong>of</strong>-life care. (MK)<br />

11. Circumstances when withholding or withdrawing care is acceptable. (MK)<br />

12. The role <strong>of</strong> federal and state legislation in governing health care. (MK)<br />

13. Circumstances when it may be unavoidable or acceptable to breach the basic ethical<br />

principles. (MK)<br />

B. SKILLS: Students should be able to demonstrate specific skills, including:<br />

1. Participating in a discussion about advance directives with a patient. (PC, CS)<br />

2. Participating in obtaining informed consent for a procedure. (PC, CS)<br />

3. Participating in explaining and obtaining informed consent for genetic testing (PC, CS)<br />

4. Participating in a preceptor’s discussion with a patient about a requested<br />

treatment that may not be considered appropriate (e.g., not cost-effective). (PC, CS)<br />

5. Participating in family and interdisciplinary team conferences discussing end<strong>of</strong>-life<br />

care and incorporating the patient’s wishes in that discussion. (PC, CS, SBP)<br />

6. Obtaining additional help from ethics experts in conflict resolution. (PC, SBP)<br />

C. ATTITUDES AND PROFESSIONAL BEHAVIORS: Students should be able to:<br />

209


1. Demonstrate ongoing commitment to self-directed learning regarding bioethics. (PLI)<br />

2. Recognize the importance <strong>of</strong> patient preferences, perspectives, and perceptions regarding<br />

health and illness. (P)<br />

3. Demonstrate a commitment to caring for all patients, regardless <strong>of</strong> the medical diagnosis,<br />

gender, race, socioeconomic status, intellect/level <strong>of</strong> education, religion, political affiliation,<br />

sexual orientation, ability to pay, or cultural background. (P)<br />

4. Recognize the importance <strong>of</strong> allowing terminally ill patients to die with comfort and dignity<br />

when that is consistent with the wishes <strong>of</strong> the patient and/or the patient’s family. (P)<br />

5. Recognize the potential conflicts between patient expectations and medically appropriate care.<br />

(P)<br />

6. Respond appropriately to patients who are nonadherent to treatment. (P)<br />

7. Demonstrate respect for the patient’s privacy and confidentiality when dealing with protected<br />

health information and follow HIPAA standards. (P)<br />

8. Appreciate the psychological impact genetic information may have on patients. (P)<br />

D. REFERENCES:<br />

Bioethics Resources on the Web<br />

Inter-Institute Bioethics Interest Group<br />

National Institutes <strong>of</strong> Health<br />

U.S. <strong>Department</strong> <strong>of</strong> Health and Human Services<br />

www.nih.gov/sigs/bioethics/<br />

<strong>University</strong> <strong>of</strong> Pennsylvania Center for Bioethics<br />

www.bioethics.upenn.edu<br />

Snyder L, Leffler C, Ethics and Human Rights Committee, American College <strong>of</strong> Physicians.<br />

Ethics manual: fifth edition. Ann Intern Med. 2005;142:560-82.<br />

World Medical Association Ethics Unit<br />

www.wma.net/e/ethicsunit/resources.htm<br />

Ethics in <strong>Medicine</strong> <strong>University</strong> <strong>of</strong> Washington School <strong>of</strong> <strong>Medicine</strong><br />

eduserv.hscer.washington.edu/bioethics<br />

Program in Ethics In Science and <strong>Medicine</strong> <strong>University</strong> <strong>of</strong> Texas Southwestern Medical Center<br />

www3.utsouthwestern.edu/ethics/<br />

Virtual Mentor American Medical Association<br />

www.ama-assn.org/ama/pub/category/3040.html<br />

Bioethics Interest Group American Medical Student Association<br />

www.amsa.org/bio/index.cfm<br />

210


#8 SELF-DIRECTED LEARNING<br />

GENERAL CLINICAL CORE COMPETENCIES<br />

RATIONALE:<br />

Because <strong>of</strong> the breadth <strong>of</strong> the problems encountered in clinical practice, internists face an<br />

extraordinary challenge to keep up with the burgeoning amount <strong>of</strong> new information relevant to<br />

providing high quality care. Therefore, they must master and practice self-directed life-long learning,<br />

including the ability to access and utilize information systems and resources efficiently.<br />

PREREQUISITES:<br />

Prior knowledge, skills, and attitudes acquired during the pre-clinical experience should include:<br />

Basic library skills, including the ability to perform an electronic literature search.<br />

Critical appraisal skills.<br />

Understanding <strong>of</strong> basic concepts <strong>of</strong> biostatistics and clinical epidemiology including: sensitivity,<br />

specificity, positive predictive value, negative predictive value, absolute risk reduction, relative<br />

risk reduction, number needed to treat, likelihood/odds ratios, and tests <strong>of</strong> significance.<br />

SPECIFIC LEARNING OBJECTIVES:<br />

A. KNOWLEDGE: Students should be able to define, describe, and discuss:<br />

1. Key sources for obtaining updated information on issues relevant to the medical management<br />

<strong>of</strong> adult patients. (MK, PLI)<br />

2. A system for managing information from a variety <strong>of</strong> sources. (MK, PLI)<br />

3. The concept <strong>of</strong> the focused clinical question. (MK, PLI)<br />

4. Key questions to ask when critically appraising articles on diagnostic tests:<br />

Was there an independent, blind comparison with a reference (“gold”) standard? (MK,<br />

PLI)<br />

Was the diagnostic test evaluated in an appropriate spectrum <strong>of</strong> patients (like those in<br />

whom it would be used in practice)? (MK, PLI)<br />

Was the reference standard applied regardless <strong>of</strong> the diagnostic test result? (MK, PLI)<br />

What were the results <strong>of</strong> the study (e.g. sensitivity, specificity, likelihood ratios, and/or<br />

pre- and post-test probabilities)? (MK, PLI)<br />

5. Key questions to ask when critically appraising articles on medical therapeutics:<br />

Was the assignment <strong>of</strong> patients to treatments randomized? (MK, PLI)<br />

Were all patients who entered the trial properly accounted for at the conclusion <strong>of</strong> the<br />

study and analyzed in the group they were randomized to? (MK, PLI)<br />

Were patients and study personnel blind to the treatment? (MK, PLI)<br />

Were the groups similar at the start <strong>of</strong> the trial? (MK, PLI)<br />

Aside from the experimental intervention, were the groups treated equally? (MK, PLI)<br />

What were the results <strong>of</strong> the trial (e.g. relative risk reduction, absolute risk reduction, and<br />

“number needed to treat”)? (MK, PLI)<br />

B. SKILLS: Students should be able to demonstrate specific skills, including:<br />

211


1. Performing a computerized literature search to find articles pertinent to a focused clinical<br />

question. (PLI)<br />

2. Demonstrating critical review skills. (PLI)<br />

3. Reading critically about issues pertinent to their patients. (PLI)<br />

4. Assessing the limits <strong>of</strong> medical knowledge in relation to patient problems (PLI)<br />

5. Using information from consultants critically. (PLI)<br />

6. Recognizing when additional information is needed to care for the patient (PLI)<br />

7. Asking colleagues (students, residents, nurses, faculty) for help when needed (PLI, SBP)<br />

8. Making use <strong>of</strong> available instruments to assess one’s own knowledge base (PLI, P)<br />

9. Summarizing and presenting to colleagues what was learned from consulting the medical<br />

literature. (PLI, CS)<br />

C. ATTITUDES AND PROFESSIONAL BEHAVIORS: Students should be able to:<br />

1. Demonstrate self-directed learning in every case. (PLI, P)<br />

2. Acknowledge gaps in knowledge to both colleagues and patients and request help. (PLI, P)<br />

3. Seek feedback regularly and respond appropriately and productively. (P)<br />

4. Recognize the value and limitations <strong>of</strong> other health care pr<strong>of</strong>essionals when confronted with a<br />

knowledge gap. (PLI, P, SBP)<br />

D. REFERENCES:<br />

Users' guides to the medical literature. I. How to get started. The Evidence-Based <strong>Medicine</strong><br />

Working Group. JAMA. 1993;270:2093-5.<br />

Users' guides to the medical literature. II. How to use an article about therapy or prevention. A.<br />

Are the results <strong>of</strong> the study valid? Evidence-Based <strong>Medicine</strong> Working Group. JAMA.<br />

1993;270:2598-601.<br />

Users' guides to the medical literature. II. How to use an article about therapy or prevention. B.<br />

What were the results and will they help me in caring for my patients? Evidence-Based<br />

<strong>Medicine</strong> Working Group. JAMA. 1994;271:59-63.<br />

Users' guides to the medical literature. III. How to use an article about a diagnostic test. A. Are<br />

the results <strong>of</strong> the study valid? Evidence-Based <strong>Medicine</strong> Working Group. JAMA.<br />

1994;271:389-91.<br />

Users' guides to the medical literature. III. How to use an article about a diagnostic test. B.<br />

What are the results and will they help me in caring for my patients? The Evidence-Based<br />

<strong>Medicine</strong> Working Group. JAMA. 1994;271:703-7.<br />

Strauss SE, Richardson WS, Glasziou P, Haynes RB. Evidence Based <strong>Medicine</strong>: How to<br />

Practice and Teach EBM. 3rd ed. New York, NY: Churchill Livingstone; 2005.<br />

Advancing Education in Practice-Based Learning and Improvement An Educational Resource<br />

from the ACGME Outcome Project<br />

www.acgme.org/outcome/implement/complete_PBLIBooklet.pdf<br />

#9 PREVENTION<br />

GENERAL CLINICAL CORE COMPETENCIES<br />

RATIONALE:<br />

One <strong>of</strong> the most important responsibilities <strong>of</strong> primary care physicians is to promote health and prevent<br />

disease in a cost-effective manner. Appropriate care by internists includes not only recognition and<br />

212


treatment <strong>of</strong> disease but also the routine incorporation <strong>of</strong> the principles <strong>of</strong> preventive health care into<br />

clinical practice. All physicians should be familiar with the principles <strong>of</strong> preventive health care to<br />

ensure their patients receive appropriate preventive services.<br />

PREREQUISITES:<br />

Prior knowledge, skills, and attitudes acquired during the pre-clinical experience should include:<br />

Introductory course in clinical epidemiology and biostatistics.<br />

Introductory course in health promotion and disease prevention.<br />

Ability to perform a complete medical history and physical exam.<br />

Ability to communicate with patients <strong>of</strong> diverse backgrounds.<br />

SPECIFIC LEARNING OBJECTIVES:<br />

A. KNOWLEDGE: Students should be able to define, describe, and discuss:<br />

1. Primary, secondary, and tertiary prevention. (MK)<br />

2. Criteria for determining whether or not a screening test should be incorporated into the periodic<br />

health assessment <strong>of</strong> adults. (MK)<br />

3. General types <strong>of</strong> preventive health care issues that should be addressed on a routine basis in<br />

adult patients (i.e., cancer screening; prevention <strong>of</strong> infectious diseases, coronary artery<br />

disease, osteoporosis, and injuries; and identification <strong>of</strong> substance abuse). (MK)<br />

4. Vaccines that have been recommended for routine use in at least some adults (i.e., influenza,<br />

pneumococcal, measles, mumps, rubella, tetanus-diphtheria, hepatitis). (MK)<br />

5. Indications for endocarditis prophylaxis. (MK)<br />

6. Methods for counseling patients about risk-factor modification, including the “stages <strong>of</strong> change”<br />

approach to helping patients change behavior. (MK)<br />

7. Influence <strong>of</strong> age and clinical status on approach to prevention. (MK)<br />

8. General categories <strong>of</strong> high-risk patients in whom routine preventative health care must be<br />

modified or enhanced (e.g., family history, travel to an underdeveloped area, occupational<br />

exposures, etc.). (MK)<br />

9. The major areas <strong>of</strong> controversy in screening. (MK)<br />

10. The potential roles and limitations <strong>of</strong> genetic testing in disease<br />

prevention/early detection. (MK)<br />

B. SKILLS: Students should be able to demonstrate specific skills, including:<br />

1. Obtaining a patient history, including a detailed family history, vaccination history, travel<br />

history, sexual history, and occupational exposures. (PC)<br />

2. Identifying patients at high risk for developing diabetes, dyslipidemia, coronary artery disease,<br />

cancer, osteoporosis, influenza, pneumonia, hepatitis, HIV infection, and tuberculosis by<br />

screening for major risk factors. (PC)<br />

3. Obtaining a Pap smear and interpreting its results. (PC)<br />

4. Performing a breast examination. (PC)<br />

5. Instructing patients to perform breast self-examination. (PC, CS)<br />

6. Interpreting the results <strong>of</strong> a mammogram. (PC)<br />

7. Performing a digital rectal examination. (PC)<br />

8. Interpreting the results <strong>of</strong> a PSA test and understand its limitations. (PC)<br />

9. Performing a testicular examination. (PC)<br />

10. Interpreting the results <strong>of</strong> a bone densitometry test. (PC)<br />

11. Interpreting the results <strong>of</strong> a fasting lipid pr<strong>of</strong>ile. (PC)<br />

213


12. Interpreting the results <strong>of</strong> a fasting glucose test. (PC)<br />

13. Counseling patients about safe-sex practices, smoking cessation, alcohol abuse, weight loss,<br />

healthy diet, exercise, and seat belt use. (PC, CS)<br />

14. place and interpret a PPD. (PC)<br />

15. Locating recently published recommendations as well as original data regarding measures that<br />

should be incorporated into the periodic health assessment <strong>of</strong> adults. (PLI)<br />

C. ATTITUDES AND PROFESSIONAL BEHAVIORS: Students should be able to:<br />

1. Address preventive health care issues as a routine part <strong>of</strong> their assessment <strong>of</strong> patients. (P)<br />

2. Encourage patients to share responsibility for health promotion and disease prevention. (P)<br />

3. Recognize the importance <strong>of</strong> patient preferences when recommending preventive health<br />

measures. (P)<br />

4. Understand the patient’s right to refuse preventive health measures. (P)<br />

5. Demonstrate commitment to using risk-benefit, cost-benefit, and evidence-based<br />

considerations in the selection preventive health measures. (PLI, P)<br />

6. Demonstrate ongoing commitment to self-directed learning regarding preventive health<br />

measures. (PLI, P)<br />

D. REFERENCES:<br />

Pomrehn PR, Davis MV, Chen DW, Barker W. Prevention for the 21st century: setting the<br />

context through undergraduate medical education. Acad Med. 2000;75(7 Suppl):S5-13.<br />

Guide to Clinical Preventive Services<br />

U.S. Preventative Services Task Force (USPSTF)<br />

Agency for Healthcare Research and Quality<br />

U.S. <strong>Department</strong> <strong>of</strong> Health and Human Services<br />

USPSTF Recommendation: Screening for Cancer<br />

www.ahrq.gov/clinic/cps3dix.htm#cancer<br />

USPSTF Recommendation: Screening for Lipid Disorders<br />

www.ahrq.gov/clinic/ajpmsuppl/lipidrr.htm<br />

USPSTF Recommendation: Screening for High Blood Pressure<br />

www.ahrq/clinic/3rduspstf/highbloodsc/hibloodrr.htm<br />

USPSTF Recommendations Statement: Counseling to prevent tobacco use and tobaccocaused<br />

disease<br />

www.ahrq.gov/clinic/3rduspstf/tobaccoun/tobcounrs.htm<br />

Screening for Prostate Cancer. American College <strong>of</strong> Physicians. Ann Int Med 1997; 126: 480-<br />

484.<br />

Summary <strong>of</strong> Recommendations for Adult Immunization Immunization Action Coalition Bulletin<br />

Adapted from the recommendations <strong>of</strong> the Advisory Commttee on Immunization Practices<br />

(ACIP), August 2005<br />

www.immunize.org/acip<br />

Martin GJ. Screening and prevention <strong>of</strong> disease. In Kasper DL, Braunwald EB, Fauci AS,<br />

Hauser SL, Longo DL, and Jameson JL eds. Harrison’s Principles <strong>of</strong> Internal <strong>Medicine</strong>. 16th<br />

ed. New York, NY: McGraw-Hill; 2005:26-28.<br />

214


#10 COORDINATION OF CARE<br />

GENERAL CLINICAL CORE COMPETENCIES<br />

RATIONALE:<br />

The task <strong>of</strong> coordinating a patient’s care is central to the role <strong>of</strong> the internist, and involves<br />

communication with the patient and his or her family, colleagues, consultants, nurses, social workers,<br />

and community-based agencies. It is essential for the student to learn that the physician’s<br />

responsibility toward the patient does not stop at the end <strong>of</strong> the <strong>of</strong>fice visit or hospitalization but<br />

continues in collaboration with other pr<strong>of</strong>essionals to ensure that the patient receives optimal care.<br />

PREREQUISITES:<br />

Prior knowledge, skills, and attitudes acquired during the pre-clinical experience should<br />

include:<br />

Ability to perform patient-centered interviewing to determine the patients’ needs and<br />

communicate about diagnostic and therapeutic plans, transitions <strong>of</strong> care, and end-<strong>of</strong>-life care.<br />

Ability to identify community resources for care and strategies for coordination <strong>of</strong> care.<br />

Health Information Portability and Accountability Act (HIPAA) training to promote patient<br />

privacy.<br />

Required introductory courses in interviewing/physical examination with emphasis on doctorpatient<br />

communication and health care delivery.<br />

SPECIFIC LEARNING OBJECTIVES:<br />

A. KNOWLEDGE: Students should be able to define, describe and discuss:<br />

1. The role <strong>of</strong> consultants and their limits in the care <strong>of</strong> a patient. (MK, SBP)<br />

2. Key personnel and programs in and out <strong>of</strong> the hospital that may be able to contribute to the<br />

ongoing care <strong>of</strong> an individual patient for whom the student has responsibility (e.g. home health<br />

providers, social workers, case coordinators/managers, community health organizations, etc.).<br />

(MK, SBP)<br />

3. The role <strong>of</strong> the primary care physician in coordinating the comprehensive and longitudinal<br />

patient care plan, including communicating with the patient and family (directly, telephone, or<br />

email) and evaluating patient well-being through home health and other care providers. (MK,<br />

SBP)<br />

4. HIPAA guidelines to promote patient privacy. (MK, SBP)<br />

5. The role <strong>of</strong> the primary care physician in the coordination <strong>of</strong> care during key transitions<br />

(e.g. outpatient to inpatient, inpatient to skilled nursing facility, inpatient to hospice, etc.). (MK,<br />

SBP)<br />

6. The role <strong>of</strong> clinical nurse specialists, nurse practitioners, physicians assistants, and other<br />

allied health pr<strong>of</strong>essionals in co-managing patients in the outpatient and inpatient setting. (MK,<br />

SBP)<br />

7. The importance <strong>of</strong> reconciliation <strong>of</strong> medications at all transition points <strong>of</strong> patient care. (MK,<br />

SBP)<br />

8. The rationale for a standardized approach to all “hand <strong>of</strong>f” communications (MK, SBP)<br />

B. SKILLS: Students should be able to demonstrate specific skills, including:<br />

1. Discussing with the patient and their family ongoing health care needs; using appropriate<br />

language, avoiding jargon, and medical terminology. (PC, CS)<br />

215


2. Participating in requesting a consultation and identifying the specific question to be<br />

addressed. (PC, CS, .SBP)<br />

3. Participating in the discussion <strong>of</strong> the consultant’s recommendations. (PC, CS, SBP)<br />

4. Participating in developing a coordinated, ongoing care plan in the community. (PC, SBP)<br />

5. Obtaining a social history that identifies potential limitations in the home setting which may<br />

require an alteration in the medical care plan to protect the patient’s welfare. (PC, CS)<br />

6. Reconciling patient medications at key transition points in care. (PC, SBP)<br />

7. Conveying accurately vital patient information at all care “hand-<strong>of</strong>f” points (PC, CS, SBP)<br />

C. ATTITUDES AND PROFESSIONAL BEHAVIORS: Students should be able to:<br />

1. Demonstrate teamwork and respect toward all members <strong>of</strong> the health care team. (P, SBP)<br />

2. Demonstrate responsibility for patients’ overall welfare. (P)<br />

3. Participate, whenever possible, in coordination <strong>of</strong> care and in the provision <strong>of</strong> continuity. (P,<br />

SBP)<br />

D. REFERENCES:<br />

Goldman L, Lee, T, Rudd P. Ten commandments <strong>of</strong> effective consultation. Arch Intern Med.<br />

1983;143:1753-5.<br />

Stille CJ, Jerant A, Bell D, Meltzer D, Elmore JG. Coordinating care across diseases, settings,<br />

and clinicians: a key role for the generalist in practice. Ann Intern Med. 2005;142:700-708.<br />

Haggerty JL, Reid RJ, Freeman GK, Starfield BH, Adair CE, McKendry R. Continuity <strong>of</strong> care:<br />

a multidisciplinary review. BMJ. 2003;327:1219-21.<br />

Wenger NS, Young R. Quality indicators <strong>of</strong> continuity and coordination <strong>of</strong> care for vulnerable<br />

elder persons. Rand Corporation, 2004.<br />

www.rand.org/pubs/working_papers/2004/RAND_WR176.pdf<br />

Building a Case for Medication Reconciliation Institute for Safe Medication Practices<br />

www.ismp.org/Newsletters/acutecare/articles/20050421.asp<br />

Reconcile Medications at All Transition Points Institute for Healthcare Improvement<br />

www.ihi.org/IHI/Topics/PatientSafety/MedicationSystems/Changes/Reco<br />

ncile+Medications+at+All+Transition+Points.htm<br />

Healthcare Communications Toolkit to Improve Transitions <strong>of</strong> Care <strong>Department</strong> <strong>of</strong> Defense<br />

Patient Safety Program<br />

https://patientsafety.satx.disa.mil/ ContentStore/2005_12-<br />

8%20Hand<strong>of</strong>f%20Toolkit%20FINAL.htm<br />

216


GENERAL CLINICAL CORE COMPETENCIES<br />

#11 GERIATRIC CARE<br />

RATIONALE:<br />

Geriatric patients <strong>of</strong>ten have multiple, chronic illnesses which may present with atypical symptoms.<br />

Management strategies need to take into account the effects <strong>of</strong> aging on multiple organ systems and<br />

socioeconomic factors faced by our elderly society. As the number <strong>of</strong> geriatrics patients steadily<br />

rises, the internist will devote more time to the care <strong>of</strong> these patients.<br />

PREREQUISITES:<br />

Required courses in anatomy, physiology, pathophysiology, physical examination, and nutrition<br />

with attention to specific considerations in the elderly.<br />

Ability to perform a complete medical history and physical.<br />

Ability to communicate with patients <strong>of</strong> diverse backgrounds.<br />

SPECIFIC LEARNING OBJECTIVES:<br />

A. KNOWLEDGE: Students should be able to define, describe, and discuss:<br />

1. Functional implications <strong>of</strong> aging on each major organ system. (MK)<br />

2. Nutritional needs <strong>of</strong> the elderly and adaptations needed in the presence <strong>of</strong> chronic illness.<br />

(MK)<br />

3. Key illnesses in the elderly, focusing on their <strong>of</strong>ten atypical presentation, including:<br />

Cardiovascular and cerebrovascular disease. (MK)<br />

Diabetes. (MK)<br />

Urinary tract infection. (MK)<br />

Pneumonia. (MK)<br />

Substance abuse. (MK)<br />

Depression. (MK)<br />

Thyroid disease. (MK)<br />

Fluid and electrolyte disturbances. (MK)<br />

Arthritis. (MK)<br />

Constipation. (MK)<br />

Acute abdomen. (MK)<br />

Depression. (MK)<br />

4. The common “geriatric syndromes” (i.e. symptoms and conditions common in the elderly and<br />

<strong>of</strong>ten multifactorial in origin), including:<br />

Immobility. (MK)<br />

Falls/gait and balance problems. ( MK)<br />

Dizziness. (MK)<br />

Incontinence. (MK)<br />

Weight loss/failure to thrive/malnutrition. (MK)<br />

Sleep disturbance. (MK)<br />

Dementia/delirium. (MK)<br />

Osteoporosis. (MK)<br />

Hearing and visual impairment. (MK)<br />

Pressure ulcers. (MK)<br />

217


5. Basic treatment plans for illness in the elderly, with an awareness <strong>of</strong> the pharmacokinetic and<br />

pharmacodynamic changes seen as we age. (MK)<br />

6. Principles <strong>of</strong> screening in the elderly, including immunizations, cardiovascular risk, cancer,<br />

substance abuse, mental illness, osteoporosis, and functional assessment. (MK)<br />

7. Factors that contribute to polypharmacy in the elderly. (MK)<br />

8. Principles <strong>of</strong> Medicare (including who and what services are covered) and prescription drug<br />

coverage (who and what drugs are covered). (MK, SBP)<br />

B. SKILLS: Students should be able to demonstrate specific skills, including:<br />

1. Taking a complete and focused history from a geriatric patient with attention to current<br />

symptoms, chronic illnesses, and physical and mental functioning (PC, CS)<br />

2. Always obtaining historical information from collateral source, whenever possible. (PC, CS)<br />

3. Performing a physical examination and functional assessment on an elderly patient, adapting it<br />

to a patient's symptoms, chronic illness, and possible conditions <strong>of</strong> frailty, immobility, hearing<br />

loss, memory loss, and other impairments. (PC)<br />

4. Performing a mental status examination to evaluate confusion and/or memory loss in an<br />

elderly patient. (PC)<br />

5. Identifying patients at high risk for falling. (PC)<br />

6. Developing a diagnostic and management plan for patients with the with symptoms/conditions<br />

common in the geriatric population. (PC, MK)<br />

7. Communicating the diagnosis, treatment plan, and subsequent follow-up to the patient and<br />

their family. (PC, CS)<br />

8. Eliciting input and questions from the patient and their family about the diagnostic and<br />

management plan. (PC, CS)<br />

9. With guidance and direct supervision, participating in discussing basic issues regarding<br />

advance directives with patients and their families. (CS)<br />

10. With guidance and direct supervision participating in discussing basic end-<strong>of</strong>-life issues with<br />

patients and their families. (CS)<br />

11. Actively attempting to limit polypharmacy whenever possible. (PC)<br />

12. Participating in an interdisciplinary approach to management and rehabilitation <strong>of</strong> elderly<br />

patients. (PC, SBP)<br />

13. Determine when to obtain consultation from a geriatric specialist. (PC, SBP)<br />

14. Accessing and using appropriate information systems and resources to help delineate issues<br />

related to the common geriatric syndromes. (PC, PLI)<br />

15. Incorporating patient needs and preferences. (PC, P)<br />

C. ATTITUDES AND PROFESSIONAL BEHAVIORS: Students should be able to:<br />

1. Respect the increased risk for iatrogenic complications among elderly patients by always<br />

taking into account risks and monitoring closely for complications (P)<br />

2. Demonstrate respect to older patients, particularly those with disabilities, by making efforts to<br />

preserve their dignity and modesty. (P)<br />

3. Always treat cognitively impaired patients and patients at the end <strong>of</strong> their lives with utmost<br />

respect and dignity. (P)<br />

4. Demonstrate commitment to using risk-benefit, cost-benefit, and evidence-based<br />

considerations in the selection <strong>of</strong> diagnostic and therapeutic interventions for the common<br />

geriatric syndromes. (PLI, P)<br />

5. Recognize the importance <strong>of</strong> patient needs and preferences when selecting among diagnostic<br />

and therapeutic options for the common geriatric syndromes. (P)<br />

218


6. Demonstrate ongoing commitment to self-directed learning regarding care <strong>of</strong> the geriatric<br />

patient. (P, PLI)<br />

7. Appreciate the impact the common geriatric syndromes have on a patient’s quality <strong>of</strong> life, wellbeing,<br />

and the family. (P)<br />

8. Recognize the importance <strong>of</strong> and demonstrate a commitment to the utilization <strong>of</strong> other health<br />

care pr<strong>of</strong>essionals in the diagnosis and treatment <strong>of</strong> geriatric patients. (P, SBP)<br />

D. REFERENCES:<br />

The American Geriatrics Society<br />

www.americangeriatrics.org/<br />

Guidelines and Position Statements<br />

www.americangeriatrics.org/products/positionpapers/<br />

Portal <strong>of</strong> Geriatric Online Education (POGOe)<br />

In association with AAMC MedEdPORTAL<br />

www.pogoe.org<br />

Tinetti ME. Clinical practice. Preventing falls in elderly persons. N Engl J Med.<br />

2003;348(1):42-9.<br />

Fuller GF. Falls in the elderly. Am Fam Physician. 2000;61:2159-68, 2173-4<br />

Miller KE, Zylstra RG, Standridge JB. The geriatric patient: a systematic approach to<br />

maintaining health. Am Fam Physician. 2000;61:1089-104.<br />

Willlams CM. Using medications appropriately in older adults. Am Fam Physician.<br />

2002;66:1917-24.<br />

Huffman GB. Evaluating and treating unintentional weight loss in the elderly. Am Fam<br />

Physician. 2002;65:640-50.<br />

Sloane PD, Coeytaux RR, Beck RS, Dallara J. Dizziness: state <strong>of</strong> the science. Ann Intern<br />

Med. 2001;134:823-32.<br />

Cummings JL. Alzheimer's disease. N Engl J Med. 2004;351:56-67.<br />

Lembo A, Camilleri M. Chronic constipation. N Engl J Med. 2003;349:1360-8.<br />

Resnick NM, Dosa D. Geriatric medicine. In Kasper DL, Braunwald EB, Fauci AS, Hauser SL,<br />

Longo DL, Jameson JL eds. Harrison’s Principles <strong>of</strong> Internal <strong>Medicine</strong>. 16th ed. New York,<br />

NY: McGraw-Hill; 2005:43-53.<br />

219


#12 BASIC PROCEDURES<br />

GENERAL CLINICAL CORE COMPETENCIES<br />

RATIONALE:<br />

For many students, the internal medicine <strong>clerkship</strong> is where the basic procedural skills required in<br />

other <strong>clerkship</strong>s, subinternships, and residencies are learned.<br />

PREREQUISITES:<br />

Prior knowledge, skills, and attitudes acquired during the pre-<strong>clerkship</strong> experience should include:<br />

Pertinent anatomic considerations, including venous anatomy <strong>of</strong> the extremities (for<br />

venipuncture and IV placement), arterial anatomy <strong>of</strong> the wrist and groin (for blood gases),<br />

vaginal/vulvar anatomy (for urethral catheterization in women as well as pap smear) and<br />

prostate anatomy in men (for prostate exam), rectal anatomy (for digital rectal exam) and<br />

surface anatomy and electrical vector orientation <strong>of</strong> the heart (for EKG placement).<br />

The fundamental tenants <strong>of</strong> informed consent.<br />

Basic training in body substance isolation procedures and sterile technique.<br />

SPECIFIC LEARNING OBJECTIVES:<br />

A. KNOWLEDGE: Students should be able to define, describe, and discuss:<br />

1. Key indications, contraindications, risks to patients and health care providers, benefits, and<br />

techniques for each <strong>of</strong> the following basic procedures:<br />

Venipuncture. (MK)<br />

Blood culture. (MK)<br />

ABG. (MK)<br />

ECG. (MK)<br />

Chest radiography. (MK)<br />

Nasogastric tube placement. (MK)<br />

Urethral catheterization. (MK)<br />

Peripheral intravenous catheter insertion. (MK)<br />

Throat culture. (MK)<br />

PAP smear. (MK)<br />

Digital rectal examination. (MK)<br />

Urine dipstick. (MK)<br />

Stool occult blood testing. (MK)<br />

Subcutaneous injection. (MK)<br />

Intramuscular injection. (MK)<br />

Wound culture. (MK)<br />

Dressing change. (MK)<br />

PPD placement. (MK)<br />

2. Alternatives to a given procedure. (MK)<br />

3. The patient’s experience <strong>of</strong> the procedure. (MK)<br />

B. SKILLS: Students should be able to demonstrate specific skills, including:<br />

220


1. Obtaining informed consent, when necessary, for basic procedures, including the explanation<br />

<strong>of</strong> the purpose, possible complications, alternative approaches, and conditions necessary to<br />

make the procedure as comfortable, safe, and interpretable as possible. (PC, CS)<br />

2. Explaining what the patient’s experience is likely to be in understandable terms. (CS)<br />

3. Demonstrating step-by-step performance <strong>of</strong> basic procedures with technical pr<strong>of</strong>iciency. (PC)<br />

4. Demonstrating proper sterile technique and body substance isolation procedures. (PC)<br />

5. Appropriately documenting, when required, how the procedure was done, any complications,<br />

and results. (CS)<br />

C. ATTITUDES AND PROFESSIONAL BEHAVIORS: Students should be able to:<br />

1. Appreciate the fear and anxiety many patients have regarding even simple procedures. (P)<br />

2. Make efforts to maximize patient comfort during a procedure. (P)<br />

3. Appreciate the patient’s right to refuse procedures. (P)<br />

4. Regularly seek feedback regarding procedural skills and respond appropriately and<br />

productively. (P)<br />

D. REFERENCES:<br />

Guide to procedures. In Lin GA, Lin TL, Sakurai KA, De Fer TM, eds. The Washington Manual<br />

Internship Survival Guide. 2nd ed. Philadelphia, PA: Lippincott Williams and Wilkins; 2005:178-<br />

210.<br />

Chen H, Sonneday CJ, Lillemoe KD eds. Manual <strong>of</strong> Common Bedside Surgical Procedures.<br />

2nd ed. Philadelphia, PA: Lippincott Williams and Wilkins; 2000.<br />

Ferri FF. Procedures and interpretation <strong>of</strong> results. In Ferri FF, ed. Practical Guide to the Care<br />

<strong>of</strong> the Medical Patient. 6th ed. St. Louis, MO: Mosby; 2004:903-934.<br />

221


GENERAL CLINICAL CORE COMPETENCIES<br />

#13 NUTRITION<br />

RATIONALE:<br />

Despite the importance <strong>of</strong> nutritional factors in health and illness, physicians frequently have been<br />

criticized for giving these factors inadequate attention. Internists, by virtue <strong>of</strong> their dedication to<br />

providing comprehensive care to their patients, must assess nutritional factors on a routine basis.<br />

Medical students should be prepared to provide patients with basic advice regarding ways to optimize<br />

their nutritional status. Students also need to have at least a basic working knowledge <strong>of</strong> the<br />

principles <strong>of</strong> nutritional assessment and intervention.<br />

PREREQUISITES:<br />

Prior knowledge, skills, and attitudes acquired during the pre-<strong>clerkship</strong> experience should include:<br />

Ability to perform a complete medical history.<br />

Ability to communicate with patients <strong>of</strong> diverse backgrounds.<br />

Knowledge <strong>of</strong> body metabolism, the respective roles <strong>of</strong> dietary fats, carbohydrates, and<br />

protein, and the need for vitamins and minerals for maintenance <strong>of</strong> health.<br />

SPECIFIC LEARNING OBJECTIVES:<br />

A. KNOWLEDGE: Each student should be able to define, describe, and discuss:<br />

1. The relationship between diet and disease. (MK)<br />

2. Common medical problems that can cause nutritional deficiencies. (MK)<br />

3. Contributions <strong>of</strong> nutrition to medical problems such as obesity,<br />

hyperlipidemia, diabetes, and hypertension. (MK)<br />

4. How to perform a nutritional assessment and assist the patient in setting goals for dietary<br />

improvement. (MK)<br />

5. Daily caloric, fat, carbohydrate, protein, mineral, and vitamin requirements; adequacy <strong>of</strong> diets<br />

in providing such requirements; evidence <strong>of</strong> need for and potential risks <strong>of</strong> supplements (e.g.<br />

calcium, antioxidants). (MK)<br />

6. Common dietary supplements and their known adverse and beneficial effects on health. (MK)<br />

7. The consequences <strong>of</strong> poor nutrition on a critically ill patient, such as poor wound healing,<br />

increased risk <strong>of</strong> infection, and increased mortality. (MK)<br />

8. Nutritional needs <strong>of</strong> the elderly and adaptations needed in the presence <strong>of</strong> chronic illness.<br />

(MK)<br />

9. The indications for enteral and parenteral nutrition. (MK)<br />

B. SKILLS: Student should be able to demonstrate specific skills, including:<br />

1. Obtaining a nutritional history for all patients, with additional focus on those with chronic<br />

disease (obesity, hyperlipidemia, diabetes mellitus, hypertension, alcoholism, cancer, COPD,<br />

CHF, renal, and GI disease), giving attention to weight change, appetite, eating habits,<br />

digestive problems, dental problems, physical handicaps, psychiatric problems, socioeconomic<br />

factors, alcohol use, medications, and physical activity. (PC, CS)<br />

2. Identifying physical exam abnormalities that may suggest malnutrition, such as muscle<br />

wasting, decreased adipose stores, as well as stigmata <strong>of</strong> vitamin/mineral or protein-calorie<br />

222


malnutrition (e.g. alopecia, ecchymoses, angular chelosis, glossitis, peripheral neuropathy,<br />

edema, etc.). (PC)<br />

3. Calculating a patient’s body mass index (BMI) and measuring waist circumference. (PC)<br />

4. Ordering appropriate tests for evaluating a patient’s nutritional status, including albumin,<br />

prealbumin, serum chemistries and coagulation pr<strong>of</strong>ile(PC)<br />

5. Performing basic nutritional counseling with patients with obesity, diabetes mellitus,<br />

hyperlipidemia, hypertension, heart failure, and coronary artery disease. (PC, CS)<br />

6. Identifying barriers that prevent a patient from successfully adhering to a recommended diet.<br />

(PC, CS)<br />

7. Determining when to obtain consultation from a dietician. (PC, SBP)<br />

8. Incorporating patient needs and preferences. (PC, P)<br />

C. ATTITUDES AND PROFESSIONAL BEHAVIORS: Students should be able to:<br />

1. Demonstrate commitment to using risk-benefit, cost-benefit, and evidence-based<br />

considerations in the selection <strong>of</strong> diagnostic and therapeutic interventions for malnutrition. (PLI,<br />

P)<br />

2. Recognize the importance <strong>of</strong> patient preferences and cultural factors when selecting nutritional<br />

counseling. (PLI, P)<br />

3. Respond to patients who are non-adherent to recommendations for appropriate nutritional<br />

intake. (CS, P)<br />

4. Demonstrate ongoing commitment to self-directed learning regarding nutrition. (PLI, P)<br />

5. Appreciate the impact malnutrition has on a patient’s quality <strong>of</strong> life, wellbeing, ability to work,<br />

and the family. (P)<br />

6. Recognize the importance <strong>of</strong> involving other healthcare pr<strong>of</strong>essionals when appropriate. (P,<br />

SBP)<br />

D. REFERENCES:<br />

Division <strong>of</strong> Nutrition and Physical Activity National Center for Chronic Disease Prevention and<br />

Health Promotion<br />

Centers for Disease Control and Prevention<br />

U.S. <strong>Department</strong> <strong>of</strong> Health and Human Serviced<br />

www.cdc.gov/nccdphp/dnpa/<br />

Dietary Guidelines for Americans<br />

U.S. <strong>Department</strong> <strong>of</strong> Agriculture<br />

U.S. <strong>Department</strong> <strong>of</strong> Health and Human Services<br />

www.health.gov/dietaryguidelines/<br />

American Dietetic Association<br />

www.eatrigh.org<br />

Food and Nutrition Information Center<br />

U.S. <strong>Department</strong> <strong>of</strong> Agriculture<br />

www.nal.usda.gov/fnic/<br />

Dwyer J. Nutritional requirements and dietary assessment. In Kasper DL, Braunwald EB,<br />

Fauci AS, Hauser SL, Longo DL, Jameson JL eds. Harrison’s Principles <strong>of</strong> Internal <strong>Medicine</strong>.<br />

16th ed. New York, NY: McGraw-Hill; 2005:399-403.<br />

Halsted CH. Malnutrition and nutritional assessment. In Kasper DL, Braunwald EB, Fauci AS,<br />

Hauser SL, Longo DL, Jameson JL eds. Harrison’s Principles <strong>of</strong> Internal <strong>Medicine</strong>. 16th ed.<br />

New York, NY: McGraw-Hill; 2005:411-415.<br />

Howard L. Enteral and parenteral nutrition therapy. In Kasper DL, Braunwald EB, Fauci AS,<br />

223


Hauser SL, Longo DL, Jameson JL eds. Harrison’s Principles <strong>of</strong> Internal <strong>Medicine</strong>. 16th ed.<br />

New York, NY: McGraw-Hill; 2005:415-422.<br />

224


#14 COMMUNITY HEALTH CARE<br />

GENERAL CLINICAL CORE COMPETENCIES<br />

RATIONALE:<br />

The increasing number <strong>of</strong> physicians practicing under managed care and in community-oriented<br />

primary care practices necessitates expanding medical education to prepare graduates for<br />

population-based clinical practice. In a managed care setting, population-based clinical practice<br />

includes the health <strong>of</strong> an enrolled population. In a community-based setting, population-based clinical<br />

practice includes the health <strong>of</strong> a population in addition to the health <strong>of</strong> the individual patient through<br />

concern with resource allocation, epidemiology, and the care <strong>of</strong> patients whose needs are not<br />

currently met by the health care system.<br />

PREREQUISITES:<br />

Prior knowledge, skills, and attitudes acquired during the pre-<strong>clerkship</strong> experience should<br />

include:<br />

Required introductory coursework in health care delivery (with an emphasis on medical<br />

sociology and health care delivery to at risk populations).<br />

Required introductory course in clinical epidemiology and biostatistics.<br />

Required introductory coursework in population health (with an emphasis on differences<br />

between individuals and populations).<br />

SPECIFIC LEARNING OBJECTIVES:<br />

A. KNOWLEDGE: Students should be able to define, describe, and discuss:<br />

1. The concepts <strong>of</strong> rate, incidence, and prevalence to characterize the health <strong>of</strong> a population.<br />

(MK)<br />

2. How to gather health information about a population. (MK)<br />

3. How disease epidemiology in a community differs from that experienced in an <strong>of</strong>fice or hospital<br />

practice. (MK)<br />

4. How health care financing and health care delivery systems affect individual physicians,<br />

patients, and communities. (MK, SBP)<br />

5. How community and individual responses to health problems may be affected by both<br />

individual and community socio-cultural characteristics. (MK)<br />

6. Local government, social service, or community organizations that provide links between the<br />

underserved members <strong>of</strong> the community and the medical care systems. (MK, SBP)<br />

7. barriers faced by his or her patients in the community setting. (MK)<br />

B. SKILLS: Students should be able to demonstrate specific skills, including:<br />

1. Defining and describing a population, its demography, culture, socioeconomic makeup, and<br />

health status. (PC)<br />

2. Identifying the unique characteristics <strong>of</strong> a population that affect the health <strong>of</strong> the population and<br />

individuals within that population. (PC)<br />

3. Considering how the socio-cultural characteristics <strong>of</strong> a particular community may affect that<br />

population’s attitudes toward health care. (PC)<br />

4. Using, in daily patient care, an understanding <strong>of</strong> the community and sociocultural context that<br />

may affect an individual patient’s health care decisions and health-related behaviors. (PC)<br />

225


5. Identifying patients whose illnesses may put the community at risk. (PC, MK)<br />

6. Incorporating a population-based perspective in analyzing clinical problems (PC)<br />

7. Reading critically clinical studies and applying findings to health care decisions involving real<br />

patients and populations <strong>of</strong> patients. (PC, MK, PLI)<br />

8. Incorporating principles <strong>of</strong> disease prevention and behavioral change appropriate for specific<br />

populations <strong>of</strong> patients within a community. (PC, MK)<br />

9. Attempting to develop solutions for barriers to health care delivery (e.g. sociocultural, financial,<br />

and system-based) that affect individual patients. (PC, SBP)<br />

10. Functioning effectively as a member <strong>of</strong> a health care team. (PC, P, SBP)<br />

11. Using, when appropriate, local government, social service, and community organizations to<br />

improve the health <strong>of</strong> individuals and populations. (PC, SBP)<br />

12. Accessing and utilizing appropriate information systems and resources to help delineate issues<br />

related to population health. (PC, PLI)<br />

C. ATTITUDES AND PROFESSIONAL BEHAVIORS: Students should be able to:<br />

1. Demonstrate respect for cultural and socioeconomic diversity. (P)<br />

2. Show willingness to accept at least partial responsibility for the health <strong>of</strong> populations. (P)<br />

3. Respond non-judgmentally to an individual whose socio-cultural and community-based<br />

background result in seemingly counterproductive heath care decisions and health-related<br />

behaviors. (P)<br />

4. Value the unique contributions <strong>of</strong> all members <strong>of</strong> the health care team. (P)<br />

5. Demonstrate ongoing commitment to self-directed learning regarding population/community<br />

health issues. (PLI, P)<br />

D. REFERENCES:<br />

Contemporary Issues in Medical Education: Quality <strong>of</strong> Care<br />

Medical Informatics and Population Health, June, 1998<br />

American Association <strong>of</strong> Medical Colleges<br />

www.aamc.org/meded/msop/msop2.pdf<br />

Population Health Forum<br />

<strong>University</strong> <strong>of</strong> Washington<br />

School <strong>of</strong> Public Health and Community <strong>Medicine</strong><br />

depts.washington.edu/eqhlth/index.htm<br />

Behavioral Risk Factor Surveillance System Division <strong>of</strong> Adult and Community Health National<br />

Center for Chronic Disease Prevention and Health Promotion Centers for Disease Control and<br />

Prevention<br />

U.S. <strong>Department</strong> <strong>of</strong> Health and Human Services<br />

www.cdc.gov/brfss/index.htm<br />

Healthy People 2010 National Center for Health Statistics<br />

Centers for Disease Control and Prevention<br />

U.S. <strong>Department</strong> <strong>of</strong> Health and Human Services<br />

www.cdc.gov/nchs/hphome.htm<br />

Rhyne RL, Bogue R, Kukulka G, Fulmer H, eds. Community-Oriented Primary Care: Health<br />

Care for the 21st Century. Washington, DC: American Public Health Association; 1998.<br />

Strelnick AH. Community-oriented primary care: the state <strong>of</strong> an art. Arch Fam Med.<br />

1999;8:550-2.<br />

Fox DM. The relevance <strong>of</strong> population health to academic medicine. Acad Med. 2001;76:6-7.<br />

Kindig D, Stoddart G. What is population health?. Am J Public Health. 2003;93:380-3.<br />

226


GENERAL CLINICAL CORE COMPETENCIES<br />

#15 CONTINUOUS IMPROVEMENT IN SYSTEMS OF MEDICAL PRACTICE<br />

RATIONALE:<br />

In the past clinical education had emphasized the role <strong>of</strong> the physician as an individual decision<br />

maker. Problems with cost and quality <strong>of</strong> care had usually been attributed to errors in individual<br />

decision making. In recent years, it has become clear that the individual does not function in isolation<br />

but within the context <strong>of</strong> a health care system and a health care team whose structure ranges from<br />

simple to complex. The way the system functions is critical to achieving high quality patient care,<br />

ensuring patient safety, reducing sources <strong>of</strong> errors in medicine, and promoting an environment that<br />

respects disclosure without blame. Furthermore, we have begun to focus on the patient as the center<br />

<strong>of</strong> the health care delivery system and to assess quality from the perspectives <strong>of</strong> the patient and the<br />

physician. With the patient as the center <strong>of</strong> the health care delivery system, the physician becomes a<br />

collaborative partner with other health pr<strong>of</strong>essionals who share a common goal <strong>of</strong> providing safe,<br />

accessible, high quality, evidence-based care.<br />

PREREQUISITES:<br />

Prior knowledge, skills and attitudes acquired during the pre-<strong>clerkship</strong> experience should include:<br />

Required introductory course in clinical epidemiology and biostatistics.<br />

Required introductory course in health care delivery.<br />

Required introductory course in bioethics and pr<strong>of</strong>essionalism.<br />

SPECIFIC LEARNING OBJECTIVES:<br />

A. KNOWLEDGE: Students should be able to define, describe, and discuss:<br />

1. The concept <strong>of</strong> systems-based practice. (MK, SBP)<br />

2. How patient care is affected by other pr<strong>of</strong>essionals, organizations, and society. (MK, SBP)<br />

3. The principles <strong>of</strong> clinical quality improvement, including the notion <strong>of</strong> variation in practice as a<br />

quality issue and the concept <strong>of</strong> medical care as a process which can be studied and<br />

improved. (MK, SBP)<br />

4. The analysis and improvement <strong>of</strong> systems to address common quality problems (e.g.,<br />

treatment delays, medication errors, failure to use evidence-based diagnostics/treatments,<br />

failure to provide preventive care, etc.). (MK, SBP)<br />

5. Principles <strong>of</strong> medical record organization in both inpatient and ambulatory settings. (MK, SBP)<br />

6. The importance <strong>of</strong> complete medical documentation in the context <strong>of</strong> measuring quality <strong>of</strong><br />

care, avoiding redundancy, preventing medical errors, and improving patient safety. (MK, SBP)<br />

7. The need for a multidimensional approach to the assessment <strong>of</strong> quality, including the patient’s<br />

perspective <strong>of</strong> quality. (MK, SBP)<br />

8. The relationship <strong>of</strong> quality and cost in health care from the standpoint <strong>of</strong> the individual, health<br />

care systems, and society. (MK, SBP)<br />

9. Major health care safety concerns (e.g., medication errors, wrong-site procedures, patient<br />

misidentification, miscommunication among health care givers, nosocomial infections, falls,<br />

use <strong>of</strong> restraints, etc.). (MK, SBP)<br />

10. Potential benefits and pitfalls <strong>of</strong> critical pathways/practice guidelines intended to improve the<br />

quality <strong>of</strong> care. (MK, SBP)<br />

11. Basic organizational structures and financing streams <strong>of</strong> the U.S. health care system. (MK,<br />

SBP)<br />

227


12. The fundamentals <strong>of</strong> the various type <strong>of</strong> health insurance (e.g., fee-for-service, preferred<br />

provider organization, health maintenance organization, point-<strong>of</strong>service). (MK, SBP)<br />

13. The fundamentals <strong>of</strong> Medicare and Medicaid. (MK, SBP)<br />

B. SKILLS: Students should be able to demonstrate specific skills, including:<br />

1. Using hospital-based support systems to assist in making clinical decisions (e.g., antibiotic<br />

control program, critical pathways/practice guidelines, etc.). (PC, PLI, SBP)<br />

2. Recognizing system flaws in the delivery <strong>of</strong> care (e.g., inability to arrange a post-discharge<br />

appointment within a needed time frame, delays in obtaining test results, inaccessibility <strong>of</strong><br />

medical records, etc.). (SBP)<br />

3. Using patient education materials to facilitate patients’ participation in their own care. (CS,<br />

SBP)<br />

4. Using the medical records system efficiently to produce medical notes that communicate<br />

information clearly. (PC, CS, SBP)<br />

5. Maintaining accurate documentation <strong>of</strong> preventive health measures. (PC, CS, SBP)<br />

6. Working collaboratively with other health pr<strong>of</strong>essionals in the delivery <strong>of</strong> quality care. (PC, P,<br />

SBP)<br />

7. Assessing the patients’ needs from the standpoint <strong>of</strong> the individual, family, and community.<br />

(PC, SBP)<br />

8. Identifying resource available to patients within the health care system. (PC, SBP)<br />

9. Reporting patient safety concerns and medical errors to the appropriate individuals. (CS, SBP)<br />

10. Using resources, appropriate information systems, and the tenants <strong>of</strong> evidence-based<br />

medicine to assess systems-based practice issues. (PLI, SBP)<br />

C. ATTITUDES AND PROFESSIONAL BEHAVIORS: Students should be able to:<br />

1. Recognize the importance <strong>of</strong> systems, particularly inter-pr<strong>of</strong>essional collaboration, in delivering<br />

high quality patient care. (P, SBP)<br />

2. Strive to improve the timeliness diagnostic and therapeutic decision making in order to improve<br />

quality <strong>of</strong> care, increase patient satisfaction, and reduce health care costs. (PLI, P, SBP)<br />

3. View the patient as the center <strong>of</strong> the health care delivery system. (P, SBP)<br />

4. Advocate for patients in the health care system. (P, SBP)<br />

5. Appreciate that medical error prevention and patient safety are the responsibility <strong>of</strong> all health<br />

care providers and systems and accept the appropriate degree <strong>of</strong> responsibility at the medical<br />

student level. (P, SBP)<br />

6. Appreciate the importance teamwork in delivering high quality care. (P, SBP)<br />

7. Respect other health care pr<strong>of</strong>essionals as colleagues on a patient-centered health delivery<br />

team and as mutual contributors to high quality patient care. (P, SBP)<br />

D. REFERENCES:<br />

GENERAL:<br />

Agency for Healthcare Research and Quality<br />

U.S. <strong>Department</strong> <strong>of</strong> Health and Human Services<br />

ahrq.gov<br />

QUALITY OF CARE:<br />

Institute for Healthcare Improvement<br />

228


www.ihi.org/ihi<br />

Crossing the Quality Chasm: A New Health System for the 21st Century<br />

Committee on Quality Health Care in America<br />

Institute <strong>of</strong> <strong>Medicine</strong><br />

National Academies Press, 2001<br />

National Committee for Quality Assurance<br />

www.ncqa.org<br />

National Guideline Clearing House<br />

Agency for Healthcare Research and Quality<br />

U.S. <strong>Department</strong> <strong>of</strong> Health and Human Services<br />

www.guideline.gov<br />

MEDIAL ERRORS AND PATIENT SAFETY:<br />

To Err Is Human: Building a Safer Health System<br />

Institute <strong>of</strong> <strong>Medicine</strong><br />

www.iom.edu/?id=4117&redirect=0<br />

Patient Safety Network<br />

Agency for Healthcare Research and Quality<br />

U.S. <strong>Department</strong> <strong>of</strong> Health and Human Services<br />

psnet.ahrq.gov<br />

National Patient Safety Foundation<br />

www.npsf.org<br />

Facts About Patient Safety<br />

Joint Commission on Accreditation <strong>of</strong> Healthcare Organizations<br />

www.jcaho.org/accredited+organizations/patient+safety/facts+about+pati<br />

ent+safety.htm<br />

HEALTH INSURANCE AND FINANCE:<br />

Understanding Managed Care<br />

Institute for Health Care Studies<br />

Michigan State <strong>University</strong><br />

www.ihcs.msu.edu/modules/UMC2003/UMC2003.pdf<br />

The Official U.S. Government Site for People with Medicare<br />

U.S. <strong>Department</strong> <strong>of</strong> Health and Human Services<br />

www.medicare.gov<br />

Checkup on Health Insurance Choices<br />

Agency for Healthcare Research and Quality<br />

www.ahrq.gov/consumer/insuranc.htm<br />

229


#16 OCCUPATIONAL HEALTH CARE<br />

GENERAL CLINICAL CORE COMPETENCIES<br />

RATIONALE:<br />

Despite increasing recognition <strong>of</strong> the health hazards found in living and working environments,<br />

physicians have traditionally received little formal training in the assessment and management <strong>of</strong><br />

occupational and environmental health problems.<br />

PREREQUISITES:<br />

Prior knowledge, skills, and attitudes acquired during the pre-<strong>clerkship</strong> experience should<br />

include:<br />

Required introductory course work in clinical epidemiology and biostatistics.<br />

Required introductory course work in the fundamental principles <strong>of</strong> public health.<br />

Ability to perform a complete medical history and physical exam.<br />

Ability to communicate with patients <strong>of</strong> diverse backgrounds.<br />

SPECIFIC LEARNING OBJECTIVES:<br />

A. KNOWLEDGE: Each student should be able to define, describe, and discuss :<br />

1. Common environmental diseases that are likely to be encountered by an internist and the<br />

principal etiologic agents associated with them. (MK)<br />

2. Pathogenesis <strong>of</strong> specific occupational diseases and the types <strong>of</strong> risks that may be encountered<br />

in the home or at the work site:<br />

Musculoskeletal/ergonomic or “repetitive stress” disorders (e.g. low back pain, carpal<br />

tunnel syndrome, etc.). (MK)<br />

Work related lung disorders (e.g. occupational asthma, particulate inhalation, etc.). (MK)<br />

Noise related hearing loss. (MK)<br />

Skin disorders (e.g. latex allergy and other forms <strong>of</strong> occupational dermatitis). (MK)<br />

Infectious disease exposure (e.g. hepatitis, HIV, TB, etc.). (MK)<br />

Psychological/stress related disorders (MK)<br />

3. Information sources for determining the risk <strong>of</strong> specific environmental and occupational health<br />

hazards. (MK)<br />

4. Purpose <strong>of</strong> Occupational Safety and Health Act (OSHA) regulations and the function <strong>of</strong> the<br />

National Institute for Occupational Safety and Health. (NIOSH). (MK, SBP)<br />

B. SKILLS: Students should be able to demonstrate specific skills, including:<br />

1. Obtaining an appropriate occupational history on all patients and identifying those patients<br />

whose health may have been adversely affected by their living conditions or work environment.<br />

(PC, CS)<br />

2. Considering the possibility that the patient’s illness may be related to their home or work<br />

environment. (PC)<br />

3. Providing patients with sound advice on the prevention <strong>of</strong> occupational and environmentalrelated<br />

diseases. (PC, CS)<br />

4. Accurately diagnosing and developing a cost-effective basic management plan for common<br />

occupational health problems (e.g. carpal tunnel syndrome, asthma, asbestosis). (PC, MK,<br />

SBP)<br />

230


5. Determining when to obtain consultation from an environmental and occupational medicine<br />

specialist. (PC, SBP)<br />

6. Accessing and utilizing appropriate information systems and resources to help delineate issues<br />

related to occupational health problems. (PC, PLI)<br />

C. ATTITUDES AND PROFESSIONAL BEHAVIORS: Students should be able to:<br />

1. Demonstrate an understanding that physicians have a duty and pr<strong>of</strong>essional responsibility to<br />

follow-up on conditions that are suspected <strong>of</strong> causing occupational or environmental-related<br />

illnesses. (P, SBP)<br />

2. Demonstrate commitment to using risk-benefit, cost-benefit, and evidence-based<br />

considerations in the selection <strong>of</strong> diagnostic and therapeutic interventions for occupational<br />

health problems. (PLI, P)<br />

3. Recognize the importance <strong>of</strong> patient needs and preferences when selecting among diagnostic<br />

and therapeutic options for occupational health problems. (P)<br />

4. Demonstrate ongoing commitment to self-directed learning regarding occupational health<br />

problems. (PLI, P)<br />

5. Appreciate the impact occupational health problems have on a patient’s quality <strong>of</strong> life, wellbeing,<br />

ability to work, and the family. (P)<br />

6. Recognize the importance <strong>of</strong> and demonstrate a commitment to the utilization <strong>of</strong> other health<br />

care pr<strong>of</strong>essionals in the diagnosis and treatment <strong>of</strong> occupational health problems. (P, SBP)<br />

D. REFERENCES:<br />

Occupational Safety and Health Administration<br />

U.S. <strong>Department</strong> <strong>of</strong> Labor<br />

www.osha.gov<br />

National Institute for Occupational Safety and Health<br />

Centers for Disease Control and Prevention<br />

U.S. <strong>Department</strong> <strong>of</strong> Health and Human Services<br />

www.cdc.gov/niosh/homepage.html<br />

231


#17 ADVANCED PROCEDURES<br />

GENERAL CLINICAL CORE COMPETENCIES<br />

RATIONALE:<br />

A number <strong>of</strong> advanced procedures may be performed by general internists, and occasionally thirdyear<br />

medical students under their supervision. In either case, knowledge <strong>of</strong> the key indications,<br />

contraindications, risks, and benefits <strong>of</strong> these procedures is essential for high quality patient care.<br />

Physicians, regardless <strong>of</strong> specialty, must be able to explain to their patients, in understandable terms,<br />

what will be experienced during a procedure.<br />

PREREQUISITES:<br />

Prior knowledge, skills, and attitudes acquired during the pre-<strong>clerkship</strong> experience should include:<br />

Pertinent anatomic considerations, including vascular anatomy <strong>of</strong> the extremities, wrist/hand,<br />

neck, subclavian area and groin.<br />

Pertinent anatomic landmarks important for the safe performance <strong>of</strong> thoracentesis,<br />

paracentesis, lumbar puncture, and arthrocentesis.<br />

Required introductory course in interviewing and physical examination.<br />

The fundamental tenants <strong>of</strong> informed consent.<br />

Basic training in body substance isolation procedures and sterile technique.<br />

SPECIFIC LEARNING OBJECTIVES:<br />

A. KNOWLEDGE: Students should be able to define, describe, and discuss:<br />

1. Key indications, contraindications, risks, benefits, techniques <strong>of</strong> each <strong>of</strong> the following advanced<br />

procedures:<br />

Arthrocentesis. (MK)<br />

o Elbow (olecranon bursa). (MK)<br />

o Wrist. (MK)<br />

o Knee. (MK)<br />

o Ankle. (MK)<br />

Central venous catheterization. (MK)<br />

o Internal jugular vein. (MK)<br />

o Subclavian vein. (MK)<br />

o Femoral vein. (MK)<br />

Arterial line placement. (MK)<br />

o Radial artery. (MK)<br />

o Femoral artery. (MK)<br />

Lumbar puncture. (MK)<br />

Thoracentesis. (MK)<br />

Paracentesis. (MK)<br />

2. Potential alternatives to the listed procedures. (MK)<br />

3. The patient’s probable experience during these procedures. (MK)<br />

4. Indications for and efficacy <strong>of</strong> intra-articular corticosteroid injections. (MK)<br />

B. SKILLS: Each student should be able to demonstrate specific skills, including:<br />

232


1. Participating in obtaining informed consent for advanced procedures, including the explanation<br />

<strong>of</strong> the purpose, possible complications, alternative approaches, and conditions necessary to<br />

make the procedure as comfortable, safe, and interpretable as possible. (PC, CS)<br />

2. Explaining the patient’s probable experience during the procedure in understandable terms.<br />

(PC, CS)<br />

3. Helping to position the patient and make them as comfortable as possible during the<br />

procedure. (PC)<br />

4. Assisting (under supervision, when appropriate) in the performance <strong>of</strong> the procedure. (PC)<br />

5. Demonstrating proper sterile technique and body substance isolation procedures. (PC)<br />

6. Appropriately documenting, when required, how the procedure was done as well as any<br />

complications and results. (CS)<br />

7. Ordering and interpreting appropriate diagnostic tests on fluids removed from the patient (e.g.<br />

synovial fluid, cerebrospinal fluid, pleural fluid, and ascitic fluid). (PC, MK)<br />

C. ATTITUDES AND PROFESSIONAL BEHAVIOR: Students should be able to:<br />

1. Demonstrate commitment to using risk-benefit, cost-benefit, and evidence-based<br />

considerations in the selection <strong>of</strong> procedures to be performed. (PLI, P)<br />

2. Appreciate the fear and anxiety many patients have regarding these procedures. (P)<br />

3. Make efforts to maximize patient comfort during a procedure. (P)<br />

4. Appreciate the patient’s right to refuse procedures. (P)<br />

5. Seek feedback regularly regarding procedural skills and respond appropriately and<br />

productively. (P)<br />

D. REFERENCES:<br />

Guide to procedures. In Lin GA, Lin TL, Sakurai KA, De Fer TM, eds. The Washington Manual<br />

Internship Survival Guide. 2nd ed. Philadelphia, PA: Lippincott Williams and Wilkins; 2005:178-<br />

210.<br />

Chen H, Sonneday CJ, Lillemoe KD eds. Manual <strong>of</strong> Common Bedside Surgical Procedures.<br />

2nd ed. Philadelphia, PA: Lippincott Williams and Wilkins; 2000.<br />

Ferri FF. Procedures and interpretation <strong>of</strong> results. In Ferri FF, ed. Practical Guide to the Care<br />

<strong>of</strong> the Medical Patient. 6th ed. St. Louis, MO: Mosby; 2004:903-934.<br />

233


LIST OF ERROR-PRONE ABBREVIATIONS, SYMBOLS, AND DOSE<br />

DESIGNATIONS<br />

234


November 27, 2003 Volume 8 Issue 24<br />

ISMP List <strong>of</strong> Error-Prone Abbreviations, Symbols, and<br />

Dose Designations<br />

It’s been over 2 years since we published a list <strong>of</strong> abbreviations, symbols, the organization’s list, we’ve highlighted these items with a<br />

double and dose designations that have contributed to medication errors. Now, asterisk (**). Also, effective April 1, 2004, each<br />

organization must include with the 2004 JCAHO National Patient Safety Goals calling for organiza-at least three additional items on<br />

their list. However, we hope that you tional compliance with a list <strong>of</strong> prohibited “dangerous” abbreviations, will consider others<br />

beyond the minimum JCAHO requirement. Selections acronyms and symbols, we thought an updated list would be useful. can be<br />

made from the attached list. These items should be considered Since JCAHO has specified that certain abbreviations must appear on<br />

for handwritten, preprinted, and electronic forms <strong>of</strong> communication.<br />

Abbreviations Intended Meaning Misinterpretation Correction<br />

µg Microgram Mistaken as “mg” Use “mcg”<br />

AD, AS, AU<br />

Right ear, left ear, each Mistaken as OD, OS, OU (right eye, left eye, each Use “right ear,” “left ear,” or<br />

ear<br />

eye)<br />

“each ear”<br />

OD, OS, OU<br />

Right eye, left eye, each Mistaken as AD, AS, AU (right ear, left ear, each Use “right eye,” “left eye,” or<br />

eye<br />

ear)<br />

“each eye”<br />

BT Bedtime Mistaken as “BID” (twice daily) Use “bedtime”<br />

cc Cubic centimeters Mistaken as “u” (units) Use “mL”<br />

D/C Discharge or<br />

discontinue<br />

Premature discontinuation <strong>of</strong> medications if D/C<br />

(intended to mean “discharge”) has been<br />

misinterpreted as “discontinued” when followed by<br />

a list <strong>of</strong> discharge medications<br />

235<br />

Use “discharge” and<br />

“discontinue”<br />

IJ Injection Mistaken as “IV” or “intrajugular” Use “injection”<br />

IN Intranasal Mistaken as “IM” or “IV” Use “intranasal” or “NAS”<br />

HS hs Half-strength At Mistaken as bedtime Mistaken as half-strength Use “half-strength” or “bedtime”<br />

bedtime, hours <strong>of</strong> sleep<br />

IU** International unit Mistaken as IV (intravenous) or 10 (ten) Use “units”<br />

o.d. or OD Once daily Mistaken as “right eye” (OD-oculus dexter), leading Use “daily”<br />

to oral liquid medications administered in the eye<br />

OJ Orange juice Mistaken as OD or OS (right or left eye); drugs Use "orange juice"<br />

meant to be diluted in orange juice may be given in<br />

the eye<br />

Per os By mouth, orally<br />

The “os” can be mistaken as “left eye” (OS-oculus Use “PO,” “by mouth,” or<br />

sinister)<br />

“orally”<br />

q.d. or QD** Every day Mistaken as q.i.d., especially if the period after the Use “daily”<br />

“q” or the tail <strong>of</strong> the “q” is misunderstood as an “i”<br />

qhs At bedtime Mistaken as “qhr” or every hour Use “at bedtime”<br />

qn Nightly Mistaken as “qh” (every hour) Use “nightly”<br />

q.o.d. or QOD** Every other day Mistaken as “q.d.” (daily) or “q.i.d. (four times Use “every other day”<br />

daily) if the “o” is poorly written<br />

q1d Daily Mistaken as q.i.d. (four times daily) Use “daily”<br />

q6PM, etc. Every evening at 6 PM Mistaken as every 6 hours<br />

Use “6 PM nightly” or “6 PM<br />

daily”<br />

SC, SQ, sub q Subcutaneous SC mistaken as SL (sublingual); SQ mistaken as “5<br />

every;” the “q” in “sub q” has been mistaken as<br />

Use “subcut” or<br />

“subcutaneously”<br />

“every” (e.g., a heparin dose ordered “sub q 2 hours<br />

before surgery” misunderstood as every 2 hours<br />

before surgery)<br />

ss Sliding scale (insulin) or<br />

½ (apothecary)<br />

Mistaken as “55”<br />

Spell out “sliding scale;” use<br />

“one-half” or “½”<br />

SSRI SSI<br />

Sliding scale regular<br />

insulin Sliding scale<br />

insulin<br />

Mistaken as selective-serotonin reuptake inhibitor<br />

Mistaken as Strong Solution <strong>of</strong> Iodine (Lugol's)<br />

Spell out “sliding scale (insulin)”<br />

i/d One daily Mistaken as “tid” Use “1 daily”<br />

TIW or tiw 3 times a week Mistaken as “3 times a day” or “twice in a week” Use “3 times weekly”


U or u** Unit Mistaken as the number 0 or 4, causing a 10-fold Use “unit”<br />

overdose or greater (e.g., 4U seen as “40” or 4u seen<br />

as “44”); mistaken as “cc” so dose given in volume<br />

instead <strong>of</strong> units (e.g., 4u seen as 4cc)<br />

Dose Designations<br />

and Other<br />

Information<br />

Intended Meaning Misinterpretation Correction<br />

Trailing zero<br />

after decimal<br />

point (e.g., 1.0<br />

mg)**<br />

No leading zero<br />

before a decimal<br />

dose (e.g., .5<br />

mg)**<br />

1 mg Mistaken as 10 mg if the decimal point is not seen<br />

0.5 mg Mistaken as 5 mg if the decimal point is not seen<br />

Do not use trailing zeros for<br />

doses expressed in whole<br />

numbers<br />

Use zero before a decimal point<br />

when the dose is less than a<br />

whole unit<br />

_ ISMP 2003<br />

ISMP MedicationSafetyAlert!<br />

November 27, 2003 Volume 8 Issue 24<br />

Dose Designations Intended Meaning Misinterpretation Correction<br />

and Other Information<br />

Drug name and dose<br />

run together<br />

(especially<br />

Inderal 40 mg Tegretol<br />

300 mg<br />

Mistaken as Inderal 140 mg Mistaken as<br />

Tegretol 1300 mg<br />

Place adequate space between the<br />

drug name, dose, and unit <strong>of</strong><br />

measure<br />

problematic for drug<br />

names that end in<br />

“L” such as<br />

Inderal40 mg;<br />

Tegretol300 mg)<br />

Numerical dose and<br />

The “m” is sometimes mistaken as a zero or Place adequate space between the<br />

unit <strong>of</strong> measure run<br />

two zeros, risking a 10- to 100-fold overdose dose and unit <strong>of</strong> measure<br />

10 mg 100 mL<br />

together (e.g., 10mg,<br />

100mL)<br />

Abbreviations such<br />

The period is unnecessary and could be Use mg, mL, etc. without a<br />

as mg. or mL. with a<br />

mistaken as the number 1 if written poorly terminal period<br />

mg mL<br />

period following the<br />

abbreviation<br />

Large doses without<br />

properly placed<br />

commas (e.g., 100000<br />

units; 1000000 units)<br />

100,000 units 1,000,000<br />

units<br />

100000 has been mistaken as 10,000 or<br />

1,000,000; 1000000 has been mistaken as<br />

100,000<br />

Use commas for dosing units at<br />

or above 1,000, or use words<br />

such as 100 "thousand" or 1<br />

"million" to improve readability<br />

Drug Name<br />

Abbreviations<br />

Intended Meaning Misinterpretation Correction<br />

ARA A vidarabine Mistaken as cytarabine (ARA C) Use complete drug name<br />

AZT zidovudine (Retrovir) Mistaken as azathioprine or aztreonam Use complete drug name<br />

CPZ<br />

Compazine<br />

(prochlorperazine)<br />

Mistaken as chlorpromazine<br />

Use complete drug name<br />

DPT<br />

Demerol-Phenergan- Mistaken as diphtheria-pertussis-tetanus<br />

Thorazine<br />

(vaccine)<br />

Use complete drug name<br />

DTO Diluted tincture <strong>of</strong> opium, Mistaken as tincture <strong>of</strong> opium<br />

Use complete drug name<br />

or deodorized tincture <strong>of</strong><br />

opium (Paregoric)<br />

HCl hydrochloric acid or<br />

hydrochloride<br />

Mistaken as potassium chloride (The “H” is<br />

misinterpreted as “K”)<br />

Use complete drug name unless<br />

expressed as a salt <strong>of</strong> a drug<br />

HCT hydrocortisone Mistaken as hydrochlorothiazide Use complete drug name<br />

HCTZ hydrochlorothiazide<br />

Mistaken as hydrocortisone (seen as HCT250<br />

mg)<br />

Use complete drug name<br />

MgSO4** magnesium sulfate Mistaken as morphine sulfate Use complete drug name<br />

MS, MSO4** morphine sulfate Mistaken as magnesium sulfate Use complete drug name<br />

MTX methotrexate Mistaken as mitoxantrone Use complete drug name<br />

PCA procainamide Mistaken as Patient Controlled Analgesia Use complete drug name<br />

236


PTU propylthiouracil Mistaken as mercaptopurine Use complete drug name<br />

T3<br />

Tylenol with codeine No.<br />

3<br />

Mistaken as liothyronine<br />

Use complete drug name<br />

TAC triamcinolone Mistaken as tetracaine, Adrenalin, cocaine Use complete drug name<br />

TNK TNKase Mistaken as “TPA” Use complete drug name<br />

ZnSO4 zinc sulfate Mistaken as morphine sulfate Use complete drug name<br />

Stemmed Drug Names Intended Meaning Misinterpretation Correction<br />

“Nitro” drip nitroglycerin infusion Mistaken as sodium nitroprusside infusion Use complete drug name<br />

“Norflox” norfloxacin Mistaken as Norflex Use complete drug name<br />

“IV Vanc” intravenous vancomycin Mistaken as Invanz Use complete drug name<br />

Symbols Intended Meaning Misinterpretation Correction<br />

Dram Minim<br />

Symbol for dram mistaken as “3” Symbol for Use the metric system<br />

minim mistaken as “mL”<br />

x3d For three days Mistaken as “3 doses” Use “for three days”<br />

> and < Greater than and less than Mistaken as opposite <strong>of</strong> intended; mistakenly Use “greater than” or “less than”<br />

use incorrect symbol; “< 10” mistaken as “40”<br />

/ (slash mark) Separates two doses or<br />

indicates “per”<br />

Mistaken as the number 1 (e.g., “25 units/10<br />

units” misread as “25 units and 110” units)<br />

Use “per” rather than a slash<br />

mark to separate doses<br />

@ At Mistaken as “2” Use “at”<br />

& And Mistaken as “2” Use “and”<br />

+ Plus or and Mistaken as “4” Use “and”<br />

° Hour Mistaken as a zero (e.g., q2° seen as q 20) Use “hr,” “h,” or “hour”<br />

_ ISMP 2003<br />

** Identified abbreviations above are also included on the JCAHO's "minimum list" <strong>of</strong> dangerous abbreviations, acronyms and symbols<br />

that must be included on an organization's "Do NotUse" list, effective January 1, 2004. An updated list <strong>of</strong> frequently asked questions<br />

about this JCAHO requirement can be found on their website at www.jcaho.org.<br />

ISMP MedicationSafetyAlert!<br />

237


COMPREHENSIVE WRITE-UPS<br />

Definition <strong>of</strong> Comprehensive Write-up<br />

A Comprehensive Write-up is a complete history and physical or a problem-focused note in<br />

standard Problem-Oriented Medical Record form (see following examples) and includes:<br />

a) a complete Problem List, with problems designated as “active” or<br />

“inactive/resolved” with dates <strong>of</strong> onset and resolution, respectively<br />

b) a comprehensive Assessment <strong>of</strong> at least three (3) problems from the Problem<br />

List, with discussion <strong>of</strong> differential diagnoses <strong>of</strong> undiagnosed problems (including rationale for<br />

including/excluding diagnoses) or discussion <strong>of</strong> diagnosed problems (such as course,<br />

complications, control and compliance)<br />

c) Plans, divided into Diagnostic, Therapeutic and Education Plans<br />

d) Resources and References<br />

e) Student’s name, printed and signed, followed by “MS3”<br />

f) Abbreviations: Since medical records communicate important information and<br />

may be scrutinized, abbreviations should not be used because their meaning is not universal<br />

among all readers <strong>of</strong> the medical records. Specifically, students should not use error-prone<br />

abbreviations, symbols, and dose designations (see Appendix).<br />

Submission Requirements <strong>of</strong> Comprehensive Write-ups<br />

a) 6B and 6L students on Inpatient <strong>Medicine</strong> are required to submit 3 write-ups by halfway<br />

through the inpatient rotation (averaging 1 write-up each week). The Hospital Site Coordinator<br />

will decide whether these 3 write-ups are satisfactory. If they are deemed satisfactory, then<br />

the student will not be required to submit any more write-ups. If, however, they are deemed<br />

unsatisfactory, then the student will be required to submit 1 – 3 additional write-ups, as<br />

determined by the Hospital Site Coordinator, up to a maximum <strong>of</strong> 6 write-ups.<br />

b) 6B students on Ambulatory <strong>Medicine</strong> are required to submit 2 write-ups each week for a<br />

total <strong>of</strong> 10 write-ups to their Ambulatory Preceptor.<br />

c) 6L students on Ambulatory <strong>Medicine</strong> are required to submit 2 write-ups each month for<br />

a total <strong>of</strong> 10 write-ups to their Ambulatory Preceptor.<br />

d) All write-ups, either originals with corrections/comments from the preceptor or copies <strong>of</strong><br />

the originals with corrections/comments, must be turned in by the last day <strong>of</strong> the respective<br />

inpatient or ambulatory component <strong>of</strong> the <strong>clerkship</strong>.<br />

238


Example <strong>of</strong> Inpatient History and Physical<br />

Date <strong>of</strong> Admission: 11/01/02<br />

Date <strong>of</strong> Exam: 11/01/02<br />

ID: 78-yo Japanese woman who is a widow and a retired hotel worker<br />

S/R: Patient and her daughter, who are fair historians. Medical records not available.<br />

RE: Admission to Progressive Care Unit at Kuakini Medical Center<br />

CC: Three episodes <strong>of</strong> "Bloody stools" since last night<br />

HPI: The patient is a 78-year old woman with history <strong>of</strong> hypertension, hypercholesterolemia and two<br />

previous "mild strokes", who was in her usual state <strong>of</strong> health until October 29, 2002, three days prior<br />

to admission, when she began passing bright red blood per rectum along with "dark black clots" and<br />

"black stools", The patient claims to have had more than 10 bowel movements <strong>of</strong> this kind within 10<br />

hours. She admits to feeling weak at this time with a "near-fainting" episode after which she found<br />

herself drenched with sweat, as if "someone dumped a bucket <strong>of</strong> water over my head". The patient's<br />

daughter claims that the patient may have experienced a brief loss <strong>of</strong> consciousness (less than 30<br />

seconds.) during this "near-fainting" episode, as she stopped talking for a short period <strong>of</strong> time. On the<br />

morning <strong>of</strong> October 30, 2002, the patient went in to see her physician, Dr. Shozo Ogawa. At this time<br />

the bleeding had ceased and her only complaint was weakness. Dr. Ogawa found her to be stable,<br />

and found her hemoglobin to be 12.2, and arranged for her to follow-up with a gastroenterologist. On<br />

October 31 st , at around 5:00pm, the patient again began to pass bright red blood per rectum, but<br />

without black clots. She says she passed 3 bloody bowel movements since that time until she<br />

presented to the Emergency Room on the morning <strong>of</strong> November 1st. She admits to feeling weak and<br />

"lousy" and again feeling faint, but not actually fainting.<br />

The patient denies any prior episodes <strong>of</strong> rectal bleeding. She had experienced some constipation the<br />

week before, and had used Metamucil, which had given relief. She denies fever, nausea, vomiting,<br />

diarrhea, sick contacts, chest pain, shortness <strong>of</strong> breath, recent weight changes or changes in<br />

appetite. She claims to have a chronic mild right lower quadrant abdominal pain which she attributes<br />

to her degenerative vertebral disc disease. She also admits to having "sour stomach" every few<br />

months, but no history <strong>of</strong> previously diagnosed GERD or peptic ulcer disease.<br />

In 2000, the patient was diagnosed with "degenerative disc disease" in her lower spine. She claims to<br />

have had back pain since age 17, and utilizes a back brace and cane to assist in mobilization. She<br />

has been using aspirin for the pain, with her last dose taken 10 days ago. She was told by her<br />

physician at the <strong>of</strong>fice visit three days ago that "aspirin wasn't good for her stomach" and so she has<br />

since been taking Extra-Strength Tylenol, with gives her only partial relief.<br />

PMH:<br />

Childhood illnesses: Not asked<br />

Immunizations: Not asked<br />

Adult illnesses: Two "mild strokes" (Had slurred speech) in 1970s<br />

Cervical cancer – had possible cone biopsy in 1970s<br />

Hypertension- first aware <strong>of</strong> diagnosis in 1970s<br />

Degenerative disc disease- told <strong>of</strong> diagnosis in 2000; uses back brace, cane, occasional<br />

acupuncture<br />

Cataracts bilaterally- date <strong>of</strong> diagnosis unknown<br />

Hypercholesterolemia- date <strong>of</strong> diagnosis unknown<br />

The patient has no history <strong>of</strong> bleeding disorder, liver disease, diabetes mellitus, myocardial infarction<br />

or renal disease.<br />

Hospitalizations/Surgeries:<br />

1950s Kapiolani Medical Center- birth <strong>of</strong> her children<br />

239


1960s Kapiolani Medical Center- Hysterectomy, reason for surgery unknown.<br />

1998 Queens Medical Center- "surgery for the insides coming out the vagina"<br />

Transfusions: Not asked<br />

Current medications: Covera HS (verapamil) 240mg qd<br />

Tylenol 1 tab every 4-6 hrs pm back pain, last dose taken 1 day ago<br />

Aspirin- dosage unknown, last dose taken 10 days ago<br />

Allergies: Penicillin reaction unknown; told by doctor not to take penicillin<br />

Sulfa acute onset <strong>of</strong> "red rash and skin peeling in sheets" (1960s)<br />

Cortisone face swelling<br />

FH: Patient's father died in his 40s from “stomach cancer." Mother died in her 90s <strong>of</strong> a "stroke, and<br />

had diabetes mellitus. The patient has numerous siblings, some <strong>of</strong> whom are step-siblings. One sister<br />

has diabetes. Two <strong>of</strong> the step-siblings have died, causes unknown. Health status <strong>of</strong> the other siblings<br />

are unknown. There is no family history <strong>of</strong> bleeding disorders.<br />

40s<br />

“Stomach ca”<br />

90<br />

“Stroke”<br />

“DM”<br />

Personal Pr<strong>of</strong>ile/SH: The patient is a retired hotel worker and widow. She now lives with her<br />

daughter. She denies the use <strong>of</strong> cigarettes, alcohol or illicit drugs. Her diet consists mostly <strong>of</strong> cereals,<br />

oatmeal and bread. She doesn't each much fruit or vegetables.<br />

ROS:<br />

General: See HPI. Denies weight loss, fever.<br />

Skin: Has no rash, itching, bruising.<br />

Eyes: Reports no blurry vision, other visual disturbances.<br />

Ears: Reports no hearing loss, tinnitus, pain, discharge, vertigo.<br />

Nose: Has "allergies” which cause runny nose, sneezing, cough.<br />

Mouth: Has no gingivitis, sore tongue, taste changes, dental problems<br />

Throat: Reports no pain, voice changes<br />

Pulmonary: Reports no chest pain, pneumonia, SOB, DOE, wheezing, sputum, hemoptysis<br />

Circulatory system: Has no chest pain, palpitations, dyspnea, PND, orthopnea, edema, syncope<br />

GI: See HPI; has no change in appetite, dysphagia, nausea, vomiting, rectal pain, hematemesis,<br />

diarrhea, peptic ulcer disease<br />

GU: Reports no frequency, nocturia, polyuria, urgency, dysuria, hematuria, hesitancy, urinary flow<br />

240


changes, retention, incontinence; has no history <strong>of</strong> kidney problems<br />

Female genitalia: See PMH.<br />

Breast: Not asked<br />

Sexual Hx: Not asked<br />

Endocrine system: Reports no neck mass, thyroid problems, exophthalmos, heat/cold intolerance,<br />

thirst changes<br />

Hemopoietic system: Has no lymph node enlargement, excessive bleeding, bruising, anemia<br />

Musculoskeletal system: See HPI and PMH. Has other joint or muscle pain.<br />

Nervous system: Has no history <strong>of</strong> head trauma, headaches, numbness, paralysis, convulsions,<br />

seizures, tremor, gait disturbances, coordination changes<br />

Mood: Not asked.<br />

PE:<br />

General Appearance: Patient appears well-nourished, appearing her stated age. She is lying<br />

comfortably in bed, in no evident distress. She is alert, oriented and cooperative.<br />

Vital Signs: Temp 96.0; Respirations 11; Oxygen sat 99% on Room Air; Supine- HR 89, BP 147/64;<br />

Standing- HR 110, BP 131/54<br />

Skin: Warm, dry, pale<br />

Head: Nontender over scalp<br />

Eyes: Acuity not tested. PERRL. Extraocular mucles function intact. Fundi not visualized due to<br />

cataracts.<br />

Ears: Acuity not tested. Pinna- no lesions, nontender. Canals- no bleeding. TMs not visualized due to<br />

cerumen obscuring view. Nose: normal pink mucosa nontender. no discharge.<br />

Mouth: Mucosa pink, moist, slightly pale. No lesions or bleeding. No tonsillar erythema or exudates.<br />

Neck: Supple. No thyromegaly, enlarged lymph nodes, jugular venous distention; no carotid bruits.<br />

Pulmonary: Lungs clear to percussion and ausculatation bilaterally, anteriorly and posteriorly<br />

Cardiac: No thrills, lifts or heaves. PMI palpated in left 5th ICS at the midclavicular line, non-bounding.<br />

Rate and rhythm are regular, normal S1 and S2. No murmurs, extra heart sounds heard.<br />

Abdomen: S<strong>of</strong>t, non-tender, non-distended. Normoactive bowel sounds in four quadrants. No<br />

hepatosplenomegaly by palpation..<br />

Rectal (done by ER physician- reported as showing no masses, with “pink stool”. Bright red blood on<br />

glove which tested positive with hemoccult.<br />

Extremities: Full motion in all extremities. No clubbing, cyanosis, edema. Patient was slow to stand,<br />

and had an antalgic gait she attributed to back pain.<br />

Neuro: Alert and oriented x 3. Cranial Nerves: II - XII grossly intact. Speech slightly slurred, difficult to<br />

comprehend at times. Sensation normal to light touch and 10 gram mon<strong>of</strong>ilament; motor 5/5 in all<br />

extremities. DTRs 2+ in biceps, triceps, knees and ankles. No Babinski response is noted.<br />

Admission lab results:<br />

CBC: WBC 7.9, differential: Bands 7, Segs 50, Lymphs 37, Monocytes 5, Eos O, Baso 1<br />

Hgb 9.8 (was noted to have been 12.2, 2 days PTA),<br />

Hct 28.4<br />

MCV 93.9<br />

Platelet count 238<br />

BMP: Na 140 BUN 20 PT 12.3<br />

K 3.6 Creatinine 0.7 INR 1.0<br />

Cl 110 Glucose 135 PTT 22<br />

Bicarb 25<br />

241


Problem List<br />

Problem<br />

No.<br />

Date Onset Active Problems Date Resolved Inactive/Resolved<br />

Problems<br />

1. 1960s Allergic reaction to sulfa (rash,<br />

peeling skin)<br />

2. 1970s 1970s Hx <strong>of</strong> cervical cancer<br />

s/p TAHBSO<br />

3. 1970s Hx <strong>of</strong> “mild stroke x2<br />

slurred speech<br />

4. 1970s Hypertension<br />

5. 1998 Bladder prolapse s/p<br />

corrective surgery<br />

6. 2000 Degenerative disc disease <br />

chronic lower back pain<br />

7. 10-29-02 GI bleed<br />

8. 11-01-02 Anemia<br />

9. Unknown Hypercholesterolemia<br />

10. Unknown Unknown Bilateral cataracts<br />

s/p cataract surgery<br />

11. Unknown Allergy to penicillin<br />

(unknown reaction), cortisone<br />

(facial swelling)<br />

Problem #1: Bleeding per rectum<br />

Assessment: The patient has experienced 2 episodes <strong>of</strong> bloody stools in the past three days, during<br />

which time she has had more many bloody bowel movements. During the first episode, she describes<br />

bright red blood as well as “black clots.” In the most recent episode, she reports only bright red<br />

blood. She has been feeling weak since these episodes. In addition, there is evidence that this patient<br />

has had substantial blood loss including: (1) a decrease in hemoglobin from 12.2 to 9.8 in two days<br />

with normal MCV, which suggest an acute bleed; (2) orthostatic changes with an increase in HR <strong>of</strong><br />

>20 and decrease in Systolic BP > 15mmHg suggest that the patient is hypovolemic, with a blood<br />

loss <strong>of</strong> greater than 1 Liter. In trying to identify the source <strong>of</strong> the bleed we must attempt to differentiate<br />

an upper GI bleed from a lower GI bleed, as well as confirm that the bleeding is from the rectum and<br />

not from the vagina or urethral orifice. Because the rectal exam confirms that there is blood in the<br />

rectum, we can assume the patient does have a GI bleed. With the reports <strong>of</strong> “bright red blood", it is<br />

likely that the patient is having a lower GI bleed, although a brisk upper GI bleed could also present<br />

as bright red blood per rectum. The “black clots” seen in the first episode suggest a possible upper GI<br />

bleed, in which the “clots” may have been melenic stool.<br />

The differential diagnosis <strong>of</strong> lower GI bleed include diverticulosis, colon cancer or polyps, ulcerative<br />

colitis, angiodysplasia, and hemorrhoids. Diverticulosis is likely as it most common in the elderly and<br />

can present<br />

with painless bright red blood per rectum that can result in massive hemorrhage. A diverticular<br />

242


hemorrhage is usually caused by erosion <strong>of</strong> a blood vessel by a fecalith within the diverticular sac.<br />

The patient did have some recent constipation which could have lead to the development <strong>of</strong> a<br />

fecolith. She took Metamucil for relief. It is unknown if the patient then had diarrhea, which can occur<br />

with Metamucil; but if she did, it is possible that the<br />

diarrhea in its rapid transit through the colon could have dislodged the fecalith resulting in injury to a<br />

blood<br />

vessel and hemorrhage. Diverticular bleeding stops spontaneously in the majority <strong>of</strong> patients. This<br />

was not the case for this patient. This could be due to the rupture <strong>of</strong> a large blood vessel in the colon,<br />

or it could be due to the patient's chronic use <strong>of</strong> aspirin (which can inhibit platelet aggregation<br />

resulting in decreased clotting ability for about 7-10 days, however, she claims to have not had<br />

aspirin for 10 days prior to admission).<br />

Colon cancer is also possible. Like diverticular disease, it is more common in the elderly. The patient<br />

displays signs and symptoms that are characteristic <strong>of</strong> a left colonic cancer- change in bowel habits<br />

(constipation, diarrhea) and bright red blood per rectum. Intestinal obstruction is also common with a<br />

left colon cancer, however the patient did not complain <strong>of</strong> abdominal pain or tenderness which would<br />

be expected with obstruction. A right colon carcinoma would present with an iron deficiency anemia<br />

due to chronic blood loss, which can be occult and thus unknown to the patient, and a dull vague<br />

abdominal pain. The patient did have a chronic dull lower right quadrant abdominal pain. She also<br />

had anemia (Hgb 9.8), however this anemia appears to be due to the acute blood loss and not iron<br />

deficiency as her Hgb was 12.2 two days prior to admission, and her MCV was normal.<br />

Ulcerative colitis should also be considered, although more commonly diagnosed in younger adults,<br />

there is also a small peak incidence among the elderly, ages 50-65. Although the patient is beyond<br />

this peak, UC must still be considered given that it's hallmark is bloody diarrhea. Most patients with<br />

UC will also have fever and weight loss, which this patient did not have. Angiodysplasia/AVM is also<br />

possible as it is most commonly seen in the elderly and presents with painless bright red blood per<br />

rectum. Hemorrhoids can be a cause <strong>of</strong> rectal bleeding, however there is usually associated pain<br />

and a palpable rectal mass, which this patient did not have. Infectious colitis is less likely in this<br />

patient as she is has no fever, abdominal pain or leukocytosis, all <strong>of</strong> which would be expected with an<br />

infectious process. The patient also denies any sick contacts.<br />

A brisk Upper GI bleed could also be responsible for this patient's rectal bleeding, however with such<br />

a large amount <strong>of</strong> blood loss, one would also expect some hematemesis, which did not occur. The<br />

differential would include gastritis, vascular ectasia, peptic ulcer disease and ruptured varices.<br />

Gastritis is very likely given the patient's chronic use <strong>of</strong> aspirin. However, she does not have<br />

abdominal pain which might be expected if it were severe enough to have caused this patient's<br />

bleeding. Peptic ulcer disease is less likely as pain is its predominant symptom, and the patient<br />

denied any abdominal or chest pain. It is important to rule-out a gastric ulcer in this patient, or to<br />

diagnose and treat it, as an untreated ulcer can increase her risk <strong>of</strong> developing gastric cancer. She is<br />

already at risk given her Japanese ethnicity and positive family history <strong>of</strong> gastric cancer. Ruptured<br />

varices is unlikely as the patient has no hematemesis and no history <strong>of</strong> liver disease which would<br />

cause the formation <strong>of</strong> varices. Her normal coagulation studies make liver disease unlikely. Given the<br />

numerous possible diagnoses for this patient's GI bleed it is imperative to have the patient to undergo<br />

both upper endoscopy and colonoscopy, especially given her history <strong>of</strong> both bright red blood per<br />

rectum and “black clots".<br />

Plan: Admit to acute care hospital with careful monitoring <strong>of</strong> vital signs.<br />

Plans:<br />

Diagnostic:<br />

-Place NG tube to assess for gastric bleeding<br />

243


-Consult a gastroenterologist for upper endoscopy and colonoscopy<br />

Treatment:<br />

-Type and crossmatch. Transfuse 2 units packed red blood cells to replace blood loss, since she is at<br />

risk to cntinue bleeding<br />

-No food or drink in preparation for endoscopy according to gastroenterologist instructions<br />

-Further treatment dependent on endoscopic findings. Consider initiate gastric acid blocking regimen<br />

prophylactically<br />

Patient education:<br />

-Inform patient <strong>of</strong> the possible diagnoses and the need for careful monitoring and testing<br />

-Inform patient <strong>of</strong> endoscopic procedures, explain risks and benefits, obtain informed consent<br />

-Inform patient <strong>of</strong> need for transfusion, explain risks and benefits, obtain informed consent<br />

Problem #2: Anemia<br />

Assessment: The patient has developed an acute anemia. Her hemoglobin had dropped 2.4 G/dL<br />

(from 12.2 to 9.8), which correlates with a decrease in hematocrit <strong>of</strong> approximately 7%. Hematocrit<br />

generally falls 2-3 points for every 500 mL <strong>of</strong> blood lost, making the estimated blood loss in this<br />

patient 3.5L. The normal MCV also suggests that the anemia is due to an acute blood loss. The<br />

normal coagulation studies rule-out a coagulopathy that may complicate the patient's GI bleed,<br />

although platelet dysfunction due to the patient's chronic aspirin use may exacerbate the bleed. The<br />

patient's orthostatic changes in heart rate and systolic BP is consistent with blood loss <strong>of</strong> greater than<br />

1 liter resulting in hypovolemia. The patient appears pale with pale mucous membranes and has<br />

complaints <strong>of</strong> weakness and light-headedness, which are all due to her anemia. It is important to<br />

transfuse this patient and increase her hemoglobin in order to avoid the complications associated with<br />

anemia and hypovolemia, such as high output cardiac failure and organ hypoperfusion, which may be<br />

<strong>of</strong> greater risk in the elderly.<br />

Plans:<br />

Diagnostic:<br />

-Monitor blood count every 4 hrs for continued bleeding and decrease in hemoglobin<br />

Treatment:<br />

- Place adequate intravenous access (2 large bore peripheral catheters)<br />

-Type and crossmatch. Transfuse 1 unit packed red cellss now.<br />

-Intravenous fluids: normal saline at 100cc/hour<br />

-Transfuse packed red cells to keep hemoglobin > 10 G/dL<br />

Patient education:<br />

-Inform patient <strong>of</strong> need for transfusion, explain risks and benefits, obtain informed consent<br />

-Inform patient <strong>of</strong> signs and symptoms <strong>of</strong> worsening anemia that she should be aware <strong>of</strong>, such as<br />

worsening orthostatic hypotension, weakness, faintness, pallor, tachycardia.<br />

Problem #3: Back pain<br />

Assessment: The patient has been previously diagnosed with degenerative disc disease and suffers<br />

from chronic back pain. She regularly uses a back brace and cane to assist with mobilization. She<br />

occasionally has acupuncture treatments to relieve the pain. She had been taking aspirin on a regular<br />

basis for pain, and recently changed to Extra-Strength Tylenol which gives only partial relief. Given<br />

the possibility <strong>of</strong> gastritis, it is best for the patient to avoid aspirin and any other NSAID, especially<br />

with an active GI bleed. Once the bleeding has resolved, a specific COX-2 inhibitor could be<br />

considered, although there is still a risk <strong>of</strong> GI irritation with COX2 inhibitors, the risk is less than that <strong>of</strong><br />

nonspecific NSAIDs. Given the patient's allergic reaction to sulfa, celecoxib is contraindicated;<br />

however, r<strong>of</strong>ecoxib can be used. In clinical trials with r<strong>of</strong>ecoxib, 3.9% <strong>of</strong> the patients had a reported<br />

sulfa sensitivity, none developed anaphylactic reactions. Another option would be to administer<br />

misoprostol along with the NSAID, in order to further protect the gastric mucosa. In order to<br />

244


completely avoid NSAIDs but still provide complete relief, tramadol could be considered. tramadol is<br />

a centrally acting analgesic which acts as an opiate agonist, although it is not opioid-derived.<br />

Tramadol does not irritate gastric mucosa, thus not causing an increased risk for GI bleed. Side<br />

effects reported for tramadol include constipation, dizziness, nausea, dry mouth, sweating and<br />

minimal cardiovascular effects including hypotension, tachycardia and syncope. Given the patients<br />

current gastritis, recent GI bleed and sulfa allergy, tramadol might be the best choice for pain relief in<br />

this patient should she request more complete relief than she is currently receiving with Tylenol; while<br />

being aware <strong>of</strong> possible hypotensive effects <strong>of</strong> this drug.<br />

Plans:<br />

Diagnostic: No further diagnostic studies at this time.<br />

Treatment:<br />

-Continue Tylenol 1 gram every 4-6 hrs as needed for back pain, not to exceed 4g daily<br />

-If better pain control is desired, consider starting tramadol 25 mg daily<br />

Patient education:<br />

-Inform patient <strong>of</strong> the need to avoid NSAIDs<br />

-Inform patient <strong>of</strong> the daily maximum dose <strong>of</strong> Tylenol (4 G per day) and the adverse effects that could<br />

occur in the event <strong>of</strong> toxicity<br />

-Inform patient that a trial <strong>of</strong> tramadol could be started, with attention to added side effects, if better<br />

pain control is needed<br />

--------------------------------------------------------------------------------------<br />

Resources:<br />

AHFS Drug Information, 2002<br />

Ferri. Practical Guide to the Care <strong>of</strong> the Medical Patient, 5th ed., 2001<br />

Myers. NMS <strong>Medicine</strong>, 4th ed" 2001<br />

<br />

Toby Best, MS 3<br />

245


Example <strong>of</strong> Ambulatory Note<br />

Problem List<br />

Problem<br />

#<br />

Date <strong>of</strong><br />

onset<br />

Active Problems Date<br />

resolved<br />

Inactive/Resolved<br />

Problems<br />

1. 9/14/04 Pharyngitis<br />

2. 1998 Hypertension<br />

3. 1994 Obesity<br />

4. 1973 S/P Appendectomy<br />

Date <strong>of</strong> Examination: September 17, 2004<br />

ID: 40 year old Part <strong>Hawaii</strong>an woman<br />

SR: History obtained from the patient and medical records, which are reliable.<br />

RE: Scheduled <strong>of</strong>fice visit<br />

CC: Sore throat and fever x3 days<br />

Problem #1: Pharyngitis<br />

S: This 40 year old woman presents to the <strong>of</strong>fice per chief complaint. 3 days ago patient began<br />

having fever, chills, sore throat, rhinorrhea, myalgia, and fatigue. She denies cough, dyspnea,<br />

wheezing, otalgia, dysphagia, abdominal pain, nausea, vomiting, diarrhea or constipation. She has<br />

taken Advil, which improves symptoms. There are no sick contacts. Patient is able to swallow solids<br />

and liquids.<br />

Patient is taking oral contraceptives, and has no medication allergies.<br />

O:<br />

General: Well-groomed, cheerful woman who appears stated age.<br />

Not in acute distress.<br />

VS: T 97.5°F; P 78; R 12; BP 112/68; Wt 159 lbs.; Ht. 61 in.<br />

Skin: Warm, dry, intact. No rashes, petechiae, or bruising noted.<br />

Ears: Symmetrical auricles, canals nontender. Auditory canals clear and<br />

non-erythematous bilaterally. Tympanic membrane intact and pearly with clear cone <strong>of</strong> light<br />

bilaterally.<br />

Nose:<br />

Nares patent, septum midline, clear discharge.<br />

Mouth/Throat: Erythema and yellowish exudate present on posterior pharynx and<br />

tonsils bilaterally. Mucous membranes moist. Lips without cyanosis or lesions. Uvula midline.<br />

Tongue pink, moist. Parotids<br />

nonpalpable.<br />

Neck:<br />

Supple, no lymphadenopathy, no masses, non-tender to palpation.<br />

Trachea midline.<br />

Lungs: Clear to percussion and auscultation bilaterally. No wheezes, rales or coarse breath<br />

sounds.<br />

Heart:<br />

No thrills or heaves felt. RRR. S1 and S2 <strong>of</strong> equal intensity. No<br />

murmurs or rubs heard.<br />

246


A: While the differential diagnosis <strong>of</strong> acute pharyngitis in adults includes a number <strong>of</strong> bacterial and<br />

viral pathogens, the objective in evaluating a patient with pharyngitis is to identify those with group A<br />

streptococcus (GAS) pharyngitis. The patient’s symptoms and physical findings suggest possible<br />

group A streptococcal pharygitis. Although viruses are the most common cause <strong>of</strong> acute pharyngitis,<br />

Group A strep pharyngitis accounts for approximately 10% <strong>of</strong> acute pharyngitis in adults. It presents<br />

with the sudden onset <strong>of</strong> sore throat, tonsillar exudate, tender cervical adenopathy, and fever.<br />

Malaise, headache, abdominal pain and vomiting may also occur. The absence <strong>of</strong> fever or presence<br />

<strong>of</strong> cough, conjunctivitis, hoarseness, coryza, viral exanthem, and diarrhea strongly suggest a viral<br />

rather than a streptococcal etiology. The Centor criteria for predicting GAS have been the most widely<br />

used and accepted and include, 1) tonsillar exudate; 2) tender anterior cervical adenopathy; 3) fever<br />

by history; 4) absence <strong>of</strong> cough. If three or four <strong>of</strong> these criteria are met, the positive predictive value<br />

is 40% to 60%. However, the presence <strong>of</strong> less than three has a negative predictive value <strong>of</strong> 80%. By<br />

these criteria, this patient has a 40-60% probability <strong>of</strong> having GAS pharyngitis.<br />

Throat cultures have always been the "gold standard" for diagnosing GAS pharyngitis with a<br />

sensitivity <strong>of</strong> 90% and specificity <strong>of</strong> 95-99% if collected properly. However, cultures take 24 to 48<br />

hours to grow, therefore cannot be used to decide which patients merit antibiotic therapy. Thus, rapid<br />

antigen detection test (RADT) has assumed greater importance in managing acute pharyngitis.<br />

RADT uses enzyme or acid extraction <strong>of</strong> antigen from throat swabs followed by latex agglutination,<br />

co-agglutination, or enzyme linked immunoabsorbent assay (ELISA) procedures to demonstrate<br />

antigen-antibody complexes. The majority <strong>of</strong> RADTs that are currently available have a sensitivity <strong>of</strong><br />

80-90% and specificity <strong>of</strong> >95%. It is also advantageous because it can yield results within<br />

approximately 10 minutes. Rapid identification and treatment <strong>of</strong> those with strep pharyngitis can<br />

reduce the risk <strong>of</strong> its spread, allowing the patient to return to school or work sooner, result in faster<br />

resolution <strong>of</strong> signs and symptoms (by 1-2 days), and can reduce the risk <strong>of</strong> suppurative complications<br />

and acute rheumatic fever. A positive result <strong>of</strong> either throat culture or RADT provides adequate<br />

confirmation <strong>of</strong> the presence <strong>of</strong> GAS in the pharynx. In children, a negative RADT should be<br />

confirmed with a throat culture result while in adults a negative RADT is sufficient to exclude this<br />

diagnosis, because the risk for acute rheumatic fever in adults is extremely low. Serologic tests for<br />

antistreptococcal antibody titers reflect past and not present immunologic events and are <strong>of</strong> no value<br />

in the diagnosis <strong>of</strong> acute pharyngitis.<br />

The major goal <strong>of</strong> identifying patients with GAS pharyngitis is to prescribe antibiotics to these patients<br />

and not treat the others, especially those with pharyngitis caused by a virus in order to limit antibiotic<br />

resistance. Recent guidelines from the Infectious Diseases Society <strong>of</strong> America (IDSA) regarding<br />

principles <strong>of</strong> management in cases <strong>of</strong> sore throat include: (1) use <strong>of</strong> clinical and epidemiologic<br />

features to distinguish patients who may have GAS pharyngitis; and (2) antibacterial treatment only<br />

for cases confirmed with a laboratory test (culture or rapid test). In contrast, a position paper by the<br />

American College <strong>of</strong> Physicians–American Society <strong>of</strong> Internal <strong>Medicine</strong>/American Academy <strong>of</strong> Family<br />

Physicians/US Centers for Disease Control and Prevention, while endorsing the IDSA approach in<br />

children, recommends a departure from the principle <strong>of</strong> laboratory confirmation <strong>of</strong> all adult cases. The<br />

suggested strategies are: empirically treat patients who have all four Centor clinical criteria (fever,<br />

tonsillar exudate, tender anterior cervical adenopathy, and absence <strong>of</strong> cough); do not treat nor<br />

perform diagnostic tests on patients with zero or one criterion; perform RADT on those with two or<br />

three criteria and use antibiotic treatment for patients with positive RADT results. Empiric treatment<br />

<strong>of</strong> all patients with 3 or 4 Centor criteria can result in unnecessary antibiotic exposure to at least 50<br />

percent <strong>of</strong> patients. Some authorities, including the Infectious Diseases Society <strong>of</strong> America, find this<br />

unacceptable and recommend antibiotics only if there is a positive RADT or culture. This criterion will<br />

result in undertreatment <strong>of</strong> 10-20%, but the consequences <strong>of</strong> undertreatment in adults are rare. This<br />

patient has three out <strong>of</strong> the four Centor criteria, so under both current recommendations patient<br />

should have a RADT done to determine if antibiotic therapy is warranted.<br />

247


Most oral antibiotics must be administered for the conventional 10-day regimen. Penicillin remains<br />

the treatment <strong>of</strong> choice in treating acute streptococcal pharyngitis because <strong>of</strong> its proven efficacy,<br />

safety, narrow spectrum, and low cost. Amoxicillin is <strong>of</strong>ten used in young children, while erythromycin<br />

is a suitable alternative for patients who are allergic to penicillin. First-generation cephalosporins are<br />

also acceptable for patients allergic to penicillin who do not manifest immediate-type hypersensitivity<br />

to beta-lactam antibiotics. Intramuscular benzathine penicillin G is preferred for patients who are<br />

unlikely to complete a full 10-day course <strong>of</strong> oral therapy. Antibiotics can affect the metabolism and/or<br />

efficacy <strong>of</strong> oral contraceptives, which the patient is using. Antibiotic therapy with penicillin should be<br />

started if patient’s RADT is positive.<br />

Symptomatic treatment, including antipyretics, fluids, and gargles, can be helpful. One study showed<br />

that herbal tea (Throat coat), containing ingredients such as licorice root that may relieve irritation,<br />

provided significant temporary relief to the intensity <strong>of</strong> sore throat with swallowing compared to<br />

placebo tea. However herbal teas should be used with caution in this patient because <strong>of</strong> her<br />

hypertension and potential medication interactions. Licorice may cause hypokalemia, so should<br />

probably not be used by a patient taking a thiazide diuretic. Regardless <strong>of</strong> RADT results, patient<br />

should be advised to use acetaminophen for pain and fever, drink increased amounts <strong>of</strong> fluid, and<br />

possibly gargle for symptomatic relief. Side effects <strong>of</strong> NSAIDs can worsen hypertension, and there<br />

are potential drug interactions with antihypertensive medications including β-blockers. Also, use<br />

caution in recommending a decongestant for this patient because <strong>of</strong> her hypertension. However, her<br />

hypertension is well-controlled, so it would probably be safe to use medications such as<br />

pseudephedrine.<br />

Although complications <strong>of</strong> GAS are rare in adults, they include acute rheumatic fever, acute<br />

glomerulonephritis, toxic shock syndrome, tonsillopharyngeal abscess, otitis media, sinusitis,<br />

necrotizing fasciitis, bacteremia, and meningitis. If patient tests positive for GAS, she should be<br />

educated on the possible complications and the signs and symptoms to watch for. She should also<br />

be reminded to return if her symptoms become worse or do not improve within a few days.<br />

P. Diagnostic<br />

Run a rapid antigen detection test to check for Group A Streptococcus pharyngitis.<br />

Therapeutic<br />

If RADT positive: oral penicillin 250 mg PO three times a day for 10 days. Pt should be cautioned use<br />

a back up method <strong>of</strong> contraception while taking the antibiotics because <strong>of</strong> the potential drug<br />

interactions.<br />

Acetaminophen 500 mg PO every 4-6 hours as needed for fever<br />

Salt gargles, tea to relieve sore throat.<br />

Patient Education<br />

If RADT negative: Explain rationale for not using antibiotics.<br />

If RADT positive: Stress importance <strong>of</strong> finishing the 10-day course <strong>of</strong> antibiotics, remind patient that<br />

she is still contagious until she has been on antibiotics for 24 hours, and educate her on the signs and<br />

symptoms <strong>of</strong> possible complications.<br />

Encourage patient to drink adequate amounts <strong>of</strong> fluid. Educate on “recipe” <strong>of</strong> NaCl solution for<br />

gargle- ¼ tsp table salt to a full cup <strong>of</strong> water, and remind her not to swallow this salty solution!.<br />

Reassure patient that asymptomatic household contacts do not need a routine culture <strong>of</strong> throat swab<br />

specimen, unless there is a “ping-pong” spreading <strong>of</strong> GAS pharyngitis within the family.<br />

Return for a follow-up visit if symptoms do not improve within a few days.<br />

Follow-up in 2-3 days if symptoms do not improve.<br />

248


Problem #2: Hypertension<br />

S. No chest pain, palpitations, dizziness, headaches, shortness-<strong>of</strong>-breath, wheezing, orthopnea,<br />

PND or edema. Pt is taking Ziac (bisoprolol/hydrochlorothiazide) 5 mg/6.25 mg, two tablets in the<br />

morning, and denies any symptomatic side effects from the medication. She exercises by walking for<br />

30 minutes three times a week. She eats a well-balanced diet and limits her sodium intake.<br />

O:<br />

VS: As above<br />

Eyes: Sclera clear, conjunctiva pink, no discharge. Extraocular muscles<br />

intact, pupils equal, round, reactive to light and accommodation.<br />

No papilledema, nicking, or cotton wool spots.<br />

Neck: No jugular venous distention<br />

Lungs:<br />

As above<br />

Heart: As above<br />

Circulation: Posterior tibial and pedal pulses 2+ bilaterally. No jugular venous<br />

distention.<br />

Extremities: No edema.<br />

A: Patient’s blood pressure is well-controlled with lifestyle modifications and Ziac. The JNC VII<br />

guidelines indicate that she is below goal blood pressure <strong>of</strong>


W., et al. JAMA. 2004 Apr 7;291(13):1587-95.<br />

“Principles <strong>of</strong> Appropriate Antibiotic Use for Acute Pharyngitis in Adults” Snow et al. Ann Intern<br />

Med.2001; 134: 506-508.<br />

Practice Guidelines for the Diagnosis and Management <strong>of</strong> Group A Streptococcal Pharyngitis,<br />

Infectious Disease Society <strong>of</strong> America.<br />

Harrison’s Online<br />

<br />

N.O. Itall, MS 3<br />

250


CLINICAL SKILLS EXAMINATION (CSE)<br />

STUDENT INSTRUCTIONS<br />

(Revised 6/12)<br />

DESCRIPTION<br />

The CSE is a practical examination consisting <strong>of</strong> 5 - 10 clinical problems including patient<br />

encounters and writing stations. The examination requires you to interact with standardized patients<br />

who volunteer for this exercise. These patients may have real clinical findings or are carefully trained<br />

to simulate patients with “real” medical problems. You should interview and examine these patients<br />

as if they are being seen in an actual patient care situation.<br />

GENERAL PROCEDURES<br />

You will have 15 minutes with each patient. You are responsible for pacing your visit with the patient.<br />

The timer will announce the beginning <strong>of</strong> each patient session. You will be warned when there are 5<br />

minutes remaining in each station, when to rotate to and begin the next station. Do not enter an exam<br />

room until told to begin. Each <strong>of</strong> you will start at a different point along the exam route and will<br />

continue to rotate until all stations are completed. If 2 (or more) students end up in the same station<br />

at the same time, notify the exam monitor immediately.<br />

Please knock before entering a patient's room. If you are finished evaluating the patient before the<br />

15 minutes are up, you may leave the station and begin answering questions in the writing station.<br />

Introduce yourself to the standardized patients as you would introduce yourself to any patient. End<br />

the encounter as you would end any encounter. On the door <strong>of</strong> every patient's room you will find<br />

student instructions in a RED FOLDER. Do not open the RED FOLDER until the timer says, “begin<br />

the encounter.” Reading the student instructions is considered part <strong>of</strong> your 15 minutes with the<br />

patient. In general, there will be enough time to review the instructions for 1 to 2 minutes and still<br />

complete your task with the patient.<br />

The instructions will give you the patient’s name, age, gender and reason for the visit. The<br />

instructions may also indicate where you are seeing the patient (outpatient clinic, emergency room,<br />

etc.), and the time <strong>of</strong> day (if it is different from the current time). The vital sign including the<br />

temperature, heart rate, blood pressure and respiratory rate may also be given. You should accept<br />

the vital signs as accurate and you do not need to repeat them unless you feel the case specifically<br />

requires it.<br />

Your task with the patient will be specifically defined. For example the instructions may tell you to<br />

take an appropriate history and counsel the patient . . . or take an appropriate history and perform a<br />

focused physical examination. You are to perform a focused examination <strong>of</strong> each patient that is<br />

consistent with the instructions provided. Hand washing is an essential practice when seeing patients<br />

and should be considered as a requirement during this examination. Facilities for washing your<br />

hands are available in each examining room.<br />

READ THE INSTRUCTIONS CAREFULLY BEFORE ENTERING EACH STATION. Do not take the<br />

RED FOLDER containing the student instructions into the examination room. A copy <strong>of</strong> the<br />

instructions will be placed in each room for your reference. If you feel it is necessary, you may take<br />

notes on the blank sheets provided on your clipboard. PLEASE DO NOT WRITE ON THE STUDENT<br />

INSTRUCTIONS.<br />

251


You should not perform rectal, genital, breast or corneal reflex examinations on your patients. If you<br />

feel that one or more <strong>of</strong> these maneuvers should be included as part <strong>of</strong> the work up for the patient<br />

you should inform the patient (e.g. “Mrs. Smith, the next step would be to do a pelvic and rectal<br />

examination on you”) and/or include it as part <strong>of</strong> your proposed diagnostic evaluation. As part <strong>of</strong> the<br />

encounter, you should provide the patient with your initial impression and initial management plan.<br />

End the encounter with the patient as you would end any patient encounter.<br />

You will receive a supply <strong>of</strong> “BUSINESS CARDS” pre-printed with your Examinee Identification<br />

Number. Before leaving the patient's rooms please leave one card with the patient (e.g. "If you have<br />

any questions or need to get in touch with me, here is my card"). This will allow the standardized<br />

patient to complete your evaluation forms. If you forget to give your card to the patient please leave<br />

it with the monitor and he/she will bring it in to the patient. ONCE YOU LEAVE THE PATIENT'S<br />

ROOM, YOU ARE NOT ALLOWED TO RE-ENTER FOR ANY REASON.<br />

WRITING STATIONS (PATIENT NOTE):<br />

Immediately after each patient encounter will begin a “writing station” where you will answer specific<br />

questions related to the encounter. Depending on the station, you will be writing your answers on<br />

paper or entering them on a computer.<br />

For stations requiring paper and pen, take one TEST BOOKLET from the blue folder located at the<br />

writing station. WRITE IN YOUR ASSIGNED STUDENT EXAMINEE ID# ON YOUR TEST<br />

BOOKLET and then begin answering the questions. You will have 10 minutes to answer the<br />

questions and complete any evaluation forms. Remember to write clearly, as we must be able to<br />

read your writing to grade it. When you have finished, turn your paper faced down on the desk. A<br />

proctor will pick up these forms.<br />

For stations requiring computer entry, log on to the computer using your UH USERNAME AND<br />

PASSWORD. Enter the Patient’s ID Number (located in a blue folder next to the computer monitor)<br />

to access the entry form for the station and begin answering the questions. You will have 10 minutes<br />

to answer the questions and complete any evaluation forms. When you have finished, remember to<br />

LOGOUT.<br />

In most stations, you will be asked to describe the significant positive and negative clinical findings<br />

uncovered during your interaction with the patient that will allow you to make clinical decisions, your<br />

differential diagnosis in the order <strong>of</strong> likelihood, and/or your INITIAL DIAGNOSTIC management plan<br />

(See Attached Sample Patient Note form). LIST only one item for each numbered blank line. Be as<br />

specific as possible.<br />

BREAKS<br />

You are asked not to leave the examination area during the break times and to refrain from<br />

discussing the cases with each other.<br />

CONTENT OF EXAM<br />

The clinical situations for this examination reflect the “General Core Clinical Competencies and<br />

“Training Problems” described in the <strong>Medicine</strong> 531/532 Student Handbook. You are being evaluated<br />

based on checklists and rating scales completed by the patient and by the responses you provide in<br />

the test booklet regarding your findings (writing stations). You are not allowed to consult any medical<br />

references during the exam.<br />

252


PAGERS/CELLULAR PHONES<br />

Cellular phones and pagers will not be allowed inside the examination area. We can hold<br />

them for you while you take the examination, but we will not be responsible for loss or damages.<br />

Please plan accordingly.<br />

EQUIPMENT<br />

1. STETHOSCOPE<br />

2. REFLEX HAMMER<br />

3. PENS with black ink<br />

4. NAME TAG<br />

5. PROPER ATTIRE. You should dress as you would to see patients.<br />

6. Otoscopes, ophthalmoscopes, sphygmomanometers, tongue blades, reflex hammers, drapes, clip<br />

boards and other supplies will be available in each examination room.<br />

GRADING AND FEEDBACK<br />

Student performance on each station is scored based on the following component scores, weighted<br />

as follows:<br />

Communication/Interpersonal Skills 30-40%<br />

Hx-Taking/Physical Exam/Counseling 40-50%<br />

Problem Solving (writing stations) 10-30%<br />

The Total Examination Score is calculated based on the average <strong>of</strong> the scores on each station.<br />

Students must achieve Clerkship Level Competency on this examination to receive credit for Med<br />

531, and Graduation Level Competency to receive credit for Med 541.<br />

Each student will receive a feedback letter with their Total Score and breakdown <strong>of</strong> their component<br />

scores. In addition, the average ratings by the standardized patients on specific communication skills<br />

(introducing self, active listening, showing interest etc), will also be provided. Unfortunately, we<br />

cannot return the actual checklists and written test materials without compromising the cases which<br />

may be used on future examinations.<br />

IMPORTANT REMINDER:<br />

You must have your UH Login and Password to complete this examination.<br />

253


FREQUENTLY ASKED QUESTIONS:<br />

Q1: How can I best prepare for this examination?<br />

A1: Review the Training Problems and General Core Clinical Competencies sections <strong>of</strong> your<br />

Med531/532 Student Handbook. For each <strong>of</strong> the Training Problems listed, be able take an<br />

appropriate history and focused physical examination. Understand how specific signs and symptoms<br />

relate to the differential diagnosis for each problem. Most importantly, review and practice good<br />

communication and interpersonal skills when seeing patients during the rotation.<br />

Q2: Are all the stations equal in length and difficulty?<br />

A2: Some stations, especially those requiring both a focused history and focused physical<br />

examination, seem to take everyone longer than other stations, although there is great variability<br />

among students. All <strong>of</strong> the stations are designed to be completed in the time allotted. The more<br />

difficult stations require students to focus their examination on obtaining the most relevant clinical<br />

information. The passing score for each station is adjusted for case difficulty.<br />

Q3: What does the interstation exercise (writing station or patient note) look like?<br />

A3: See example below.<br />

254


JOHN DOE -- PATIENT NOTE<br />

EXAMINEE ID # ____ ____<br />

DATE: __/__/__<br />

HISTORY: Include significant positives and negatives from history <strong>of</strong> present illness, past medical<br />

history, review <strong>of</strong> system(s), social history and family history.<br />

PHYSICAL EXAMINATION: Indicate only pertinent positive and negative findings related to the<br />

patient’s chief complaint.<br />

DIFFERENTIAL DIAGNOSES: In order <strong>of</strong><br />

likelihood (with 1 being most likely), list up to<br />

5 potential or possible diagnoses for this<br />

patient’s presentation (in many cases, fewer<br />

than 5 diagnoses are likely).<br />

1.<br />

2.<br />

3.<br />

4.<br />

5.<br />

DIAGNOSTIC WORK UP: List immediate<br />

plans (up to 5) for further diagnostic workup.<br />

1.<br />

2.<br />

3.<br />

4.<br />

5.<br />

255


NBME SUBJECT EXAM IN INTERNAL MEDICINE<br />

256


MEDICINE<br />

General Principles 1%-5%<br />

Organ Systems 95%-99%<br />

Immunologic Disorders 5%-10%<br />

Diseases <strong>of</strong> the Blood and Blood-forming Organs<br />

5%-10% Diseases <strong>of</strong> the Nervous System and Special Senses<br />

5%-10% Cardiovascular Disorders<br />

15%-20% Diseases <strong>of</strong> the Respiratory System<br />

15%-20% Nutritional and Digestive Disorders<br />

10%-15% Gynecologic Disorders<br />

1%-5%<br />

Renal, Urinary, and Male Reproductive System<br />

10%-15% Disorders <strong>of</strong> the Skin and Subcutaneous Tissues<br />

5%-10% Diseases <strong>of</strong> the Musculoskeletal System and Connective<br />

Tissue 5%-10% Endocrine and Metabolic Disorders<br />

Physician Tasks 5%-10%<br />

Promoting Health and Health Maintenance<br />

10%-15% Understanding Mechanisms <strong>of</strong> Disease<br />

20%-25% Establishing a Diagnosis<br />

40%-45% Applying Principles <strong>of</strong> Management<br />

20%-25%<br />

1. A 22-year-old woman with a 10-year history <strong>of</strong><br />

asthma comes to the physician because she has had to<br />

increase her use <strong>of</strong> her albuterol inhaler during the past 6<br />

weeks. Her asthma was previously well controlled with<br />

inhaled glucocorticoids. She has a 2-year history <strong>of</strong><br />

generalized anxiety disorder controlled with fluoxetine and a<br />

5-year history <strong>of</strong> migraines. The migraines were well<br />

controlled with sumatriptan until 4 months ago when she<br />

began to have headaches twice weekly; propranolol was<br />

added to her regimen at that time. She has been taking an<br />

oral contraceptive for the past year. She says she has been<br />

under increased stress at graduate school and in her<br />

personal life during the past 3 months; during this period,<br />

she has been drinking an average <strong>of</strong> four cups <strong>of</strong> c<strong>of</strong>fee<br />

daily (compared with her usual one cup daily). She does not<br />

drink alcohol or use illicit drugs. She appears mildly anxious<br />

but is not in respiratory distress. Scattered end-expiratory<br />

wheezes are heard. The remainder <strong>of</strong> the examination<br />

shows no abnormalities. Which <strong>of</strong> the following is the most<br />

likely cause <strong>of</strong> the exacerbation <strong>of</strong> this patient's asthma?<br />

(A) Fluoxetine therapy<br />

(B) Increased caffeine intake<br />

(C) Oral contraceptive therapy<br />

(D) Propranolol therapy<br />

(E) Sumatriptan therapy<br />

2. A 28-year-old woman has palpitations that occur<br />

approximately once a week, last 1B5 minutes, and consist <strong>of</strong><br />

rapid, regular heart pounding. The episodes start and stop<br />

suddenly and have not been associated with chest discomfort<br />

or dyspnea. There is no history <strong>of</strong> heart problems. She drinks<br />

two to three cups <strong>of</strong> c<strong>of</strong>fee daily. She rarely drinks alcohol<br />

and does not smoke. Her pulse is 96/min and regular, and<br />

blood pressure is 120/88 mm Hg. A stare and lid lag are<br />

noted. The thyroid gland is firm and 1.5 times larger than<br />

normal. There is a midsystolic click at the apex and a grade<br />

2/6, early systolic murmur at the upper left sternal border. An<br />

ECG is normal except for evidence <strong>of</strong> sinus tachycardia.<br />

Which <strong>of</strong> the following is the most appropriate next step in<br />

diagnosis?<br />

(A) Ambulatory ECG monitoring<br />

(B) Measurement <strong>of</strong> serum thyroidstimulating<br />

hormone concentration<br />

(C) Measurement <strong>of</strong> urine<br />

catecholamine concentration<br />

(D) MUGA scan<br />

(E) Echocardiography<br />

257


3. A study is conducted to assess the benefits <strong>of</strong> a new drug to 5. Two days after receiving 3 units <strong>of</strong> packed red blood cells for<br />

reduce the recurrence <strong>of</strong> colonic polyps. The results postpartum hemorrhage, a 24-year-old woman has<br />

show a number needed to treat (NNT) <strong>of</strong> 16. Which <strong>of</strong> fatigue and slight jaundice. Laboratory studies show:<br />

the following is the most accurate interpretation <strong>of</strong> this<br />

result?<br />

Hemoglobin 8.8 g/dL<br />

(A) For every 16 patients treated, 1 would<br />

Hematocrit 28% Serum total<br />

benefit from the new drug<br />

bilirubin 5 mg/dL<br />

(B) For every 100 patients treated, 16 would<br />

benefit from the new drug<br />

(C) The new drug is 1.6 times more<br />

beneficial than a placebo<br />

(D) 93% <strong>of</strong> patients taking the new drug<br />

would benefit from it<br />

(E) 84% <strong>of</strong> patients taking the new drug<br />

would not have any benefit from it<br />

4. A previously healthy 57-year-old woman comes to<br />

the physician 1 week after noticing a lump under her right<br />

arm. She is concerned that it is breast cancer because both<br />

her mother and maternal aunt died <strong>of</strong> breast cancer. She<br />

does not smoke, drink alcohol, or use illicit drugs. She has<br />

avoided the sun for the past 10 years. She notes that her skin<br />

has never tanned but always burned and freckled when<br />

exposed to the sun. She exercises daily on a stationary<br />

bicycle and eats a well-balanced diet. Her temperature is<br />

37°C (98.6°F), pulse is 82/min and regular, respirations are<br />

14/min, and blood pressure is 130/74 mm Hg. There are<br />

numerous freckles over the entire body. Examination <strong>of</strong> the<br />

right breast shows a 0.6-cm, flat, brown lesion; the lesion is<br />

mottled with deep purple and black areas and has an irregular<br />

border. There are no breast masses, dimpling, peau d'orange,<br />

or nipple discharge. The patient says that the lesion has been<br />

present for 1 year, but she has never had it examined. There<br />

is a firm, nontender mass in the right axilla. Examination<br />

shows no other abnormalities. Which <strong>of</strong> the following is the<br />

most likely diagnosis?<br />

(A) Basal cell carcinoma<br />

(B) Fibrocystic changes <strong>of</strong> the<br />

breast (C) Malignant melanoma<br />

(D) Mastitis<br />

(E) Pigmented nevus<br />

(F) Port-wine stain<br />

(G) Squamous cell<br />

carcinoma (H) Superficial breast<br />

carcinoma<br />

Liver tests are otherwise within normal limits. Which <strong>of</strong><br />

the following is the most appropriate next step in<br />

diagnosis?<br />

(A) Cytomegalovirus antibody titer<br />

(B) Direct and indirect antiglobulin<br />

(Coombs) tests<br />

(C) Monospot test<br />

(D) Serology for hepatitis B<br />

markers (E) Ultrasonography <strong>of</strong> the<br />

gallbladder<br />

6. A 30-year-old man has had nausea, vomiting, and<br />

severe colicky right flank pain radiating into the thigh for 4<br />

hours. He is afebrile. There is right costovertebral angle<br />

tenderness. Urinalysis shows RBCs too numerous to count<br />

and no bacteria. Which <strong>of</strong> the following is the most likely<br />

diagnosis?<br />

(A) Acute glomerulonephritis<br />

(B) Bacterial cystitis<br />

(C) Benign prostatic hyperplasia<br />

(D) Bladder carcinoma<br />

(E) Renal cell carcinoma<br />

(F) Urinary tract tuberculosis<br />

(G) Urolithiasis<br />

7. A 66-year-old woman comes to the emergency<br />

department 1 hour after the sudden onset <strong>of</strong> retrosternal<br />

chest discomfort accompanied by nausea and diaphoresis.<br />

She has hypotension, jugular venous distention, and a<br />

murmur <strong>of</strong> tricuspid regurgitation. An ECG shows STsegment<br />

elevation in the right precordial leads. Which <strong>of</strong> the<br />

following is the most likely diagnosis?<br />

(A) Constrictive pericarditis<br />

(B) Dissecting aortic aneurysm<br />

(C) Pericardial tamponade<br />

(D) Pulmonary emboli<br />

(E) Right ventricular infarction<br />

258


8. A 20-year-old African American woman comes to the physician because <strong>of</strong> a 6-month history <strong>of</strong> diffuse joint pain,<br />

especially in her hips and knees. During this period, she occasionally has had a rash on her nose and cheeks. She has no<br />

history <strong>of</strong> serious illness and takes no medications. Her temperature is 38.1°C (100.5°F). Examination shows warmth and<br />

swelling <strong>of</strong> the knees. Laboratory studies show:<br />

Which <strong>of</strong> the following is the most likely diagnosis?<br />

(A) Ankylosing spondylitis<br />

(B) Gouty arthritis<br />

(C) Psoriatic arthritis<br />

(D) Reactive arthritis<br />

(E) Rheumatoid arthritis<br />

(F) Septic arthritis<br />

(G) Systemic lupus<br />

erythematosus<br />

Hemoglobin 10.5 g/dL Erythrocyte sedimentation rate 40 mm/h<br />

Serum<br />

Urea nitrogen 30 mg/dL Creatinine 1.8<br />

mg/dL<br />

9. A 37-year-old man with type 1 diabetes mellitus<br />

comes to the physician for a routine examination. His only<br />

medication is insulin. His pulse is 72/min, respirations are<br />

12/min, and blood pressure is 138/88 mm Hg. Funduscopic<br />

examination shows microaneurysms and hemorrhages.<br />

Sensation to vibration and light touch is decreased over the<br />

lower extremities. His serum creatinine concentration is 1.6<br />

mg/dL. A 24-hour urine collection shows 550 mg <strong>of</strong> protein.<br />

Treatment with which <strong>of</strong> the following is most likely to slow<br />

progression <strong>of</strong> this patient's renal disease?<br />

(A) Atenolol<br />

(B) Clonidine<br />

(C) Hydralazine<br />

(D) Hydrochlorothiazide<br />

(E) Lisinopril<br />

10. A 50-year-old man is admitted to the hospital within<br />

2 hours <strong>of</strong> the onset <strong>of</strong> nausea, vomiting, and acute<br />

crushing pain in the left anterior chest. He has a family<br />

history <strong>of</strong> early coronary artery disease. The pain does not<br />

subside with the administration <strong>of</strong> nitroglycerin, sublingually.<br />

An ECG shows ST-segment elevation in leads aVL and V2<br />

through V4. Which <strong>of</strong> the following is the most appropriate<br />

management to decrease myocardial damage and<br />

mortality?<br />

(A) Administration <strong>of</strong> digitalis<br />

(B) Administration <strong>of</strong> lidocaine<br />

(C) Administration <strong>of</strong> quinidine<br />

(D) Coronary artery bypass grafting within 1<br />

week<br />

(E) Thrombolytic therapy<br />

11. A previously healthy 67-year-old woman comes to<br />

the physician with her husband because <strong>of</strong> a 4-month history<br />

<strong>of</strong> a resting tremor <strong>of</strong> her right arm. Her husband reports that<br />

her movements have been slower and that she appears less<br />

stable while walking. Examination shows increased muscle<br />

tone in the upper extremities that is greater on the right than<br />

on the left. There is decreased right arm swing. Her gait is<br />

slow and shuffling. Which <strong>of</strong> the following is the most likely<br />

explanation for this patient's symptoms?<br />

(A) Bilateral frontal lobe<br />

degeneration<br />

(B) Decreased dopaminergic input to the<br />

(C) striatum Decreased serotonergic activity in the brain<br />

stem<br />

(D) Excessive output <strong>of</strong> oxytocin<br />

(E) Excessive thalamic output <strong>of</strong><br />

norepinephrine<br />

12. A 47-year-old man comes to the physician because<br />

<strong>of</strong> a 4week history <strong>of</strong> increased thirst and urination. He has<br />

had a 23-kg (50-lb) weight gain during the past 2 years. He<br />

has no history <strong>of</strong> serious illness and takes no medications.<br />

His mother and maternal grandfather have type 2 diabetes<br />

mellitus. The patient does not smoke and drinks one beer<br />

every night. He is 175 cm (5 ft 9 in) tall and now weighs 104<br />

kg (230 lb); BMI is 34 kg/m 2 . His pulse is 90/min, and blood<br />

pressure is 150/88 mm Hg. The remainder <strong>of</strong> the<br />

examination shows no abnormalities. His serum glucose<br />

concentration is 330 mg/dL. Which <strong>of</strong> the following is the<br />

most likely underlying cause <strong>of</strong> this patient's increased<br />

serum glucose concentration?<br />

(A) Autoimmune destruction <strong>of</strong> islet cells<br />

(B) Chronic pancreatitis<br />

(C) Exogenous production <strong>of</strong><br />

corticosteroids<br />

(D) Insulin resistance<br />

(E) Pancreatic cancer<br />

259


13. A previously healthy 39-year-old woman is brought<br />

to the physician because <strong>of</strong> a tingling sensation in her<br />

fingers and toes for 2 days and rapidly progressive<br />

weakness <strong>of</strong> her legs. She had an upper respiratory tract<br />

infection 2 weeks ago. She was unable to get up from bed<br />

this morning. Examination shows weakness <strong>of</strong> all four<br />

extremities, distal greater than proximal. Deep tendon<br />

reflexes are absent. Sensation is mildly decreased over the<br />

feet. Which <strong>of</strong> the following is the most likely diagnosis?<br />

(A) Guillain-Barré syndrome<br />

(B) Multiple sclerosis<br />

(C) Myasthenia gravis<br />

(D) Poliomyelitis<br />

(E) Tick paralysis<br />

14. A previously healthy 77-year-old woman who<br />

resides in a skilled nursing care facility is brought to the<br />

emergency department 6 hours after the onset <strong>of</strong> acute<br />

midback pain that began while lifting a box. The pain does<br />

not radiate, and she has no other symptoms. She continues<br />

to carry out her daily activities. She appears to be in mild<br />

distress. She is 157 cm (5 ft 2 in) tall and weighs 47 kg (104<br />

lb); BMI is 19 kg/m 2 . Examination shows mild tenderness<br />

over T11. There is no tremor. Serum studies show a calcium<br />

concentration <strong>of</strong> 9.1 mg/dL, a urea nitrogen concentration <strong>of</strong><br />

12 mg/dL, and a creatinine concentration <strong>of</strong> 0.5 mg/dL. An x-<br />

ray <strong>of</strong> the dorsal and lumbar spine shows an anterior wedge<br />

fracture <strong>of</strong> T11. In addition to treating the pain,<br />

supplementation with which <strong>of</strong> the following is most likely to<br />

improve this patient's underlying condition?<br />

(A) 25-Hydroxycholecalciferol<br />

(B) Levothyroxine<br />

(C) Selenium<br />

(D) Vitamin C<br />

(E) Vitamin E<br />

15. A 52-year-old woman comes to the physician<br />

because <strong>of</strong> a 3-month history <strong>of</strong> diarrhea and intermittent<br />

abdominal pain that radiates to her back. The pain is<br />

exacerbated by eating. She describes her stools as greasy,<br />

foul-smelling, and difficult to flush. She has had a 4.5-kg<br />

(10-lb) weight loss during the past 4 months. She has a<br />

history <strong>of</strong> chronic alcohol abuse. Examination shows mild<br />

epigastric tenderness. An x-ray <strong>of</strong> the abdomen shows<br />

calcifications in the epigastrium. Which <strong>of</strong> the following is<br />

the most likely diagnosis?<br />

(A) Bacterial overgrowth<br />

(B) Celiac disease<br />

(C) Lactose intolerance<br />

(D) Malabsorption <strong>of</strong> bile salts<br />

(E) Pancreatic insufficiency<br />

16. A 67-year-old woman comes to the physician<br />

because <strong>of</strong> an 8-month history <strong>of</strong> progressive shortness <strong>of</strong><br />

breath. The shortness <strong>of</strong> breath initially occurred only with<br />

walking long distances but now occurs after walking ¼ mile<br />

to her mailbox. She also has a daily morning cough<br />

productive <strong>of</strong> whitish tan sputum. She has had no chest pain,<br />

palpitations, orthopnea, or paroxysmal nocturnal dyspnea.<br />

She has smoked one pack <strong>of</strong> cigarettes daily for 52 years.<br />

Her pulse is 88/min, respirations are 20/min, and blood<br />

pressure is 144/90 mm Hg. Examination shows a barrelshaped<br />

chest. Breath sounds are decreased, and faint<br />

expiratory wheezes are heard in all lung fields. There is no<br />

peripheral edema. An x-ray <strong>of</strong> the chest shows no<br />

abnormalities except for hyperinflation. Which <strong>of</strong> the<br />

following is the most likely diagnosis?<br />

(A) Angina pectoris<br />

(B) Asthma<br />

(C) Chronic obstructive pulmonary<br />

disease<br />

(D) Chronic pulmonary embolism<br />

(E) Congestive heart failure<br />

(F) Panic disorder<br />

17. A 22-year-old woman comes to the physician<br />

because <strong>of</strong> a 10-day history <strong>of</strong> pain in multiple joints. She<br />

first had pain in her right elbow, and then her right shoulder,<br />

and now has pain, redness, and swelling in her left knee that<br />

began 2 days ago. She currently has no pain in the right<br />

shoulder and elbow. There is no history <strong>of</strong> trauma. She is<br />

sexually active, and she and her partner use condoms for<br />

contraception inconsistently. Examination <strong>of</strong> the left knee<br />

shows warmth, erythema, tenderness, and s<strong>of</strong>t-tissue<br />

swelling. Range <strong>of</strong> motion <strong>of</strong> the knee is limited to 10<br />

degrees <strong>of</strong> flexion. The remainder <strong>of</strong> the examination,<br />

including pelvic examination, shows no abnormalities.<br />

Arthrocentesis <strong>of</strong> the knee joint yields 10 mL <strong>of</strong> cloudy fluid<br />

with a leukocyte count <strong>of</strong> 18,300/mm 3 (97% segmented<br />

neutrophils). Microscopic examination <strong>of</strong> the leukocytes<br />

within the joint fluid is most likely to show which <strong>of</strong> the<br />

following?<br />

(A) Acid-fast bacteria<br />

(B) Cuboidal positively birefringent crystals<br />

(C) Gram-negative diplococci<br />

(D) Gram-positive cocci in clusters<br />

(E) Needle-shaped negatively birefringent<br />

crystals<br />

18. A 47-year-old woman comes to the physician for a<br />

routine health maintenance examination. She feels well and<br />

has no history <strong>of</strong> serious illness. Her mother, brother, and<br />

sister have hypertension. The patient's pulse is 84/min, and<br />

blood pressure is 138/85 mm Hg. Examination shows no<br />

abnormalities. The most appropriate recommendation is<br />

decreased intake <strong>of</strong> which <strong>of</strong> the following?<br />

(A) Calcium<br />

(B) Carbohydrates<br />

(C) Potassium<br />

(D) Protein<br />

(E) Sodium<br />

260


19. A 32-year-old man comes to the physician because<br />

<strong>of</strong> a 12day history <strong>of</strong> abdominal cramps and bloating,<br />

diarrhea, and flatulence. He says that he started a new<br />

exercise program 2 weeks ago and has been consuming a<br />

high quantity <strong>of</strong> yogurt bars, peanut butter, and protein- and<br />

calorie-enriched milk shakes to "bulk up." He has no history<br />

<strong>of</strong> serious illness and takes no medications. His temperature<br />

is 37°C (98.6°F). The abdomen is distended, nontender, and<br />

tympanitic to percussion. Bowel sounds are increased. The<br />

remainder <strong>of</strong> the examination shows no abnormalities. Which<br />

<strong>of</strong> the following is the most likely cause <strong>of</strong> this patient's<br />

symptoms?<br />

(A) Allergy to peanuts<br />

(B) Fungal overgrowth in the small<br />

bowel<br />

(C) Incarcerated hernia<br />

(D) Irritable bowel syndrome<br />

(E) Lactase deficiency<br />

20. A 22-year-old college student comes to student<br />

health services because <strong>of</strong> a 7-day history <strong>of</strong> low-grade<br />

fever, sore throat, fatigue, and general malaise. One month<br />

ago, she had a painless vulvar ulcer that resolved<br />

spontaneously; she has been otherwise healthy. Her last<br />

menstrual period was 3 weeks ago; she uses tampons<br />

regularly. She is sexually active and has had three partners<br />

since the age <strong>of</strong> 15 years; she uses an oral contraceptive.<br />

Her temperature is 38°C (100.4°F), pulse is 100/min, and<br />

blood pressure is 110/60 mm Hg. Examination shows a rash<br />

over the palms and soles and mild cervical<br />

lymphadenopathy. Pelvic examination shows no<br />

abnormalities. Which <strong>of</strong> the following is the most appropriate<br />

pharmacotherapy?<br />

(A) Acyclovir<br />

(B) Dexamethasone<br />

(C) Interferon<br />

(D) Penicillin<br />

(E) Zidovudine (AZT)<br />

261


Answer Form for <strong>Medicine</strong> Subject Test Sample Questions<br />

(Questions 1-20)<br />

1. _____<br />

2. _____<br />

3. _____<br />

4. _____<br />

5. _____<br />

6. _____<br />

7. _____<br />

8. _____<br />

9. _____<br />

10. _____<br />

11.<br />

12.<br />

13.<br />

14.<br />

15.<br />

16.<br />

17.<br />

18.<br />

19.<br />

20.<br />

262


Answer Key for <strong>Medicine</strong> Subject Test Sample Questions<br />

(Questions 1-20)<br />

1. D 11. B<br />

2. B 12. D<br />

3. A 13. A<br />

4. C 14. A<br />

5. B 15. E<br />

6. G 16. C<br />

7. E 17. C<br />

8. G 18. E<br />

9. E 19. E<br />

10. E 20. D<br />

263

Hooray! Your file is uploaded and ready to be published.

Saved successfully!

Ooh no, something went wrong!